Download as pdf or txt
Download as pdf or txt
You are on page 1of 175

IJSO STAGE-I & II

MATHEMATICS
PREFACE

Dear Student,

In the current competitive environment, there are certain students who always aspire for more.
These students are those who have extremely good fundamentals and do not face any problem in
attempting the regular school book problems. This study material is for those students.

This material is aimed at equipping the students with first and second stage of International

Junior Science Olympiad (IJSO) i.e. National Standard Examination in Junior Science (NSEJS)
and INJSO (Indian National Junior Science Olympiad). The best use of this material is for those
students who have natural interest in problem solving and want to aspire for more.

As this material is exam oriented rather than class oriented, there is no specific class wis target.
However, based on our prior experience, this material can be best utilized by students of class
VIII, IX and X.

We hope you this material in its accuracy and appropriateness.

Team Resonance.

Pre-foundation Career Care Programmes (PCCP) Division

Every effort has been taken to make our study material error free, however any suggestion to
improve is welcome in this regard.
SUBJECT : MATHEMATICS IJSO (STAGE-1 & 2)

CONTENTS
S. NO. TOPICS PAGE NO.

1. Number System 1 - 16

2. Commercial Mathematics 17 - 37

3. Polynomials 38 - 49

4. Linear Equation In Two Variables 50 - 55

5. Linear Inequations 56 - 59

6. Quadratic equations 60 - 66

7. Progressions 67 - 73

8. Trigonometry 74 - 83

9. Co-ordinate Geometry 84 - 97

10. Sets 98 - 104

11. Plane Geometry - I 905 - 128

12. Plane Geometry - II 129 - 145

13. Mensuration 146 - 160

14. Permutation & Combination 161 - 167

15. Answer key 168 - 172

© Copyright reserved

All right reserved. Any photocopying, publishing or reproduction of full or any part of this study
material is strictly prohibited. This material belongs to only the enrolled student of RESONANCE.
Any sale/resale of this material is punishable under law.
NUMBER SYSTEM

 Pre-requisite : Before going through this chapter, you Fundamental Laws of Logarithm :
should be thorough with the basic concepts of the
Logarithm to any base a (where a > 0 and a  1 ).
same chapter explained in IX NCERT.
(i) loga a = 1
LOGARITHM
If ‘a’ is a positive real number, other than 1 and x is a (ii) loga 0 = not defined
rational number such that ax = N, then x is the [As an = 0 is not possible, where n is any number]
logarithm of N to the base a.
(iii) loga (–ve no.) = not defined.
 If ax = N then loga N = x.
[As in loga N, N will always be (+ ve)]
[ Remember N will be +ve]
Systems of Logarithm : (iv) loga (mn) = loga m + logan
There are two systems of logarithm which are generally [Where m and n are +ve numbers]
used.
m
(i) Common logarithm : In this system base is always (v) loga   = logam – logan
n
taken as 10.
(ii) Natural logarithm : In this system the base of the (vi) loga(m)n = n logam
logarithm is taken as ‘e’. Where ‘e’ is an irrational
number lying between 2 and 3. (The approximate value log m
b
of e upto two decimal places is e = 2.73) (vii) logam 
log a
b
Some Useful Results :
(i) If a > 1 then (viii)logam . logma = 1
(a) loga x < 0 [for all x satisfying 0 < x < 1]
(b) loga x = 0 for x = 1 (ix) If ‘a’ is a positive real number and ‘n’ is a positive
(c) loga x > 0 for x > 1 rational number, then
(d) x > y  loga x > loga y i.e. logax is an increasing a loga n  n
function.
(x) If ‘a’ is a positive real number and ‘n’ is a positive
 Graph of y = loga x, a > 1 rational number, then
y
logaq np  p log n
a
y = logax, a > 1 q

x' 0 (1,0) x (xi) ploga q  qloga p

(xii) logax = logay  x = y


y'
Ex.1 If log3a = 4, find value of a.
(ii) If 0 < a < 1, then Sol.  log3a = 4
(a) loga x < 0 for all x > 1
 a = 34
(b) loga x = 0 for x = 1
(c) logax > 0 for all x satisfying 0 < x < 1  a = 81.
(d) x > y  logax < loga y i.e. loga x is a decreasing 9 27 3
function. Ex.2 Find the value of log – log  log
8 32 4
Sol. Given :
 Graph of y = loga x, 0 < a < 1.
y 9 27 3  9 27  3
log – log  log  log    log
y = logax, 0 < a < 1. 8 32 4  8 32  4

(1,0)  9 32 3 
 log   
x' 0 x  8 27 4 

= log1 = 0. [ loga1 = 0]

y'

PAGE # 1
Ex.3 If 2log4x = 1 + log4(x – 1), find the value of x. FACTORS AND MULTIPLES
Sol. Given 2log4x = 1 + log4(x – 1)
 log4x2 – log4(x – 1) = 1 Factors : ‘a’ is a factor of ‘b’ if there exists a relation
such that a × n = b, where ‘n’ is any natural number.
x2 x2
 log4 =1  41 =  1 is a factor of all numbers as 1 × b = b.
x –1 x –1
 x2 = 4x – 4  x2 – 4x + 4 = 0  Factor of a number cannot be greater than the number
 (x – 2) = 0 2
 x = 2. (in fact the largest factor will be the number itself).
Thus factors of any number will lie between 1 and the
Ex.4 Evaluate : 3 2 – log3 5 . number itself (both inclusive) and they are limited.

Multiples : ‘a’ is a multiple of ‘b’ if there exists a relation


Sol. Given 3 2 – log3 5 = 3 2.3 – log3 5 [ am + n = am.an]
of the type b × n = a. Thus the multiples of 6 are
log3 5 –1 6 × 1 = 6, 6 × 2 = 12, 6 × 3 = 18, 6 × 4 = 24, and so on.
= 9. 3
= 9 × 5–1  The smallest multiple will be the number itself and the
number of multiples would be infinite.
9
= .
5  NOTE :
To understand what multiples are, let’s just take an
log11 13 log11 7 example of multiples of 3. The multiples are 3, 6, 9, 12,
Ex.5 If A = log27625 + 7 and B = log9125 + 13 ,
.... so on. We find that every successive multiples
then find the relation between A and B.
appears as the third number after the previous.
log11 13 4 log 13
Sol. A = log27625 + 7 = log 3 3 5 + 7 11 So if one wishes to find the number of multiples of 6
less than 255, we could arrive at the number through
4 log 13 255
or, A = log35 + 7 11 ....(i) = 42 (and the remainder 3). The remainder is of
3 6
no consequence to us. So in all there are 42 multiples.
log11 7
and,B = log9125 + 13 255
If one wishes to find the multiples of 36, find =7
36
log11 13
or, B = log 3 2 5 3 + 7 (and the remainder is 3).
Hence, there are 7 multiples of 36.
3 log 13
or, B = log35 + 7 11 ...(ii)
2 Ex.7 How many numbers from 200 to 600 are divisible by
By (i) and (ii) we have, 4, 5, 6 ?
Sol. Every such number must be divisible by L.C.M. of
4 3
A– log35 = B – log35 (4, 5, 6) = 60.
3 2
 600   200 
4 3  60  –  60  = 10 – 3 = 7.
   
 log35 < log35
3 2
Such numbers are 240, 300, 360, 420, 480, 540 and
 A < B. 600.
Clearly, there are 7 such numbers.
Ex.6 Find the value of log25125 – log84
Sol. Given, log25125 – log84 Factorisation : It is the process of splitting any number
into a form where it is expressed only in terms of the
= log 5 2 5 3 – log 2 3 2 2
most basic prime factors.
For example, 36 = 22 × 3 2. It is expressed in the
3 2 factorised form in terms of its basic prime factors.
= log55 – log2 2
2 3
Number of factors : For any composite number C,
3 2 which can be expressed as C = ap × bq × cr ×....., where
= – [ loga a  1 ]
2 3 a, b, c ..... are all prime factors and p, q, r are positive
integers, the number of factors is equal to
5
= . (p + 1) × (q + 1) × (r + 1).... e.g. 36 = 22 × 32. So the
6
factors of 36 = (2 +1) × (2 + 1) = 3 × 3 = 9.

PAGE # 2
Ex.8 If N = 123 × 34 ×52, find the total number of even factors (i) No. of closed lockers = No. of non-perfect square
of N. numbers from 1 to 1000 = 1000 – 31 = 969.
Sol. The factorised form of N is (ii) Upto 500 students they can go to two or more than two
(22 × 31)3 × 34 × 52  26 × 37 × 52. lockers, while the rest 500 can go to only one locker.
Hence, the total number of factors of N is (iii) The 31 perfect squares ( the last being 312 = 961)
(6 + 1) (7 + 1) (2 + 1) = 7 × 8 × 3 = 168. will be open while the lockers from 971 to 1000 is yet
Some of these are odd multiples and some are even. to be accessed last time so they all are open. The total
The odd multiples are formed only with the combination being = 31 + 30 = 61
of 3s and 5s. (iv) The no. of students that have gone to locker no.
So, the total number of odd factors is 840 is same as the no. of factors of 840.
(7 + 1) (2 + 1) = 24. 840 = 23 × 3 × 5 × 7.
Therefore, the number of even factors is So, the no. of factors = (3 + 1) (1 + 1) (1 + 1) (1 + 1) = 32.
168 – 24 = 144.
HCF AND LCM
Ex.9 A number N when factorised can be written
N = a4 × b3 × c7. Find the number of perfect squares LCM (least Common Multiple) : The LCM of given
which are factors of N (The three prime numbers numbers, as the name suggests is the smallest
a, b, c > 2). positive number which is a multiple of each of the given
Sol. In order that the perfect square divides N, the powers numbers
of ‘a’ can be 0, 2 or 4, i.e. 3.
HCF (Highest Common factor) : The HCF of given
Powers of ‘b’ can be 0, 2, i.e. 2. Power of ‘c’ can be 0, 2,
numbers, as the name suggests is the largest factor
4 or 6, i.e. 4.
of the given set of numbers.
Hence, a combination of these powers given 3 × 2 × 4
i.e. 24 numbers. Consider the numbers 12, 20 and 30. The factors and
So, there are 24 perfect squares that divides N. the multiples are

Ex.10 Directions : (i to iv) Answer the questions based on Factors


Given
Multiples
numbers
the given information.
1, 2, 3, 4, 6, 12 12 12, 24, 36, 48, 60, 72, 84, 96, 108, 120....
There are one thousand lockers and one thousand 1, 2, 4, 5, 10, 20 20 20, 40, 60, 80, 100, 120.....
students in a school. The principal asks the first student 1, 2, 3, 5, 6, 10, 15, 30 30 30, 60, 90, 120....

to go to each locker and open it. Then he asks the The common factors are 1 and 2 and the common
second student go to every second locker and close it. multiples are 60, 120...
The third student goes to every third locker, and if it is Thus the highest common factor is 2 and the least
closed, he opens it, and it is open, he closes it. The common multiple meaning of HCF it is the largest
fourth student does it to every fourth locker and so on. number that divides all the given numbers.
The process is completed with all the thousand Also since a number divides its multiple, the meaning
students. of LCM is that it is the smallest number which can be
(i) How many lockers are closed at the end of the divided by the given numbers.
process ?
(ii) How many students can go to only one locker ?  HCF will be lesser than or equal to the least of the
(iii) How many lockers are open after 970 students numbers and LCM will be greater than or equal to the
have done their job ? greatest of the numbers.
(iv) How many student go to locker no. 840 ? Ex.11 Find a number greater than 3 which when divided by
Sol. (i to iv) : Whether the locker is open or not depends on 4, 5, and 6 always leaves the same remainder 3.
the number of times it is accessed. If it is accessed odd Sol. The smallest number which, when divided by 4, 5 and
number of times, then it is open while if it is accessed 6, leaves the remainder 3 in each case is
even number of times then it is closed. LCM (4, 5 and 6) + 3 = 60 + 3 = 63.
How many times a locker will be accessed depends
on the locker no. If it contains odd number of factors, Ex.12 In a school 437 boys and 342 girls have been divided
then it will be open and if it contains even number of into classes, so that each class has the same number
factors. Then it will be closed. We know that a perfect of students and no class has boys and girls mixed.
square contains odd number of factors while a What is the least number of classes needed?
non-perfect square contains even number of factors. Sol. We should have the maximum number of students in
Thus the lockers with perfect square number will be a class. So we have to find HCF (437, 342) = 19.
open and the number of these perfect squares from 1 HCF is also the factor of difference of the number.
to 1000 determines the no. of open lockers. 437 342
 Number of classes = +
19 19
` = 23 + 18 = 41 classes.

PAGE # 3
 For any two numbers x and y,
DIVISIBLITY
x × y = HCF (x, y) × LCM (x, y).
Division Algorithm : General representation of result
HCF and LCM of fractions :
is,
LCM of numerators
LCM of fractions = HCF of deno min ators Dividend Re mainder
 Quotient 
Divisor Divisor
HCF of numerators
HCF of fractions = LCM of deno min ators
Dividend = (Divisor × Quotient ) + Remainder
Make sure the fractions are in the most reducible form.
Ex.18 On dividing 15968 by a certain number, the quotient
8 16 2 10 is 89 and the remainder is 37. Find the divisor.
Ex.13 Find the H.C.F. and L.C.M. of , , and .
9 81 3 27
Dividend  Re mainder 15968  37
Sol. Divisor = 
H.C.F. of (2, 8,16,10 ) 2 Quotient 89
Sol. H.C.F. of given fractions = = ,
L.C.M. of (3, 9, 81, 27 ) 81 = 179.
 NOTE :
L.C.M. of (2, 8,16,10 ) 80 (i) (xn – an) is divisible by (x – a) for all the values of n.
L.C.M. of given fractions = = .
H.C.F. of (3, 9, 81, 27 ) 3 (ii) (xn – an) is divisible by (x + a) and (x – a) for all the
Ex.14 Find the least number which when divided by 6, 7, 8, 9 even values of n.
and 10 leaves remainder 1. (iii) (xn + an) is divisible by (x + a) for all the odd values of n.
Sol. As the remainder is same
Required number = LCM of divisors + Remainder Test of Divisibility :
= LCM (6, 7, 8, 9, 10) +1 No. Divisiblity Te st
= 2520 + 1 2 U nit digit s hould be 0 or even
= 2521. 3 The s um of digits of no. s hould be divis ible by 3

Ex.15 Six bells start tolling together and they toll at intervals 4 The no form ed by las t 2 digits of given no. s hould be divis ible by 4.

of 2, 4, 6, 8, 10, 12 sec. respectively, find 5 U nit digit s hould be 0 or 5.


(i) after how much time will all six of them toll together ? 6 N o s hould be divis ible by 2 & 3 both
(ii) how many times will they toll together in 30 min ? 8 The num ber form ed by las t 3 digits of given no. s hould be divis ible by 8.
Sol. The time after which all six bells will toll together must
9 Sum of digits of given no. s hould be divis ible by 9
be multiple of 2, 4, 6, 8, 10, 12. The difference betw een s um s of the digits at even & at odd places
11
Therefore, required time = LCM of time intervals. s hould be zero or m ultiple of 11.
25 Las t 2 digits of the num ber s hould be 00, 25, 50 or 75.
= LCM (2, 4, 6, 8, 10, 12) = 120 sec.
Therefore after 120 s all six bells will toll together. Rule for 7 : Double the last digit of given number and
After each 120 s, i.e. 2 min, all bell are tolling together.
subtract from remaining number the result should be
 30  zero or divisible by 7.
Therefore in 30 min they will toll together   1
 2 
Ex.19 Check whether 413 is divisible by 7 or not.
= 16 times
1 is added as all the bells are tolling together at the Sol. Last digit = 3, remaining number = 41, 41 – (3 x 2) = 35
start also, i.e. 0th second. (divisible by 7). i.e. 413 is divisible by 7.
This rule can also be used for number having more
Ex.16 LCM of two distinct natural numbers is 211. What is
than 3 digits.
their HCF ?
Sol. 211 is a prime number. So there is only one pair of Ex.20 Check whether 6545 is divisible by 7 or not.
distinct numbers possible whose LCM is 211, Sol. Last digit = 5, remaining number 654, 654 – (5 x 2)
i.e. 1 and 211. HCF of 1 and 211 is 1. = 644; 64 – (4 x 2) = 56 divisible by 7. i.e. 6545 is
Ex.17 An orchard has 48 apple trees, 60 mango trees and divisible by 7.
96 banana trees. These have to be arranged in rows
Rule for 13 : Four times the last digit and add to
such that each row has the same number of trees and
remaining number the result should be divisible by
all are of the same type. Find the minimum number of
13.
such rows that can be formed.
Sol. Total number of trees are 204 and each of the trees Ex.21 Check whether 234 is divisible by 13 or not .
are exactly divisible by 12. HCF of (48, 60, 96). Sol. 234, (4 x 4) + 23 = 39 (divisible by 13), i.e. 234 is divisible
204 by 13.
 = 17 such rows are possible.
12

PAGE # 4
Rule for 17 : Five times the last digit of the number and
REMAINDERS
subtract from previous number the result obtained
should be either 0 or divisible by 17. The method of finding the remainder without actually

Ex.22 Check whether 357 is divisible by 17 or not. performing the process of division is termed as
Sol. 357, (7 x 5) – 35 = 0, i.e. 357 is divisible by 17. remainder theorem.

Rule for 19 : Double the last digit of given number and Remainder should always be positive. For example if
add to remaining number The result obtained should we divide –22 by 7, generally we get –3 as quotient
be divisible by 19. and –1 as remainder. But this is wrong because
remainder is never be negative hence the quotient
Ex.23 Check whether 589 is divisible by 19 or not.
Sol. 589, (9 x 2) + 58 = 76 (divisible by 19), i.e. the number should be –4 and remainder is +6. We can also get
is divisible by 19. remainder 6 by adding –1 to divisor 7 ( 7–1 = 6).

Ex.24 Find the smallest number of six digits which is exactly Ex.28 Two numbers, x and y, are such that when divided by
divisible by 111. 6, they leave remainders 4 and 5 respectively. Find the
Sol. Smallest number of 6 digits is 100000. remainder when (x2 + y2) is divided by 6.
On dividing 100000 by 111, we get 100 as remainder. Sol. Suppose x = 6k1 + 4 and y = 6k2 + 5
 Number to be added = (111 – 100) = 11. x2 + y2 = (6k1 + 4)2 + (6k2 + 5)2
Hence, required number = 100011. = 36k12 + 48k1 + 16 + 36k22 + 60k2 + 25
Ex.25 Find the largest four digit number which when = 36k12 + 48k1 + 36k22 + 60k2 + 41
reduced by 54, is perfectly divisible by all even natural Obviously when this is divided by 6, the remainder will
numbers less than 20. be 5.
Sol. Even natural numbers less than 20 are 2, 4, 6, 8, 10,
Ex.29 A number when divided by 259 leaves a remainder
12, 14, 16, 18.
139. What will be the remainder when the same
Their LCM = 2 × LCM of first 9 natural numbers
number is divided by 37 ?
= 2 × 2520 = 5040.
This happens to be the largest four-digit number Sol. Let the number be P.
divisible by all even natural numbers less than 20. 54 So, P – 139 is divisible by 259.
was subtracted from our required number to get this
P  139
number. Let Q be the quotient then, =Q
259
Hence, (required number – 54) = 5040
 Required number = 5094.  P = 259Q + 139

Ex.26 Ajay multiplied 484 by a certain number to get the P 259 Q  139
 =
result 3823a. Find the value of ‘a’. 37 37
Sol. 3823a is divisible by 484, and 484 is a factor of 3823a.
 259 is divisible by 37,
4 is a factor of 484 and 11 is also a factor of 484.
 When 139 divided by 37, leaves a remainder of 28.
Hence, 3823a is divisible by both 4 and 11.
To be divisible by 4, the last two digits have to be Ex.30 A number being successively divided by 3, 5 and 8
divisible by 4. leaves remainders 1, 4 and 7 respectively. Find the
‘a’ can take two values 2 and 6. respective remainders if the order of divisors be
38232 is not divisible by 11, but 38236 is divisible by reversed.
11. Sol.
Hence, 6 is the correct choice. 3 x
5 y 1
Ex.27 Which digits should come in place of  and $ if the 8 z 4
number 62684$ is divisible by both 8 and 5 ? 1 7
Sol. Since the given number is divisible by 5, so 0 or 5 must
 z = (8 × 1 + 7) = 15 ; y = (5z + 4) = (5 × 15 + 4) = 79 ;
come in place of $. But, a number ending with 5 in
x = (3y + 1) = (3 × 79 + 1) = 238.
never divisible by 8. So, 0 will replace $.
Now,
Now, the number formed by the last three digits is 40, 8 238
which becomes divisible by 8, if  is replaced by 4 or 8. 5 29 6
Hence, digits in place of  and $ are (4 or 8 or 0) and 0 3 5 4
1 2
respectively.
 Respective remainders are 6, 4, 2.

PAGE # 5
16
Ex.31 A number was divided successively in order by 4, 5 Ex.36 What is the remainder when 1415 is divided by 5 ?
and 6. The remainders were respectively 2, 3 and 4. 16
15
Then find out the number. Sol. 14 = (15 –1)odd = 15n + (–1)odd, i.e. a (multiple of 5)
Sol. 4 x
–1. Thus when divided by 5 the remainder will be (–1),
5 y 2 i.e. 4.
6 z 3
1 4 Ex.37 What is the remainder when 357 + 27 is divided by
28?
 z = (6 × 1 + 4) = 10 Sol. 357 = (33)19
 y = (5 × z + 3) = (5 × 10 + 3) = 53  357 + 27 = (27)19 + 27
 x = (4 × y + 2) = (4 × 53 + 2) = 214 = (28 – 1)19 + 27
Hence, the required number is 214. = 28M + (–1)19 + 27 [Expand by binomial theorem]
Ex.32 In dividing a number by 585, a student employed the = 28M – 1 + 27
method of short division. He divided the number = 28M + 26
successively by 5, 9 and 13 (factors of 585) and got the When 28M + 26 divided by 28, the remainder is 26.
remainders 4, 8 and 12. If he had divided number by Hence, the required remainder is 26.
585, then find out the remainder. Ex.38 What is the remainder when 82361 + 83361 + 84361
Sol.
5 x + 85361 + 86361 is divided by 7?
9 y 4 Sol. 82 361 + 83 361 + 84 361 + 85 361 + 86 361 = [(84 – 2) 361
13 z 8 + (84 – 1)361 + 84361 + (84 + 1)361 + (84 + 2)361]
1 12
Since, 84 is a multiple of 7, then the remainder will be
Now, 1169 when divided by 585 gives remainder when, (– 2)361 + (–1)361 + 1361 + 2361 is divided by 7 is
= 584. (– 2)361 + (–1)361 + 1361 + 2361 = 0. So the remainder is
To find the remainder of big number zero.

 NOTE :
CYCLICITY
(i) Binomial Expansion :
n n(n  1) We are having 10 digits in our number systems and
(a + b)n = an + an–1b + an – 2b2 + .... + bn, or some of them shows special characteristics like they,
1! 2!
repeat their unit digit after a cycle, for example 1 repeat
n n(n  1)
(a – b)n = an – an–1b + an– 2b2 – ......+ (– 1)nbn. its unit digit after every consecutive power. So, its
1! 2! cyclicity is 1 on the other hand digit 2 repeat its unit
Hence, first term is purely of a i.e an and last digit is
digit after every four power, hence the cyclicity of 2 is
purely of b, i.e. bn.
four. The cyclicity of digits are as follows :
(ii) Total number of terms in the expansion of (a + b)n is
(n + 1). Digit Cyclicity
0, 1, 5 and 6 1
Ex.33 What is the remainder when 738 is divided by 48.
4 and 9 2
19 19 19
7 38 72  49  48  1 2, 3, 7 and 8 4
Sol. = = = so by using
48 48 48 48
binomial expansion, we can say that 18 terms are So, if we want to find the last digit of 245, divide 45 by 4.
completely divisible by 48 but the last term which is The remainder is 1 so the last digit of 245 would be
same as the last digit of 21 which is 2.
 119
is not divisible. So, 119 = 1 is the remainder..
48 To Fi n d t h e U n i t Di g i t i n Ex p o n e n t i a l
Ex p r e s s i o n s :
Ex.34 What is the remainder if 725 is divided by 4?
Sol. 725 can be written (8–1)25. There are 26 terms in all and (i) When there is any digit of cyclicity 4 in unit’s place.
first 25 terms are divisible by 8, hence also by 4. The Since, when there is 2 in unit’s place then in 21 unit
last term is (–1)25. Hence, (8 –1)25 can be written digit is 2, in 22 unit digit is 4, in 23 unit digit is 8, in 24 unit
8X – 1 or 4Y –1 ( where Y = 2X). So, 4Y – 1 divided by digit is 6, after that the unit’s digit repeats. e.g. unit digit
4 leaves the remainder 3. (12)12 is equal to the unit digit of, 24 i.e.6.
Ex.35 What is the remainder if 345 is divided by 8 ?
Ex.39 In (32)33 unit digit is equal to the unit digit of 21 i.e. 2.
Sol. 345 can be written as 922 × 3. 9 can be written as (8 + 1).
Hence, any power of 9 can be written as 8N + 1. In Ex.40 In (23)15 unit digit is equal to the unit digit of 33 i.e. 7.
other words, any power of 9 is 1 more than a multiple
Ex.41 In (57)9 unit digit is equal to the unit digit of 71 i.e. 7.
of 8. Hence, (8N + 1) × 3 leaves remainder 3 when
divided by 8. Ex.42 In (678)22 unit digit is equal to the unit digit of 82 i.e. 4.

PAGE # 6
(ii) When there is any digit of cyclicity 2 in unit’s place.
Since, when there is 4 in unit’s place then in 41 unit HIGHEST POWER DIVIDING A FACTORIAL
digit is 4, in 42 unit digit is 6 and so on.
Factorial n : Product of n consecutive natural numbers
33
Ex.43 In (34) unit digit is 4. is known as ‘factorial n’ it is denoted by ‘n!’.
So, n! = n(n – 1)(n – 2)...321. e.g. 5! = 5 × 4 × 3 × 2 × 1 = 120.
Ex.44 In (29)15 unit digit is 9.
 The value of factorial zero is equal to the value of
Ex.45 In (49)18 unit digit is 1.
factorial one. Hence 0! = 1 = 1!
(iii) When there is any digit of cyclicity 1 in unit’s The approach to finding the highest power of x dividing
place.
Since, when there is 5 in unit’s place then in 51 unit y  y   y 
y! is     2    3  ......., where [ ] represents just
digit is 5, in 52 unit digit is 5 and so on. x x  x 

Ex.46 In (25)15 unit digit is 5. the integral part of the answer and ignoring the fractional
part.
Ex.47 In (46)13 unit digit is 6.
Ex.52 What is the highest power of 2 that divides 20!
Ex.48 Find the last digit of
completely?
(i) 357 (ii) 1359
Sol. 20! = 1 × 2 × 3 × 4 ×....× 18 × 19 × 20 = 1 × (21) × 3 × (22)
57
Sol. (i) The cyclicity of 3 is 4. Hence, gives the remainder × 5 × (21 × 31) × 7 × (23) × ..... so on. In order to find the
4
highest power of 2 that divides the above product, we
1. So, the last digit of 357 is same as the last digit of 31,
need to find the sum of the powers of all 2 in this
i.e. 3.
expansion. All numbers that are divisible by 21 will
(ii) The number of digits in the base will not make a
difference to the last digit. It is last digit of the contribute 1 to the exponent of 2 in the product
base which decides the last digit of the number itself.
20
59 = 10. Hence, 10 numbers contribute 21 to the
For 1359, we find which gives a remainder 3. So 21
4 product. Similarly, all numbers that are divisible by
the last digit of 1359 is same as the last digit of 33, i.e. 7. 22 will contribute an extra 1 to the exponent of 2 in the
Ex.49 Find unit’s digit in y = 717 + 734 20
product, i.e = 5. Hence, 5 numbers contribute an
Sol. 717 + 734 = 71 + 72 = 56, Hence the unit digit is 6 22
6476
extra 1 to exponents. Similarly, there are 2 numbers
Ex.50 What will be the last digit of (73 )75 that are divisible by 23 and 1 number that is divisible
by 24. Hence, the total 1s contributed to the exponent
6476 76
Sol. Let (73 )75 = (73)x where x = 75 64 = (75)even power of 2 in 20! is the sum of ( 10 + 5 +2 +1) = 18. Hence,
 Cyclicity of 3 is 4 group of all 2s in 20! gives 218 x (N), where N is not
 To find the last digit we have to find the remainder divisible by 2.
when x is divided by 4. If 20! is divided by 2x then maximum value of x is 18.
x = (75)even power = (76 – 1)even power , where n is divided by
Ex.53 What is the highest power of 5 that divides of
4 so remainder will be 1.
x = 100! = 100 × 99 × 98 × ...... × 3 × 2 × 1.
6476
Therefore, the last digit of (73 )75 will be 31 = 3. Sol. Calculating contributions of the different powers of 5,
100 100
75 63
55
we have 1 = 20, = 4.
Ex.51 What will be the unit digit of (87 ) . 5 52
Hence, the total contributions to the power of 5 is 24, or
6355 55
75
Sol. Let (87 ) = (87)x where x = 75 63 = (75)odd the number 100! is divisible by 524.
 Cyclicity of 7 is 4. Ex.54 How many zeros at the end of first 100 multiples
 To find the last digit we have to find the remainder of 10.
when x is divided by 4. Sol. First 100 multiple of 10 are = 10 × 20 × 30 × ......× 1000
x = (75)odd power = (76 – 1)odd power
= 10100 (1 × 2 × 3 × .......× 100)
where x is divided by 4 so remainder will be –1 or 3, but
= 10100 × 1024 × N
remainder should be always positive.
= 10124 × N
6355
Therefore, the last digit of (87 )75 will be 73 = 343. Where N is not divisible by 10
So, there are 124 zero at the end of first 100 multiple of
6355
Hence, the last digit is of (87 )75 is 3. 10.

PAGE # 7
Ex.55 What is the highest power of 6 that divides 9! (i) Conversion from base 10 to any other base :
9 9
Sol. By the normal method. = 1 and 2 = 0. Thus Ex.58 Convert (122)10 to base 8 system.
6 6
answers we get is 1 which is wrong. True there is just Sol.
8 122
one multiple of 6 from 1 to 9 but the product 2 × 3 = 6 8 15 2
and also 4 × 9 = 36, can further be divided by 6. Thus, 8 1 7
when the divisor is a composite number find the 0 1
highest power of its prime factors and then proceed. In
The number in decimal is consecutively divided by the
this case, 9! can be divided by 27 and 34 and thus by 64
(In this case we need not have checked power of 2 as number of the base to which we are converting the

it would definitely be greater than that of 3). decimal number. Then list down all the remainders in
the reverse sequence to get the number in that base.
Ex.56 What is the largest power of 12 that would divide 49! ?
So, here (122) 10 = (172)8.
Sol. To check the highest power of 12 in 49!, we need to
check the highest powers of 4 and 3 in it. Ex.59 Convert (169)10 in base 7.
Highest power of 3 in 49! = 22
Highest power of 2 in 49! = 46 7 169
7 24 1
46 3
 Highest power of 4 in 49! = = 23 Sol. 3
7
2 0 3
 Highest power of 12 will be 22. (Since the common
(169)10 =(331)7
power between 3 and 4 is 22).
Ex.60 Convert (0.3125)10 to binary equivalent.
Ex.57 How many zeros will be there at the end of 36!36! ?
Sol.
Sol. Highest power of 5 in 36! is 8. Integer
2 ´ 0.3125 = [0.625] 0
So, there will be 8 zeros at the end of 36!.
2 ´ 0.625 = [1.25] 1
So, at the end of 36!36! , there will be 8 × 36! zeros. 2 ´ 0.25 = [0.50] 0
2 ´ 0.50 = [1.00] 1
Thus
BASE SYSTEM (0.3125)10 = (0.0101)2

The number system that we work in is called the Ex.61 Convert (1987.725)10  (........)8
‘decimal system’. This is because there are 10 digits Sol. First convert non-decimal part into base 8.
in the system 0-9. There can be alternative system that
8 1987
can be used for arithmetic operations. Some of the 8 248 3
most commonly used systems are : binary, octal and 8 31 0
hexadecimal. 8 3 7
These systems find applications in computing. 0 3
Binary system has 2 digits : 0, 1.
Octal system has 8 digits : 0, 1,..., 7.  (1987)10 = (3703)8
Hexadecimal system has 16 digits : 0, 1, 2,..., 9, A , B, Now we have to convert (0.725)10 (........)8
C, D, E, F. Multiply
After 9, we use the letters to indicate digits. For instance, 0.725 × 8 = [5.8] ...5
A has a value 10, B has a value 11, C has a value 12,... 0.8 × 8 = [6.4] ...6
0.4 × 8 = [3.2] ...3
so on in all base systems.
0.2 × 8 = [1.6] ...1
The counting sequences in each of the systems would 0.6 × 8 = [4.8] ...4
be different though they follow the same principle. Keep on accomplishing integral parts after

Conversion : Conversion of numbers from (i) decimal multiplication with decimal part till decimal part is zero.

system to other base system. (ii) other base system to  (0.725)10 = (0.56314...)8
decimal system.  (1987.725)10 = (3703.56314...)8

PAGE # 8
(ii) Conversion from any other base to decimal Ex.68 The sum of first n natural numbers is a three-digit
system :
number, all of whose digits are the same. What is the
Ex.62 Convert (231)8 into decimal system. value of n?
Sol. (231)8 , the value of the position of each of the numbers
( as in decimal system) is : Sol. In 5 seconds, you can solve the equation
1 = 80 × 1 n(n  1)
= aaa (111, 222, etc) . How do you proceed
3 = 81 × 3 2
2 = 82 × 2 next ? If you think it's hit-and-trial from this point, you
Hence, (231)8 = (80 × 1 + 81 × 3 + 82 × 2)10
are wrong. Here goes the simple logic. It might strike
(231)8 = (1 + 24 + 128)10
(231)8 = (153)10 you instantly if you have been working with numbers:

Ex.63 Convert (0.03125)10 to base 16. n(n  1)


= aaa = a × 111 = a × 3 × 37
Sol. 16  0.03125 = 0.5 0 2
16  0.5 = 8.0 8  n(n + 1) = 6a x 37
So (0.03125)10 = (0.08)16
Look at the L.H.S. of the equation, n(n + 1) is a product
Ex.64 Convert (761.56)8  (......)16 of two consecutive natural numbers. Therefore, R.H.S.
Sol. In such conversion which are standard form
should also be a product of two consecutive natural
conversions, it is easier to
(761.56)8  (.....)2  (.....)16 numbers. One of the numbers is 37. Therefore, what
Converting every digit in base 8 to base 2, could the other number 6a, consecutive to 37 be? It can
(111110001.101110)2  (1F1.B8)16
only be 36, giving a = 6 and n = 36. Therefore, 36
Ex.65 Convert (3C8.08)16 to decimal numbers have been summed up and their sum is equal
Sol. (3C8.08)16 = 3  162 + C  161 + 8  16 0+ 0  16–1 + 8  16–2
to 666.
= 768 + 192 + 8 + 0 + 0.03125
= (968.03125)10 Ex.69 If ABC x CBA = 65125, where A, B and C are single
So, (3C8.08)16 = (968.03125)10
digits, then A + B + C = ?
Sol. As the unit digit of the product is 5, therefore, the unit
ALPHA NUMERICS NUMBERS
digit of one of the numbers is 5 and the unit digit of the
other number is odd. Therefore, AB5 x 5BA = 65125,
aa
where A = 1, 3, 5, 7 or 9.
b b
Ex.66 If a – b = 2, and then find the value of a, b and c. As the product of two three-digit numbers is a five-digit
cc 0
number, and not a six-digit number, A can only be equal
Sol. These problems involve basic number to 1. IB5 x 5B1 = 65125.
(i) aa + bb = 11(a + b) (ii) aa, bb are two-digit numbers. The digit sums of both numbers, 1B5 and 5B1 will be
Hence, their sum cannot exceed 198. So, c must be 1.
(iii) Hence, cc0 = 110. This implies a + b = 10 or a = 6 same. Therefore, the product would give digit sum of a
and b = 4. perfect square. The digit sum on the R.H.S. is 1.
Such problems are part of a category of problems called Therefore, the digit sum of each number can be 1or 8.
alpha numerics.
Correspondingly B will be 4 or 2 (as digit sum cannot
a 3b be equal to 1).
 a c Keeping B = 2, we can see that 125 x 521 = 65125.
Ex.67 If _____ then find a, b and c if each of them is
a a 9
Ex.70 Find the four-digit number ABCD such that
distinctly different digit.
Sol. (i) since the first digit of (a 3 b) is written as it is after ABCD x 4 = DCBA.
subtracting ac carry over from a to 3. Sol. Any number multiplied by 4 will give us an even number.
(ii) there must be a carry over from 3 to b, because if no Hence, the digit D when multiplied by 4 will give us an
carry over is there, it means 3 – a = a.
even number.
3
 2a = 3  a = Since A is the unit digit of the product it is even. Hence,
2
which is not possible because a is a digit. For a carry A = 2, 4, 6 or 8 (It cannot be 0). A is also the first digit of
over 1, 2 – a = a the multiplicand and if A = 4, 6 or 8 the product
 a=1
(iii) it means b and c are consecutive digit (2, 3), ABCD x 4 will become a 5 digit number. Hence A = 2.
(3, 4),.... (8, 9) Writing the value of A we get 2BCD x 4 = DCB2.

PAGE # 9
Now for the value of D looking at the first and last digits 4. The digit at the 100 th place in the decimal
6
of the multiplicand, we can see that 4 x D gives the unit representation of , is :
7
digit of 2 and 4 x 2 gives the first digit of D. Yes, you got (A) 1 (B) 2
(C) 4 (D) 5
it right D = 8. Writing the multiplication again with the
value of D we get 2BC8 x 4 = 8CB2. PRIME NUMBER

Now for the value of B. A number is divisible by 4 if the 5. If a, a + 2, and a + 4 are prime numbers, then the
number of possible solution for a is :
number formed by the last two digits is divisible by 4.
(A) three (B) two
Since the number 8CB2 is a multiple of 4, the number (C) one (D) more than three
B2 should be divisible by 4. Or, the number B2 = 12,
6. The number of positive n in the range 12  n  40 such
32, 52, 72 or 92. Hence the original number ABCD is that the product (n –1) (n – 2).... 3.2.1 is not divisible by
21C8, 23C8, 25C8, 27C8 or 29C8. But the last 4 n is :
(A) 5 (B) 7
numbers when multiplied by 4 will not give you the first
(C) 13 (D) 14
digit of 8 in the product. Therefore B = 1 and the original
7. If least prime factor of a number m is 3 and least prime
number is 21C8. We write the multiplication again
factor of another number n is 7, then least prime factor
21C8 x 4 = 8C12. of the number (m + n) is :
Now for the value of C notice that when you multiply 8, (A) 2 (B) 3
(C) 5 (D) 7
the unit digit of 21C8, by 4 you write 2 in the unit digit of
8. There are four prime numbers written in ascending
the product and carry 3. The tenth digit of the product is order. The product of the first three is 385 and that of
1. Therefore, 4 x C + 3 (carry over) gives a unit digit of 1. the last three is 1001. The last number is :
Hence, C is 2 or 7. You can easily check by the hundreds (A) 11 (B) 13
(C) 17 (D) 19
digit in the product (which is C again) that C = 7.
9. The positive integers A, B, A – B and A + B are all prime
Therefore, our answer is 2178 x 4 = 8712.
numbers. The sum of these four primes is
(A) even (B) divisible by 3
(C) divisible by 5 (D) prime
SQUARES AND SQUARE ROOTS, CUBES
NOTE : More than one correct option may be possible.
AND CUBE ROOTS
CLASSIFICATION OF NUMBER

1. Convert 0.225 in to form p/q. 10. If (12 + 22 + 32 + .....+ 102) = 385, then the value of
3 9 (22 + 42 + 62 +...... + 202).
(A) (B) (A) 770 (B) 1155
10 40
(C) 1540 (D) (385 × 385)
9 9
(C) (D)
50 400 11. If n2 = 12345678987654321, find the value of n ?
(A) 12344321 (B) 1235789
2. Arrange the following rational number in ascending
(C) 11111111 (D) 111111111
3 4 7 1
order , , , .
7 5 9 2
12. Find the square root of 7 – 4 3 .
4 7 3 1 3 1 7 4
(A) , , , (B) , , , (A) 2 – 3 (B) 5 – 3
5 5 9 2 7 2 9 5
(C) 2 – 5 (D) None of these
4 7 1 3 1 3 7 4
(C) , , , (D) , , ,
5 9 2 7 2 7 9 5 13. The value of is :
10  25  108  154  225
3. Let D be a recurring decimal of the form D = 0. a1 a2 a1
a2 a1 a2 ....., where digits a1 and a2 lie between 0 and 9. (A) 4 (B) 6
Further, at most one of them is zero. Which of the (C) 8 (D) 10
following numbers necessarily produces an integer, FACTORS & MULTIPLES
when multiplied by D?
14. The number of prime factors of (3 × 5)12 (2 × 7)10 (10)25
(A) 18 (B) 108
is :
(C) 198 (D) 288
(A) 47 (B) 60
(C) 72 (D) 94

PAGE # 10
Directions : (15 to 19) Read the following information 23. The HCF of 2 numbers is 11 and their LCM is 693.
carefully and answer the questions given below. If their sum is 176, find the numbers.
In a big hostel, there are 1,000 rooms. In that hostel (A) 99,77 (B) 110, 66
only even numbers are used for room numbers, i.e. (C) 88,77 (D) 121, 44
the room numbers are 2, 4, 6, ...., 1998, 2000. All the
rooms have one resident each. One fine morning, the 24. If P is a prime number, then the LCM of P and (P + 1) is
warden calls all the residents and tells them to go (A) P(P +1) (B) (P + 2)P
back to their rooms as well as multiples of their room (C) (P + 1)(P – 1) (D) None of these
numbers. When a guy visits a room and finds the door
1 3
open, he closes it, and if the door is closed, he opens 25. Three pieces of cakes of weights 4 Ibs, 6 Ibs and
2 4
it, All 1,000 guys do this operation. All the doors were 1
open initially. 7 Ibs respectively are to be divided into parts of equal
5
15. The last room that is closed is room number ? weights. Further, each must be as heavy as possible.
(A) 1936 (B) 2000 If one such part is served to each guest, then what is
(C) 1922 (D) None of these the maximum number of guests that could be
entertained ?
16. The 38th room that is open is room number :
(A) 54 (B) 72
(A) 80 (B) 88
(C) 20 (D) 41
(C) 76 (D) None of these

17. If only 500 guys, i.e. residents of room number 2 to 26. The LCM of two numbers is 567 and their HCF is 9. If
1000 do the task, then the last room that is closed is the difference between the two numbers is 18, find the
room number two numbers :
(A) 2000 (B) 1936 (A) 36 and 18 (B) 78 and 60
(C) 1849 (D) None of these (C) 63 and 81 (D) 52 and 34

18. In the case of the previous question, how many rooms DIVISIBILITY
will be closed in all ? 27. How many numbers between 200 and 600 are divisible
(A) 513 (B) 31 by 14?
(C) 13 (D) 315 (A) 28 (B) 29
19. If you are a lazy person, you would like to stay in a room (C) 27 (D) None of these
whose number is :
28. How many natural numbers between 200 and 400 are
(A) more than 500 (B) more than 1000
there which are divisible by
(C) 500 (D) 2000
i. Both 4 and 5?
20. Find the total number of prime factors in the expression ii. 4 or 5 or 8 or 10 ?
(4)11 × (7)5 × (11)2. (A) 9, 79 (B) 10, 80
(A) 37 (B) 33 (C) 10, 81 (D) None of these
(C) 26 (D) 29
29. 461 + 462 + 463 + 464 is divisible by :
HCF AND LCM
(A) 3 (B) 10
21. How many three-digit numbers would you find, which (C) 11 (D) 13
when divided by 3, 4, 5, 6, 7 leave the remainders 1, 2,
30. If x is a whole number, then x2 (x2 – 1) is always divisible
3, 4, and 5 respectively ?
by :
(A) 4 (B) 3 (A) 12 (B) 24
(C) 2 (D) 1 (C) 12 – x (D) Multiple of 12
22. Six strings of violin start vibrating simultaneously and 31. If 653 xy is exactly divisible by 80, then the find the value
they vibrate at 3, 4, 5, 6,10 and 12 times in a minute, of (x + y).
find : (A) 2 (B) 3
i. After how much time will all six of them vibrate (C) 4 (D) 6
together ?
32. When a number P is divided by 4 it leaves remainder
ii. How many times will they vibrate together in 30 min ?
3. If the twice of the number P is divided by the same
(A) 60 min, 31 times (B) 60 sec, 31 times
divisor 4 than what will be the remainder ?
(C) 120 sec, 15 times (D) None of these
(A) 0 (B) 1
(C) 2 (D) 6

PAGE # 11
33. If (232 +1) is divisible by a certain number then which of 44. In a number system, the product of 44 and 11 is 3414.
the following is also divisible by that number. The number 3111 of this system, when converted to
(A) (216 – 1) (B) 216 + 1 the decimal number system, becomes :
(C) 296 + 1 (D) None of these (A) 406 (B) 1086
(C) 213 (D) 691
34. Smallest number which should be added to the num-
ber 803642 so that it is divisible by 11. 45. Convert the number 1982 from base 10 to base 12.
(A) 40 (B) 4 The results is :
(C) 23 (D) 7 (A) 1182 (B) 1912
(C) 1192 (D) 1292
35. If the number 357y25x is divisible by both 3 and 5, then
46. What is the decimal equivalent of the 25 digits of
find the missing digit in the unit’s place and the
hexadecimal number (100.....001)16 ?
thousand place respectively are :
(A) 223 + 1 (B) 224 + 1
(A) 0, 6 (B) 5, 6
(C) 292 + 1 (D) 296 + 1
(C) 5, 4 (D) None of these
47. If the decimal number 2111 is written in the octal system,
36. 113 + 213 + 313 + .....+ 6013 is divisible by : then what is its unit place digit ?
(A) 61 (B) 63 (A) 0 (B) 1
(B) 65 (D) 59 (C) 2 (D) 3
37. A number when divided by 342 gives a remainder 47. REMAINDER
When the same number is divided by 19, what would
48. What is the remainder when 9875347 × 7435789
be the remainder ?
× 5789743 is divided by 4 ?
(A) 3 (B) 5
(A) 1 (B) 2
(C) 9 (D) None of these
(C) 3 (D) None of these
38. A 4-digit number is formed by repeating a 2-digit
49. What is remainder when 784 is divided by 2402?
number such as 2525, 3232 etc. Any number of this
(A) 1 (B) 6
from is exactly divisible by :
(C) 2401 (D) None of these
(A) 7 (B) 11
(C) 13 50. P is a prime number greater than 5. What is the
(D) Smallest 3-digit prime number remainder when P is divided by 6?
(A) 5 (B) 1
39. The largest number which exactly divides the product
(C) 1 or 5 (D) None of these
of any four consecutive natural numbers is :
(A) 6 (B) 12 51. What is the remainder when 3040 is divided by 17?
(C) 24 (D) 120 (A) 1 (B) 16
(C) 13 (D) 4
40. The largest natural number by which the product of
three consecutive even natural numbers is always 52. What is the remainder when 650 is divided by 215?
divisible, is : (A) 1 (B) 36
(A) 6 (B) 24 (C) 5 (D) 214
(C) 48 (D) 96 53. What is the remainder when 7413 – 4113 + 7513 – 4213 is
41. Let N = 553 + 173 – 723, then N is divisible by : divided by 66?
(A) both 7 and 13 (B) both 3 and 13 (A) 2 (B) 64
(C) both 17 and 7 (D) both 3 and 17 (C) 1 (D) 0

42. 76n – 66n, where n is an integer > 0, is divisible by : 54. A number when divided successively by 4 and 5 leaves
(A) 13 (B) 127 remainders 1 and 4 respectively. W hen it is
(C) 559 (D) All of these successively divided by 5 and 4, then the respective
remainders will be :
BASE SYSTEM
(A) 1, 2 (B) 2, 3
43. When (55) 10 is represented in base 25 then the (C) 3, 2 (D) 4, 1
expression is :
55. Let N = 1421 × 1423 × 1425. What is the remainder
(A) (25)25 (B) (35)25
when N is divided by 12?
(C) (55)25 (D) none of these
(A) 0 (B) 9
(C) 3 (D) 6

PAGE # 12
CYCLICITY
 1  1  1
56. Find the unit digit of (795 – 358). 67. log 10  1   + log 10  1   + log 10  1   + ... +
 2   3   4 
(A) 6 (B) 4
(C) 3 (D) None of these  1 
log10 1   . When simplified has the value equal
 1999 
57. If n = 67 then find the unit digit of [3n + 2n ].
to :
(A) 1 (B) 10
(A) 1 (B) 3
(C) 5 (D) None
(C) 10 (D) 100

58. The last digit of (13 + 23 + 33 + ... 103)64 is : 68. If log 10N  2.5 then, find out total number of digits in N.
(A) 2 (B) 5 (A) 3 (B) 4
(C) 9 (D) 0 (C) 5
(D) cannot be determine
59. Sum of the Unit digit of the number 22006 and 52007 is
equal to 69. If log x = n then 2n is equal to :
(A) 6 (B) 9 (A) log (x2) (B) (logx)2
(C) 5 (D) 1 (C) log (x+2) (D) log 2x

60. What is the last digit of the number equal to the sum 1 70. Given log2 = 0.3010, then log 16 is :
+ 6 + 62 + ----- + 6100 ? (A) 2.4080 (B) 1.2040
(A) 0 (B) 1 (C) 0.2408 (D) 1.9030
(C) 2 (D) 6
71. The value of [log10 (5 log10 100)]2 is :
ALPHANUMERIC PUZZLE AND WORD (A) 0 (B) 1
PROBLEM (C) 2 (D) 10
61. Find out (A + B + C + D) such that AB x CB = DDD, where 72. If log10 [log10(log10x)] = 0.
AB and CB are two-digit numbers and DDD is a three- (A) x = 103 (B) x = 1010
digit number. (C) x = 155 (D) None
(A) 21 (B) 19
(C) 17 (D) 18 1
73. If log 3 M  3 log 3 N = 1+ log 5, then :
3 0.008

62. A 3-digit number 4a3 is added to another 3-digit


number 984 to give the four-digit number 13b7, which 9 9 9 9
(A) M  (B) N 
is divisible by 11. Then ,(a + b) is : N M
3 3 9 3
(A) 10 (B) 11 (C) M  (D) N 
N M
(C) 12 (D) 15
74. The value of x, when log3(log2 x) + 2 log9(log7 8) = 2, is :
63. If aabb is a four digit number and also a perfect square
(A) 243 (B) 27
then the value of a + b is :
(C) 343 (D) 64
(A) 12 (B) 11
(C) 10 (D) 9 75. Find x if log10 1250 + log1080 = x.
(A) 5 (B) 4
64. The sum of the digits of two digit number is 11, if the digits
are reversed the number decreases by 45. The number (C) 8 (D) 7
is : MISC ELL ANEOUS
(A) 38 (B) 65
(C) 74 (D) 83 76. Which of the following surds is greatest in magnitude
6
LO GA R I T H M 17 , 2,12 25 , 3 4 .

65. If logxy = 100 and log2x = 10, then the value of y is : (A) 6 17 (B) 12
25
(A) 21000 (B) 2100
2000 (C) 3
4 (D) 2
(C) 2 (D) 210000
77. W hen Sholey screened on the TV there was a
66. Find the value of ‘x’ if 2logx 7 + log7x 7 + 3log49x 7 = 0
commercial break of 5 min after every 15 min of the
4 movie. If from the start of the movie to the end of the
(A) x = (B) x = 7–1/2
3 movie there was in all 60 min of commercials that was
screened what is the duration the movie ?
(C) x = 7–4/3 (D) Either (B) or (C)
(A) 180 min (B) 195 min
(C) 169 min (D) 165 min

PAGE # 13
78. How many numbers between 400 and 600 begin with
or end with a digit of 5 ?
(A) 40 (B) 100
(C) 110 (D) 120 COMPETITIVE EXAM PREVIOUS YEAR

79. Anita had to do a multiplication. Instead of taking 35 as QUESTION


one of the multipliers, she took 53. As a result, the
product went up by 540. What is the new product? 1. P, Q and R are three natural numbers such that P and
(A) 1050 (B) 540 Q are primes and Q divides PR. Then out of the
(C) 1440 (D) 1590 following the correct statement is : [IJSO-2008]
(A) Q divides R (B) P divides R
80. Three friends, returning from a movie, stopped to eat
(C) P divides QR (D) P divides PQ
at a restaurant. After dinner, they paid their bill and
noticed a bowl of mints at the front counter. Sita took 2. It is required to decide if 1107 is a prime number or
1/3 of the mints, but returned four because she had a not. The number of trials of division necessary is :
monetary pang of guilt. Fatima then took 1/4 of what [IJSO-2008]
was left but returned three for similar reasons. Eswari (A) 10 (B) 11
then took half of the remainder but threw two back into (C) 12 (D) 235
the bowl. The bowl had only 17 mints left when the raid
was over. How many mints were originally in the bowl? 3. The number of integers between – 8 and 32 is :
(A) 38 (B) 31 [NSTSE-2009]
(C) 41 (D) 48 (A) 5 (B) 6
81. A set of consecutive positive integers beginning with 1 (C) 7 (D) 8
is written on the blackboard. A student came and erased
4. When expanded, the number of zeros in 100010 is :
one number. The average of the remaining numbers
[NSTSE-2009]
7 (A) 13 (B) 30
is 35 . What was the number erased?
17 (C) 4 (D) 10
(A) 7 (B) 8
(C) 9 (D) None of these 5. Let N = 28, the sum of All distinct factors of N is :
82. W hat is the value of the following expression [IJSO-2009]
 1   1   1   1  (A) 27 (B) 28
 (2 2  1)    ( 4 2  1)    (6 2  1)   .....   (20 2  1)  ?
        (C) 55 (D) 56
       
9 10 6. The units digit of (1 + 9 + 92 + 93 + --------- + 92009) is :
(A) (B) [IJSO-2009]
19 19
10 11 (A) 0 (B) 1
(C) (D)
21 21 (C) 9 (D) 3

 1  1  1  1  1  1  1 7. The biggest among the following is : [IJSO-2009]


1    1   1    1   1    1   1  
83.  2  3  4  5  6  7  8 (A) 21/2 (B) 31/3
1/6
is equal to : (C) 6 (D) 81/8
(A) 9 (B) 8
8. If a, b  1, ab > 0, a  b and logba = logab, then ab = ?
(C) 4.5 (D) None of these
[IAO- 2009]
84. How many even integers n, where 100  n  200, are (A) 1/2 (B) 1
divisible neither by seven nor by nine ? (C) 2 (D) 10
(A) 40 (B) 37
(C) 39 (D) 38 9. If 2009 = pa.qb, where "p" and "q" are prime numbers,
then find the value of p + q. [NSTSE 2009]
85. A rich merchant had collected many gold coins. He did
(A) 3 (B) 48
not want any body to know about him. One day, his
(C) 51 (D) 2009
wife asked, “How many gold coins do we have?” After
pausing a moment he replied, “Well ! if divide the coins 10. If HCF (p, q) = 12 and p × q = 1800 n then LCM (p, q) is :
into two unequal numbers, then 48 times the difference [NSTSE 2010]
between the two numbers equals the difference (A) 3600 (B) 900
between the square of the two numbers. “ The wife (C) 150 (D) 90
looked puzzled. Can you help the merchant’s wife by
11. The value of log10(3/2) + log10 (4/3) + ......... up to 99
finding out how many gold coins the merchant has ?
terms. [IAO 2008]
(A) 96 (B) 53
(A) 0 (B) 2
(C) 43 (D) 48
(C) 2.5 (D) None of the above

PAGE # 14
12. In the familiar decimal number system the base is 10. 22. The number 5 41 lies between [IJSO-2013]
In another number system using base 4, the counting
(A) 29 and 30 (B) 30 and 31
proceeds as 1, 2, 3, 10, 11, 12, 13, 20, 21 .... The
twentieth number in this system will be [IJSO-2010] (C) 31 and 32 (D) 32 and 33
(A) 40 (B) 320
23. Number plate of a vehicle consists of 4 digits. The first
(C) 210 (D) 110 digit is the square of second. The third digit is thrice
the second and the fourth digit is twice the second.
2 3 The sum of all 4 digits is thrice the first. The number is
13. Given 600 = 24.49, the value of is [IJSO-2014]
3– 2
(A) 1132 (B) 4264
(C) 1642 (D) 9396
[IJSO-2011]
(A) 9.978 (B) 8.989 24. If the highest common factor of a,b and c is 1, where
(C) 9.989 (D) 9.898 a,b and c belong to the set of natural numbers, then
the highest common factor of (a × b) and c is
[IJSO-2014]
14. If x < 0 and log7 (x2 – 5x – 65) = 0, then x is (A) c (B) a × b
(C) 1 (D) insfficient data
[IJSO-2011]
(A) –13 (B) –11 25. A number x is a rational number if there exists integers
(C) – 6 (D) – 5 p and q such that x = p/q. This is definition of rational
numbers in which, [IJSO-2014]
(A) both p & q can be zero
15. If 22x+1 + 21–2x = 2, then the value of x is [IJSO-2011] (B) both p & q should not be zero
(A) 0.5 (B) –0.5 (C) q can be zero but not p
(D) p can be zero but not q
(C) 1 (D) 0
26. The least positive integer, n, such that 2 divides n, 3
16. If the eight digit number 2575d568 is divisible by 54 divides n + 1, 4 divides n + 2, 5 divides n + 3 and 6
and 87, the value of the digit ‘d’ is [IJSO-2011] divides n + 4 is [IJSO-2014]
(A) 62 (B) 120
(A) 4. (B) 7. (C) 720 (D) 52
(C) 0. (D) 8.
27. The sum of 2 digits x and y is divisible by 7. What can
17. What will be the remainder if the number 72012 is divided one say about a 3 digit number formed by these two
by 25? [IJSO-2012]
digits. [IJSO-2014]
(A) 24 (B) 18
(C) 7 (D) 1 (A) xxy is divisible by 7 (B) xyx is divisible by 7
(C) xyx is divisible by 72 (D) yyx is divisible by 7
18. The product of three consecutive natural numbers is
124850054994. What is their average? 28. What will be the remainder if the number 72015 is divided
[IJSO-2012] by 25? [IJSO-2015]
(A) 4993 (B) 4994 (A) 1 (B) 7
(C) 4997 (D) 4998 (C) 18 (D) 24

29. What is the sum of all three digit even numbers divisible
x y
19. If 75 = 45 = z 15 , then which of the statement is by seventeen? [IJSO-2015]
true [IJSO-2012] (A) 18846 (B) 18684
(A) x + y = 2z (B) x + y = 3z (C) 14688 (D) 16848
(C)x – y = 2z (D) x – y = 3Z
30. 8888888 * 8888888 this fifteen digit number is divisible
20. In a certain number system 363 + 1056 = 1452. Find by 22. Find the eighth digit in the number.
the value of (654 – 456) in the same number system [IJSO-2015]
[IJSO-2012] (A) 1 (B) 3
(A) 156 (B) 165 (C) 5 (D) 8
(C) 178 (D) 198
31. (41)16 - (14)16 is a multiple of [IJSO-2015]
(A) 1485 (B) 1584
21. Let T be the number of 4– digit integers, each ending
(C) 1845 (D) 1854
in 3 (in units place) and each divisible by 11.Then
[IJSO-2013]
32. What will be the remainder if the number (1000000)3
(A) 20 T  79 (B) 80 T  89 is divided by 143? [IJSO-2015]
(C) 90 T  99 (D) T 100 (A) 9 (B) 6
(C) 1 (D) 0

PAGE # 15
38. How many three digit numbers are divisible by 13 and
33. If 338 – 288 = m then m = ? [IJSO-2015]
having middle digit 5 ? [IJSO-2015]
(A) 5 (B) 7
(A) 50 (B) 32 (C) 10 (D) 13
(C) 18 (D) 2 39. Number of numbers less than 40 having exactly four
divisors is [IJSO-2016]
34. How many four digit numbers divisible by twenty nine (A) 15 (B) 12
have the sum of their digits 29? [IJSO-2015] (C) 11 (D) 14
(A) 4 (B) 5
(C)) 13 (D) none of these 40. Tenth term in the sequence 12, 18, 20, 28, ....... is
[IJSO-2016]
35. Select any three distinct digits. Form a three digit (A) 336 (B) 63
number. Form the another number by reversing the (C) 216 (D) 68
digits. Find the difference of these two numbers. What
is the G.C.D of all such differences? [IJSO-2015] 41. Number of integers n such that the number 1 + n is a
(A) 9 (B) 11 divisor of the number 1 + n2 is [IJSO-2016]
(C) 33 (D) 99 (A) 0 (B) 1
(C) 4 (D) 2
36. 5901AB04 is an eight digit number divisible by 792.
Find A + B = ? [IJSO-2015] 42. The number of integers a,b,c for which
(A) 6 (B) 7 a2 + b2 – 8c = 3 is [IJSO-2016]
(C) 8 (D) 9 (A) 2 (B) infinite
(C) 0 (D) 4
37. What is the smallest natural number when multiplied
by 15 and divided by 63 gives remainder 21?
[IJSO-2015]
(A) 13 (B) 14
(C) 17 (D) 20



PAGE # 16
COMMERCIAL MATHEMATICS

 Percentage increase/decrease when a quantity


‘a’ is increased/decreased to become another
quantity ‘b’.
The word 'percentage' literally means 'per hundred' Percentage Increase/Decrease
or 'for every hundred.' Therefore, whenever we Increase / Decrease
calculate something as a part of 100, that part is = × 100
Initial Value
numerically termed as percentage.
b – a
In other words, percentage is a ratio whose second  a  100, when b  a ; (increase)
term is equal to 100. i.e. 1 : 4 can be written as 25 : 100 
= a – b
or 25%, 3 : 8 can be written as 37.5 : 100 or 37.5%,  a  100, when b  a ; ( decrease)
3 : 2 can be written as 150 : 100 or 150%, and so on. Therefore new quantity b

 To express a% as a fraction divide it by 100.   percentage increase 


a   1  
i.e. a% = a/100   100 
=   percentage decrease 
 To express a fraction (x/y) as a percent multiply it by a  1 – 
  100 
100.
i.e. x/y = (x/y x 100)% Ex.5 A dealer buys products for Rs.80 and hikes up the
price to Rs.125. He sells it to the customer after
Basic Formula of Percentage : giving a discount of Rs.5. Find his profit percentage.
p Sol. Profit percentage = Percentage Increase/Decrease
 p% of a number N is = N × . in his income.
100
Pr ofit
Ex.1 What is 37.5% of 648 ? = Cost Pr ice ×100
37.5 120 – 80
Sol. 37.5% of 648 = × 648 = ×100 = 50%.
100 80
375
= × 648 = 3 × 81 = 243. Ex.6 A dealer sells goods priced at Rs.180 after giving a
1000 discount of 25%. Find his selling price.
Ex.2 What is 20% of 50% of 60% of 200 ?  25 
Sol. Selling price after discount = 180 1 –  = 135.
 100 
 60  50  20
Sol. Required percentage :  200  100   100  × = 12.
   100  If one quantity A is x% more or less than another
 To increase or decrease a number by x %, multiply quantity B, then B is less or more than A by :
 x 
[100  x]    100
the number by .  100  x 
100
Where, (+)  Increase, (–)  Decrease. Ex.7 The salary of Ramesh is 25% more than that of
Anil’s salary. By what percentage is Anil’s salary
REMARK : less than that of Ramesh’s ?
Sol. Anil’s salary is less than that of Ramesh’s by
To solve these type of problems calculate x % of given
x
number & add or subtract the value from given number = × 100
100  x
for increase or decrease respectively.
25
= × 100 = 20%.
 To calculate what percentage of a is b, use the 125
Ex.8 Vijay’s salary was reduced by 50%. Again the reduced
b
formula : Percentage = ×100. salary was increased by 50%. Then, what will be the %
a loss in salary ?
Sol. Say, salary was Rs.100
Ex.3 What percentage of 240 is 90 ? Reduction 50%
90 Now salary = Rs. 50
Sol. Percentage = × 100 = 37.5%. Increase = 50%
240
50  150
Ex.4 What percentage of 75 is 125 ?  = Rs.75
100
125 100  75
Sol. Percentage ×100 = 166.66%. Loss % = = 25
75 100
Hence, loss is 25%.

PAGE # 17
Ex.9 Entry fee in an exhibition was Rs.1. Later, this was Ex.11 A ball drops from a height of 4802 m. Thereafter, it
reduced by 25% which increased the sale by 20%. bounces every time to a height which is 14.28% less
than its previous height. What height will the ball reach
Then, find the percentage of slump in business.
on its 4th bounce ?
Sol. Let the total original sale be Rs. 100.
1
Then, original number of visitors = 100. Sol. I wonder how many of you will notice that 14.28% = .
7
120 th
New number of visitors = = 160. 1
0.75 Therefore, the ball is rising up to a height which is
7
 Increase % = 60%.
less than the previous height. Or, the ball is rising up to
6
a height which is of the previous height.
7
Therefore, on its 4th bounce the ball will reach a height

6 6 6 6
Conversion of Fractions into Percentages : = 4802 × × × × = 2592 m.
7 7 7 7
Knowing conversion of common fractions into
Ex.12 A man spends 75% of his income. If his income is
percentages helps to convert many fractions into increased by 20% and he increased his expenditure
percentage immediately. For example, knowing that by 10%. By what % will saving increased ?
Sol. Let his income be Rs 100, Expenditure = Rs. 75.
1 3 Now, Income is increased by 20%.
= 12.5% will help to convert fractions like New income = 120,
8 8
Expenditure is increased by 10%
5
or into percentages immediately.. 75  110
8 = = Rs. 82.50
100
 Given below are the fractions converted into Saving = 120 – 82.50 = 37.50
Earlier saving = 100 – 75 = 25
percentage.
37.50  25
Increase in saving = × 100 = 50%.
Fraction Percentage Fraction Percentage Fraction Percentage 25
1 1 1
2
50%
10
10%
18
5.55% Ex.13 A students scores 40 marks in an examination and
1 1 1 fails by 26 marks. If the passing percentage is 33 then
33.33% 9.09% 5.26%
3 11 19
find the maximum marks in the examination.
1
25%
1
8.33%
1
5% Sol. Let, the maximum marks in the examination is 100.
4 12 20
Then he needs 33 marks to pass.
1 1 1
5
20%
13
7.69% 21
4.76% But, passing marks required are 40 + 26 = 66 marks.
1 1 1 33 marks are required to pass if maximum marks are
16.66% 14
7.14% 22
4.54%
6 100.
1
14.28%
1
15
6.66%
1
23
4.34% Here, 66 marks are required to pass, then maximum
7

1 1 100  66
1 12.50% 16
6.25% 24
4.16% marks are = 200 marks.
8 33
1 1 1
11.11% 17
5.88% 25
4%
9

If a quantity x is increased or decreased successively


Ex.10 The salary of Sachin Tendulkar is 20% more than
by A%, B%, C% then the final value of x will be
that of Ricky Pointing. By what percentage is Ricky’s
 A  B  C 
salary less than that of Sachin’s ? = x 1   1   1  .
 100   100   100 
Sol. As, Sachin Tendulkar’s salary is 20% more than Ricky
 Let the present population of town be P and let there
Ponting’s salary then Ricky Ponting’s salary is less
be an increase or decrease of R% per annum.
than Sachin Tendulkar’s salary by n
 R 
20 Then, population after n years = P  1  
 100 
=  100
20  100  If length & breadth of a rectangle is changed by a % &
b% respectively, then % change in area will be :
20
=  100  (a  b ) 
120 = a  b  %
 100 
= 16.67% (use +ve for increase & -ve for decrease)

PAGE # 18
Ex.14 The population of a variety of tiny bush in an  Overhead :
experimental field increased by 10% in the first year, Sometimes, after purchasing an article, we have to
increased by 8% in the second year but decreased by pay some more money for things like transportation,
10% in the third year. If the present number of bushes labour charges, repairing charges, local taxes, etc.
in the experimental field is 26730, then find the number These extra expenses are called overhead. For
of bushes in the beginning. calculating the total cost price, we add overhead to the
Sol. Let the number of bushes in the beginning is P so, purchase price.
 10   8  10  Ex.15 A grocer buys 20 kg of sugar at a cost of Rs 18 per kg
P × 1   1   1   = 26730
and 30 kg of an inferior sugar at a cost of
 100   100   100 
Rs 15 per kg. He mixes the two kinds of sugar and
26730 sells the mixture at a cost of Rs 16.50 per kg. Find his
 P=
 10   8  10  profit or loss percent.
1   1   1  
 100   100   100  Sol. C.P. of 20 kg of sugar = 18 × 20 = Rs.360
C.P. of 30 kg of sugar = 15 × 30 = Rs.450
 10 25 10  Total C.P. = 360 + 450 = Rs.810
 P =  26730    
S.P. of (20 + 30) kg = 50 kg of sugar = 16.50 × 50
 11 27 9 
= Rs.825
 P = 25000.
Profit = S.P. – C.P. = 825 – 810 = Rs.15
15 50 23
PROFIT, LOSS & DISCOUNT Profit percent =  100 = =1 .
810 27 27
DEFINITION : 23
Hence, the required profit = 1 %.
27
(i) Cost price (C.P.) : The amount for which an article is
Ex.16 If the selling price of 20 articles is the same as the
bought is called its cost price, abbreviated to CP.
cost price of 23 articles, find the profit or loss percent
(ii) Selling price (S.P.) : The amount for which an article in the transaction.
is sold is called its selling price, abbreviated to SP. Sol Let the C.P. of an article be Rs x.
Then, C.P. of 23 articles = Rs 23x
(iii) Gain : When S.P. > C.P. then there is a gain. and C.P. of 20 articles = Rs 20x.
Gain = S.P. – C.P. S.P. of 20 articles = C.P. of 23 articles = Rs 23x.
Since, S.P. of 20 articles > C.P. of 20 articles, hence
(iv) Loss : When S.P. < C.P. then there is a loss.
there is a profit in the transaction,
Loss = C.P. – S.P. Hence, profit on 20 articles = S.P. – C.P.
 REMARK = Rs (23x – 20x) = Rs 3x.
The gain or loss is always calculated on the cost price. 3x
Profit percent =  100 = 15%
20 x
Required profit = 15%.
Ex.17 A man bought 2 boxes for Rs.1300. He sold one box
(i) Gain = S.P. – C.P. at a profit of 20% and other box at a loss of 12%.
If the selling price of both the boxes is the same, find
(ii) Loss = C.P. – S.P. the cost price of each box.
 Gain  Sol. Let the C.P. of the first box which was sold at a profit of
(iii) Gain% =   100 %
 C.P.  20% be Rs.x. Then the C.P. of the second box which
 Loss  was sold at a loss of 12% will be Rs.(1300 – x).
(iv) Loss% =   100 % Since the first box was sold a profit of 20%, its S.P.
 C.P. 
120
(v) To find S.P. when C.P. and gain% or loss% are given. = Rs. x.
100
(100  Gain %) 88 1300  x 
(a) S.P. = × C.P.. S.P. of second box = Rs.
100 100
Since, the S.P. of both the boxes are same.
(100  Loss %)
(b) S.P. =  C.P. We have,
100
120 x 88 1300  x 
(vi) To find C.P. when S.P. and gain% or loss% are =
100 100
given :  15x = 11 (1300 – x)
100  15x + 11x = 11 × 1300
(a) C.P. = 100  Gain %  S.P. 11  1300
 x= = 550.
100 26
(b) C.P. =  S.P. Hence, C.P. of the first box = Rs.550.
100  Loss%
And that of the second box = Rs.(1300 – 550) = Rs.750

PAGE # 19
Ex.18 Even after reducing the marked price of a transistor
by Rs. 32, a shopkeeper makes a profit of 15 %. If the DISCOUNT
cost price be Rs. 320, what percentage of profit would
(i) Marked price : In big shops and department stores,
he have made if he had sold the transistor at the
marked price ? every article is tagged with a card and its price is written
Sol. C.P. = Rs. 320, profit = 15% on it. This is called the marked price of that article,
 115  abbreviated to MP. For books, the printed price is the
S.P. = Rs.   320  = Rs. 368. marked price.
 100 
Marked price = Rs. (368 + 32) = Rs. 400. (ii) List price : Items which are manufactured in a factory
 80  are marked with a price according to the list supplied
 Required profit% =   100  % = 25%.
 320  by the factory, at which the retailer is supposed to sell
them. This price is known as the list price of the article.
Ex.19 A man buys an article and sells it at a profit of 20%.
If he would buy it at 20% less and sell it for Rs.75 less, (iii) Discount : In order to increase the sale or clear the
he would have gained 25%. What is the cost price of
old stock, sometimes the shopkeepers offer a certain
the article ?
Sol Let the C.P. of the article be Rs. x. percentage of rebate on the marked price. This rebate
He makes a profit of 20%. is known as discount.

120x 6x  An important fact : The discount is always calculated


Hence, S.P. = Rs. = Rs.
100 5 on the marked price.
If he would buy it at 20% less, then Clearly, Selling Price = Marked Price – Discount
 20   1 Ex.21 The marked price of a woolen coat is Rs. 2000. It is
Then, new C.P. = Rs.x 1  100  = Rs. x 1  5 
    sold at a discount of 15%. The shopkeeper has allowed
4x a further discount of 5% due to off season. Find the
= Rs.
5 selling price of the coat.
He would sell it for Rs 75 less, Sol. Marked price = Rs. 2000
Ist discount = 15% of Rs. 2000
 6x 
Then, the new S.P. = Rs.   75 
 5  15
= Rs.  2000 = Rs. 300
125 4 x 100
He gains 25%, then the new S.P. = Rs.  = Rs. x.
100 5  The reduced marked price after the 1st discount
= Rs.2000 – Rs 300 = Rs. 1700
6x
Hence,  75 = x 2nd discount due to off-season = 5% of Rs 1700
5
6x 5
  x = 75 = Rs.  1700 = Rs. 85
5 100
x
 = 75 Hence, the final reduced price after the 2nd discount
5
Hence, the required C.P. = Rs 375. = Rs.1700 – Rs.85
= Rs.1615 = S.P.
Ex.20 A vendor bought oranges at 20 for Rs.56 and sold
them at Rs.35 per dozen. Find his gain or loss percent. Hence, the required S.P. of the coat is Rs.1615.
Sol. Let the number of oranges bought
Ex.22 Find a single discount equivalent to the discount
= LCM of 20 and 12 = 60
C.P. of 20 oranges = Rs 56. series 25%, 20% and 10%.
Sol. Let the marked price of the article be Rs.100
56
 C.P. of 1 orange = Rs. . Then a single discount equivalent to the discount
20
series is
 56 
Hence, the C.P. of 60 oranges = Rs.   60  100  25 100  20 100  10
 20  = (100 – × × × 100)%
= Rs.168 100 100 100
S.P. of 12 oranges = Rs.35 75 80 90
= (100 – × × × 100)%
35 100 100 100
 S.P. of 1 orange = Rs. .
12
 35  3 4 9
Hence, the S.P. of 60 oranges =   60  = Rs.175 = (100 – × × × 100)%
 12  4 5 10
Thus, C.P. = Rs 168 and S.P. = Rs.175. = (100 – 54)%
Since, (S.P.) > (C.P.), the vendor has made a gain.
= 46%
Gain = (175 – 168) = Rs. 7
Hence, the given discount series is equivalent to a
 Gain   7  1 single discount of 46%.
 Gain% =   100 % =   100 % = 4 % .
 C.P.   168  6

PAGE # 20
Ex.23 A person marks his goods 10 % above his cost price. Some other ratios :
He then sells it by allowing a discount of 10%. What is Compounded Ratio : The compounded ratio of the
his profit or, loss percent? ratios (a : b), (c : d), (e : f) is (ace : bdf).
Sol. Let his cost price be Rs.x
Duplicate ratio : The duplicate ratio of (a : b) is (a2 : b2).
 10 x  11x
Then, his marked price = Rs.  x   = Rs. . Sub-duplicate ratio : The sub-duplicate ratio of (a : b)
 100  10
is ( a : b ).
He then sells it at a discount of 10% on this marked
price. Triplicate ratio : The triplicate ratio of (a : b) is (a3 : b3).
11x Sub-triplicate ratio : The sub-triplicate ratio of (a : b) is
Discount = 10% of Rs.
10
11x 10 11x  1 1
= Rs.  = Rs.  a 3 : b 3 .
10 100 100  
 
 11x 11x 
His S.P. = Rs.   
 10 100  a c
Componendo : If  then, the componendo is
110 x  11x b d
= Rs.
100 ab c d
 .
99 x b d
= Rs.
100 a c
Dividendo : If  then, the dividendo is
Since, his C.P. > S.P., hence there will be a loss. b d
a–b c –d
 99 x  x  .
And loss = C.P. – S.P. = Rs.  x   = Rs. b d
 100  100 a c
Componendo and Dividendo : If  , then the
x 1 b d
 Loss percent =   100 = 1.
100 x componendo-dividendo is a  b  c  d .
Hence, the required loss = 1%. a–b c–d
Variation :
(i) We say that x is directly proportional to y, if x = ky for
Ratio : The comparison of two quantities a and b of some constant k and we write, x  y.
similar kind is represented as a : b is called a ratio (ii) We say that x is inversely proportional to y, if xy = k
a
also it can be represented as . 1
b for some constant k and we write, x  .
In the ratio a : b, we call a as the first term or y
antecedent and b, the second term or consequent. Ex.24 If a : b = 5 : 9 and b : c = 4 : 7, find a : b : c.
5 9  9
For example : The ratio 5 : 9 represents , with  63
9 Sol. a : b = 5 : 9 and b : c = 4 : 7 =  4   :  7   = 9 :
 4  4 4
antecedent = 5 and consequent = 9.
63
 The multiplication or division of each term of a ratio by  a:b:c=5:9: = 20 : 36 : 63.
4
the same non-zero number does not affect the ratio.
Ex.25 Find out :
For example : 4 : 5 = 8 : 10 = 12 : 15 etc. Also, 4 : 6 = 2 : 3. (i) the fourth proportional to 4, 9, 12.
Proportion : The equality of two ratios is called (ii) the third proportional to 16 and 36.
proportion. (iii) the mean proportional between 0.08 and 0.18.
Sol. (i) Let the fourth proportional to 4, 9, 12 be x.
If a : b = c : d, we write, a : b : : c : d and we say that a, b,
Then, 4 : 9 : : 12 : x
c, d are in proportion.
 4 × x = 9 × 12
where, a is called first proportional, b is called second
9  12
proportional, c is called third proportional and d is  x= = 27.
4
called fourth proportional.
 Fourth proportional to 4, 9, 12 is 27.
 Law of Proportion : (ii) Let the third proportional to 16 and 36 is x.
Product of means = Product of extremes Then, 16 : 36 : : 36 : x  16 × x = 36 × 36
Thus, if a : b : : c : d  (b × c) = (a × d), 36  36
Here a and d are called extremes, while b and c are  x= = 81.
16
called mean terms.
 Mean proportional of two given numbers a and b is  Third proportional to 16 and 36 is 81.
ab .

PAGE # 21
(iii) Mean proportional between 0.08 and 0.18 We can also represent this thing as under :
= C.P. of a unit quantity of cheaper C.P. of a unit quantity of dearer
0.08  0.18
(c) (d)
8 18 Mean price
  (m)
100 100
(d – m) (m – c)
144 12
= = = 0.12
100  100 100 Suppose a container contains x units of liquid from
which y units are taken out and replaced by water. After
Ex.26 If x : y = 3 : 4, find (4x + 5y) : (5x – 2y).
n operations, the quantity of pure liquid :
x 3
Sol.    n
y 4 y 
=  x 1 – x   units.
x 3    
4   5  4   5 
4 x  5y  y  4   (3  5)  32 . Ex.30 The cost of Type- 1 rice is Rs.15 per kg and Type-2
  
5 x – 2y x  3  7 7 rice is Rs.20 per kg. If both Type-1 and Type-2 are mixed
5  – 2  5  – 2   
y
   4   4  in ratio of 2 : 3, then find the price per kg of the mixed
Ex.27 Divide Rs. 1162 among A, B, C in the ratio 35 : 28 : 20. variety of rice.
Sol. Sum of ratio terms = (35 + 28 + 20) = 83. Sol. Let the price of the mixed variety be Rs. x per kg.
By the rule of alligation, we have :
 35 
A’s share = Rs. 1162   = Rs. 490;
 83  Cost of 1 kg of Type 1 rice Cost of 1 kg of Type 2 rice

 28  Rs. 15 Rs. 20
B’s share = Rs. 1162   = Rs. 392; Mean price
 83  Rs. x
 20 
C’s share = Rs. 1162   = Rs. 280. (20 – x) (x – 15)
 83 
Ex.28 A bag contains 50 p, 25 p and 10 p coins in the ratio (20 – x ) 2
 =
5 : 9 : 4, amounting to Rs. 206. Find the number of ( x – 15 ) 3
coins of each type.  60 – 3x = 2x – 30
Sol. Let the number of 50 p, 25 p and 10 p coins be 5x, 9x
 5x = 90
and 4x respectively.
 x = 18.
5x 9 x 4x So, price of the mixture is Rs.18 per kg.
Then,   = 206
2 4 10
 50x + 45x + 8x = 4120 Ex.31 A milk vendor has 2 cans of milk. The first contains
 103x = 4120 25% water and the rest milk. The second contains
 x = 40. 50% water. How much milk should he mix from each
 Number of 50 p coins = (5 × 40) = 200; of the containers so as to get 12 litres of milk such that
Number of 25 p coins = (9 × 40) = 360; the ratio of water to milk is 3 : 5 ?
Number of 10 p coins = (4 × 40) = 160. Sol. Let cost of 1 litre milk be Re.1.
Ex.29 If a man goes from a place A to another place B 100 3
m apart in 4 hours at a certain speed. With the same Milk in 1 litre mixture in 1st can = litre,
4
speed going from B to C 400 m apart, what time will he 3
take ? C.P. of 1 litre mixture in 1st can = Rs. .
Sol. d = st, where d is distance in m, s is speed in m/sec., 4
nd
1
t is time in seconds. Speed is same  d  t. Milk in 1 litre mixture in 2 can = litre,
2
New distance is 4 times, now the time will be 4 times
the time it takes from A to B .So, the time taken from B 1
C.P. of 1 litre mixture in 2nd can = Rs. .
to C is 4 × 4 = 16 hours. 2
5
Milk in 1 litre of final mixture = litre,
8
5
Alligation : It is the rule that enables us to find the ratio Mean price = Rs. .
8
in which two or more ingredients at the given price By the rule of alligation, we have :
must be mixed to produce a mixture of a desired price.
x 3/4  5 / 8 x 1/ 8 1
Mean Price : The cost price of a unit quantity of mixture y = 5 / 8  1/ 2 ; y = 1/ 8 = 1 .
is called the mean price.
C.P. of 1 litre mixture in 1st can C.P. of 1 litre mixture in 2nd can
Rule of Alligation : If two ingredients are mixed, then, 3/4 1/2
Mean price
Quantity of cheaper C.P. of dearer  – Mean price 
 5/8
Quantity of dearer Mean price  – C.P. of cheaper 
1/8 1/8
 We will mix 6 from each can.

PAGE # 22
Ex.32 Tea worth Rs.126 per kg and Rs.135 per kg are mixed
with a third variety in the ratio 1 : 1 : 2. If the mixture is
worth Rs. 153 per kg, then find the price of the third When two or more persons jointly start a business with
variety per kg. an objective to earn money. This is called partnership.
Sol. Since first and second varieties are mixed in equal These persons are called partners and the money
 126  135  invested in the business is known as capital.
proportions, so their average price = Rs. 
 2  Distribution of Profit/Loss when unequal capital is
= Rs.130.50 invested for equal interval of time :
So, the mixture is formed by mixing two varieties, one When partners invest different amounts of money, for
at Rs. 130.50 per kg and the other at say, Rs. x per kg equal interval of time, then profit/loss is divided in the
in the ratio 2 : 2, i.e., 1 : 1. We have to find x. ratio of their investment.
By the rule of alligation, we have :
Ex.35 A and B invested Rs. 3600 and Rs. 4800 respectively
Cost of 1 kg tea of 1st kind Cost of 1 kg tea of 2nd kind to open a shop. At the end of the year B’s profit was
130.50 Rs. x Rs. 1208. Find A’s profit.
Mean price
Rs. 153 Sol. Profit sharing ratio = 3600 : 4800 = 3 : 4
Profit of A 3
(x – 153) 22.50 
Profit of B 4
x  153 3
 1=  Profit of A = Profit of B
22.5 4
 153 + 22.5 = x 3
 Profit of A = × 1208 = Rs. 906
 x = Rs.175.50 4

Ex.33 A jar full of whisky contains 40% alcohol . A part of this Distribution of P/L when equal capital is invested for
whisky is replaced by another containing 19% alcohol different intervals of time :
and now the percentage of alcohol was found to be Ex.36 Govind & Murari started a business with equal
26%. Find the quantity of whisky replaced. capitals. Govind terminated the partnership after
Sol. By the rule of alligation, we have : 7 months. At the end of the year, they earned a profit of
Strength of first jar Strength of 2nd jar Rs. 7600. Find the profit of each of them.
40% 19% Sol. Govind invested for 7 month, Murari invested for
Mean strength 12 month.
26% Since investment is same for both (Let it be Rs. x)
 Profit sharing ratio = 7x : 12x = 7 : 12
7 14
So, ratio of 1st and 2nd quantities = 7 : 14 = 1 : 2. 7
 Govind’s profit = × 7600 = 2800
7  12
2 12
 Required quantity replaced = . Murari’s profit =
3 7  12
× 7600 = 4800.
Ex.34 A vessel is filled with liquid, 3 parts of which are water Ex.37 Ramesh started a business by investing Rs. 25000.
and 5 parts syrup. How much of the mixture must be 3 months later Mahesh joined the business by invest-
drown off and replaced with water so that the mixture ing Rs. 25000. At the end of the year Ramesh got Rs.
may be half water and half syrup ? 1000 more than Mahesh out of the profit. Find the total
Sol. Suppose the vessel initially contains 8 litres of liquid. profit.
Let x litres of this liquid be replaced with water. Sol. Ramesh invested for 12 month, Mahesh invested for 9
 3x  month.
Quantity of water in new mixture =  3 –  x  litres.
 Profit sharing ratio = 12x : 9x = 12 : 9 = 4 : 3.
 8 
Let Capital be Rs P.
 5x 
Quantity of syrup in new mixture =  5 –  litres. 4
 8  Profit of Ramesh = P
7
 3x   5x 
 3 –  x = 5 –  3
 8   8  Profit of Mahesh = P
7
 5x + 24 = 40 – 5x 4 3
 P = P + 1000
7 7
8
 10x = 16  x = . 4 3
5  P – P = 1000
7 7
 8 1 1
So, part of the mixture replaced =    = . P
5 8 5  = 1000  P = Rs.7000.
7

PAGE # 23
Distribution of P/L when capital and time both are Ex.41 Tanoj & Manoj started a business by investing
unequal : Rs. 75000 and Rs. 90000 respectively. It was decided
to pay Tanoj a monthly salary of Rs. 1875 as he was
Ex.38 Suresh & Ramesh entered into a partnership by
the active partner. At the end of the year if the total profit
investing Rs.14000 and Rs. 18000 respectively.
is Rs. 39000, find the profit of each.
Suresh with drew his money after 4 months. If the total
Sol. Profit sharing ratio = 75000 : 90000 = 5 : 6
profit at the end of a year is Rs. 12240, find the profit of
Total profit = Rs. 39000
each.
Salary of Tanoj = 12 × 1875 = Rs. 22500
Sol. Profit sharing ratio = 14000 × 4 : 18000 × 12 = 7 : 27
Profit left = Rs.39000 – Rs. 22500 = Rs.16500.
7 5
Suresh’s profit = × 12240 = Rs. 2520 Tanoj’s profit = × 16500 = 7500.
34 11
27  Total profit of Tanoj = 22500 + 7,500 = Rs. 30,000
Ramesh’s profit = × 12240 = Rs. 9720
34
6
Ex.39 David started a business establishment by investing Manoj’s profit = × 16500 = Rs. 9,000
11
Rs.15000. After 4 months William entered into a
Change in invested capital :
partnership by investing a certain amount. At the end
of the year; the profit was shared in the ratio 9 : 8. Find Ex.42 Rajeev & Sanjeev entered into a partnership and
how much money was invested by william. invested Rs. 36000 and Rs. 40000 respectively. After
Sol. Let william invested Rs. x 8 months Rajeev invested an additional capital of
Profit sharing ratio = 15000 × 12 : 8x = 1,80,000 : 8x Rs. 4000, Sanjeev withdrew Rs. 4000 after 9 months.

Also profit ratio = 9 : 8 At the end of the year total profit was Rs. 45800. Find

 ATQ, 180000 : 8x = 9 : 8 the profit of each.


Sol. Rajeev’s capital = 36000 × 8 + (36000 + 4000) × 4
180000  8 = Rs. 448000
 =x
98
Sanjeev’s capital = 40000 × 9 + (40000 – 4000) × 3

 x = Rs. 20,000 = Rs. 468000


Profit sharing ratio = 448000 : 468000 = 112 : 117
Working and Sleeping partner :
112
Rajeev’s profit = × 45800 = Rs. 22400
Active Partner : A partner who manages the business 229

is known as active or working partner. 117


Sanjeev’s profit = × 45800 = Rs. 23400.
Sleeping Partner : A partner who only invests the money 229

is known as sleeping partner. Ex.43 A, B and C start a business each investing Rs. 20000.
Ex.40 Nitesh & Jitesh invested Rs.15000 and Rs.18000 After 5 months A withdrew Rs. 5000, B withdrew
respectively in a business. If the total profit at the end Rs. 4000 and C invests Rs. 6000 more. At the end of
of the year is Rs. 8800 and Nitesh, being an active the year, a total profit of Rs. 69900 was recorded. Find
partner, gets an additional 12.5% of the profit, find the the share of each.
total profit of Nitesh. Sol. Ratio of the capitals of A, B and C

Sol. Profit sharing ratio = 15000 : 18000 = 5 : 6 = 20000 × 5 + 15000 × 7 : 20000 x 5 + 16000 × 7 : 20000 × 5

Total profit = 8800 + 26000 × 7

12.5 = 205000 : 212000 : 282000 = 205 : 212 : 282.


Nitesh gets 12.5% of the profit = × 8800
100 205
= Rs. 1100  A’s share = Rs. (69900 × ) = Rs. 20500;
699
Net profit = 8800 – 1100 = Rs. 7700
212
B's share = Rs. (69900 × ) = Rs. 21200;
5 699
Nitesh share in profit = × 7700 = Rs. 3500
56
282
C’s share = Rs. (69900 x ) = Rs. 28200.
Total profit of Nitesh = 3500 + 1100 = Rs. 4600 699

PAGE # 24
 A can finish a work in x days and B is k times as

efficient as A (i.e. B will complete the work in


Work is defined as the amount of job assigned or the
amount of job actually done. x
Work is always considered as a whole or 1. days) Then time taken by both A & B working
k
Units of work : Work is measured by many units i.e. x
together to finish the job will be .
men-days, men-hours, men-minutes, machine-hours k 1
or in general person-time, machine-time.
 If A is k times as good as B and takes x days less
 If A and B can do a piece of work in x and y days than B to finish the work. Then the amount of time
respectively while working alone, then they will take kx
required by A and B working together is 2 days.
 xy  k –1
  days to complete the work if both are
xy Ex.46 A is thrice as good a work man as B and takes 60
working together. days less than B for doing a job. Find the time in which
1 1 they can do it together.
Proof : A’s one day work = and B’s one day work =
x y Sol. Here k = 3, x = 60
1 1  Time in which they can do it together
and (A + B)’s one day work = +
x y
3  60 3  60 1
xy =   22 days.
(A + B)’s one day work = 32 – 1 8 2
xy
 Time taken by both A and B (working together) to Ex.47 25 men were employed to do a piece of work in
xy 24 days. After 15 days, 10 more men were engaged
complete the work = .
xy and the work was finished a day too soon. In what time
 If A, B, C can do a piece of work in x, y, z days could they finish the work if extra men were not
respectively while working alone, then they will employed.
1 Sol. Actual work done = (25 × 15) + (25 + 10) × 8 = 655 man days.
together take days to complete the work. 655
1 1 1
  Time required by 25 men to complete this work is =
x y z 25
= 26.2 days.
Ex.44 A, B and C together can finish a piece of work in
4 days. A alone can do it in 9 days and B alone in Ex.48 If 12 men or 18 women can reap a field in 7 days, in
18 days. How many days will be taken by C to do it what time can 4 men & 8 women reap the same field.
alone. Sol. 12 men = 18 women
Sol. Let’s time taken by C alone to complete the work in  4 men = 6 women
x days
4 men + 8 women = 6 women + 8 women = 14 women
1 1 1 1
 =   Total work done = 18 × 7 women-days
4 9 18 x
 No. of days required to complete this work by 4 men
 x = 12 days.
18  7
Ex.45 A, B and C can do a piece of work 6, 8 and 12 days and 8 women = 14 women is = = 9 days.
14
respectively. B and C work together for 2 days, then
A takes C’s place. How long will it take to finish the
work.
1 1  Here the work done is in terms of filling or emptying a
Sol. Work done by B & C in 2 days = 2 ×    cistern.
 8 12 
th
5 Inlet pipe : It is the pipe connected to cistern which fill
= part of the work.
12 the cistern (time taken is in + ve).
th th Outlet pipe : It is the pipe connected to cistern which
 5  7
Remaining work =  1 –   part. empties the cistern. (time taken is – ve).
 12  12 th
th th  1
 1 1  7   If an inlet pipe fills a cistern in ‘a’ hours, then  
A and B’s one day work =      part. a
6 8  24  part is filled in 1 hr.
th
7
So, part of work is completed in 1 day..  If two inlet pipes A & B can fill a cistern in ‘m’ & ‘n’ hours
24
th  mn 
7
 part of work will be completed in respectively then together they will take   hrs. to
12 mn
1 7 24 7 fill the cistern.
  
= 7 12 7 12 = 2 days.
24

PAGE # 25
 If an inlet pipe fills a cistern in ‘m’ hours and an outlet
pipe empties it in ‘n’ hours, then the net part filled in

 1 1 dis tan ce
1 hr. When both the pipes are opened is  –  hours  Speed =
m n time
mn distan ce
and the cistern will get filled in hours, for cistern  Time =
n–m speed
to get filled, m < n.
 Distance = Speed × time
 If m > n, the cistern will never get filled, in this case a
 If a certain distance (from A to B) is covered at u km/hr
 mn 
completely filled cistern gets emptied in   hours. and the same distance (from B to A) is covered at
m – n v km/hr. then the average speed during the whole
 If an inlet pipe fills a cistern in m hrs. and takes n hrs. 2uv
journey is = km/hr..
longer to fill the cistern due to leak in the cistern, then u v
the time in which the leak will empty the cistern in Average speed :
 m If a body travels d1, d2, d3,........, dn distances with speeds
m × 1   .
 n s1, s2, ......., sn,........ respectively, then the average speed
Ex.49 A tank is emptied by 2 pipes and filled by a third. If the of the body through the total distance is given by :
1st two can empty the tank in 2 and 3 hrs. respectively Total distance covered
and third can fill it in 4 hours. How much time will it take Average speed = Total time taken
th
4 d1  d2  d3  ..........  dn
to empty the tank   full when all three are open.
5 = t1  t 2  t 3  .......  t n
Sol. Let the time taken to completely empty the tank is x hrs.
d1 d
Where, t1 = , t2 = 2 ...
1 1 1  1 s1 s2
   – 
x 2 3  4
Ex.51 A man travels Ist 50 km at 25 km/hr, next 40 km with
20 km/hr. and then 90 km at 15 km/hr. Then, find his
1 7
  average speed for the whole journey (in km/hr).
x 12

12 50  40  90
 x= hrs. Sol. Avg. Speed = = 18 km/hr..
7 50 40 90
 
25 20 15
12
Complete tank will be emptied in hrs. Ex.52 If a man travels @ 10 km/hr from A to B and again
7
@ 15 km/hr. from B to A. Find the average speed of
th
4 12 4 man for complete journey.
   tank will be emptied in =  5
5 7
2  10  15 2  10  15
Sol. Avg. speed =  = 12 km/hr..
48 10  15 25
= hrs.
35
PROBLEMS OF TRAINS
Ex.50 Two pipes M & N can fill a cistern in 12 & 16 hrs.
(i) Time taken by a train of length ‘a’ metres to pass a
respectively. If both the pipes are opened together, then
pole or a standing man or a signal post is equal to the
after how many minutes N should be closed so that
time taken by the train to cover ‘a’ metres.
the tank is full in 9 hrs.
Sol. Let N be closed after x hrs. Then, (ii) Time taken by a train of length ‘a’ metres to pass a
stationary object of length ‘b’ metres is the time taken
 1 1  1 by the train to cover (a + b) metres.
x   + (9 – x) =1
 12 16  12
(iii) Suppose two trains or two bodies are moving in
the same direction at u m/s and v m/s, where u > v,
3 then their relative speed = (u – v) m/s.
 x = 16 ×
12
(iv) Suppose two trains or two bodies are moving in
 x = 4 hrs = 240 minutes. opposite direction at u m/s and v m/s, then their relative
speed = (u + v) m/s.

PAGE # 26
(v) If two trains of length ‘a’ metres and ‘b’ metres are Ex.56 A train 125 m long passes a man, running at 5 kmph
moving in opposite directions at u m/s and v m/s, then in the same direction in which the train is going, in
10 seconds. Find the speed of the train.
time taken by the trains to cross each other
 125 
(a  b ) Sol. Speed of the train relative to man =   m/sec
= sec.  10 
(u  v )
 25 
=   m/sec
(vi) It two trains of length ‘a’ metres and ‘b’ metres are  2 
moving in the same direction at u m/s and v m/s then  25 18 
=    km/hr
the time taken by the faster train to cross the slower  2 5 
= 45 km/hr.
(a  b )
train = sec. Let the speed of the train be x kmph. Then, relative
(u  v ) speed = (x – 5) km/hr
(vii) If two trains (or bodies) start at the same time  x – 5 = 45 or x = 50 km/hr.

from points A and B towards each other and after Ex.57 Two goods train each 500 m long, are running in
crossing they take ‘a’ and ‘b’ sec in reaching B and A opposite directions on parallel tracks. Their speeds
are 45 km/hr and 30 km/hr respectively. Find the time
respectively, then (A’s speed) : (B’s speed) = ( b : a ) . taken by the slower train to pass the driver of the faster
Ex.53 Two trains running in the same direction at 40 km/hr one.
and 22 km/hr completely pass one another in 1 minute. Sol. Relative speed = (45 + 30) km/hr
If the length of the Ist train is 125 m., then what will be  5   125 
=  75   m/sec =   m/sec
the length of II nd
train.  18   6 
Sol. Relative speed of trains = 40 – 22 = 18 km/hr. Distance covered = (500 + 500) m = 1000 m.
18 km/hr. = 5 m/sec.  6 
Required time =  1000 

 sec = 48 sec.
Let the length of second train = L m.  125 
L  125
Time taken to cross each other = BOATS AND STREAMS
5

L  125 (i) In water, the direction along the stream is called


 = 60  L = 175 m. downstream. And, the direction against the stream is
5
called upstream.
Ex.54 A train passes a station platform in 36 seconds and (ii) If the speed of a boat in still water is u km/hr and the
a man standing on the platform in 20 seconds. If the speed of the stream is v km/hr, then :
speed of the train is 54 km/hr, what is the length of the Speed downstream = (u + v) km/hr.
platform ? Speed upstream = (u – v) km/hr.
 5  (iii) If the speed downstream is a km/hr and the speed
Sol. Speed =  54   m/sec = 15 m/sec
upstream is b km/hr, then :
 18 
Length of the train = (15 × 20) m = 300 m. 1
Speed of boat in still water = (a + b) km/hr
Let the length of the platform be x metres. 2
1
x  300 Rate of stream = (a – b) km/hr..
Then, = 15 2
36
Ex.58 A man can row three-quarters of a kilometre against the
 x + 300 = 540  x = 240 m. 1
stream in 11 minutes and covered the same distance
4
Ex.55 Two trains are moving in opposite directions @ 60 km/hr 1
and 90 km/hr. Their lengths are 1.10 km and 0.9 km with the stream in 7 min. Find the speed (in km/hr) of
2
respectively. Find the time taken by the slower train to the man in still water.
cross the faster train in seconds.  750  10
Sol. Relative speed = (60 + 90) km/hr Sol. Rate upstream =   m/sec = m/sec ;
 675  9

 5   125   750  5
= 150   m/sec =   m/sec.
Rate downstream =   m/sec = m/sec
 18   3   450  3

Distance covered = (1.10 + 0.9) km = 2 km = 2000 m. 1  10 5 


 Rate in still water =    m/sec
3  2 9 3

Required time =  2000   sec = 48 sec.
 125  25  25 18 
= m/sec =    km/hr = 5 km/hr..
18  18 5 

PAGE # 27
Ex.59 A man’s speed with the current is 15 km/hr and the (vii) Simple Interest : If the principal remains the same
speed of the current is 2.5 km/hr. Find the man’s speed throughout the loan period, then the interest paid by
against the current. the borrower is called simple interest.
Sol. Man’s rate in still water = (15 – 2.5) km/hr = 12.5 km/hr. PRT
Man’s rate against the current = (12.5 – 2.5) km/hr S.I. =
100
= 10 km/hr.
1
(viii) Compound Interest : If the borrower and the lender
Ex.60 A man can row 9 kmph in still water and finds that agree to fix up a certain interval of time (Say, a year or a
3
it takes him thrice as much time to row up than as to half year or a quarter of year etc.) so that the Amount
row down the same distance in the river. Find the speed (= Principal + Interest) at the end of an interval becomes
of the current. the principal for the next interval, then the total interest
Sol. Let speed upstream be x kmph. over all the intervals calculated in this way is called the
Then, speed downstream = 3x kmph. compound interest and is abbreviated as C.I.
1
Speed in still water = (3x + x) kmph = 2x kmph. NOTE : S.I. and C.I. are equal for Ist year.
2
28 14
 2x = x = .
3 3
14 To find simple interest and the amount when rate of
So, Speed upstream = km/hr ;
3 interest is given as percent per year :
Speed downstream = 14 km/hr.
Ex.62 Find the simple interest and the amount on
1 14 
Hence, speed of the current =  14 –  km/hr Rs. 2400 for 3 years 5 months and 15 days at the rate
2 3  of 9%.
14 2 Sol. Given : Principal (P) = Rs. 2400, Rate (R) = 9%.
= km/hr = 4 km/hr..
3 3 83
Time (T) = 3 years 5 months and 15 days = years.
Ex.61 A boat covers a certain distance downstream in 24
To find : Simple interest and the amount
1 PRT
1 hour, while it comes back in 1 hours. If the speed Simple interest =
2 100
of the stream be 3 kmph, what is the speed of the boat
9 83
in still water ? = Rs. 2400 × 
100 24
Sol. Let the speed of the boat in still water be x kmph. Then, = Rs. 747
Speed downstream = (x + 3) kmph, And the amount = Rs. 2400 + Rs. 747
Speed upstream = (x – 3) kmph. = Rs. 3147.
3
 (x + 3) × 1 = (x – 3) ×  2x + 6 = 3x – 9
2 INVERSE QUESTIONS ON SIMPLE INTEREST
 x = 15 kmph.
Ex.63 Find the rate of interest when Rs. 640 amounts to
SIMPLE INTEREST & COMPOUND INTEREST Rs. 841 and 60 paise for the period 2 years 7 months
and 15 days at a simple rate of interest.
DEFINITION : Sol. Given :
(i) Principal : The money borrowed or lent out is called Principal = Rs. 640
principal. Amount = Rs. 841.60
Interest = Rs. (841.60 – 640)
(ii) Interest : The additional money paid by the
= Rs. 201.60
borrower is called the interest.
21
(iii) Amount : The total money (interest + principal) Time = 2 years 7 months 15 days = years
8
paid by the borrower is called the amount. [ A  P  I] To find : Rate (R)
PRT
(iv) Rate of interest : If the borrower paid interest of Simple interest (I) =
100
Rs. x on Rs.100 for 1 year, then the rate of interest is
x percent per annum. R 21
 201.60 = 640 × 
100 8
(v) Time : The period for which the sum is borrowed
21 R 20160
is called the time.  640 ×  =
8 100 100
(vi) Conversion Period : The fixed interval of time at
20160 1 8 100
the end of which the interest is calculated and added  R= × × ×
100 640 21 1
to the principal at the beginning of the interval is called
 R = 12 %.
the conversion period.
Hence, rate of interest = 12%.

PAGE # 28
SOME SPECIAL QUESTIONS ON SIMPLE INTEREST COMPOUND INTEREST
Ex.64 A sum of money amount to Rs. 1237.50 and Computation of Compound Interest when
Rs. 1443.75 in 4 and 6 years respectively at a simple
Interest is compounded Annually.
rate of interest. Find principal and the rate of interest.
Sol. Principal + interest of 4 years = Rs. 1237.50 Ex.66 Find the compound interest on Rs. 8000 for 3 year at
Same principal + interest of 6 years = Rs. 1443.75 5% per annum.
 2 years interest on the given principal Sol. Principal for the first year = Rs. 8000, Rate = 5% per annum,
= Rs (1443.75 – 1237.50) = Rs. 206.25
T = 1 year.
 4 years interest on the given principal
PRT  8000  5  1
206.25 Interest for the first year = = Rs.  
= Rs.  4 = 412.50 100  100 
2
 Principal = Amount of 4 year – Interest of 4 year = Rs. 400
= Rs. 1237.50 – 412.50
 Amount at the end of the first year = Rs. (8000 + 400)
= Rs. 825
= Rs. 8400
To find : Rate (R)
Given : Principal = 825 Now, principal for the second year = Rs. 8400
Interest = 412.50 PRT
Interest for the second year =
Time = 4 years 100
From the formulae,  8400  5  1
= Rs.  
PRT  100 
Simple interest =
100
R = Rs. 420
 412.50 = 825 × ×4
100  Amount at the end of the second year
R 41250 = Rs. (8400 + 420) = Rs. 8820
 825 × ×4 =
100 100
PRT
41250 100 1 1 Interest for the third year =
 R=    100
100 1 825 4
 R = 12.5 8820  5  1
= Rs. = Rs. 441
Hence, the required rate is 12.5%. 100

Ex.65 Madhav lent out Rs. 7953 for 2 years and Rs. 1800  Amount at the end of the third year
for 3 years at the same rate of simple interest. If he got = Rs. (8820 + 441) = Rs. 9261
Rs. 2343.66 as total interest then find the percent rate Now, we know that total C.I. = Amount – Principal
of interest. = Rs. (9261 – 8000) = Rs. 1261
Sol. Let the percent rate of interest be x %
We can also find the C.I. as follows
7953  2  x 15906 Total C.I. = Interest for the first year + Interest for the
=  ×x
100 100 second year + Interest for third year
and interest on Rs. 1800 for 3 years at the rate of x% = Rs. (400 + 420 + 441) = Rs. 1261

1800  3  x Computation of Compound Interest When


= = 54 x
100 Interest is compounded Half yearly.
1
According to problem, Ex.67 Find the compound interest on Rs. 8000 for 1
2
15906 x years at 10% per annum, interest being payable half
+ 54x = 2343.66
100 yearly.
15906 x  5400 x 234366 Sol. We have
 =
100 100 Rate of interest = 10% per annum = 5% per half year,
 21306x = 234366 1
Time = 1 years = 3 half year..
234366 2
 x= Original principal = Rs. 8000
21306
 8000  5  1
 x = 11% Interest for the first half year = Rs.  
 100 
Hence, the required rate of interest = 11%.
= Rs. 400

PAGE # 29
Amount at the end of the first half year COMPUTATION OF COMPOUND INTEREST
= Rs. 8000 + 400 = Rs. 8400 BY USING FORMULAE
Principal for the second half year = Rs. 8400
(i) Let P be the principal and the rate of interest be R%
 8400  5  1  per annum. If the interest is compounded annually
Interest for the second half year = Rs.   then the amount A and the compound interest C.I. at
 100 
the end of n years.
= Rs. 420 n
 R 
Amount at the end of the second half year Given by A = P 1  
= Rs. 8400 + Rs. 420 = Rs. 8820  100 
 n 
Principal for the third half year = Rs. 8820 R 
and C.I. = A – P = P 1  100   1 respectively..
  
 8820  5  1 
Interest for the third half year = Rs.   (ii) Let P be the principal and the rate of interest be
 100  R% per annum. If the interest is compounded k times
= Rs. 441 in a year annually, then the amount A and the compound
Amount at the end of third half year interest. C.I. at the end of n years is given by
nk
= Rs. 8820 + Rs. 441 = Rs. 9261  R 
A = P 1  
 Compound interest  100k 
= Rs. 9261 – Rs. 8000 = Rs. 1261.  R 
nk 
and C.I. = A – P =   1   – 1 P respectively..
Computation of compound Interest when  100k  
Interest is Compounded Quarterly :
(iii) Let P be the principal and the rate of interest be
Ex.68 Find the compound interest on Rs. 10,000 for 1 year R1% for first year, R2% for second year, R3% for third
at 20% per annum interest being payable quarterly. yearand so on and in the last Rn% for the nth year.
Then, the amount A and the compound interest C.I. at
Sol. We have Rate of interest = 20% per annum
the end of n years are given by
20
= = 5% per quarter  R1  R   R 
4 A = P 1   1  2 ..........1  n 
Time = 1 year = 4 quarters.  100  100   100 
and C.I. = (A – P) respectively
Principal for the first quarter = Rs. 10000
(iv) Let P be the principal and the rate of interest be
 10000  5  1 
Interest for the first quarter = Rs.   R% per annum. If the interest is compounded annually
 100 
1
= Rs. 500 but time is the fraction of a year, say 5 year, then
Amount at the end of first quarter = Rs. 10000 + Rs. 500 4
amount A is given by
= Rs. 10500 5
 R   R/4 
Principal for the second quarter = Rs. 10500 A = P 1   1   and C.I. = A – P..
 100   100 
 10500  5  1  Ex.69 Find the compound interest on Rs. 12000 for 3 years
Interest for the second quarter = Rs.  
at 10% per annum compounded annually.
 100 
= Rs. 525 Sol. We know that the amount A at the end of n years at the
Amount at the end of second quarter rate of R % per annum when the interest is
compounded annually is given by :
= Rs. 10500 + Rs. 525 = Rs. 11025
n
 R 
Principal for the third quarter = Rs. 11025 A = P 1  
 100 
11025  5  1 Here, P = Rs. 12000, R = 10% per annum and n = 3.
Interest for the third quarter = Rs.
100  Amount after 3 years
= Rs. 551.25 3 3
 R   10 
Amount at the end of the third quarter = P 1   = Rs. 12000 ×  1  
= Rs. 11025 + Rs. 551.25  100   100 
3
= Rs. 11576.25  1 
= Rs. 12000 ×  1  
Principal for the fourth quarter = Rs. 11576.25  10 
3
Interest for the fourth quarter  11 
= Rs. 12000 ×  
 11576.25  5  1   10 
= Rs.   = Rs. 578.8125 11 11 11
 100  = Rs. 12000 × × ×
Amount at the end of the fourth quarter 10 10 10
= Rs (12 × 11 × 11 × 11) = Rs. 15972.
= Rs. 11576.25 + Rs. 578.8125
Now, Compound interest = A – P
= Rs. 12155.0625
 Compound interest = Rs.15972 – Rs.12000
Compound interest = Rs. 12155.0625 – Rs. 10000
= Rs. 3972.
= Rs. 2155.0625

PAGE # 30
1
10. 37 % of the candidates in an examination were girls,
2
NOTE : More than one correct option may be possible. 1
75% of the boys and 62 % of the girls passed and
P E R C E N TAG E 2
342 girls failed. The number of boys failed was :
1. Two-fifth of one-third of three-seventh of a number is
(A) 350 (B) 360
15. What is 40 percent of that number ?
(A) 72 (B) 84 (C) 370 (D) 380
(C) 105 (D) 140 5
11. part of the population in a village are males. If 30%
2. An organization has 32 members. It hopes to 9
increase the membership by 50% each year. If this of the males are married, the percentage of unmarried
is done, how many members will the organization females in the total population is :
have in 5 years ?
(A) 243 (B) 200 7
(A) 20% (B) 27 %
(C) 162 (D) 112 9
(D) 40% (D) 70%
3. When any number is divided by 12, then quotient
1 12. What percent decrease in salaries would exactly cancel
becomes th of the other number By how much out the 20 percent increase ?
4
percent first number is greater than the second 2
number ? (A) 16 % (B) 18%
3
(A) 150% (B) 200% 1
(C) 300% (D) Date inadequate (C) 20% (D) 33 %
3
4
4. A tempo is insured to the extent of of its original 13. The price of a T.V. set is decreased by 25% as a result
5
value. If the premium on it at the rate of 1.3 percent of which the sale increased by 20%. What will be the
amounts to Rs.910, the original value of the tempo is : effect on the total revenue of the shop ?
(A) Rs.78500 (B) Rs.80000 (A) No effect (B) 5% decrease
(C) Rs.82500 (D) Rs.87500 (C) 10% increase (D) None of these
5. Gauri went to the stationers and bought things worth 14. In a fraction, if numerator is increased by 40% and
Rs.25, out of which 30 paise went on sales tax on
denominator is increased by 80%, then what fraction
taxable purchases. If the tax rate was 6%, then what
of the original is the new fraction ?
was the cost of the tax free items ?
(A) Rs. 15 (B) Rs. 15.70 1 7
(A) (B)
(C) Rs. 19.70 (D) Rs. 20 2 9
7
6. A student has to obtain 33% of the total marks to pass. (C) (D) Date inadequate
18
He got 125 marks and failed by 40 marks. The
maximum marks are : 15. The price of wheat falls by 16%. By what percentage a
(A) 300 (B) 500 person can increase the consumption of wheat so that
(C) 800 (D) 1000 his overall budget does not change ?
7. 10% of the voters did not cast their votes in an election (A) 16% (B) 18%
between two candidates. 10% of the votes polled were (C) 18.5% (D) 19%
found invalid. The successful candidate got 54% of
the valid votes and won by a margin of 1620 votes. The 16. A’s income is 25% more than B’s income B’s income
number of voters enrolled in the voters’ list was : in terms of A’s income is :
(A) 25000 (B) 33000 (A) 75% (B) 80%
(C) 35000 (D) 40000 (C) 90% (D) 96%
8. A scored 30% marks and failed by 15 marks. B scored
17. Fresh fruit contains 68% water and dry fruit contains
40% marks and obtained 35 marks more than those
20% water. How much dry fruit can be obtained from
required to pass. The pass percentage is :
(A) 33% (B) 38% 100 kg of fresh fruits ?
(C) 43% (D) 46% (A) 32 kg (B) 40 kg
9. In a recent survey, 40% houses contained two or more (C) 52 kg (D) 80 kg
people. Of those houses containing only one person, 18. Milk contains 5% water. What quantity of pure milk should
25% were having only a male. What is the percentage
be added to 10 litres of milk to reduce this to 2%.
of all houses, which contain exactly one female and no
(A) 5 liters (B) 7 litres
males ?
(A) 15 (B) 45 (C) Cannot be determined (D) None of these
(C) 75 (D) Can’t be determined

PAGE # 31
19. If half of x is y and one-third of y is z then : 28. A book was sold for Rs. 27.50 with a profit of 10%. If it
(A) z = 6% of x (B) z = 16.66% of x were sold for Rs. 25.75, that what would have been the
(C) z = 60% of x (D) z = 30% of x percentage of profit or loss ?
20. The total income of A and B is Rs 6000. A spends 60 % (A) 4.5% (B) 5.0%
of his income and B spends 80 % of his income. If (C) 3.0% (D) 3.25%
their savings are equal, then what is the income of A ? 29. The C.P of 21 articles is equal to S.P. of 18 articles.
(A) 2000 (B) 2400
Find the gain or loss percent.
(C) 2600 (D) 2800
2
(A) 16 % (B) 8%
21. One bacteria splits into eight bacteria of the next 3
generation. But due to environment, only 50 % of one (C) 10% (D) 12%
generation can produce the next generation. If the
30. By selling 33 metres of cloth, one gains the selling of
seventh generation number is 4096 million, what is
11 metres. Find the gain percent.
the number in first generation ?
(A) 17.5% (B) 33.33%
(A) 1 million (B) 2 million
(C) 40% (D) 50%
(C) 4 million (D) 8 million
31. A vendor bought bananas at 6 for Rs.10 and sold them
22. A man earns x % on the first Rs.2,000 and y % on the
at 4 for Rs.6. Find his loss percent.
rest of his income. If he earns Rs. 700 from Rs. 4,000
and Rs. 900 from Rs. 5,000 of income, find x %. (A) 12% (B) 8%
(A) 20 % (B) 15 % (C) 6% (D) 10%
(C) 25 % (D) None of these 32. If the manufacturer gains 10%, the wholesale dealer
23. I bought 5 pens, 7 pencils and 4 erasers. Rajan bought 15% and the retailer 25%, then find the cost of
6 pens, 8 erasers and 14 pencils for an amount which production of a table, the retail price of which is
was half more what I had paid. What percent of the Rs. 1265 ?
total amount paid by me was paid for the pens? (A) 600 (B) 800
(A) 37·5 % (B) 62·5 % (C) 700 (D) 900
(C) 50 % (D) None of these
33. After getting two successive discounts, a shirt with a
PROFIT, LOSS & DISCOUNT list price of Rs. 150 is available at Rs.105. If the second
24. The price of a Maruti car rises by 30 % while the sales discount is 12.5%, find the first discount.
of the car comes down by 20 %. What is the percentage (A) 25% (B) 12%
change in the total revenue ? (C) 15% (D) 20%
(A) – 4 % (B) – 2 % 1
34. A shopkeeper expects a gain of 22 % on his cost
(C) + 4 % (D) + 2 % 2
price. If in a week, his sale was of Rs. 392, what was
25. A person who has a certain amount with him goes to
his profit ?
market. He can buy 50 oranges or 40 mangoes. He
(A) Rs. 18.20 (B) Rs. 70
retains 10% of the amount for taxi fares and buys 20
(C) Rs. 72 (D) Rs. 88.25
mangoes and of the balance, he purchases oranges.
Number of oranges he can purchase is : 35. Jacob bought a scooter for a certain sum of money. He
(A) 36 (B) 40 spent 10% of the cost on repairs and sold the scooter
(C) 15 (D) 20 for a profit of Rs. 1100. How much did he spend on
26. A shopkeeper sells 25 articles at Rs. 45 per article repairs if he made a profit of 20% ?
after giving 10% discount and earns 50% profit. If the (A) Rs. 400 (B) Rs. 440
discount is not given, the profit gained is : (C) Rs.500 (D) Rs.550
2
(A) 60% (B) 60 % 36. If the selling price of 50 articles is equal to the cost
3
price of 40 articles, then the loss or gain percent is :
2
(C) 66% (D) 66 % (A) 20% gain (B) 20% loss
3
27. A person incurs 5 % loss by selling a watch for (C) 25% loss (D) 25% gain
Rs. 1140. At what price should the watch be sold to 37. A man bought a number of clips at 3 for a rupee and an
earn 5 % profit ? equal number at 2 for a rupee. At what price per dozen
(A) Rs. 1275 (B) Rs. 1280 should he sell them to make a profit of 20% ?
(C) Rs. 1210 (D) Rs. 1260
(A) Rs. 4 (B) Rs. 5
(C) Rs. 6 (D) Rs. 7

PAGE # 32
38. On selling a T.V. at 5% gain and a fridge at 10% gain, a
1 1 1 1
shopkeeper gains Rs 2000. But if he sells the T.V. at 46. If : = : , then the value of x is :
5 x x 125
10% gain and the fridge at 5% loss. He gains Rs 1500
on the transaction. Find the actual prices of T.V. and (A) 15 (B) 20
fridge. (C) 25 (D) 35
(A) Rs.20,000 & Rs.15,000
47. If x : y = 5 : 2, then (8x + 9y) : (8x + 2y) is :
(B) Rs.20,000 & Rs.10,000
(C) Rs.10,000 & Rs.12,000 (A) 22 : 29 (B) 26 : 61
(D) Rs.15,000 & Rs.18,000 (C) 29 : 22 (D) 61 : 26

39. Garima purchased a briefcase with an additional 10 48. The salaries of A, B, C are in the ratio 2 : 3 : 5. If the
% discount on the reduced price after deducting 20 % increments of 15%, 10% and 20% are allowed
on the labelled price. If the labelled price was Rs. 1400, respectively in their salaries, then what will be the new
at what price did she purchase the briefcase ? ratio of their salaries?
(A) Rs. 980 (B) Rs. 1008 (A) 3 : 3 : 10
(C) Rs. 1056 (D) Rs.1120 (B) 10 : 11 : 20
(C) 23 : 33 : 60
40. A shopkeeper purchased 150 identical pieces of (D) Cannot be determined
calculators at the rate of Rs. 250 each. He spent an
amount of Rs. 2500 on transport and packing. He fixed 49. Two numbers are in the ratio 3 : 5. If 9 is subtracted
the labelled price of each calculator at Rs. 320. from each, the new numbers are in the ratio 12 : 23.
However, he decided to give a discount of 5 % on the The smaller number is :
labelled price. What is the percentage profit earned by (A) 27 (B) 33
him? (C) 49 (D) 55
(A) 14 % (B) 15 % 50. The ratio of three numbers is 3 : 4 : 7 and their product
(C) 16 % (D) 20 % is 18144. The numbers are :
41. By selling an umbrella for Rs. 300, a shopkeeper gains (A) 9, 12, 21 (B) 15, 20, 25
20 %. During a clearance sale, the shopkeeper allows (C) 18, 24, 42 (D) None of these
a discount of 10 % on the marked price. His gain 51. Two numbers are respectively 20% and 50% more
percent during the sale is : than a third number. The ratio of the two numbers is :
(A) 7 (B) 7.5 (A) 2 : 5 (B) 3 : 5
(C) 8 (D) 9 (C) 4 : 5 (D) 6 : 7
42. A shopkeeper sells a badminton racket, whose marked 52. The ratio of the number of boys and girls in a college is
price is Rs.30, at a discount of 15 % and gives a shuttle 7 : 8. If the percentage increase in the number of boys
cock costing Rs.1.50 free with each racket. Even then and girls be 20 % and 10 % respectively, what will be
makes a profit of 20 %. His cost price per racket is : the new ratio?
(A) Rs. 19.75 (B) Rs. 20 (A) 8 : 9
(C) Rs. 21 (D) Rs. 21.25 (B) 17 : 18
RATIO AND PROPORTION (C) 21 : 22
(D) Cannot be determined
43. If 25% of a number is subtracted from a second
number, the second number reduce to its five-sixth. 53. The prices of a scooter and a TV are in the ratio 7 : 5. If
What is the ratio of the first number to the second the scooter costs Rs. 8000 more than a TV set, then
number ? the price of a T.V set is :
(A) 1 : 3 (B) 2 : 3 (A) Rs. 20,000 (B) Rs. 24,000
(C) 3 : 2 (D) Data inadequate (C) Rs. 28,000 (D) Rs. 32,000
44. A trader mixes three varieties of groundnuts costing 54. An amount of Rs. 735 was divided between A, B and C.
Rs.50, Rs. 20 and Rs. 30 per kg in the ratio 2 : 4 : 3 in If each of them had received Rs. 25 less, their shares
terms of-weight, and sells the mixture at Rs.33 per kg. would have been in the ratio of 1 : 3 : 2, The money
What percentage of profit does he make ? received by C was :
(A) 8 % (B) 9 % (A) Rs. 195 (B) Rs. 200
(C) 10 % (D) None of these (C) Rs. 225 (D) Rs. 245
45. The cash difference between the selling prices of an 55. What is the ratio whose terms differ by 40 and the
article at a profit of 4 % and 6 % is Rs. 3. The ratio of the 2
two selling prices is : measure of which is ?
7
(A) 51 : 52 (B) 52 : 53 (A) 16 : 56 (B) 14 : 56
(C) 51 : 53 (D) 52 : 55 (C) 15 : 56 (D) 16 : 72

PAGE # 33
56. The speeds of three cars are in the ratio 5 : 4 : 6. The 65. A invested Rs. 76000 in a business. After few months,
ratio between the time taken by them to travel the same B joined him with Rs. 57000. At the end of the year, the
distance is : total profit was divided between them in the ratio of
(A) 5 : 4 : 6 (B) 6 : 4 : 5 2 : 1. After how many months did B join ?
(C) 10 : 12 : 15 (D) 12 : 15 : 10 (A) 4 (B) 8
MIXTURE & ALLIGATION (C) 9 (D) None of these

57. Arun purchased 30 kg of wheat at the rate of 66. Simran started a software business by investing
Rs.11.50 per kg and 20 kg of wheat at the rate of Rs. 50,000. After six months, Nanda joined her with a
Rs.14.25 per kg. He mixed the two and sold the mixture. capital of Rs. 80,000. After 3 years, they earned a profit
Approximately what price per kg should he sell the of Rs. 24,500. What was Simran's share in the profit ?
mixture to make 30% profit ? (A) Rs. 9423 (B) Rs. 10,500
(A) Rs. 14.80 (B) Rs. 15.40 (C) Rs. 12,500 (D) Rs. 14,000
(C) Rs. 15.60 (D) Rs. 16.30 67. A, B and C enter into a partnership. They invest Rs.
58. Padam purchased 30 kg of rice at the rate of 40,000, Rs. 80,000 and Rs. 1,20,000 respectively. At
Rs.17.50 per kg and another 30 kg rice at a certain the end of the first year, B withdraws Rs. 40,000, while
rate. He mixed the two and sold the entire quantity at at the end of the second year, C withdraws Rs.80,000.
the rate of Rs.18.60 per kg and made 20% overall profit. In what ratio will the profit be shared at the end of
At what price per kg did he purchase the lot of another 3 years ?
30 kg rice ? (A) 2 : 3 : 5 (B) 3 : 4 : 7
(A) Rs. 12.50 (B) Rs. 13.50 (C) 4 : 5 : 9 (D) None of these
(C) Rs. 14.50 (D) Rs. 15.50
68. A, B and C enter into partnership A invests some money
59. How many kg. of wheat costing Rs.8 per kg must be at the beginning, B invests double the amount after
mixed with 36 kg of rice costing Rs.5.40 per kg so that 6 months and C invests thrice the amount after
20% gain may be obtained by selling the mixture at 8 months. If the annual profit be Rs. 27,000, C's share
Rs.7.20 per kg ? is :
(A) 10.08 kg (B) 8.6 kg (A) Rs. 8625 (B) Rs. 9000
(C) 9.2 kg (D) 10.8 kg (C) Rs.10,800 (D) Rs. 11,250
60. In what ratio must water be mixed with milk costing 69. In a business, A and C invested amounts in the ratio
Rs.12 per litre to obtain a mixture worth of Rs. 8 per litre ? 2 : 1, whereas the ratio between amounts invested by
(A) 1 : 2 (B) 2 : 1 A and B was 3 : 2. If Rs. 1,57,300 was their profit, how
(C) 2 : 3 (D) 3 : 2 much amount did B receive ?
61. In what ratio must a grocer mix two varieties of tea (A) Rs.24,200 (B) Rs.36,300
worth Rs. 60 per kg and Rs. 65 per kg so that by selling (C) Rs.48,400 (D) Rs.72,600
the mixture at Rs. 68.20 a kg he may gain 10% ? 70. A and B started a partnership business investing some
(A) 3 : 2 (B) 3 : 4
amount in the ratio of 3 : 5. C joined them after six
(C) 3 : 5 (D) 4 : 5
months with an amount equal to that of B. In what
P ARTN E RS HI P
proportion should the profit at the end of one year be
62. An amount, of Rs. 2430 is divided among A, B and C distributed among A, B and C ?
such that if their shares be reduced by Rs. 5, Rs. 10 (A) 3 : 5 : 2 (B) 3 : 5 : 5
and Rs.15 respectively, the remainders shall be in the
(C) 6 : 10 : 5 (D) Data inadequate
ratio of 3 : 4 : 5. Then, B's share was :
(A) Rs. 605 (B) Rs. 790 71. A, B, C started a business with their investments in the
(C) Rs. 800 (D) Rs. 810 ratio 1 : 3 : 5. After 4 months, A invested the same
63. Sum of Rs. 53 is divided among A, B, C in such a way amount as before and B as well as C withdrew half of
that A gets Rs. 7 more than what B gets and B gets their investments. The ratio of their profits at the end of
Rs. 8 more than what C gets. The ratio of their shares the year is :
is :
(A) 4 : 3 : 5 (B) 5 : 6 : 10
(A) 16 : 9 : 18 (B) 25 : 18 : 10
(C) 6 : 5 : 10 (D) 10 : 5 : 6
(C) 18 : 25 : 10 (D) 15 : 8 : 30
64. A, B and C enter into partnership. A invests 3 times as 72. A and B entered into partnership with capitals in the
much as B invests and B invests two-third of what 1
ratio 4 : 5. After 3 months, A withdrew of his capital
C invests. At the end of the year, the profit earned is 4
Rs. 6600. What is the share of B ? 1
and B withdrew of his capital. The gain at the end of
(A) Rs. 3600 (B) Rs. 1800 5
(C) Rs. 1200 (D) None of these 10 months was Rs. 760. A’s share in this profit is :
(A) Rs.330 (B) Rs.360
(C) Rs.380 (D) Rs.430

PAGE # 34
73. A starts business with Rs. 3500 and after 5 months, B 82. A is thrice as efficient as B, and B is twice as efficient
joins with A as his partner. After a year, the profit is as C. If A, B and C work together, how long will they take
divided in the ratio 2 : 3. What is B's contribution in the to complete a job which B completes in 10 days ?
capital? 20 11
(A) Rs.7500 (B) Rs.8000 (A) days (B) days
9 9
(C) Rs.8500 (D) Rs.9000 (C) 3 days (D) None of these
TIME & WORK 83. A can do a piece of work in 7 days of 9 hr each, and
4 B can do it in 6 days of 7 hr each. How long will they
74. A tin of oil was full. When six bottles of oil were taken
5 42
3 take to do it working together hr a day ?
out and four bottles of oil were poured in, it was full. 5
4 (A) 3 days (B) 4 days
How many bottles of oil did the tin contain initially ? (C) 4.5 days (D) None of these
(A) 16 (B) 40
(C) 32 (D) None of these 84. A can do a piece of work in 24 days. If B is 60% more
efficient then the number of days required by B to do
75. If 5 men can cane 5 chairs in 5 hours, then 1 man
the twice as large as the earlier work is :
shall cane 1 chair in how many hours.
(A) 24 (B) 36
(A) 1 hour (B) 15 hours
(C) 15 (D) 30
(C) 5 hours (D) 10 hours
85. A works twice as fast as B. If B can complete a work in
76. A can do a piece of work in 9 days and B in 18 days.
12 days independently, then the number of days in
They began the work together but 3 days before the
which A and B can together finish the work is :
completion of work, A leaves. The time taken to
(A) 4 days (B) 6 days
complete the work is :
(C) 8 days (D) 18 days
(A) 7 days (B) 5 days
(C) 8 days (D) 11 days 86. Taps A and B can fill a tank in 12 min and 15 min
77. If 4 workers can dig 20 m long ditch in 6 days, find out respectively. If both are opened and A is closed after
the part dug by 2 workers in a day (in metre) ? 3 min, how long will it take for B to fill the tank ?
(A) 7 min 45 sec (B) 7 min 15 sec
2 2
(A) 1 (B) 3 (C) 8 min 5 sec (D) 8 min 15 sec
3 4
2 3 87. A leak in the bottom of a tank can empty it in 6 hr. A pipe
(C) 3 (D) 1
3 2 fills the tank at 4 L/min. When the tank is full, the inlet is
opened, but due to the leak the tank is emptied in 8 hr.
78. A and B together can complete a work in 12 days. A
What is the capacity of the tank ?
alone can complete it in 20 days. If B now does the
(A) 5,260 L (B) 5,760 L
work for half a day daily. Then in how many days A and
B together will complete the work ? (C) 5,846 L (D) 6,970 L
(A) 15 days (B) 20 days 88. Because of a leak, a tank which usually is filled in
(C) 45 days (D) 30 days 3 hrs, now takes 3.5 hrs to be filled. The leak alone will
79. Harpal is thrice as good a workman as Kewal and empty the filled tank in :
takes 10 days less to do a piece of work than Kewal (A) 18 hrs (B) 21 hrs
takes. Then Kewal can do that work in : (C) 15 hrs
(A) 8 days (B) 12 days (D) Cannot be determined
(C) 13 days (D) 15 days 89. In a camp of 100 students, there is ration which lasts
80. A takes 10 days less than the time taken by B to finish a for 8 days. After the first 2 days, 50 more students join
piece of work. If both A and B together can finish the work them. How long will the ration last now ? (Assume all
in 12 days, the time taken by B to finish the work is : the students have equal eating capacity)
(A) 20 days (B) 30 days (A) 4 days (B) 6 days
(C) 40 days (D) 50 days (C) 8 days (D) None

81. A, B and C can complete a work, working alone in TIME,SPEED AND DISTANCE
10, 15 and 20 days respectively. If all of them work 90. Two trains of equal length are running on parallel lines
together to complete the work, what fraction of the work in the same direction at 46 km/hr and 36 km/hr. The
would have done by B ? faster train passes the slower train in 36 seconds.
4 1 The length of each train is :
(A) (B)
13 2 (A) 50 m (B) 72 m
1 6 (C) 80 m (D) 82 m
(C) (D)
3 13

PAGE # 35
91. A train X speeding with 120 kmph crosses another 100. What sum will become Rs 9826 in 18 months if the
train Y, running in the same direction, in 2 minutes. 1
If the lengths of the trains X and Y be 100 m and 200 m rate of interest is 2 % per annum and the interest is
2
respectively, what is the speed of train Y ? compounded half-yearly ?
(A) 111 km/hr (B) 123 km/hr (A) Rs. 9466.54 (B) Rs. 9646.54
(C) 127 km/hr (D) 129 km/hr (C) Rs. 9566.54 (D) Rs. 9456.54
92. A train overtakes two persons walking along a railway 101. In what time will Rs 64000 amount to Rs 68921 at
track. The first one walks at 4.5 km/hr. The other one
5% per annum, interest being compounded
walks at 5.4 km/ hr. The train needs 8.4 and 8.5 seconds
half-yearly ?
respectively to overtake them. What is the speed of the
train if both the persons are walking in the same 1 2
(A) 1 years (B) years
direction as the train ? 2 3
(A) 66 km/hr (B) 72 km/hr (C) 2 years (D) None of these
(C) 78 km/hr (D) 81 km/hr
102. Reena borrowed from Kamila certain sum for two years
93. Two trains running in opposite directions cross a man at simple interest. Reena lent this to Hamid at the same
standing on the platform in 27 seconds and 17 seconds rate for two years compound interest. At the end of two
respectively and they cross each other in 23 seconds.
years she received Rs. 110 as compound interest but
The ratio of their speeds is :
paid Rs. 100 as simple interest. Find the sum and rate
(A) 1 : 3 (B) 3 : 2
of interest.
(C) 3 : 4 (D) None of these
(A) Rs 250 and 20 % (B) Rs 250 and 25 %
94. A railway passenger counts the telegraph posts on the (C) Rs 200 and 20 % (D) Rs 200 and 25 %
line as he passes them while travelling; If the poles
are 50 m apart and the train is going at the speed of 103. The value of refrigerator which was purchased 2 years
48 km/hr. How many posts will be passed by the train ago depreciates at 12 % p.a. If its present value is
per minute ? Rs 9680, for how much was it purchased ?
(A) 15 (B) 16 (A) Rs. 11500 (B) Rs. 10500
(C) 18 (D) 20 (C) Rs. 12000 (D) Rs. 12500
95. A man is traveling by car at the rate of 40 km/hr. After 104. The compound interest on Rs. 10,000 in 2 years at
every 80 km, he rests for 20 min. How long will he take 4 % per annum, being compounded half yearly is :
to cover a distance of 240 km ? (A) Rs. 832.24 (B) Rs. 828.82
(A) 6 hr 40 min (B) 6 hr (C) Rs. 824.32 (D) Rs. 912.86
(C) 6 hr 20 min (D) 7 hr
105. A sum of Rs. 500 was lent for two years at 2 %
SIMPLE INTEREST, COMPOUND INTEREST
compound interest. The interest for two years will be :
96. At what percent per annum, simple interest will double (A) Rs. 20.00 (B) Rs. 25.00
a sum of money in 12 years ? (C) Rs. 50.20 (D) Rs. 20.20
1 106. Compute the compound interest on Rs. 1000 for
(A) 8 % (B) 24 %
3 2 years at 10 % per annum when compounded
25 half-yearly.
(C) % (D) 8.25 % (A) Rs. 315.15 (B) Rs. 200
4
(C) Rs. 215.50 (D) None of these
97. Find the amount of Rs 12000 after 2 years
compounded annually, the rate of interest being 5 % 107. Find the compound interest on Rs 24000 at 15 % per
p.a. during the first year and 6 % p.a. during the second 1
annum for 2 years.
year, also find the compound interest. 3
(A) Rs. 1356 (B) Rs. 1200 (A) Rs. 9327 (B) Rs. 9723
(C) Rs. 1256 (D) None of these (C) Rs. 9372 (D) Rs. 9237
98. Find the compound interest on Rs 31250 at 8 % per
3
annum for 2 years.
4
(A) Rs. 8637 (B) Rs. 7387
(C) Rs. 1250 (D) None of these
99. A certain sum amounts to Rs 72900 in 2 years at
8 % per annum compound interest. Find the sum.
(A) Rs. 72000 (B) Rs. 62500
(C) Rs. 135000 (D) None of these

PAGE # 36
5. Scienctists in an R & D company made three design
improvements on a car : the first saves 50% of fuel, the
second saves 30% of fuel and the third saves 20%. If
COMPETITIVE EXAM PREVIOUS YEAR
the company implements all three design changes at
QUESTION
once, the new car will consume fuel that is ___% of the
1. If a, b, c are in continued proportion, the expression fuel consumption of normal car [IJSO - 2014]
(A) 50% (B) 100%
a 2  ab  b 2 (C) 28% (D) 20%
can be simplified to : [IJSO-2010]
b 2  bc  c 2 6. The diagram shows a road network. All vehicles drive
in one direction from A to B. Numbers represent the
ac a
(A) (B) maximum flow rate (capacity of roads) in vehicles per
ab c hour. The maximum number of vehicles that can drive
through the network every hour is [IJSO - 2014]
c ac
(C) (D)
a b

2. In a school, the average score of all 1400 students at


an examination was found to be 69.5 The average score
of boys in the school was 68 and that of girls was 72.
Hence, the number of boys and the number of girls
respectively are [IJSO-2011]
(A) 315 (B) 240
(A) 680 and 720 (B) 720 and 680
(C) 215 (D) 340
(C) 875 and 525. (D) 800 and 600.
7. A water filter advertisement claims to provide 8 litres of
3. The distance between two spots A & B on the same water per hour. How much time does it take to fill four
bank of the river is 75km. Speed of the boat in still bottles of 1.5 litres each ? [IJSO - 2014]
water is twice as much as that of the speed of the (A) 2 hr (B) 1 hr
water current of river. The boat travels in the river from (C) 30 min (D) 45 min
A to B and returns back to the spot in 16 hour. What is
the speed of the boat in still water? [IJSO-2012] 8. If a : b = c : d then how many of the following statements
(A) 12.5kmph (B) 15kmph are true? [IJSO - 2015]
(C) 16kmph (D) 18kmph (i) c(a + b) = a(c + d)
(ii) d(a – b) = b(c – d)
4. Y varies inversely as x. If x is increased by 25%, then (iii) (a2 + b2) (ac – bd) = (a2 – b2) (ac + bd)
the value of percentage change to y is [IJSO-2012]  a2   c2   2ac 
(A) 80% (B) 75%    
(iv)  2  +  2  =  
(C) 60% (D) 62.5%  b   d   bd 
(A) 1 (B) 2
(C) 3 (D) All



PAGE # 37
POLYNOMIALS

(iii) Quadratic polynomial :

A polynomial of degree two is called a quadratic


(a) Definition : polynomial. The general form of a quadratic polynomial

An algebraic expression f(x) of the form is ax2 + bx + c where a  0

f(x) = a0 + a1x + a2x2 + ..........+ anxn, Where a0 ,a1, a2.....an e.g. x2 + x + 1, 2x2 + 1, 3x2 + 2x + 1 etc.
are real numbers and all the indices of x are non
(iv) Cubic polynomial :
negative integers is called a polynomial in x and the
A polynomial of degree three is called a cubic
highest index n is called the degree of the polynomial,
polynomial. The general form of a cubic polynomial is
if an  0. Here a0 , a1x, a2x2 .....,anxn are called the terms
ax3 + bx2 + cx + d, where a  0
of the polynomial and a0, a1, a2, ...... an are called various
e.g. x3 + x2 + x + 1, x3 + 2x + 1, 2x3 + 1 etc.
co-efficients of the polynomial f(x). A polynomial in x is
said to be in its standard form when the terms are (v) Biquadratic polynomial :
written either in increasing order or decreasing order A polynomial of degree four is called a biquadratic or
of the indices of x in various terms. quartic polynomial. The general form of biquadratic

EXAMPLES : polynomial is ax4 + bx3 + cx2 + dx + e where a  0


e.g. x4 + x3 + x2 + x + 1 , x4 + x2 + 1 etc.
3 2
(i) 2x + 4x + x + 1 is a polynomial of degree 3.
NOTE :
(ii) x7 + x5 + x2 + 1 is a polynomial of degree 7. A polynomial of degree five or more than five does not
have any particular name. Such a polynomial is usually
(iii) x3/2 + x2 + 1 is not a polynomial as the indices of x called a polynomial of degree five or six or ..... etc.
are not all non negative integer
(b) Polynomial Based on Terms :

(iv) x2 + 2 x + 1 is a polynomial of degree 2. There are three types of polynomial based on number
of terms.

(v) x–2 + x + 1 is not a polynomial as –2 is not non (i) Monomial : A polynomial is said to be a monomial if
negative. it has only one term.
For example, x, 9x2, – 5x2 are all monomials

(ii) Binomial : A polynomial is said to be a binomial if it


contains two terms.
(a) Polynomial Based on Degree :
For example 2x2 + 3x, 3 x + 5x4, – 8x3 + 3 etc are all
There are five types of polynomials based on degree.
binomials.
(i) Constant polynomial :
(iii) Trinomial : A polynomial is said to be a trinomial if
A polynomial of degree zero is called a zero degree it contains three terms.

polynomial or constant polynomial. 5


For example 3x3 – 8x + , 7 x10 + 8x4 – 3x2 etc are all
2
e.g. f(x) = 4 = 4x0 trinomials.

(ii) Linear polynomial : REMARKS :

(i) A polynomial having four or more than four terms


A polynomial of degree one is called a linear polynomial.
does not have any particular name. They are simply
The general form of a linear polynomial is ax + b, where
called polynomials.
a and b are any real numbers and a  0
(ii) A polynomial whose coefficients are all zero is called
e.g. 4x + 5, 2x + 3, 5x + 3 etc. a zero polynomial, degree of a zero polynomial
is not defined.

PAGE # 38
Ex.2 Show that x + 1 and 2x – 3 are factors of
2x3 – 9x2 + x + 12.
(a) Value of a Polynomial :
Sol. To prove that (x + 1) and (2x – 3) are factors of
The value of a polynomial f(x) at x =  is obtained by
2x3 – 9x2 + x + 12 it is sufficient to show that p(–1) and
substituting x =  in the given polynomial and is
3
denoted by f(). p   both are equal to zero.
2
Consider the polynomial f(x) = x3 – 6x2 + 11x – 6, p (– 1) = 2 (– 1)3 – 9 (– 1)2 + (– 1) + 12
If we replace x by – 2 everywhere in f(x), we get = – 2 – 9 – 1 + 12
3 2
f(– 2) = (– 2) – 6(– 2) + 11(– 2) – 6 = – 12 + 12 = 0.
f(– 2) = – 8 – 24 – 22 – 6 3 2
3 3 3 3
f(– 2) = – 60  0. and p   = 2   – 9       12
2
  2
  2
  2
So, we can say that value of f(x) at x = – 2 is – 60.
27 81 3
= –   12
(b) Zero or root of a Polynomial : 4 4 2
The real number  is a root or zero of a polynomial
27 – 81  6  48
f(x), if f( = 0. =
4

Consider the polynomial f(x) = 2x3 + x2 – 7x – 6, – 81  81


=
4
If we replace x by 2 everywhere in f(x), we get
= 0.
 f(2) = 2(2)3 + (2)2 – 7(2) – 6
Hence, (x + 1) and (2x – 3) are the factors
= 16 + 4 – 14 – 6 = 0
2x3 – 9x2 + x + 12.
Hence, x = 2 is a root of f(x).
Ex. 3 Find the values of a and b so that the polynomials
x3 – ax2 – 13x + b has (x – 1) and (x + 3) as factors.
Let ‘p(x)’ be any polynomial of degree greater than or Sol. Let f(x) = x3 – ax2 – 13x + b
equal to one and a be any real number and If p(x) is Because (x – 1) and (x + 3) are the factors of f(x),
divided by (x – a), then the remainder is equal to p(a).  f(1) = 0 and f(– 3) = 0

Ex.1 Find the remainder, when f(x) = x3 – 6x2 + 2x – 4 is f(1) = 0

divided by g(x) = 1 – 2x.  (1)3 – a(1)2 – 13(1) + b = 0


Sol. f(x) = x3 – 6x2 + 2x – 4  1 – a – 13 + b = 0
Let, 1 – 2x = 0  – a + b = 12 .... (i)
 2x = 1 f(–3) = 0
 (– 3)3 – a(– 3)2 – 13(– 3) + b = 0
 x= 1
2  – 27 – 9a + 39 + b = 0
 1  – 9a + b = –12 ...(ii)
Remainder = f  
2
Subtracting equation (ii) from equation (i)
3 2
 1  1  1  1 (– a + b) – (– 9a + b) = 12 + 12
f   =   – 6   2  – 4
2 2 2 2
 – a + 9a = 24
1 3 1 – 12  8 – 32 35  8a = 24
= –  1– 4 = – .
8 2 8 8
a = 3.

FACTOR THEOREM Put a = 3 in equation (i)


– 3 + b = 12
Let p(x) be a polynomial of degree greater than or equal b = 15.
to 1 and ‘a’ be a real number such that p(a) = 0, then Hence, a = 3 and b = 15.
(x – a) is a factor of p(x). Conversely, if (x – a) is a factor
of p(x), then p(a) = 0.

PAGE # 39
DIVISION ALGORITHM FOR POLYNOMIALS

If p(x) and g(x) are any two polynomials with g(x)  0,


then we can find polynomials r(x) and q(x) such that
p(x) = g(x) × q(x) + r(x)
i.e. Dividend = (Divisor x Quotient) + Remainder
where r(x)=0 or degree of r(x) < degree of g(x).
(i) If r(x) = 0, g(x) is a factor of p(x)
(ii) If deg(p(x)) > deg(g(x)),
then deg(q(x)) = deg(p(x)) –deg(g(x))
(iii) If deg(p(x)) = deg(g(x)),  2x4 – 2x3 – 7x2 + 3x + 6 = (2x2 – 3) (x2 – x – 2)
then deg(q(x)) = 0 and deg(r(x)) < deg(g(x)) = (2x2 – 3) (x – 2) (x + 1)
Ex.4 What must be subtracted from x3 – 6x2 – 15x + 80 so  3   3 
= 2  x  2   x  2  (x – 2) (x + 1)

that the result is exactly divisible by x2 + x – 12.
   
Sol. Let ax + b be subtracted from p(x) = x3 – 6x2 – 15x + 80
so that it is exactly divisible by x2 + x – 12. 3 3
So, the zeros are – , , 2, – 1.
2 2
 s(x) = x3 – 6x2 – 15x + 80 – (ax + b)
= x3 – 6x2 – (15 + a)x + (80 – b)
Dividend = Divisor × quotient + remainder
ALGEBRAIC IDENTITIES
But remainder will be zero.
 Dividend = Divisor × quotient Some important identities are
 s(x) = (x2 + x – 12) × quotient
(i) (a + b)2 = a2 + 2ab + b2
 s(x) = x3 – 6x2 – (15 + a)x + (80 – b)
x 7 (ii) (a – b)2 = a2 – 2ab + b2
x 2  x  12 x 3  6 x 2  x15  a   80  b
3
 x 2  (iii) a2 – b2 = (a + b) (a – b)
x  12 x __________ _____
 7 x 2  12x  15  ax  80  b (iv) (a + b + c)2 = a2 + b2 + c2 + 2ab + 2 bc + 2ca
or
 7 x 2  x  3  a  80  b (v) a3 + b3 = (a + b) (a2 – ab + b2)
 7x 2 
 7x  84
_______
x ( 4  a )  ( 4  b )  0 (vi) a3 – b3 = (a – b) (a2 + ab + b2)

Hence, x (4 – a) + (– 4 – b) = 0.x + 0 (vii) (a + b)3 = a3 + b3 + 3ab (a + b)


 4 – a = 0 & (– 4 – b) = 0
(viii) (a – b)3 = a3 – b3 – 3ab (a – b)
 a = 4 and b = – 4
Hence, if in p(x) we subtract 4x – 4 then it is exactly (ix) a3 + b3 + c3 – 3abc = (a + b + c) (a2 + b2 + c2 – ab – bc – ac)
divisible by x2 + x – 12.
Special case : if a + b + c = 0 then a3 + b3 + c3 = 3abc.
Ex.5 Find all the zeros of the polynomial
Value Form :
f(x) = 2x4 – 2x3 – 7x2 + 3x + 6, if two of its zeros are
(i) a2 + b2 = (a + b)2 – 2ab, if a + b and ab are given.
3 3
– and . (ii) a2 + b2 = (a – b)2 + 2ab, if a – b and ab are given.
2 2
3 3
Sol. Since, – and are zeros of f(x). (iii) a + b = a  b 2  4ab , if a – b and ab are given.
2 2

(iv) a – b = a  b2  4ab , if a + b and ab are given.


 3   3   2 3  2x 2  3
Therefore,  x  2   x  2  =  x   =
 2
  2 2 1  1 1
   (v) a2 + =  a   – 2, if a + is given.
a2  a  a
or 2x2 – 3 is a factor of f(x). 1
2
1  1
(vi) a2 + =  a   + 2, if a – is given.
a2  a a
(vii) a3 + b3 = (a + b)3 – 3ab (a + b), if (a + b) and ab are
given.
(viii) a3 – b3 = (a – b)3 + 3ab (a – b), if (a – b) and ab are
given.

PAGE # 40
1 1
3
1 1
(ix) a3  1 =  a   – 3  a   , if a + is given.
  = x8 – .
a 3
 a  a a x8
3 (ii) (2x + y)(2x – y)(4x2 +y2)
 11  1 1
(x) a 3  =  a   + 3  a   , if a – is given. = [(2x)2 – (y)2](4x2 + y2)
a 3 a  a  a
  = (4x2 – y2)(4x2 + y2)
(xi) a4 – b4 = (a2 + b2) (a2 – b2) = [(a + b)2 – 2ab](a + b) (a – b). = (4x2)2 – (y2)2 = 16x4 – y4.

Ex.8 Find the value of x – y when x + y = 9 & xy = 14:


Ex.6 Expand :
2
Sol. x + y = 9
 1  2 On squaring both sides
(i)  2x   (ii) 3 x 2  5 y
 
 3x  x2 + y2 + 2xy = 81
2 Putting value of xy = 14
1 1 
(iii)  2 x  3y  2x  3y  (iv)  a  b  1 x2 + y2 + 28 = 81
 4 2 
2 x2 + y2 = 81 – 28 = 53 ...(i)
 1 
Sol. (i)  2x   (x – y)2 = x2 + y2 – 2xy
 3x 
Putting xy = 14 and (i)
(x – y)2 = 53 – 2 (14) = 53 – 28
 1  1 4 1
= (2x)2 – 2(2x)   + = 4x2 – + . (x – y)2 = 25
 3x  3x 2 3 9x 2
x–y=± 25 = ±5
(ii) (3x2 + 5y)2
1  1
= (3x2)2 + 2(3x2)(5y) + (5y)2 Ex.9 If x2 + = 23, find the values of x   ,
2 x
x 
= 9x4 + 30x2y + 25y2
 1  1 
 x   and  x 4  4 .
(iii) ( 2 x – 3y)( 2 x + 3y)  x  x 
1
= ( 2 x)2 – (3y)2 Sol. 2
x + = 23 …(i)
x2
= 2x2 – 9y2
1
 x2 + + 2 = 25 [Adding 2 on both sides of (i)]
1 1 
2 x2
(iv)  a  b  1 2
4 2   1 1
2
2 2  (x ) +   + 2  x  = 25
1   1  1   1  x x
=  a  +   b  + (1)2 + 2  a    b 
 4   2  4   2  2
 1
 1  1    x   = (5)2
x
+ 2   b  (1) + 2(1)  a  
 2  4  1
1 1 2 ab a  x+ =  5
= a2 + b +1– –b+ . x
16 4 4 2
2
Ex. 7 Simplify :  1 1
 x   = x2 + –2
 x x2
 1  1  2 1  4 1 
(i)  x   x   x  2  x  4 
 x  x  x  x  2
 1
 x   = 23 – 2 = 21
(ii) 2x  y 2x  y  4 x 2  y 2    x

 1  1  1  1  1
  x    x2  2   x4  4 

Sol.  x    x   =  21 .
 x  x  x  x   x

2 1   2 1  1  1 
2
= x  2
  x  2   x4  4   2 1   4
x  2  =x  4  + 2
 x   x  x   x   x 
 2
2 2  1    4 1  1   1 
2
= (x )   2    x  4 
 4
  x  4  =  x2  2  – 2
  x    x  
 x   x 
4 1  4 1   4 1 
= x  4
 x  4    x  4  = (23)2 – 2 = 529 – 2
 x  x   x 
2
 1   4 1 
= (x4)2 –    x  4  = 527.
 x4  
 x 

PAGE # 41
1 1
 a 2 – 5ab a2 – b2  Ex.14 If x – = 5, find the value of x3 – 3 .
Ex.10 Find the value of   . x x
2 2
 a – 6ab  5b a 2  ab  1
Sol. We have, x – =5 ...(i)
x
a 2 – 5ab a2 – b2  1
3
Sol. ×
a 2 – 6ab  5b 2 a 2  ab   x   = (5)3 [Cubing both sides of (i)]
 x

1 1  1
a(a – 5b) (a – b)(a  b)  x3 – – 3x    x   = 125
= × =1 x3 x  x
(a – b )(a – 5b ) a(a  b)

1  1
7.98  7.98 – 2.02  2.02  x3 – – 3  x   = 125
Ex.11 Find the value of x3  x
5.96
1  1
7.98  7.98 – 2.02  2.02  x3 – – 3 × 5 = 125 [Substituting  x   = 5]
x3  x
Sol.
5.96
1
 x3 – – 15 = 125
(7.98  2.02)(7.98 – 2.02) 10  5.96 x3
= = = 10.
5.96 5.96
1
 x3 – = (125 + 15) = 140.
Ex.12 Simplify : x3
3 3
 2  2 Ex.15 Find the products of the following expression :
(i) (3x + 4)3 – (3x – 4)3 (ii)  x     x  
 x  x (i) (4x + 3y) (16x2 – 12xy + 9y2)
Sol. (i)(3x + 4)3 – (3x – 4)3 (ii) (5x – 2y) (25x2 +10xy + 4y2)
= [(3x)3 + (4)3 + 3 (3x) (4) (3x + 4)] – [(3x)3 – (4)3 – 3 (3x) Sol. (i) (4x + 3y) (16x2 – 12 xy + 9y2)
(4) (3x – 4)] = (4x + 3y) [(4x)2 – (4x) × (3y) + (3y)2]
= [27x3 + 64 + 36x (3x + 4)] – [27x3 – 64 – 36x (3x – 4)] = (a + b) (a2 – ab + b2) [Where a = 4x, b = 3y ]
= [27x3 + 64 + 108x2 + 144x] – [27x3 – 64 – 108x2 + 144x] =a +b3 3

= 27x3 + 64 + 108x2 + 144x – 27x3 + 64 + 108x2 – 144x = (4x)3 + (3y)3 = 64x3 + 27y3.
= 128 + 216x2.
3 3 (ii) (5x – 2y) (25x2 + 10xy + 4y2)
 2  2 = (5x – 2y) [(5x)2 + (5x) × (2y) + (2y)2]
(ii)  x   +  x  
 x  x = (a – b) (a2 + ab + b2) [Where a = 5x, b = 2y]
3 3
2 2  2 2 = a3 – b3
= x3 +   + 3 (x)    x   + x 3 –  
x x  x x = (5x)3 – (2y)3
= 125x3 – 8y3.
2  2
– 3(x)    x   Ex. 16 If a + b + c = 9 and ab + bc + ac = 26, find the value of
x  x
a3 + b3 + c3 – 3abc.
8 12 8 12
= x3 + + 6x + + x3 – 3 – 6x + Sol. We have a + b + c = 9 ...(i)
x3 x x x
 (a + b + c)2 = 81 [On squaring both sides of (i)]
24  a2 + b2 + c2 + 2(ab + bc + ac) = 81
= 2x3 + .
x  a2 + b2 + c2 + 2 × 26 = 81 [ ab + bc + ac = 26]
Ex.13 Evaluate :  a2 + b2 + c2 = (81 – 52)
(i) (1005)3 (ii) (997)3  a2 + b2 + c2 = 29.
Sol. (i) (1005)3 = (1000 + 5)3 Now, we have
= (1000)3 + (5)3 + 3 (1000) (5) (1000 + 5) a3 + b3 + c3 – 3abc
= 1000000000 + 125 + 15000 (1000 + 5) = (a + b + c) (a2 + b2 + c2 – ab – bc – ac)
= 1000000000 + 125 + 15000000 + 75000 = (a + b + c) [(a2 + b2 + c2) – (ab + bc + ac)]
= 1015075125. = 9 × [(29 – 26)]
= (9 × 3) = 27.
(ii) (997)3 = (1000 – 3)3
2 2 3 3 3
= (1000)3 – (3)3 – 3 × 1000 × 3 × (1000 – 3)
Ex.17 Simplify : a  b   b2  c 2  c 2  a2
     .
= 1000000000 – 27 – 9000 × (1000 – 3) a  b3  b  c 3  c  a 3
= 1000000000 – 27 – 9000000 + 27000
= 991026973. Sol. Here, a 2  b 2  b 2  c 2  c 2  a 2  0
     
2 3 3 3
 a  b2   b 2
 c2   c 2
 a2 

PAGE # 42
= 3 a 2  b 2 b2  c 2 c 2  a2
    (f) Factorization of a algebraic expression as the sum
or difference of two cubes :
Also, a  b   b  c   c  a   0
Ex.24 Factorize : 16a3b–250b4.
 a  b3  b  c 3  c  a 3 = 3a  b b  c c  a
Sol. 16a3b–250b4=2b(8a3–125b3) = 2b{(2a)3–(5b)3}
 Given expression
= 2b{(2a–5b)(4a2+25b2+10ab)}
2 2 2 2 2 2
=

3 a b b c c a    (g) Factorization of a algebraic expression of the form
3a  b  b  c  c  a 
a3 + b3 + c3 – 3abc :
3a  b  a  b  b  c b  c c  a c  a 
= Ex.25 Factorize :
3a  b  b  c  c  a 
(i) 2 2a 3  8b 3  27c 3  18 2abc
= a  bb  c c  a .
(ii) (x – y)3 + (y – z)3 + (z –x)3
Ex.18 Find the value of (28)3 – (78)3 + (50)3.
Sol. (i) 2 2a 3  8b 3  27c 3  18 2abc
Sol. Let a = 28, b = – 78, c = 50
Then, a + b + c = 28 – 78 + 50 = 0 3 3 3
=  2a  (2b)  (3c )  3( 2a )(2b)( 3c )
 a3 + b3 + c3 = 3abc. =  2a  2b  3c 
So, (28)3 + (–78)3 + (50)3 = 3 × 28 × (–78) × 50
 2a 2  2b 2   3c 2 
   2a2b  2b 3c    3c  2a
= – 327600. 

2
=  2a  2b  3c  2a  4b 2  9c 2  2 2ab  6bc  3 2ac 
(ii) (x – y)3 + (y – z)3 + (z –x)3
To express a given polynomial as the product of
Let a =x – y, b = y–z and c = z–x
polynomials, each of degree less than that of the given
So, (x – y)3 + (y – z)3 + (z –x)3 = a3 + b3 + c3
polynomial such that no such a factor has a factor of
Now, a + b + c = x – y + y – z + z – x = 0
lower degree, is called factorization.
So, a3 + b3 + c3 = 3abc
(a) Factorization by taking out the common factor :
 (x – y)3 + (y – z)3 + (z –x)3 = 3(x – y) (y – z)(z –x)
When each term of an expression has a common factor,
divide each term by this factor and take out as a multiple. H.C.F. AND L.C.M. OF POLYNOMIALS

Ex.19 Factorize : 6x3 + 8x2 – 10x  A polynomial D(x) is a divisor of the polynomial P(x) if it
3 2
Sol. 6x + 8x –10x = 2x (3x + 4x – 5) 2 is a factor of P(x). Where Q(x) is another polynomial
(b) Factorization by grouping : such that P(x) = D(x) × Q(x)

Ex.20 Factorize : ax + by + ay + bx  HCF/GCD (Greatest Common Divisor) : A polynomial


h(x) is called the HCF or GCD of two or more given
Sol. ax + by + ay + bx = ax + ay + bx + by = a (x + y) + b(x+y)
polynomials, if h(x) is a polynomial of highest degree
= (x + y) (a + b)
dividing each of one of the given polynomials.
(c) Factorization by making a perfect square :
 L.C.M. (Least Common Multiple ) : A polynomial P(x) is
Ex.21 Factorize : 4x2 + 12x + 9
called the LCM of two or more given polynomials, if it
Sol. 4x2 + 12x + 9 = (2x)2 + 2 (2x) (3) + 32 is a polynomial of smallest degree which is divided by
= (2x + 3)2 each one of the given polynomials. For any two
(d) Factorization the difference of two squares : polynomials P(x) and Q(x). We have :
Ex.22 Factorize : 4x2 – 25. P(x) × Q(x) = [HCF of P(x) and Q(x)]
Sol. 4x2 – 25 = (2x)2 – (5)2 = (2x – 5) (2x + 5) × [LCM of P(x) and Q (x)]
Ex.26 If p(x) = (x + 2)(x2 – 4x–21), Q(x) = (x– 7) (2x2 + x – 6)
(e) Factorization of a quadratic polynomial by splitting
find the HCF and LCM of P(x) and Q(x).
the middle term : Sol. P(x) = (x + 2) (x2 – 4x – 21)
= (x + 2) (x2 – 7x + 3x – 21)
Ex.23 Factorize : x2 + 4 2 x + 6
= (x + 2) (x – 7) (x + 3)
Sol. x2 + 4 2 x + 6 Q(x) = (x – 7) (2x2 + x – 6)
= x2 + 3 2 x + 2 x+6 = (x – 7)(2x2 + 4x – 3x – 6)
= (x – 7) [2x (x + 2) – 3 (x + 2)]
= x(x + 3 2 ) + 2 (x + 3 2 ) = (x – 7) (2x – 3) (x + 2)
HCF = (x + 2)(x– 7)
= (x + 3 2 )(x + 2 ) LCM = (x + 3)(x – 7) (2x – 3) (x + 2).

PAGE # 43
Ex.27 If HCF & LCM of P(x) and Q(x) are (x + 2) and (x + 3) cubic polynomial, cannot have more than three linear
(x2 + 9x + 14) respectively if P(x) = x2 + 5x + 6, find Q(x).
factors.
Sol. P(x) = (x2 + 5x + 6) = (x + 2) (x + 3)
LCM = (x + 3) (x2 + 9x + 14) = (x + 3)(x + 7)(x + 2)  f(x) = k(x – ) (x – ) (x – )
We know that HCF  LCM = P(x)  Q(x)  ax3 + bx2 + cx + d = k(x – ) (x – ) (x – )
( x  2)( x  3)( x  7)( x  2)  ax3 + bx2 + cx + d = k{x3 – ( +  + ) x2 + ( +  + ) x – }
Q(x) =
( x  2)( x  3)  ax3 +bx2 +cx+d =k x3 –k (+ + )x2 +k ( ++)x– k
= (x + 7) (x+2) = x2 + 9x + 14. Comparing the coefficients of x3, x2, x and constant
terms on both sides, we get
a = k, b = – k ( +  + ), c = k ( +  + ) and d = – k ()

b
 ++ =–
Let  and  be the zeros of a quadratic polynomial a
f(x) = ax2 + bx + c. By factor theorem (x – ) and (x – )
are the factors of f(x). c
  +  + =
 f(x) = k (x – ) (x – ) are the factors of f(x) a
 ax2 + bx + c = k{x2 – ( + ) x + } d
 ax2 + bx + c = kx2 – k ( + ) x + k And,= –
a
Comparing the coefficients of x2, x and constant terms
on both sides, we get b Coefficien t of x 2
 Sum of the zeros = – =–
a = k, b = – k ( + ) and c = k a Coefficien t of x 3
b c  Sum of the products of the zeros taken two at a
 +=– and  =
a a
c Coefficient of x
Coefficient of x time = =
  +  = –
a Coefficient of x 3
Coefficient of x 2
d Constant term
 Product of the zeros = – =–
Constant term a Coefficien t of x 3
and  =
Coefficien t of x 2  REMARKS :
Hence,
Coefficient of x Cubic polynomial having ,  and  as its zeros is
b
Sum of the zeros = – =–
a Coefficient of x 2 given by

Constant term f(x) = k (x – ) (x – ) (x – )


c
Product of the zeros = = . f(x) = k{x3 – ( +  + ) x2 + ( +  + ) x – },
a Coefficient of x 2
 REMARKS : where k is any non-zero real number.
If  and  are the zeros of a quadratic polynomial f(x). Ex.29 , ,  are zeros of cubic polynomial x3 – 12x2 + 44x + c.
Then, the polynomial f(x) is given by If , ,  are in A.P., find the value of c.
f(x) = k{x2 – ( + ) x + } Sol. f (x) = x3 – 12x2 + 44x + c.
or, f(x) = k{x2 – (Sum of the zeros) x + Product of the , , are in A.P.
zeros} Let  = a – d, = a, = a + d
Sum of zeros = + + 
Ex.28 Find a quadratic polynomial whose zeros are
=a–d+a+a+d
3  5 and 3  5 . = 3a
Sol. Given : = 3 + 5 and = 3 – 5 So, 3a = – (– 12)
Sum of zeros = +  a = 4.
=3+ 5 +3– 5=6 Sum of product of zeros taken two at a time
Products of zeros =   + + = 44
 (a – d) a + (a) (a + d) + (a + d) (a – d) = 44
= (3 + 5 ) (3 – 5 )
 a2 – ad + a2 + ad + a2 – d2 = 44
=9–5=4
So, quadratic polynomial isk {x2 – 6x + 4}, where k is  3a2 – d2 = 44
any constant.  3 (4)2 – d2 = 44 d2 = 4
 d=±2
product of zeros = = – c
 (a – d) a (a + d) = – c
 (4 – 2) (4) (4 + 2) = – c
Let , ,  be the zeros of a cubic polynomial  (2) (4) (6) = – c
 c = – 48.
f(x) = ax3 + bx2 + cx + d, a  0. Then, by factor theorem,
x – , x –  and x –  are factors of f(x) . Also, f(x) being a

PAGE # 44
In algebraic or in set theoretic language the graph
of a polynomial f(x) is the collection (or set) of all
points (x, y), where y = f(x). In geometrical or in
graphical language the graph of a polynomial f(x) is
a smooth free hand curve passing through points
(x 1, y1), (x 2, y2), (x 3, y3), ... etc, where y1, y2, y3, ... are
the values of the polynomial f(x) at x 1, x 2, x 3, ...
respectively.
In order to draw the graph of a polynomial f(x), follow (b) Graph of a Quadratic Polynomial :
the following algorithm. Let a, b, c be real numbers and a  0. Then
ALGORITHM : f(x) = ax2 + bx + c is known as a quadratic polynomial
Step (i) Find the values y1, y2, ...., yn, .... of polynomial in x. Graph of the quadratic polynomial i.e. the curve
f(x) on different points x 1, x 2, ...., x n, .... and prepare whose equation is y = ax2 + bx + c, a  0. Graph of a
a table that gives values of y or f(x) for various values quadratic polynomial is always a parabola.
of x. Let y = ax2 + bx + c, where a  0.
x: x1 x2 ... xn xn + 1 ...
...  4ay = 4a2x2 + 4abx + 4ac
y = f(x) : y1 = f(x1) y2 = f(x2) yn = f(xn) yn + 1 = f(xn + 1) ...
 4ay = 4a2x2 + 4abx + b2 – b2 + 4ac
Step (ii) Plot that points (x 1, y1), (x 2, y2), (x 3, y3), ...
 4ay = (2ax + b)2 – (b2 – 4ac)
(x n , yn), ... on rectangular co-ordinate system.
 4ay + (b2 – 4ac) = (2ax + b)2
In plotting these points use different scales on the
 4ay + (b2 – 4ac) = 4a2(x + b/2a)2
X and Y axes.
2
 b 2 – 4ac  2 b 
Step (iii) Draw a free hand smooth curve passing  4ay  4a 
  4a  x  
2a 
 
through points plotted in step 2 to get the graph of 2
 D   b 
the polynomial f(x).  y    a x   . ....(i)
 4a   2a 
(a) Graph of a Linear Polynomial : where, D  b2 – 4ac is the discriminant of the quadratic
Consider a linear polynomial f(x)= ax + b, a  0. equation.
Graph of y = ax + b is a straight line. That is why
 REMARKS :
f(x) = ax + b is called a linear polynomial. Since two
 b D 
points determine a straight line, so only two points Shifting the origin at  – ,–  , we have
 2a 4a 
need to plotted to draw the line y = ax + b. The line  b  (–D)
X = x – –  and Y = y – .
represented by y = ax + b crosses the X-axis at  2a  4a
 b  Substituting these values in (i), we obtain Y = aX2 ...(ii)
exactly one point, namely   , 0  . which is the standard equation of parabola.
 a 
Clearly, this is the equation of a parabola having its
Ex.30 Draw the graph of y = x – 4.
Sol. y = x – 4  b D 
vertex at  – ,– .
 2a 4a
y=x–4
The parabola opens upwards or downwards
x 0 4 5 according as a > 0 or a < 0.
y –4 0 1

Let  be a real root of ax 2 + bx + c = 0. Then,


a2 + b + c = 0. Point (, 0) lies on y = ax2 + bx + c.
Thus, every real root of ax2 + bx + c = 0 represents a
point of intersection of the parabola with the X-axis.

PAGE # 45
Conversely, if the parabola y = ax2 + bx + c intersects Let us plot the points (–4,16), (–3, 7), (–2, 0),
the X-axis at a point (, 0), then (, 0) satisfies the (–1, –5), (0, –8), (1, –9), (2, –8), (3, –5), (4, 0), (5, 7)
equation y = ax2 + bx + c
and (6, 16) on a graphs paper and draw a smooth
 a2 + b + c = 0 [ is a real root of ax2 + bx + c = 0] free hand curve passing through these points. The
Thus, the intersection of the parabola y = ax2 + bx + c curve thus obtained represents the graphs of the
with X-axis gives all the real roots of ax2 + bx + c = 0. polynomial f(x) = x2 – 2x – 8. This is called a parabola.
Following conclusions may be drawn : The lowest point P, called a minimum point, is the
(i) If D > 0, the parabola will intersect the x-axis in two vertex of the parabola. Vertical line passing through
distinct points and vice-versa. P is called the axis of the parabola. Parabola is
–b– D symmetric about the axis. So, it is also called the
The parabola meets x-axis at   and
2a line of symmetry.
–b D
 .
2a

(ii) If D = 0, the parabola will just touch the x-axis at


one point and vice-versa.

 Observations :
From the graphs of the polynomial f(x) = x2 – 2x – 8,
following observations can be drawn :
(iii) If D < 0, the parabola will not intersect x-axis at all (i) The coefficient of x2 in f(x) = x2 – 2x – 8 is 1
and vice-versa. (a positive real number) and so the parabola opens
upwards.
(ii) D = b 2 – 4ac = 4 + 32 = 36 > 0. So, the parabola
cuts X-axis at two distinct points.
(iii) On comparing the polynomial x2 – 2x – 8 with
ax2 + bx + c, we get a = 1, b = – 2 and c = – 8. The
vertex of the parabola has coordinates (1, – 9)
 b D
i.e.  ,  , where D  b 2 – 4ac.
 2a 4a 
(iv) The polynomial f(x) = x2 – 2x – 8 = (x – 4) (x + 2)
is factorizable into two distinct linear factors
(x – 4) and (x + 2). So, the parabola cuts X-axis at
 REMARKS : two d is t inc t poi nts (4 , 0 ) a nd (– 2, 0) . The
  x  R, y > 0 only if a > 0 & D  b²  4ac < 0 x-coordinates of these points are zeros of f(x).
  x  R, y < 0 only if a < 0 & D  b²  4ac < 0
Ex.31 Draw the graph of the polynomial f(x) = x2 – 2x – 8.
Sol. Let y = x2 – 2x – 8.
The following table gives the values of y or f(x) for
various values of x.
x –4 –3 –2 –1 0 1 2 3 4 5 6
2
y = x – 2x – 8 16 7 0 –5 –8 –9 –8 –5 0 7 16

PAGE # 46
ALGEBRAIC IDENTITIES AND FACTORIZATION
10. If a2 –b2 =21 and a2 + b2 = 29, which of the following
NOTE : More than one correct option may be possible. could be the value of ab ?
I. –10 II. 5 2 III. 10
FACTOR THEOREM AND REMAINDER THEOREM
(A) I only (B) II only
1. The remainder obtained when t6 +3t2 + 10 is divided by (C) III only (D) I and III only
t3 + 1 is :
11. On simplifying (a + b)3 + (a – b)3 + 6a(a2 – b2) we get :
(A) t2 – 11 (B) 3t2 + 11
3 (A) 8a2 (B) 8a2b
(C) t – 1 (D) 1 – t3 3
(C) 8a b (D) 8a3
2. If (x + a) is a factor of x2 + px + q and x2 + mx + n then the
a 3  b 3  c 3 – 3abc
value of a is : 12. Find the value of , when
ab  bc  ca – a 2 – b 2 – c 2
m–p n–q
(A) (B) a = – 5, b = – 6, c = 10.
n–q m–p (A) 1 (B) –1
nq mp (C) 2 (D) –2
(C) (D)
mp nq
13. If (x + y + z) = 1, xy + yz + zx = –1, xyz = –1, then value of
x3 + y3 + z3 is :
3. If x2 – 4 is a factor of 2x3 + ax2 + bx + 12, where a and b
(A) –1 (B) 1
are constant. Then the values of a and b are :
(C) 2 (D) –2
(A) – 3, 8 (B) 3, 8
(C) –3, – 8 (D) 3, – 8 1 1 1
14. If x 3  y 3  z 3 = 0 then which one of the following
4. If x51 + 51 is divided by (x + 1) the remainder is : expression is correct :
(A) 0 (B) 1 (A) x3 + y3 + z3 = 0
(C) 49 (D) 50 1 1 1
(B) x + y + z = 3 x 3 y 3 z 3
5. The polynomials ax3 + 3x2 – 3 and 2x3 – 5x + a when
divided by (x – 4) leaves remainders R1 & R2 respectively (C) x + y + z = 3xyz
then value of ‘a’ if 2R1 – R2 = 0. (D) x3 + y3 + z3 = 3xyz
18 18 1 1
(A) – (B) 15. If a4 + = 119, then find the value of a3 – .
127 127 a 4
a3
17 17 (A) 11 (B) 36
(C) (D) –
127 127 (C) 33 (D) 12
2
(a – b ) (b – c ) 2 (c – a ) 2
6. A quadratic polynomial is exactly divisible by (x + 1) & 16. Evaluate :   .
(b – c )(c – a ) (a – b)(c – a) (a – b )(b – c )
(x + 2) and leaves the remainder 4 after division by (A) 0 (B) 1
(x + 3) then that polynomial is :
(C) 2 (D) 3
(A) x2 + 6x + 4 (B) 2x2 + 6x + 4
(C) 2x2 + 6x – 4 (D) x2 + 6x – 4 0.76  0.76  0.76  0.24  0.24  0.24
17. The value of is :
0.76  0.76  0.76  0.24  0.24  0.24
7. The polynomial p(x) = 2x4 – x3 – 7x2 + ax + b is divisible
(A) 0.52 (B) 1
by x2 – 2x – 3 for certain values of a and b. The value of
(C) 0.01 (D) 0.1
(a + b), is :
(A) – 34 (B) – 30 18. If x + y = 3 and xy = 2, then the value of x3 – y3 is equal to
(C) – 26 (D) – 18 (A) 6 (B) 7
(C) 8 (D) 0
8. What is the remainder when the polynomial 1 1
19. If x = , then the value of x + 1
is :
p(x) = x200 – 2x199 + x50 – 2x49 + x2 + x + 1 is divided by 2
1
1
(x – 1) (x – 2) ? 1
x
(A) 1 (B) 7
5 4
(C) 2x + 1 (D) 6x – 5 (A) (B)
4 5
3
9. A cubic polynomial p(x) is such that p(1)=1, p(2)=2, (C) (D) None of these
4
p(3)= 3 and p(4) = 5, then the value of p(6) is :
(A) 16 (B) 13 a b
20. If (a2 + b2)3 = (a3 + b3)2 then + =
(C) 10 (D) 7 b a
2 3
(A) (B)
3 2
5 6
(C) (D)
6 5

PAGE # 47
21. m5 + m4 + m3 + m2 + m + 1 = (m3 + 1) × _______ 31. If ,  are the zero’s of polynomial
(A) m5 + m4 + m2 + m (B) m2 + m3 f(x) = x2 – p(x + 1) – c then ( + 1)( + 1) is equal :
3 3
(C) m + m + m + 1 (D) m2 + m + 1 (A) c – 1 (B) 1 – c
(C) c (D) 1 + c
4
4
22. If x = 2  2 , then x + x 4 is : 32. If the sum of the zeros of the quadratic polynomial
(A) 2(3 – 2 ) (B) 6 2 – 2 f(t) = kt2 + 2t + 3k is equal to their product, then the value
(C) 6 – 2 (D) 12 of k is :
23. If 4x – 5z = 16 and xz = 12, 64x3 – 125z3 = 3 3
(A) – (B)
(A) 14512 (B) 15676 2 2
(C) 25833 (D) 15616 2
(C) – (D) none of these
3
x 3  y 3 33. If ,  are the zeroes of x2 – 6x + k = 0. What is the
24.  3 1
x y  ( xy )  2  y 3 x 1 value of k if 3  + 2 = 20.
(A) x + y (B) y – x (A) –16 (B) 8
1 1 1 1 (C) – 2 (D) – 8
(C) – (D) +
x y x y
34. If ,  are zeros of quadratic polynomial kx2 + 4x + 4,
2 find the value of k such that ( + )2 – 2 = 24.
 a – b  4 ab 5
25. If = , then the value of a : b is : 2
a–b 3 (A) – 1 (B)
3
(A) 1 : 16 (B) 1 : 4
(C) both(A)and(B) (D) None of these
(C) 4 : 1 (D) 16 : 1
35. If   and  are the three zeroes of the polynomial
26. If x = 0.50, then the value of the expression p(x) = x3 – 64x – 14, what is the value of 33 + 3 ?
 2 x 3 
(1  x  x )  (A) 36 (B) 40
1  x  is :


(C) 42 (D) 64
(A) 4 (B) 2
(C) 1.50 (D) 1
36. The graph of y = ax2 + bx + c is given in figure then
27. If p = 22/3 + 21/3, then : identify the signs of a, b and c : y
(A) p3 – 6p + 6 = 0 (B) p3 – 3p – 6 = 0 (A) a < 0, b > 0, c < 0
x' x
(C) p3 – 6p – 6 = 0 (D) p3 – 3p + 6 = 0 (B) a < 0, b < 0, c > 0
A
(C) a < 0, b > 0, c > 0
RELATIONSHIP BETWEEN ZEROS AND (D) a < 0, b < 0, c < 0
COEFFICIENTS AND GRAPH OF
y'
P O LY N O MI A L
37. Minimum value for the polynomial 4x2 – 6x + 1 is :
28. If one zero of 2x2 – 3x + k is reciprocal to the other,
3 5
then the value of k is : (A) – (B) –
4 4
2
(A) 2 (B) 
3 5
3 (C) – (D) – 
(C)  (D) – 3 16
2
29. If  and  are the zeroes of x2 – 4x +1, then

1 1
   is : COMPETITIVE EXAM PREVIOUS YEAR
 
QUESTION
(A) 3 (B) 5
1. Let f (x) = x2 + x – 6. For what values of “t” does
(C) – 5 (D) – 3
f (t – 5) = 0 ? [NSTSE-2009]
30. The equation x2 + Bx + C = 0 has 5 as the sum of its (A) – 3 and 2 (B) – 2 and 3
roots, and 15 as the sum of the square of its roots. The (C) 5 (D) 2 and 7
value of ‘C’ is :
(A) 5 (B) 7.5 2. If , ,  are the roots of the equation
(C) 10 (D) 12.5 (x – 2)(x2 + 6x – 11) = 0, therefore, ( +  + ) equals :
[IJSO-2010]
(A) – 4 (B) 23/6
(C) 13 (D) – 8

PAGE # 48
10. Let a, b, c be positive real numbers such that abc  1,
x10  1
3. If x2 – 5x + 1 = 0, then has the value (ab)2 = (bc)4 = (ca)x = abc. Then x equals
x5
[IJSO-2013]
[IJSO-2010]
(A) 2524 (B) 2525 (A) 1 (B) 2
(C) 2424 (D) 2010 (C) 3 (D) 4/5

4. The graph of the equation y = 2x2 + 4x + 3 has its lowest 11. Sucharitha purchases x pencils at Rs x each, y pens at
point at : [IJSO-2010] Rs y each and z notebooks at Rs z each. She
(A) (– 1, 9) (B) (1, 9) purchases altogether 50 items and pays Rs. 1000.
(C) (– 1, 1) (D) (0, 3) The cost of y pencils, z pens and x notebooks is
[IJSO-2013]
5. Given that a (a+b) = 36 and b (a + b) = 64, where a and (A) Rs 600 (B) Rs.750
b are positive, (a – b) equals [IJSO-2011] (C) Rs.500 (D) Rs.350
(A) 2.8 (B) 3.2
(C) –2.8 (D) –2.5 12. The number of real values of a for which the cubic
equation x2 – 3ax2 + 3ax – a = 0 has all real roots one of
6. When the polynomial (6x4 + 8x3 + 17x2 + 21x + 7) is which is a itself, is [IJSO-2013]
divided by (3x2 + 4x + 1), the remainder is (ax – b). (A) 0 (B) 1
Therefore, [IJSO-2011] (C) 2 (D) 3
(A) a = 1, b = 2 (B) a = 1, b = –2
(C) a = 2, b = 1 (D) a = –1, b = –2 13. If xy2 = a3 , yz2 = b3 and zx2 = c3 then z3 equals
[IJSO-2013]
7. lf a + b + c = 1, a2 + b2 + c2 = 21 and abc = 8 then find the
value of (1– a)(1– b) (1– c) [IJSO-2012]
bc 4 b 4c
(A) –10 (B) –18 (A) (B)
(C)–24 (D)–30 a2 a2

8. Find x2 + y2 + z2 if x2 + xy + xz = 135, y2 + yz + yx = 351 and b2c 4 ab 4


z2 + zx + zy = 243 [IJSO-2012] (C) (D)
b2 c2
(A) 225 (B) 250
(C) 275 (D) 300
14. If 3x + 3y – 1, 4x2 + y – 5, 4x + 2y are the sides of an
9. If x3 = a + 1 and x + (b/x) = a ; then x equals equilateral triangle, its area is closest to the interger
[IJSO-2013] [IJSO-2013]
(A) 84 (B) 85
ab  1 ab  1
(A) (B) (C) 86 (D) 87
a2 – b a2 – b

ab  a  1 ab – a – 1
(C) (D) 15. The number 38(310 + 65) + 23(212 + 67) is
a2 – b a2 – b [IJSO-2016]
(A) A perfect square and a perfect cube
(B) Neither a perfect square nor a perfect cube
(C) A perfect cube but not a perfect square
(D) A perfect square but not a perfect cube

PAGE # 49
LINEAR EQUATION IN TW O VARIABLES

LINEAR EQUATION IN TWO VARIABLES Now substitute value of y in equation (ii)


 7x – 3 (3) = 5
An equation of the form Ax + By + C = 0 is called a linear  7x – 3 (3) = 5
equation.  7x = 14
Where A is called coefficient of x, B is called coefficient 14
of y and C is the constant term (free from x & y)  x= =2
7
A, B, C R [  belongs to, R  Real No.] So, solution is x = 2 and y = 3.
But A and B can not be simultaneously zero.
(b) Elimination by Equating the Coefficients :
If A  0, B = 0, equation will be of the form Ax + C = 0.
Ex.2 Solve : 9x – 4y = 8 & 13x + 7y = 101.
If A = 0, B  0, equation will be of the form By + C = 0. Sol. 9x – 4y = 8 ..... (i)
If A  0 , B  0, C = 0 equation will be of the for A x + By = 0. 13x + 7y = 101 .... (ii)
[line passing through origin] Multiply equation (i) by 7 and equation (ii) by 4, we get
If A  0 , B  0 , C  0 equation will be of the form A x + By + C = 0.

It is called a linear equation, because the two


unknowns (x & y) occur only in the first power, and the 460
 x = x = 4.
product of two unknown quantities does not occur. 115
Substitute x = 4 in equation (i)
Since it involves two variables, therefore a single
9 (4) – 4y = 8
equation will have infinite set of solution
 36 – 8 = 4y
i.e. indeterminate solution. So we require a pair of
equation i.e. simultaneous equations.  28 = 4y
28
Standard form of linear equation :  y = = 7.
4
Standard form refers to all positive coefficients.
So, solution is x = 4 and y = 7.
a1x + b1y + c1 = 0 ...(i)
(c) Elimination by Cross Multiplication :
a2x + b2y + c2 = 0 ...(ii)
a1x + b1y + c1 = 0
For solving such equations, we have three methods :
a2x + b2y + c2= 0
(i) Elimination by Substitution.
b1 c1 a1 b1
(ii) Elimination by equating the coefficients.
b2 c2 a2 b2
(iii) Elimination by Cross multiplication.

(a) Elimination By Substitution : (Write the coefficient in this manner)

Ex.1 Solve : x + 4y = 14 & 7x – 3y = 5. x y 1


= =
Sol. x + 4y = 14 b1c 2 – b 2c 1 a 2c 1 – a1c 2 a1b 2 – a 2b1
x = 14 – 4y ....(i)
x 1
7x – 3y = 5 ....(ii)  =
b1c 2 – b 2c 1 a1b 2 – a 2b1
Substitute the value of x in equation (ii)
 7 (14 – 4y) – 3y = 5 b1c 2 – b 2c 1
 98 – 28y – 3y = 5  x= a b –a b
1 2 2 1
 98 – 31y = 5
y 1
 93 = 31y Also a c – a c = a b – a b
2 1 1 2 1 2 2 1
93 a 2c 1 – a1c 2
 y= y = 3.
31  y= a b –a b
1 2 2 1

PAGE # 50
Ex.3 Solve : 3x + 2y + 25 = 0 & x + y + 15 = 0. 1 5 3 5 3 61
Sol. Here, a1 = 3, b1 = 2, c1 = 25 Ex.5 If 2(x + 2y ) + 3(3x – 2y ) = – 2 , 4(x + 2y ) – 5(3 x – 2y ) = 60 ,
a2 = 1, b2 = 1, c2 = 15
where x + 2y  0 and 3x – 2y  0. Then what will be the
2 25 3 2
 values of x and y ?
1 15 1 1
x y 1 1 1
Sol. Let = U, =V
= = x  2y 3 x  2y
2  15 – 25  1 25  1 – 15  3 3  1 – 2  1
x y 1 U 5V 3
 30 – 25 = 25 – 45 = 3 – 2  +
3
=–
2 2
x y 1  3U + 10V = – 9 ... (i)
 5 = – 20 = ......(i)
1 5U 3V 61
 –
5
=
60
4
x y 1
 5 = 1, – 20 =  75U – 36V = 61 ... (ii)
1
Equation (i) is multiplied by 25
 x = 5, y = – 20 75U + 250 V = – 225
So, solution is x = 5 and y = – 20. 75U – 36 V = 61
– + –
, Ex.4 Solve the following system of equations : ––––––––––––––––––––––––

6 7 1 1 Subtracting, 286V = – 286


 3, 
xy xy 2( x  y ) 3( x  y ) 1
V = – 1, U =
where x  y  0 and x  y  0 . 3
1 1 1
6 7  = , =–1
Sol. = +3 ...(i) x  2y 3 3 x  2y
xy xy x + 2y = 3, – 3x + 2y = 1

Put x + y = a, x – y = b x + 2y = 3
Now, the given equation reduces to –3x + 2y = 1
+ – –
6 7 ––––––––––––––––
= 3 ...(ii)
a b Subtracting, 4x = 2
1 1 1 5
Another equation is  ...(iii) So, x = ,y= .
2( x  y ) 3( x  y ) 2 4
1 1
= Ex.6 Solve : 3 2 x – 5 3 y + 5=0
2a 3b
3b 2 3 x+ 7 2 y–2 5 = 0.
or, a = ...(iv)
2
Put (iv) in (ii) Sol. 3 2 x – 5 3 y + 5 =0 ....(i)
6 7  3b
×2= 2 3 x+7 2 y–2 5 =0 ....(ii)
3b b
4 = 7 + 3b
Multiplying (i) by 2 3 and (iii) by 3 2
b=–1 ...(v)
Put (v) in (iv) 6 6 x – 10 × 3y = – 2 15
3
a= 
2 6 6 x+ 21 × 2y = 6 10
From our assumptions, – – –
–––––––––––––––––––––––––––––––––––––––––––––––
3
a=x+y= 
2
Subtracting, – 72y = – ( 6 10 + 2 15 )
b= x– y= –1
Adding the two equations given above, or, 72y = 6 10 + 2 15

5 5 6 10  2 15
2x =   x =  y=
2 4 72
10 15 – 7 10
5 1  x= .
 y=  +1 y=  . 72
4 4

PAGE # 51
CONDITIONS FOR SOLVABILITY OF Ex.9 Find the values of  and  for which the following
SYSTEM OF EQUATIONS system of linear equations has infinite number of
solution :2x + 3y = 7 & 2x + ( + ) y = 28.
Let the two equations be : Sol. For infinite solution :
a1x + b1y + c1 = 0 ...(i) a1 b1 c 1
 
a2x + b2y + c2 = 0 ...(ii) a2 b2 c 2
(a) Unique Solution : 2 3 7
  =
2    28
If the Denominator a1b2 – a2 b1  0, then the given
1 3 1
system of equations have unique solution (i.e. only   =  =4
one solution)   4
[From the first & third term]
a b
 a1b2 – a2 b1  0  1  1 . 3 1
a2 b2    4 +  = 12   = 8.
4 4
For two lines :lines are said to be consistent (i.e. they
meet at one point) when the given system of equation WORD PROBLEMS
has unique solution.
Ex.10 A two digit number is such that product of its digits is
(b) No Solution :
18. When 63 is subtracted from the number, the digits
If the Denominator a1b2 – a2 b1 = 0, then the given system interchange their place. Find the number.
of equations have no solution. Sol. xy = 18 ...(i)
Let the given number be 10x + y
a1 b1 c1
i.e. =  As per the question,
a2 b2 c2
(10x + y) – 63 = 10y + x
For two lines : Lines are said to be inconsistent  10x – 10y – x + y = 63
(i.e. they does not meet) when the given the system of  9x – 9y = 63
equation has no solution.  x–y=7 ...(ii)
18
(c) Many Solutions (Infinite Solutions) : Put x = y in (ii)
a1 b1 c 1 18
If = = then system of equations has many
a2 b2 c 2  y –y=7
solutions and lines are said to be consistent.  18 – y2 = 7y
For two lines : Two lines are coincident when they  y2 + 7y – 18 = 0
have many solutions.  y2 + 9y – 2y – 18 = 0
 y(y + 9) – 2 (y + 9) = 0
Ex.7 Find the value of ‘M’ for which the given system of  (y + 9) (y – 2) = 0
equation has only one solution (i.e. unique solution).  y = 2, – 9
Mx – 2y = 9 & 4x – y = 7. y = – 9 is not valid
Sol. a1 = M, b1 = –2 , c1 = 9  y = 2, x = 9.
a2 = 4, b2 = – 1, c2 = 7 So, the number = 10x + y = 10 (9) + 2 = 92.
a1 b1 Ex.11 The sum of two numbers is 2490. If 6.5 % of one
Condition for unique solution is  number is equal to 8.5 % of the other, find the numbers.
a2 b2
M 2 8 Sol. Let, the numbers be x & y.
   M  M 8. Then, x + y = 2490 ...(i)
4 1 1
M can have all real values except 8. x  6 .5 8.5
 = ×y
100 100
Ex.8 What is the value of a, for which the system of linear 8. 5
equations ax + 3y = a – 3 ; 12x + ay = a has no solution.  x= y
6.5
a1 b1 c1 17
Sol. Condition for no solution is =   x= y ...(ii)
a2 b2 c2 13
a 3 a3 Put (ii) in (i)
 
12 a a 17
y + y = 2490
2
a = 36 or, a =  6 13
a = + 6 is not possible because it gives  30y = 13 × 2490
13  2490
a1 b1 c1  y= = 1079
= = 30
a2 b2 c2
 x = 1411
or, a = – 6 is the answer. So, the numbers are x = 1411 & y = 1079.

PAGE # 52
Ex.12 A and B each has a certain number of mangoes.
A says to B, ‘if you give 30 of your mangoes I will have
twice as many as left with you.’ B replies ‘if you give me
10, I will have thrice as left with you.’ Find how many
mangoes does each have.
Sol. Say, A has x mangoes & B has y mangoes initially.
As per the statement of A to B,
x + 30 = 2 (y – 30)
or, x – 2y = – 90 ...(i)
and as per statement of B to A,
3 (x – 10) = y + 10
or, 3x – y = 40 ...(ii)
Now, we have
x – 2y = – 90
3x – y = 40
3x – 6y = – 270
3y – y = 40
– + –
––––––––––––––––––––––––––
– 5y = – 310

 y = 62
 x = 34.
So, A have 34 mangoes and B have 62 mangoes.

PAGE # 53
8. Find the values of a and b for which the following
system of linear equations has infinite number of
solutions : 2x – 3y = 7 ; (a + b) x – (a + b – 3) y = 4a + b
NOTE : More than one correct option may be possible.
(A) a = – 5, b = – 1 (B) a = 1, b = 5
METHOD OF SOLVING (C) a = – 1, b = – 5 (D) a = 5, b = 1

1. Solve the following system of equation for x and y : WORD PROBLEM


2 3 8 6 9. Sum of two numbers is 35 and their difference is 13.
  2;   2.
x y x y Find the numbers.
1 1 1 1 (A) 23, 12 (B) 24, 11
(A) , (B) , (C) 23, 10 (D) 22, 9
2 3 3 2
(C) 2, 3 (D) 3, 2 10. The digit in the ten’s place of a two-digit number is
2. The solution of the following pairs of simultaneous three times that in the one’s place. If the digits are
equation is : reversed the new number will be 36 less than the
original number. Find the number.
2x – 1 y  2 x3 x–y (A) 64 (B) 52
 4 & –  3.
3 4 2 3 (C) 62 (D) 42
(A) x = – 5, y = – 4 (B) x = 5, y = 2 11. Denominator of a rational number is 4 less than its
(C) x = – 5, y = 2 (D) x = 5, y = 4 numerator. If 11 is added to numerator and 1 is
7 8 subtracted from denominator, the new number
6 9
3. Solve for x & y : x – = 15 & x = . 7
3 2 y 3 2y becomes . Find the rational number..
3
(A) – 3, – 2 (B) – 2, – 3 27
23
(A) (B)
1 1 1 1 27 23
(C) , (D) ,
2 3 3 2 13 17
(C) (D)
4. The pair of equations 3 =81, 81 x+y x–y
= 3 has : 17 13
(A) No solution 12. The sum of the present ages of father and his son is
1 1 60 years. 6 years ago, father’s age was five times the
(B) The solution x = 2 , y = 2
2 2 age of the son. After six years son’s age will be :
(C) The solution x = 2, y = 2 (A) 20 years (B) 14 years
(C) 12 years (D) 18 years
1 7
(D) The solution x = 2 ,y= 1 13. Four years ago father's age was 6 times that of his
8 8
son. Twelve years from now, father's age will be twice
CONDITION OF SOLVABILITY
that of the son. What is the ratio of father and son's
5. Given 3x – 4y = 7 and x + cy = 13, for what value of “c” present ages ?
will the two equation not have a solution ? (A) 6 : 1 (B) 7 : 1
3 4 (C) 8 : 2 (D) 7 : 2
(A) (B) 14. The sum of a two digit number and the number formed
4 3
by interchanging its digits is 110.If 10 is subtracted
–4 from the first number, the new number is 4 more than
(C) – 4 (D) 5 times the sum of the digits in the first number. Find
3
6. For what value of K, the system of linear equations the first number.
2x + ky = 1 ; 3x – 5y = 7 has a unique solutions : (A) 46 (B) 64
(C) 55 (D) 65
10 10
(A) K  – (B) K  15. After covering a distance of 30 km with a uniform speed
3 3
10 there is some defect in a train engine and therefore, its
(C) K  – 35 (D) K = – speed is reduced to 4/5 of its original speed.
3
Consequently, the train reaches its destination late by
7. If the system of the given equation has no solution
45 minutes. Had it happened after covering
then find the value of k.
18 kilometres more, the train would have reached
2x + ky = 7 & 2kx + 3ky = 20.
9 minutes earlier. Find the speed of the train and the
(A) 1 (B) 6
distance of journey.
(C) 3 (D) None of these
(A) Speed = 40 km/hr, Distance = 160 km
(B) Speed = 20 km/hr, Distance = 100 km
(C) Speed = 30 km/hr, Distance = 120 km
(D) Speed = 35 km/hr, Distance = 140 km

PAGE # 54
4. Let "b" be a positive number such that the system
16. A railway half ticket costs half the full fare and the
reservation charge is the same on half ticket as on full
ax  3 y  1
ticket. One reserved first class ticket from Mumbai to 
Ahmedabad costs Rs. 216 and one full and one half 5 x  ay  b
reserved first class tickets cost Rs. 327. What is the has an infinite number of solutions. By rounding to the
basic first class full fare and what is the reservation
nearest hundredth, the value of "b" equals :
charge ?
[NSTSE 2009]
(A) Full Fare = Rs. 210, Reservation charge = Rs. 6
(B) Full Fare = Rs. 144, Reservation charge = Rs. 72 (A) 0.60 (B) 1.29
(C) Full Fare = Rs. 180, Reservation charge = Rs. 36 (C) 1.67 (D) 3.87
(D) Full Fare = Rs. 200, Reservation charge = Rs. 16 5. The solution set of the system of equation
4 3
17. A wizared having powers of mystic in candations and + 5y = 7, + 4 y = 5 is : [NSTSE-2010]
magical medicines seeing a cock, fight going on, spoke x x
privately to both the owners of cocks. To one he said; if 1   –1 
your bird wins, than you give me your stake-money, but (A)  3 ,–1 (B)  ,1
   3 
if you do not win, I shall give you two third of that’. Going
to the other, he promised in the same way to give three
fourths. From both of them his gain would be only  –1  1 
(C)  3 ,–1 (D)  3 ,1 
12 gold coins. Find the stake of money each of the    
cock-owners have.
6. If 2a = b, the pair of equations ax + by = 2a2 – 3b2,
(A) 27 gold coins & 30 gold coins respectively.
(B) 12 gold coins & 20 gold coins respectively. x + 2y = 2a – 6b possess : [NSTSE 2010]
(C) 33 gold coins & 30 gold coins respectively. (A) no solution (B) only one solution
(D) 42 gold coins & 40 gold coins respectively. (C) only two solutions
(D) an infinite number of solutions

7. In a rectangle ABCD the lengths of sides AB, BC, CD


COMPETITIVE EXAM PREVIOUS YEAR
and DA are (5x + 2y + 2) cm, (x + y + 4) cm,
QUESTION (2x + 5y – 7) cm and (3x + 2y – 11) cm respectively.
Which of the following statements is /are true ?
1. The equations 2x – 3y + 5 = 0 and 6y – 4x = 10, when
[IJSO-2009]
solved simultaneously, have : [IJSO-2008]
(A) One of the sides of the rectangle is 15 cm long.
(A) only one solution
(B) Each diagonal of the rectangle is 39 cm long.
(B) no solution
(C) Perimeter of the rectangle is 102 cm.
(C) only two solutions
(D) Area of the rectangle is 560 cm2
(D) infinite number of solutions
2. Given 3x – 4y = 7 and x + cy = 13, for what value of “c” 8. If 4a – 18b + 13c = 0 and 3a + 3b – 4c = 0, then the ratio
will the two equation not have a solution ? a : b : c will be : [IJSO-2010]
[NSTSE-2009] (A) 3 : 6 : 5 (B) 5 : 3 : 6
3 4 (C) 3 : 5 : 6 (D) 5 : 6 : 3
(A) (B)
4 3
–4
(C) – 4 (D) 9. A pen costs Rs 13 and a note book costs Rs. 35 Let m
3
be the maximum number of items that can be bought
Y for Rs 1000and n be the minimum number of items
that can be bought for the same amount. Then m + n is
4 [IJSO-2013]
3 (A) 76 (B) 88
3. 2 [NSTSE-2009]
(C) 96 (D) 98
1
0 1 2 3 4 X

The above diagram shows the graph of :


(A) y = x – 3 (B) y = 2x + 3

–4 x
(C) y = –x – 3 (D) y = +4
3

PAGE # 55
LINEAR INEQUATIONS

LINEAR INEQUATIONS IN ONE VARIABLE

Let a be a non-zero real number and x be a variable.


A statement involving variable (s) and the sign of inequality
Then inequations of the form ax + b < 0, ax + b  0,
viz, >,<  or  is called an inequation.
ax + b > 0 and ax + b  0 are known as linear inequations
An inequation may contain one or more variables. in one variable x.
Also, it may be linear or quadratic or cubic etc. For example : 9x – 15 > 0, 5x – 4  0, 3x + 2 < 0 and
(i) 3x – 2 < 0 (ii) 2x2 + 3x + 4 > 0 (iii) 2x + 5y  4 2x – 3  0 are linear inequations in one variable.

(a) Properties of inequalities (a) Solving linear inequations in one variable

(i) If ‘a’ is a positive no. i.e. a > 0 then for x < y Rule 1: Same number may be added to (or subtracted
from) both side of an inequation without changing the
x y sign of inequality.
  & ax < ay..
a a
Rule 2 : Both sides of an inequation can be multiplied
(ii) If ‘a’ is –ve i.e. a < 0 then for x < y (or divided) by the same positive real number without
changing the sign of inequality. However, the sign of
x y
  & ax > ay.. inequality is reversed when both sides of an inequation
a a
are multiplied or divided by a negative number.
(iii) If ‘a’ is a +ve no. i.e. a > 0 then for x > y
Rule 3 : Any term of an inequation may be taken to the
other side with its sign changed without affecting the
x y
  & ax > ay.. sign of inequality.
a a
Ex.1 Solve the inequality ax > a.
(iv) If ‘a’ is a –ve no. i.e. a < 0 then for x > y
Sol. This inequality has the parameter a that needs to be
x y investigated further.
  & ax < ay.. If a > 0, then x > 1
a a
If a < 0, then x < 0

Ex.2 Solve : 24 x < 100 when


(i) x is a natural number (ii) x is an integer.
(i) Closed interval : Let a and b be two given real
Sol. We are given 24 x < 100
numbers such that a < b. Then the set of all real
24 x 100
numbers x such that a  x  b is called closed interval  <
24 24
and is denoted by [a, b] may be graphed as :
25
 x<
6
a b 
(i) When x is a natural number, the following values of
x make the statement true. 1, 2, 3, 4.
(ii) Open interval : If a and b are two real numbers
(ii) When x is an integer, the solutions of the given
such that a < b, then the set of all real numbers equations are ....,– 3, – 2, – 1, 0, 1, 2, 3, 4.
x satisfying a < x < b is called an open interval and is The solution set of the equation is :
denoted by (a, b) or ]a, b[ and may be graphed as : { ...,– 3,– 2,– 1, 0,1, 2, 3, 4}.

Ex.3 Solve & graph the solution set of 3x + 6  9


a b
and – 5x > –15, x  R.
(iii) Semi-closed or semi-open interval : If a and b are Sol. 3x + 6  9 and –5x > –15
 3x  3  – x > – 3
two real numbers such that a < b, then the sets
 x1  x<3
(a, b] = {x  R : a < x  b} and [a, b) = {x  R : a  x < b} are
Combining the solution
known as semi-open or semi-closed intervals. (a, b]
and [a, b) are also denoted by ]a, b] and [a, b[
–3 –2 –1 0 1 2 3
respectively.
So, the solution is x  [1,3).

PAGE # 56
Ex.4 Solve & graph the solution set of – 2 < 2x – 4 2x  4
and – 2x + 5  13, x  R. Ex.7 Solve the following inequations : > 5.
x –1
Sol. 2x  4
Sol. We have, 5
–4 –3 –2 –1 0 1 2 3 x –1
– 2 < 2x – 4 2x  4
 –50
 2x – 4 > –2 and –2x + 5  13 x –1
2x > 2 and –2x  13 – 5 2x  4 – 5( x – 1)
  0
x –1
 x>1 and –2x  8
2x  4 – 5x  5
 x>1 and –x  4  0
x –1
 x>1 and x  –4
–3 x  9
 x > 1 and x  – 4 or x  (–  , – 4]  (1,  ).  0 [Multiplying both sides by – 1]
x –1
x   . 3x – 9
 0
x –1
Ex.5 Solve the following equation :
3( x – 3)
2(2x + 3) – 10 < 6 (x – 2)  0 [Dividing both sides by 3]
x –1
Sol. We have,
x–3
 2(2x + 3) – 10  6 (x – 2)  0
x –1
 4x + 6 – 10  6x – 12  1<x3
 4x – 4  6x – 12  x  (1, 3]
 4x – 6x  – 12 + 4 + – +
[Transposing – 4 to RHS and 6x to LHS] 1 3
 – 2x  – 8 Hence, the solution set of the given inequations is
(1, 3].
–2 x –8 Ex.8 Solve 5x - 3 < 3x + 1 when
 
–2 –2 (i) x is an integer, (ii) x is a real number.
Sol. We have, 5x – 3 < 3x + 1
 x4
 5x – 3 + 3 < 3x + 1 + 3
 x  [4, )
 5x < 3x + 4
Hence, the solution set of the given inequation is [4, )
 5x – 3 x < 3x + 4 – 3x
which can be graphed on real line as shown in Figure.
 2x < 4
 x<2
0 4
(i) When x is an integer, the solutions of the given
5 x – 2 7x – 3 x inequality are {..........., – 4, –3, –2, –1, 0,1}
Ex.6 Solve the following inequations : –  .
3 5 4 (ii) When x is a real number, the solutions of the
5 x – 2 7x – 3 x inequality are given by x < 2, i.e., all real number x
Sol. –  which are less than 2. Therefore, the solution set of
3 5 4
the inequality is x  (-  , 2).
5(5 x – 2) – 3(7 x – 3) x
>
15 4
25 x – 10 – 21x  9 x The function f(x) defined by
>
15 4 x, when x  0
f(x) = x  
– x, when x  0
4x – 1 x 
> is called the modulus function. It is also called the
15 4
absolute value function.
4 (4x – 1) > 15 x y
[Multiplying both sides by 60 i.e. LCM of 15 and 4]
16x – 4 > 15x f(x
)= x
16x – 15x > 4 )=
–x f(x
[Transposing 15 x to LHS and – 4 to RHS]
x>4 x' x
o
x  (4, )
Hence, the solution set of the given inequation is
(4, ). This can be graphed on the real number line as
shown in figure.
y'
0 The distance between two real numbers x and y is
4
defined as x – y .

PAGE # 57
Ex.9 Solve : x – 4 = 7

x – 4, where x – 4  0  x  4 NOTE : More than one correct option may be possible.


Sol. x – 4  
– ( x – 4 ), where x – 4  0  x  4
 x–4=7 and – (x – 4) = 7 PROBLEMS BASED ON INEQUALITY
x = 11 and –x+4=7 1. If – 5x + 1  0, then :
 –x=3
1
(A) x   (B) x   1
 x  –3 5 5
 x=–3  Ans. 
x  11 1 1
(C) x  (D) x 
5 5
Ex.10 Evaluate : 3  – 2 – 3 – 3 – – 7 .
2. If x, y are any two positive real numbers and x > y, then
Sol. 3  –2–3 –3– –7 which of the following statement is not true ?
(A) – x < y (B) – x < – y
= 3 + 5 – 3 – {–(–7)} 1 1 1 1
= 3 + {–(–5)} – 3 – 7 (C) x  y (D) x  y
= 3 + 5 – 10 = 8 – 10 = – 2.
3. Solution of the system of inequalities is :
(a) Inequations involving absolute value 3x – 4 < 8x + 6 ; 2x – 1 > 5x – 4 & 11x – 9  15x + 3
Result 1. If a is a positive real number, then (A) – 2 < x < 1 (B) – 4 < x < 1
(i) | x | < a  – a < x < a i.e. x  (– a, a) (C) – 1 < x < 1 (D) – 2 < x < 2
2x  1
4. Solution of inequality < 1 is :
–a a x 1
(A) – 1 < x < 3 (B) – 1 < x < 2
(ii) | x |  a  – a  x  a i.e. x  [–a, a]
(C) – 2 < x < 2 (D) – 3 < x < 2

–a a 1
5. Solve : < 1.
x
Result 2. If a is a positive real number, then
(A) (– , 0) U (1, + ) (B) (– , 0) U [1, + )
(i) | x | > a  x < – a or x > a (C) (1, + ) (D) (– , 0)
x 1
–a a 6. Solve :  .
x2 x
(ii) | x |  a  x  – a or x  a
(A) –3 < x  2 (B) – 2 < x  2
(C) – 2 < x  3 (D) (–2, –1]  (0, 2]
–a a
7. Solution of inequality (x – 1)2 (x + 1)3 (x – 4)  0 is :
Ex.11 Find x from 1  x  2 and represent it on number line. (A) – 1 < x < 3 (B) – 2  x 4
(C) – 1  x  4 (D) – 1 < x < 2
Sol. 1  x  x  1  x > 1 or x < –1
( x  1)2 ( x  1)3
 x  ( ,  1)  (1,  ) ...(i) 8. Solve :  0.
x 4 ( x  2)
also x  2  x < 2 or x > – 2 (A) – 1  x < 1 (B) – 1  x < 3
 x lies between – 2 & 2 (C) – 2  x < 2 (D) – 1  x < 2 – {0}
 x  (–2, 2) ...(ii) 5  2x x
9. Solve :  5.
Combining the two results, we get 3 6
(A) [8, ) (B) 7,  
1  x  2  {– 2 < x < –1}  {1 < x < 2}
i.e. x  (– 2, –1)  (1, 2) (C)  8,   (D) [8, )

3x  4 x  1
–2 –1 0 1 2 10. Solve :   1.
2 4
(A) x  1 (B) x > 1
Ex.12 Find x satisfying x – 5  3 .
(C) x  1 (D) x < 2
Sol. as x – a  r  a – r  x  a  r i.e. x  [a – r, a  r ] 11. The marks obtained by a student of Class X in first and
second unit test are 35 and 21, respectively. Find the
 x – 5  3  5 – 3  x  5  3 i.e. 2  x  8 i.e. x  [2, 8]
minimum marks he should get in the annual
examination to have an average of at least 30 marks.
2 8 (A) x  34 (B) x  34
(C) x > 34 (D) x < 34

PAGE # 58
12. Find pair of consecutive odd natural numbers, both of
which are larger than 13, such that their sum is less
than 40.
(A) (15,17) (B) (13,17) ac
1. If a,b,c are positive, is [IJSO - 2011]
(C) (19,21) (D) (21,17) bc
13. The solution set of the inequality a
x (A) always smaller than
0< < 1, x  R is b
x 1 a
(A) Set of all positive real numbers (B) always greater than
b
(B) set of all real numbers except – 1 a
(C) set of all non-negative real numbers (C) greater than only if a > b.
b
(D) set of all numbers satisfying 0  x  1, x  R
a
(D) greater than only if a < b.
PROBLEMS BASED ON MODULUS b
F UN C T IO N

14. Solve : | 2x – 3|  5. 2. Let a, b be two positive real numbers such that


(A) – 2  x  5 (B) – 2  x  4 1  1  1
(C) – 1  x  4 (D) – 1  x  5 a<b< and let x =  a  a  –  b  b  .Then
a    
15. Solve : | 1 – x | > 3. [IJSO - 2013]
(A) x > 4 or x > – 2 (B) x > 4
(A) x is always greater than zero
(C) x < – 2 (D) x > 4 or x < – 2
(B) x is always less than zero
16. The solution the equation | 3x – 10 |  5 is : (C) x = 0

5 5 (D) No such definite conclusion can be drawn about x


(A) x  5 or x  (B) x  5 or x 
3 3
x
3. The solution set of the inequality 0 < < 1, x  R is
5 5 x 1
(C) x  – 5 or x  – (D) x  – 5 or x  – [IJSO - 2014]
3 3
(A) Set of all positive real numbers
(B) set of all real numbers except – 1
17. Which of the following is a solution of 3|x+1| – 5 = – 2? (C) set of all non-negative real numbers
(A) 2 (B) 1 (D) set of all numbers satisfying 0  x  1, x  R
4
(C) (D) None of these 4. The statement “a is not less than 4” is correctly
3
represented by [IJSO - 2016]
18. Find x satisfying x – 5  3 (A) a < 4 (B) a > 4
(A) 2  x  8 (B) 3  x  10 (C) a  4 (D) a  4
(C) 2  x  8 (D) None of these



PAGE # 59
QUADRATIC EQUATIONS

QUADRATIC EQUATIONS NATURE OF ROOTS

If P(x) is quadratic expression in variable x, then Consider the quadratic equation, a x2 + b x + c = 0


P(x) = 0 is known as a quadratic equation. having  ,  as its roots and b 2  4ac is called
(a) General form of a Quadratic Equation : discriminant of roots of quadratic equation. It is
denoted by D or .
Th e g e n e r a l f o r m o f a q u a d r a t i c e q u a t i o n i s
ax2 + bx + c = 0, where a, b, c are real numbers and a  0. Roots of the given quadratic equation may be
(i) Real and unequal (ii) Real and equal
Since a  0, quadratic equations, in general, are of
(iii) Imaginary and unequal.
the following types : Let the roots of the quadratic equation ax2 + bx + c = 0
(i) b = 0, c  0, i.e., of the type ax2 + c = 0. (where a  0, b, c  R) be  and  then

(ii) b  0, c = 0, i.e., of the type ax2 + bx = 0.  b  b 2  4ac


= ... (i)
(iii) b = 0, c = 0, i.e., of the type ax2 = 0. 2a

(iv) b  0, c  0, i.e., of the type ax2 + bx + c = 0.  b  b 2  4ac


And = ... (ii)
2a
ROOTS OF A QUADRATIC EQUATION The nature of roots depends upon the value of
expression ‘b2 – 4ac’ with in the square root sign. This
The value of x which satisfies the given quadratic is known as discriminant of the given quadratic
equation is known as its root. The roots of the given equation.
equation are known as its solution.
Consider the Following Cases :
General form of a quadratic equation is :
ax2 + bx + c = 0 Case-1 When b2 – 4ac > 0, (D > 0)
 4a2x2 + 4abx + 4ac = 0 [Multiplying by 4a] In this case roots of the given equation are real and
 4a2x2 + 4abx = – 4ac distinct and are as follows
[By adding b2 both sides]
2 2 2 2
 4a x + 4abx + b = b – 4ac  b  b 2  4ac  b  b 2  4ac
= and  =
 (2ax + b)2 = b2 – 4ac 2a 2a
Taking square root of both the sides (i) When a( 0), b, c  Q and b2 – 4ac is a perfect
2ax + b = ± square
 b 2  4ac
In this case both the roots are rational and distinct.
 b  b 2  4ac (ii) When a( 0), b, c  Q and b2 – 4ac is not a perfect
 x =
2a square
Hence, roots of the quadratic equation ax2 + bx + c = 0 In this case both the roots are irrational and distinct.
 b  b 2  4ac  b  b 2  4ac [See remarks also]
are and .
2a 2a Case-2 When b2 – 4ac = 0, (D = 0)
 NOTE : b
In this case both the roots are real and equal to – .
A quadratic equation is satisfied by exactly two 2a
Case-3 When b2 – 4ac < 0, (D < 0)
values of 'x' which may be real or imaginary. The
equation, a x2 + b x + c = 0 is : In this case b2 – 4ac < 0, then 4ac – b2 > 0
 A quadratic equation if a  0 : Two roots.  b   ( 4ac  b 2 )
 =
 A linear equation if a = 0, b  0 : One root. 2a

 A contradiction if a = b = 0, c  0 : No root.  b   ( 4ac  b 2 )


and  =
2a
 An identity if a = b = c = 0 : Infinite roots.
 b  i 4ac  b 2  b  i 4ac  b 2
 A quadratic equation cannot have more than two roots. or = and  =
2a 2a
 It follows from the above statement that if a quadratic
[  1 = i]
equation is satisfied by more than two values of x, then
it is satisfied by every value of x and so it is an identity. i.e. in this case both the roots are imaginary and distinct.

PAGE # 60
 REMARKS : Ex.3 Solve the following quadratic equation by factorization
method : x2 – 2ax + a2 – b2 = 0.
 If a, b, c  Q and b2 – 4ac is positive (D > 0) but not a
Sol. Here, Factors of constant term (a2 – b2) are (a – b) and
perfect square, then the roots are irrational and they
(a + b).
always occur in conjugate pairs like 2 + 3 and Also, Coefficient of the middle term = – 2a
= – [(a – b) + (a + b)]
2 – 3 . However, if a, b, c are irrational numbers and
 x2 – 2ax + a2 – b2 = 0
b2 – 4ac is positive but not a perfect square, then the
 x2 – {(a – b) + (a + b)} x + (a – b) (a + b) = 0
roots may not occur in conjugate pairs.
 x2 – (a – b) x – (a + b) x + (a – b) (a + b) = 0
 If b2 – 4ac is negative (D < 0), then the roots are complex  x {x – (a – b)} – (a + b) {x – (a – b)} = 0
conjugate of each other. In fact, complex roots of an  {x – (a – b)} {x – (a + b)} = 0
equation with real coefficients always occur in conjugate  x – (a – b) = 0 or, x – (a + b) = 0
pairs like 2 + 3i and 2 – 3i. However, this may not be true  x = a – b or x = a + b
in case of equations with complex coefficients. For
example, x2 – 2ix – 1 = 0 has both roots equal to i. Ex.4 Solve the quadratic equation by the method of
factorisation : x2 + 8x + 7 = 0.
 If a and c are of the same sign and b has a sign opposite
Sol. We have
to that of a as well as c, then both the roots are positive,
x2 + 8x + 7 = 0
the sum as well as the product of roots is positive
 x2 + 7x + x + 7 = 0
(D  0).
 x (x + 7) + 1 (x + 7) = 0
 If a, b, c are of the same sign then both the roots are  (x + 7) (x + 1) = 0
negative, the sum of the roots is negative but the product  Either x + 7 = 0
of roots is positive (D  0).  x=–7
Ex.1 Find the roots of the equation x2 – x – 3 = 0. or x + 1 = 0
Sol. x2 – x – 3 = 0  x=–1
From the quadratic formula we can find the value of x, Hence, the required solution are x = – 7 and x = – 1.
Ex.5 Solve the quadratic equation 16x2 – 24x = 0.
 1  1  4  1( 3 ) 1 13
x= = Sol. The given equation may be written as 8x(2x – 3) = 0.
2 1 2
3
1 13 1 - 13 This gives x = 0 or x = .
So, x = , 2
2 2
Hence, the roots are Irrational. 3
x = 0, are the required solutions.
2
Ex.2 Determine the value of K for which the x = – a is a
Ex.6 Find the solutions of the quadratic equation
solution of the equation :
x2 – 2 (a + b) x + 3K = 0. x2 + 6x + 5 = 0.
Sol. Putting x = – a in the given equation, we have Sol. The quadratic polynomial x2 + 6x + 5 can be factorised
(– a)2 – 2 (a + b) (– a) + 3K = 0 as follows :
 a 2 + 2a 2 + 2ab + 3K = 0 x2 + 6x + 5
 3K = –3a 2 – 2ab = x2 + 5x + x + 5
a
= x (x + 5) + 1 (x + 5)
 K=– (3a + 2b). = (x + 5) (x + 1)
3
Therefore, the given quadratic equation becomes
METHODS OF SOLVING QUADRATIC EQUATION (x + 5) (x + 1) = 0.
This gives x = – 5 or x = – 1.
(a) By Factorization : Therefore, x = – 1, – 5 are the required solutions of the
ALGORITHM : given equation.

Step (i) Factorize the constant term of the given 2x 1 3x  9


Ex.7 Solve :   0.
quadratic equation. x – 3 2x  3 ( x – 3)(2x  3)
Sol. Obviously, the given equation is valid if x – 3  0 and
Step (ii) Express the coefficient of middle term as the
sum or difference of the factors obtained in step (i). 2x + 3  0.
Clearly, the product of these two factors will be equal to Multiplying throughout by (x – 3)(2x + 3), we get
the product of the coefficient of x2 and constant term. 2x (2x + 3) + 1(x – 3) + 3x + 9 = 0
or 4x2 + 10x + 6 = 0
Step (iii) Split the middle term in two parts obtained in or 2x2 + 5x + 3 = 0
step (ii).
or (2x + 3)(x + 1) = 0
Step (iv) Factorize the quadratic equation obtained in But 2x + 3  0, so we get x + 1 = 0.
step (iii). This gives x = – 1 as the only solution of the given
equation.

PAGE # 61
(b) By Completion of Square Method :
3 5
ALGORITHM :  x 
2 2
Step-(i) Obtain the quadratic equation. Let the quadratic
equation be ax2 + bx + c = 0, a  0. This gives x =

– 3 5 
or x =
–3 5
2 2
Step-(ii) Make the coefficient of x2 unity, if it is not unity.
b c 3 5 –3 5
i.e., obtain x2 + x+ = 0. Therefore, x = – , are the solutions of
a a 2 2
c the given equation.
Step-(iii) Shift the constant term on R.H.S. to get
a
b c Ex.10 Solve the quadratic equation by completing the
x2 + x=– .
a a squares : 3x2 – 2 15 x – 2 = 0.
Step-(iv) Add square of half of the coefficient of x.i.e. Sol. We have

2 3x2 – 2 15 x – 2 = 0
 b 
  on both sides to obtain :
 2a  15 2
 x2 – 2 x– =0
2 2 3 3
 b   b   b  c
x2 + 2   x +   =   – [Dividing both sides by the coefficient of x2]
 2a   2a   2a  a
2 2
Step-(v) Write L.H.S. as the perfect square of a binomial 15  15   15  2
2
x –2 x+    
expression and simplify R.H.S. to get

3  3  –  3  – 3 =0
   
2 [Adding and subtracting the square of half the
 b  b 2  4ac
x   = coefficient of x]
 2a  4a 2
2
 x  15   15  2 = 0
 
Step-(vi) Take square root of both sides to get  
 3 
 9 3
2
b b  4ac 2
x+ =±
 x  15  –  15  6  = 0
 
2a 4a 2
 
 3   9 
Step (vii) Obtain the values of x by shifting the constant 2
 x  15  – 21 = 0
 
b
term on RHS. 
2a

 3  9

Ex.8 Solve: 25x2 – 30x + 9 = 0. 15  21


 x– =
Sol. 25x2 – 30x + 9 = 0 3 3
 (5x)2 – 2(5x)×3 + (3)2 = 0 [Taking the square roots on both sides]
 (5x – 3)2 = 0
15  21 15  21
3 3 3  x=  =
This gives x = , or simply x = as the required 3 3 3
5 5 5
solution. Hence, the required solutions are

Ex.9 Solve: x2 + 3x + 1 = 0. 15  21 15  21
x= and x = .
Sol. We have x2 + 3x + 1 = 0 3 3
1 (c) By Using Quadratic Formula :
Add and subtract ( coefficient of x)2 in L.H.S. and get
2
Solve the quadratic equation in general form viz.
2 2
3 3 ax2 + bx + c = 0.
x2 + 3x + 1 +   –   = 0
 2  2 We have, ax2 + bx + c = 0
2 2 Step (i) By comparison with general quadratic equation,
3 3 3
 x2 + 2   x +   –   + 1 = 0 find the value of a, b and c.
2
   2  2
2 Step (ii) Find the discriminant of the quadratic equation.
 3 5
 x   –  0 D = b2 – 4ac
 2 4
Step (iii) Now find the roots of the equation by given
2 2
 3  5 equation
 x     
 2   2 
  b D b D
x= ,
2a 2a

PAGE # 62
 REMARK : APPLICATIONS OF QUADRATIC EQUATIONS
 If b2 – 4ac < 0, i.e., negative, then b 2 – 4ac is not real ALGORITHM :
and therefore, the equation does not have any real roots. The method of problem solving consists of the
2
Ex.11 Solve the quadratic equation x – 7x – 5 = 0. following three steps :
Sol. Comparing the given equation with ax2 + bx + c = 0, Step (i) Translating the word problem into symbolic
we find that a = 1, b = – 7 and c = – 5. language (mathematical statement) which means
Therefore, D = (–7)2 – 4 × 1 × (–5) = 49 + 20 = 69 > 0 identifying relationships existing in the problem and
Since, D is positive, the equation has two roots given then forming the quadratic equation.

7  69 7 – 69 Step (ii) Solving the quadratic equation thus formed.


by ,
2 2 Step (iii) Interpreting the solution of the equation, which
means translating the result of mathematical statement
7  69 7 – 69 into verbal language.
 x= , are the required solutions.
2 2
 REMARKS :
Ex.12 If the roots of the equation :
 Two consecutive odd natural numbers be 2x – 1, 2x + 1
a (b – c) x2 + b(c – a) x + c (a – b) = 0 are equal, show that
where x  N.
2 1 1  Two consecutive even natural numbers be 2x, 2x + 2
  .
b a c where x  N.
 Two consecutive even positive integers be 2x, 2x + 2
Sol. Since the roots of the given equations are equal, so
where x  Z+.
discriminant will be equal to zero.
 Consecutive multiples of 5 be 5x, 5x + 5, 5x + 10, .............
 b2(c – a)2 – 4a(b – c) . c(a – b) = 0
 b2(c2 + a2 – 2ac) – 4ac(ba – ca – b2 + bc) = 0, Ex.15 The sum of the squares of two consecutive positive
 a2b2 + b2c2 + 4a2c2 + 2b2ac – 4a2bc – 4abc2 = 0, integers is 545. Find the integers.
 (ab + bc – 2ac)2 = 0 Sol. Let x be one of the positive integers. Then the other
 ab + bc – 2ac = 0 integer is x + 1, where x  z+.
 ab + bc = 2ac Since the sum of the squares of the integers is 545,
we get x2 + (x + 1)2 = 545
1 1 2  2x2 + 2x – 544 = 0
 + =
c a b  x2 + x – 272 = 0
 x2 + 17x – 16x – 272 = 0
2 1 1  x (x + 17) – 16 (x + 17) = 0
   .. Hence Proved.
b a c  (x – 16) (x + 17) = 0.
Ex.13 If the roots of the equation (b – c) x2 + (c – a) x + (a – b) = 0 Here, x = 16 or x = – 17. But, x is a positive integer.
are equal , then prove that 2b = a + c. Therefore, reject x = – 17 and take x = 16. Hence, two
Sol. If the roots of the given equation are equal, then consecutive positive integers are 16 and (16 + 1),
discriminant is zero i.e i.e., 16 and 17.
(c – a)2 – 4 (b – c) (a – b) = 0 Ex.16 The sum of two numbers is 48 and its product is
 c2 + a2 – 2ac + 4b2 – 4ab + 4ac – 4bc = 0 432. Find the numbers.
 c2 + a2 + 4b2 + 2ac – 4ab – 4bc = 0 Sol. Let the two numbers be x and 48 – x so that their
 (c + a – 2b)2 = 0 sum is 48. It is given that the product of the two
 c + a = 2b. Hence Proved. numbers is 432. Hence, we have
x(48 – x) = 432
Ex.14 If the roots of the equation x2 – 8x + a2 – 6a = 0 are real
 48x – x2 – 432 = 0
and distinct, then find all possible values of a .
 x2 – 48x + 432 = 0
Sol. Since the roots of the given equation are real and
 x2 – 36x – 12x + 432 = 0
distinct, we must have D > 0  x (x – 36) – 12 (x – 36) = 0
 64 – 4 (a2 – 6a) > 0  4[16 – a2 + 6a ] > 0  (x – 36) (x – 12) = 0
 – 4(a2 – 6a – 16) > 0  a2 – 6a – 16 < 0  Either x – 36 = 0
 (a – 8) (a + 2) < 0  x = 36 o x – 12 = 0  x = 12.
 – 2 < a < 8.  W hen one number is 12 another number is
Hence, the roots of the given equation are real if ‘a’ lies 48 – 12 = 36 and when one number is 36, another
between – 2 and 8. number is 48 – 36 = 12.

PAGE # 63
Ex.17 The length of a hall is 5 m more than its breadth. If
the area of the floor of the hall is 84 m2, what are the
length and the breadth of the hall ?
NOTE : More than one correct option may be possible.
Sol. Let the breadth of the hall be x metres.
Then the length of the hall is (x + 5) metres. METHOD OF FINDING ROOTS
The area of the floor = x (x + 5) m2
Therefore, x (x + 5) = 84 1. The roots of the equation x2/3 + x1/3 – 2 = 0 are :
 x2 + 5x – 84 = 0 (A) 1,4 (B) 1,– 4
 (x + 12)(x – 7) = 0 (C) 1, – 8 (D) 1, 8
This gives x = 7 or x = – 12. 2. The solution of the equation 4x + 2x = 6 is/are :
Since, the breadth of the hall cannot be negative, we (A) 0, 1 (B) 2, 1
reject x = –12 and take x = 7 only. (C) 1, 0 (D) 1
Thus, breadth of the hall = 7 metres, and length of the
3. The equation x  1  x  1  0 has :
hall = (7 + 5), i.e., 12 metres.
(A) one real solution (B) two real solutions
7
Ex.18 Out of a group of swans, times the square root of (C) No solution (D) None of these
2
the total number are playing on the shore of a tank. 4. The roots of the equation 3x  1 – 1 = x are :
The two remaining ones are playing, in deep water.
(A) 0 (B) 1
What is the total number of swans ?
(C) 0,1 (D) None
Sol. Let us denote the number of swans by x.
Then, the number of swans playing on the shore of the 5. Evaluate : 6  6  6  6  ...... .
7 (A) 2.45 (B) 2.59
tank = x.
2 (C) 2.79 (D) 3
There are two remaining swans.
7 6. If p and q are the roots of the equation x2 + px + q = 0,
Therefore, x = x +2 then :
2
(A) p = 1 (B) p = 1 or 0
7
 x–2= x (C) p = – 2 (D) p = – 2 or 0
2
2 7. Solve : 2 x  9 + x = 13
7
 (x – 2)2 =   x (A) 8 (B) 20
 2
 4(x2 – 4x + 4) = 49x (C) 16 (D) 3
 4x2 – 65x + 16 = 0
 4x2 – 64x – x + 16 = 0 1
4x(x – 16) –1(x – 16) = 0 8. If x = 1 + , then the value of x will be
 1
(x – 16)(4x – 1) = 0 x
 1
x
1 2 x 1
This gives x = 16 or x = . (A) (B)
4 3 2
1
We reject x = and take x = 16. 3 3
4 (C) (D)
2 2
Hence, the total number of swans is 16.
Ex.19 The sum ‘S’ of first n natural numbers is given by the NATURE OF ROOTS
n(n  1) 9. The roots of the equation x2 + ax – 4 = 0 are, where a  R :
relation S = . Find n, if the sum is 276.
2 (A) real and distinct (B) equal
n(n  1) (C) imaginary (D) real
Sol. We have S = = 276
2 10. For what value of k, (4 – k) x2 + (2k + 4) x + (8 k + 1) = 0,
2
 n + n – 552 = 0 is a perfect square :
– 1  1  2208 – 1 – 1  2208 (A) k = 1, 2 (B) k = 1, – 2
This gives n= , (C) k = 0, 3 (D) k = – 1, – 3
2 2
11. If the equation x2 – 2kx – 2x + k 2 = 0 has equal roots,
– 1  2209 – 1 – 2209 the value of k must be :
 n= ,
2 2
1
–1  47 –1 – 47 (A) zero (B) either zero or 
 n= , 2
2 2
 n = 23, –24 1 1 1
(C)  (D) either or –
We reject n = – 24, since –24 is not a natural number. 2 2 2
Therefore, n = 23.

PAGE # 64
12. For what value of k will x2 – (3k – 1) x + 2k 2 + 2k = 11 22. If a and c are such that the quadratic equation
have equal roots ? ax2 – 5x + c = 0 has 10 as the sum of the root and also
(A) 9, – 5 (B) – 9, 5 as the product of the roots, find a and c respectively.
(C) 9, 5 (D) – 9, – 5
1 1
(A) ,6 (B) ,5
WORD PROBLEM 2 2

13. If the list price of a book is reduced by Rs. 5, a person 1


can buy 5 more books for Rs. 300. Find the original list (C) ,8 (D) None of these
2
price of the book.
(A) 15 (B) 10 23. If one root of the quadratic equation ax2 + bx + c= 0 is
(C) 20 (D) 25 double the other then which one of the following is
correct :
14. The sum of the squares of two consecutive odd
(A) b2 = 3ac (B) 2b2 = 5ac
positive integers is 290. Find them. 2
(C) 2b = 9ac (D) 2b2 > 9ac
(A) 11, 13 (B) 9, 11
(C) 13, 15 (D) 7, 9

15. The sum of the squares of two positive integers is


208. If the square of the larger number is 18 times the
smaller number, find the numbers. COMPETITIVE EXAM PREVIOUS YEAR
(A) 14, 10 (B) 12, 6 QUESTION
(C) 12, 8 (D) 16, 12 1. The quadratic equation a x2 + bx + c = 0 has real roots
16. The length of a hypotenuse of a right triangle exceeds  and . If a, b, c real and of the same sign, then :
the length of its base by 2 cm and exceeds twice the [IJSO-2008]
length of the altitude by 1 cm. Find the length of each (A)  and  are both positive
side of the triangle (in cm) : (B)  and  are both negative
(A) 6, 8, 10 (B) 7, 24, 25 (C)  and  are of opposite sign
(C) 8, 15, 17 (D) 7, 40, 41 (D) Nothing can be said about the signs of  and  as
the information is insufficient.
17. A shopkeeper buys a number of books for Rs. 80. If he
had bought 4 more for the same amount, each book 2. (2x2 + 3x + 5)1/2 + (2x2 + 3x + 20)1/2 = 15, therefore x is :
would have cost Rs. 1 less. How many books did he [IJSO-2009]
buy ? (A) (–8 / 3) (B) (14/5)
(A) 8 (B) 16 (C) (–11/2) (D) 4
(C) 24 (D) 28 3. The sum of all roots of the equations
18. Two square have sides x cm and (x + 4) cm. The sum of |x – 1|2 – 5| x – 1| + 6 = 0 is : [IJSO-2009]
their areas is 656 cm2 . Find the sides of the square : (A) 5 (B) 4
(A) 8 cm, 12 cm (B) 12 cm, 15 cm (C) 2 (D) 6
(C) 6 cm, 10 cm (D) 16 cm, 20 cm 4. Solve for x, given y = x2 – 1 and x = 2y + 1.
MISC ELL ANEOUS [NSTSE 2009]

19. If the roots of the equation x2 + px – 6 = 0 are 6 and – 1, 1  3


(A) x  {0, 1} (B) x   , 
then the value of p is : 2 4 
(A) 2 (B) 3
(C) – 5 (D) 5 1    3
(C) x   ,1 (D) x  0, 
2   4 
20. The quadratic equation having roots 1 5 and 1 5
is : 5. Find the sum of all values of "x", so that
(A) x2 – 2x – 4 = 0 2
(B) x + 2x + 4 = 0 2
16 x  = 8x
3 x 1 2
3 x  2 . [NSTSE 2009]
(C) x2 + 2x – 4 = 0 (D) x2 – 2x + 4 = 0
(A) 0 (B) 3
1/ x  1/ y  3 / 2, (C) – 3 (D) – 5
21. Solve the system of equations : 1/ x 2  1/ y 2  5 / 4

(A) (1,2) (B) (1,3) 6. The product of the roots of the equation
(C) (2, 3) (D) (2, 2) 3
8  x  3 8  x = 1 is [IJSO - 2010]
(A) – 21 (B*) – 189
(C) 9 (D) – 5

PAGE # 65
7. If one of the roots of the equation x2 – px + q = 0 is m 9. The number of natural numbers n  30 for which
times the other root then m/(1 + m2) is equal to
[IJSO - 2012] n  n  n  .......... ...... is natural number is

q p (A) 30 (B) zero


(A) (B) (C) 6 (D) 5
p 2  2q q2  2p

q p
(C) 2 (D) 2 10. The number of natural numbers n  30 for which
q  2p p  2q

n  n  n  .. is a prime number is [IJSO - 2016]


8. If a and b are two positive real numbers such that
(A) Three (B) Zero
a2  b2 a (C) Nine (D) Two
= 6 , then a positive value of lies between
ab b

[IJSO - 2013]
(A) 2 and 3 (B) 3 and 4
(C) 4 and 5 (D) 5 and 6

PAGE # 66
PROGRESSIONS

SEQUENCE GENERAL FORM OF AN A.P.

A sequence is an arrangement of numbers in a If we denote the starting number i.e. the 1st number by
definite order according to some rule. ‘a’ and a fixed number to be added is ‘d’ then
e.g. (i) 2, 5, 8, 11, ... (ii) 4, 1, – 2, – 5, ... a, a + d, a + 2d, a + 3d, a + 4d,........... forms an A.P.
(iii) 3, –9, 27, – 81, ... Ex.3 Find the A.P. whose 1st term is 10 & common difference
Types of Sequence is 5.
Sol. Given :First term (a) = 10 & Common difference (d) = 5.
On the basis of the number of terms there are two  A.P. is 10, 15, 20, 25, 30,.......
types of sequence :
(i) Finite sequences : A sequence is said to be nth FORM OF AN A.P.
finite if it has finite number of terms.
Let A.P. be a, a + d, a + 2d, a + 3d,...........
(ii) Infinite sequences : A sequence is said to be Then, First term (a1) = a + 0.d
infinite if it has infinite number of terms. Second term (a2) = a + 1.d
Ex.1 W rite down the sequence whose n th term is : Third term (a3) = a + 2.d
. .
2n 3  ( 1)n . .
(i) (ii)
n 3n . .
2n nth term (an) = a + (n – 1) d
Sol. (i) Let tn =  an = a + (n – 1) d is called the nth term.
n
Put n = 1, 2, 3, 4, .............. we get Ex.4 Determine the A.P. whose third term is 16 and the
8 difference of 5th term from 7th term is 12.
t1 = 2, t2 = 2, t3 = , t4 = 4 Sol. Given : a3 = a + (3 – 1) d = a + 2d = 16 .... (i)
3
8 a7 – a5 = 12 ..... (ii)
So the sequence is 2, 2, , 4, ........  (a + 6d) – (a + 4d) = 12
3
 a + 6d – a – 4d = 12
3  ( 1)n  2d = 12
(ii) Let tn =
3n  d=6
Put n = 1, 2, 3, 4, ...... Put d = 6 in equation (i)
So the sequence is a = 16 – 12  a = 4.
2 4 2 4  A.P. is 4, 10, 16, 22, 28,.......
, , , , ......
3 9 27 81 Ex.5 Which term of the sequence 72, 70, 68, 66,....... is 40 ?
Sol. Here 1 st term a = 72 and common difference
PROGRESSIONS d = 70 – 72 = – 2.
 For finding the value of n
Those sequence whose terms follow certain patterns an = a + (n – 1)d
are called progressions. Generally there are three types  40 = 72 + (n – 1) (–2)
of progressions.  40 – 72 = – 2n + 2
(i) Arithmetic Progression (A.P.)  – 32 = – 2n + 2
(ii) Geometric Progression (G.P.)  – 34 = – 2n
(iii) Harmonic Progression (H.P.)  n = 17
 17th term is 40.
ARITHMETIC PROGRESSION Ex.6 Is 184, a term of the sequence 3, 7, 11,.......... ?
A sequence is called an A.P., if the difference of a term Sol. Here 1 st term (a) = 3 and common difference
(d) = 7 – 3 = 4.
and the previous term is always same.
nth term (an) = a + (n – 1) d
i.e. d = tn + 1 – tn = Constant for all n  N. The constant
 184 = 3 + (n – 1) 4
difference, generally denoted by ‘d’ is called the
 181 = 4n – 4
common difference.
 185 = 4n
Ex.2 Find the common difference of the following 185
 n=
A.P. : 1, 4, 7, 10, 13, 16,...... 4
Sol. 4 – 1 = 7 – 4 = 10 – 7 = 13 – 10 = 16 – 13 = 3 (constant). Since, n is not a natural number.
 Common difference (d) = 3.  184 is not a term of the given sequence.

PAGE # 67
mth TERM OF AN A.P. FROM THE END Ex.9 Find the sum of 20 terms of the A.P. 1,4,7,10.....
Sol. a = 1, d = 3
Let ‘a’ be the 1st term and ‘d’ be the common difference
of an A.P. having n terms. Then mth term from the end n
is (n – m + 1)th term from beginning or {n – (m – 1)}th Sn = [2a + (n – 1)d]
2
term from beginning.
Ex.7 Find 20th term from the end of an A.P. 3, 7, 11........407. 20
S20 = [2(1) + (20 – 1)3]
Sol. 407 = 3 + (n – 1) 4 n = 102 2
 20th term from end m = 20 = 590.
a102 – (20 – 1) = a102 – 19 = a83 from the beginning.
a83 = 3 + (83 – 1) 4 = 331. Ex.10 Find the sum of all three digit natural numbers. Which
are divisible by 7.
SELECTION OF TERMS IN AN A.P. Sol. 1st no. is 105 and last no. is 994.
994 = 105 + (n – 1)7
Sometimes we require certain number of terms in A.P.
 n = 128
The following ways of selecting terms are generally
very convenient. 128
 Sum, S128 = [105 + 994]
2
No. of Terms Terms Common Difference
= 70336.
For 3 terms a – d, a, a + d d

For 4 terms a – 3d, a – d, a + d, a + 3d 2d


PROPERTIES OF A.P.
For 5 terms a – 2d, a – d, a, a + d, a + 2d d

For 6 terms a – 5d, a – 3d, a – d, a + d, a + 3d, a + 5d 2d (i) For any real number a and b, the sequence whose
nth term is an = an + b is always an A.P. with common
Ex.8 The sum of three numbers in A.P. is – 3 and their
difference ‘a’ (i.e. coefficient of term containing n).
product is 8. Find the numbers.
Sol. Let three no.’s in A.P. be a – d, a, a + d (ii) If a constant term is added to or subtracted from
 a – d + a + a + d = –3 each term of an A.P. then the resulting sequence is
 3a = –3 a = –1
also an A.P. with the same common difference.
& (a – d) a (a + d) = 8
 a (a2 – d2) = 8 (iii) If each term of a given A.P. is multiplied or divided
 (–1) (1 – d2) = 8 by a non-zero constant K, then the resulting
 1 – d2 = – 8 sequence is also an A.P. with common difference
 d2 = 9
d
 d= 3 Kd or respectively. W here d is the common
K
If a = – 1 & d = 3 numbers are – 4, –1, 2.
difference of the given A.P.
If a = – 1 & d = –3 numbers are 2, –1, – 4.
(iv) In a finite A.P. the sum of the terms equidistant
SUM OF n TERMS OF AN A.P. from the beginning and end is always same and is
Let A.P. be a, a + d, a + 2d, a + 3d,............., a + (n – 1)d equal to the sum of 1st and last term.
Then,
(v) If three numbers a, b, c are in A.P. , then 2b = a + c.
Sn = a + (a + d) +...+ {a + (n – 2) d} + {a + (n – 1) d} ..(i)
also Ex.11 Check whether an = 2n2 + 1 is an A.P. or not.
Sn= {a + (n – 1) d} + {a + (n – 2) d} +....+ (a + d) + a ..(ii) Sol. an = 2n2 + 1
Add (i) & (ii)
Then, an + 1 = 2(n + 1)2 + 1
 2Sn = 2a + (n – 1)d + 2a + (n – 1)d +....+ 2a + (n – 1)d  an + 1 – an = 2(n2 + 2n + 1) + 1 – 2n2 – 1
 2Sn = n [2a + (n – 1) d] = 2n2 + 4n + 2 + 1 – 2n2 – 1
= 4n + 2, which is not constant
n
 Sn  [ 2a  (n  1) d]  The above sequence is not an A.P..
2
Arit hmetic Mean ( Mean or Avera ge) (A .M.)
n n
Sn = [a + a + (n – 1)d] = [a +  ] If three terms are in A.P. then the middle term is
2 2
called the A.M. between the other two, so if a, b, c
n are in A.P., b is A.M. of a & c.
 Sn  [a  ] where,  is the last term.
2 A.M. for any n number a1, a2,..., an is;

 rth term of an A.P. when sum of first r terms is S r is


given by, tr = S r – S r – 1.

PAGE # 68
a1  a 2  a 3  .....  a n n th term = a rn1
A= .
n (ii) Sum of the first n terms.
n - A ri thm etic Means B etween Two
Numbers : 

a rn  1  , r 1
r 1

If a, b are any two given numbers & a, A1, A2,...., An, b are Sn = 
in A.P. then A1, A2,... An are the n-A.M.’s between a & b.


a 1 r n  , r 1
Total terms are n + 2.  1  r
ba (iii) Sum of an infinite G.P. when r < 1. W hen
Last term b = a + (n+2–1)d.Now, d = .
n 1
n  rn  0 if r < 1 therefore,
ba 2(b  a) a
A1 = a + , A2 = a + ,............, S = (| r |  1) .
n 1 n 1 1– r
n ( b  a) Ex.14 If the first term of G.P. is 7, its nth term is 448 and
An = a + .
n 1 sum of first n terms is 889, then find the fifth term of
 NOTE : G.P.
Sum of all n-A.M.’s inserted between a & b is equal Sol. Given : a = 7
to n times the single A.M. between a & b. tn = arn – 1 = 7(r)n – 1 = 448
n  7rn = 448 r
i.e.  Ar = nA where A is the single A.M. between a & b.
r 1
a(r n  1) 7(r n  1)
13 Also S n = =
Ex.12 Between two numbers whose sum is , an even r 1 r 1
6
number of A.M.s are inserted, the sum of these
448r  7
means exceeds their number by unity. Find the  889 = r=2
r 1
number of means.
Sol. Let a and b be two numbers and 2n A.M.s are Hence T 5 = ar4 = 7(2)4 = 112.
inserted between a and b then
1 1 1
2n Ex.15 Let S = 1 + + + + .......... find the sum of
(a + b) = 2n + 1. 2 4 8
2
(i) first 20 terms of the series
 13   13 
 n   = 2n + 1. Given a  b  6  (ii) infinite terms of the series.
 6  
 n = 6.   1  20 
1    
 Number of means = 12.  2 
  2 20  1
Sol. (i) S 20 = 1 = .
Ex.13 Insert 20 A.M. between 2 and 86. 1 219
Sol. Here 2 is the first term and 86 is the 22 nd term of A.P. 2
So, 86 = 2 + (21)d
 d=4 1
(ii) S  = = 2.
1
So, the series is 2, 6, 10, 14,......., 82, 86 1
2
 required means are 6, 10, 14,...82.

(i) If a, b, c are in G.P.  then b 2 = ac, in general if


G.P. is a sequence of numbers whose first term is a1, a2, a3, a4,......... an – 1 , an are in G.P.,
non zero & each of the succeeding terms is equal
then a1an = a2an – 1 = a3 an – 2 = ..............
to the preceding term multiplied by a constant. Thus
in a G.P. the ratio of successive terms is constant. (ii) Any three consecutive terms of a G.P. can be
This constant factor is called the common ratio of a
taken as , a , ar..
the series & is obtained by dividing any term by that r
which immediately precedes it.
(iii) Any four consecutive terms of a G.P. can be taken
1 1 1 1 a a
Example : 2, 4, 8, 16 ... & , , , ...are in G.P.
.P. as , , ar, ar3.
3 9 27 81 r3 r

(i) Therefore a, ar, ar2, ar3, ar4,...... is a G.P. with ‘a’ as (iv) If each term of a G.P. be multiplied or divided or
the first term and ‘r’ as common ratio. raised to power by the some nonzero quantity, the
resulting sequence is also a G.P.

PAGE # 69
Ex.17 If m th term of H.P. is n, while n th term is m, find its
(m + n)th term.
1
If a, b, c are in G.P., b is the G.M. between a & c. Sol. Given T m = n or = n, where a is the first
a  (m  1) d
b² = ac, therefore b = a c ; a > 0, c > 0. term an d d is th e co mm on d ifferen ce o f th e
corresponding A.P.
n-Geometric Means Between a, b : 1
So, a + (m – 1) d =
If a, b are two given numbers & a, G 1, G 2,....., G n, b n
are in G.P.. Then, 1
and, a + (n – 1) d =
G 1, G 2, G 3,...., G n are n-G.M.s between a & b. m
mn
G 1 = a(b/a)1/n+1, G 2 = a(b/a)2/n+1,......, G n = a(b/a)n/n+1  (m – n) d =
mn
 NOTE : 1
 d=
The product of n G.M.s between a & b is equal to the mn
nth power of the single G.M. between a & b 1 (m  1) 1
So, a = – =
n n mn mn
i.e.  G =  ab  = G
r
n n where G is the single G.M. 1 mn
r 1 Hence, T (m + n) = =
between a & b. a  (m  n  1) d 1 m  n  1
mn
Ex.16 Insert 4 G.M.s between 2 and 486. = .
mn
Sol. Common ratio of the series is given by 2 2
1 Ex.18 Insert 4 H.M between and .
3 13
 b  n 1
r=   = (243)1/5 = 3 Sol. Let d be the common difference of corresponding A.P.
a 13 3
Hence four G.M.s are 6, 18, 54, 162. 
2 2
So, d = = 1.
5

1 3 5 2
A sequence is said to be H.P. if the reciprocals of its  = +1=  H1 =
H1 2 2 5
terms are in A.P. If the sequence a1, a2, a3,...., an is
an H.P. then 1/a1, 1/a2,...., 1/an is an A.P. Here we do 1 3 7 2
= +2=  H2 =
not have the formula for the sum of the n terms of a H2 2 2 7
H.P. For H.P. whose first term is a and second term
1 3 9 2
is b, the n th term is tn = +3=  H3 =
H3 2 2 9
ab 1
tn = . 3 11 2
b  (n  1)(a  b) = +4=  H4 = .
H4 2 2 11
2ac a ab
If a, b, c are in H.P.  b = or = . R e l a t ion b e t w e e n me a ns :
ac c bc
If A, G, H are respectively A.M., G.M., H.M. between
 NOTE : a & b both being unequal & positive then, G² = AH
ab a i.e. A, G, H are in G.P.
(i) If a, b, c are in A.P.  =
bc a
Ex.19 The A.M. of two numbers exceeds the G.M. by
ab a
(ii) If a, b, c are in G.P.  b  c = 3 6
b and the G.M. exceeds the H.M. by ; find the
2 5
numbers.
Sol. Let the numbers be a and b, now using the relation
G 2 = A.H.
If a, b, c are in H.P., b is the H.M. between a & c, then  3  6
 G2 =  G    G  
2ac  2  5
b= .
ac 3 9
If a1, a2 , ........ an are ‘n’ non-zero numbers then H.M.  G2 = G2 + G–
10 5
H of these numbers is given by :
15
 G=6 & A=
1 1  1  1  .......  1  2
=  
H n  a1 a 2 an  . i.e. ab = 36, also a + b = 15
Hence the two numbers are 3 and 12.

PAGE # 70
 A.M.  G.M.  H.M. 7. If su m of n term s of a sequ en ce is g iven b y
Let a1, a2, a3, .......an be n positive real numbers, then S n = 2n 2 + 3n, find its 50 th term.
we define their (A) 250 (B) 225
a1  a 2  a 3  .......  a n (C) 201 (D) 205
A.M. = ,
n 8. The sum of 3 rd and 15 th elements of an arithmetic
1/n
G.M. = (a1 a2 a3 .........an) and progression is equal to the sum of 6 th, 11 th and 13 th
elements of the same progression. Then which
n element of the series should necessarily be equal
H.M. = 1 1 1 .
  .......  to zero ?
a1 a 2 an (A) 1 st (B) 9 th
(C) 12 th (D) None of the above
It can be shown that A.M.  G.M.  H.M. and equality
9. If pth, qth and rth term of an A.P. are a, b, c respectively,
holds at either places iff a1 = a2 = a3 = ..............= an.
then a (q – r) + b (r – p) + c (p – q) = ?
a b c (A) 2 (B) 1
Ex.20 If a, b, c, > 0 prove that + +  3. (C) 0 (D) None of these
b c a
Sol. Using the relation A.M.  G.M. we have 10. If m times the mth term of an A.P. is equal to n times its
a b c 1 nth term. Find the value of (m + n)th term of the A.P.
  (A) 2 (B) 1
b c a   a . b . c 
3

3  b c a  (C) 0 (D) None of these

a b c 11. Divide 600 biscuits among 5 boys so that their


    3.
b c a shares are in Arithmetic progression and the two
smallest shares together make one-seventh of what
the other three boys get. W hat is the sum of the
shares of the two boys who are getting lesser
NOTE : More than one correct option may be possible. number of biscuits, than the remaining three ?
(A) 75 (B) 85
A RI TH ME TI C PR OGR ES SI ON ( n T H TE RM ,
(C) 185 (D) 90
SELECTI ON OF TERMS, S UM OF n TER MS,
A RI TH ME TI C ME AN ) 12. If 9th term of an A.P. be zero then the ratio of its 29th and
19th term is :
1. If t54 of an A.P. is – 61 and t4 = 64, find t10. (A) 2 : 1 (B) 1 : 2
(A) 59 (B) 49 (C) 1 : 3 (D) 3 : 1
(C) 36 (D) 64
13. If four numbers in A. P. are such that their sum is 50
2. F in d th e nu mb er of terms in the seq uenc e and the greatest number is 4 times the least, then the
4, 12, 20, ........108. numbers are :
(A) 5, 10 ,15, 20 (B) 4, 10, 16, 22
(A) 12 (B) 19
(C) 3, 7, 11, 15 (D) None of these
(C) 13 (D) 14
14. If S1 is the sum of an arithmetic progression of ‘n’ odd
3. Find the sum of all natural numbers divisible by 5, number of terms and S2 the sum of the terms of the
but less than 100.
(A) 950 (B) 925 S1
series in odd places, then =
(C) 880 (D) 1050 S2
4. Find the sum of all the three digit natural numbers 2n n
(A) (B)
which on division by 7 leaves remainder 3. n1 n1
(A) 70441 (B) 70821
n1 n1
(C) 50821 (D) 40821 (C) (D)
2n n
5. The sum of three numbers in A.P. is 27 and the 15. If Sr denotes the sum of the first r terms of an A.P. Then,
sum of their squares is 293, find them. S3n : ( S2n – Sn) is
(A) 4, 9, 14 (B) 6, 9, 12 (A) n (B) 3n
(C) 7, 9, 11 (D) 8, 9, 10 (C) 3 (D) None of these
a n 1  b n 1
7n  1 16. If is the A.M. between a and b. Then,
6. The sum of n terms of two A.Ps. are in ratio
4n  27
. an  b n
th
Find the ratio of their 11 terms. find the value of n.
(A) 1 (B) 2
4 2 (C) 0 (D) 3
(A) (B)
3 3
17. The sum of three numbers in A.P. is 12 and the sum of
1 4 their cubes is 288. Find the numbers.
(C) (D) (A) 3, 4, 5 (B) 2, 4, 6
3 7
(C) 2, 5, 8 (D) 3, 6, 9

PAGE # 71
18. The angles of a quadrilateral are in A.P. whose com- 26. Which one of the following is true about A.M., G.M. and
mon difference is 10º. Find the angles. H.M. :
(A) 70º, 80º, 90º, 100º (B) 60º, 80º, 100º, 120º (A) H.M. G.M. A.M. (B) A.M. H.M. G.M.
(C) 72º, 85º, 98º, 111º (D) 75º, 85º, 95º, 105º (C) A.M. G.M. H.M. (D) G.M. H.M. A.M.
1 1 1 27. If A.M. and G.M. of two numbers is 10 and 8 respectively
19. If   and ab =c, what is the average (arithmetic
a b c then find their H.M. :
mean) of a and b ? (A) 9 (B) 12
1 (C) 6 (D) 6.4
(A) 0 (B)
2
ab
(C) 1 (D)
2c
COM PE TI TI VE E XA M PR EVIOUS Y EA R
GEOME TR I C PR OGRE S SI ON ( n T H TE RM ,
SELECTI ON OF TERMS, S UM OF n TER MS,
QU E S T I ON
GE OM ET RI C ME AN )
1. Consider the sequence 1, – 2, 3, – 4, 5, – 6, .........., n
20. Sum of n terms of the series (– 1)n + 1. What is the average of the first 300 terms of
the sequence ? [NSTSE 2009]
2  8  18  32 + .......... is
(A) – 1 (B) 0.5
n(n  1) (C) 0 (D) – 0.5
(A) (B) 2n (n + 1)
2 2. If the first four terms of an arithmetic sequence are :
a, 2a, b and a – 6 – b for some numbers "a" and "b",
n(n  1) then the value of the 100th term is : [NSTSE 2009]
(C) (D) 1 (A) – 100 (B) – 300
2
(C) 150 (D) – 150
21. The first term of an infinite G.P. is 1 and any term is 3. If l2 + m2 + n2 = 5, then (lm + mn + In) is : [IJSO-2009]
equal to the sum of all the succeeding terms. Find (A) (–5/2) (B) (–1)
the series.
(C)  5 (D)  3
1 1 1 4. Let Sn denote the sum of the first 'n' terms of an A.P.
(A) 1, 2, 4, 8, ... (B) 1, , , , ...
3 9 27 and S2n = 3Sn. Then, the ratio S3n : Sn is equal to :
[NSTSE 2010]
1 1 1 1 1 (A) 4 : 1 (B) 6 : 1
(C) 1, , , ... (D) 1, , , , ...
4 8 2 4 8 (C) 8 : 1 (D) 10 : 1
22. 3 geometric means between 1/9 and 9 is :
5. If a2 + b2 + c2 = 1 and p = ab + bc + ca, then
2 1
(A) , 1, 3 (B) , 2, 3 [IJSO-2010]
3 3
1 1 1 1 1
(C) , 1, 6 (D) , 1, 3 (A) p2 (B) – p
3 3 2 2 2
23. The first term of a G.P. is 7, the last term is 567 and
1 1
sum of terms is 847. Find the common ratio of the (C) – p 1 (D) – 1  p 
G.P. 2 2
(A) 3 (B) 2 6. If a2 + b2 + c2 + d2 = 25 , out of the following statments
(C) 4 (D) 6 the correct one is [IJSO-2011]
1 1 1 (A) ab + bc + cd + da 25
24. Find the sum the infinite G.P.: 1 +   + ......
3 9 27 (B) ab + bc +cd + da 25.
3 3 25
(A) (B) (C) ab + bc + cd + da 
5 2 2
49 8 25
(C) (D) (D) ab + bc + cd + da 
27 5 2

M IS CE LLA N EOU S bc –a ca–b ab–c


7. If , and are in A.P. and
1 1 1 1 1 a b c
25. Find the 10th term of the H.P. :1, , , , , ,..... a + b + c  0, then [IJSO-2011]
3 5 7 9 11
2ac 2ac
1 (A) b = (B) b =
(A) 19 (B) ac ac
19
ac
1 (C) b = (D) b = ac
(C) 21 (D) 2
21

PAGE # 72
8. Find the value 14. Along a road lie an odd number of stones placed at
2 2 2 2 2 intervals of 10 metre. These stones have to be as-
    ........  [IJSO-2012]
15 35 63 99 9999 sembled around the middle stone. A person can carry
(A) 8/33 (B) 2/11 only one stone at a time. If a man starts from one of the
(C) 98/303 (D) 222/909 end stones, and by carrying them in succession he
covers 3 km to pile all stones at the centre. The num-
9. x + y + z = 1 ; x  z ; x, y, z are real numbers and
ber of stones is then : [IJSO-2014]
1 1 1 (A) 12 (B) 15
  = m. How many of the following values of
x y z (C) 30 (D) 25
m are always possible. [IJSO-2012]
(i) m = 6, (ii) m = 8, 3 3 3 3 3
(iii) m = 10 and (iv) m = 12 15. + + + + .... + =?
(A) 1 (B) 2 4 28 70 130 9700
(C) 3 (D) all [IJSO-2015]
(A) 0.97 (B) 0.99
(C) 1 (D) 1.03
1 1 1 1 1
10. The sum 1– + – + – +....................– 16. The sum of first four terms of an A.P is 56. The sum of
2 3 4 5 6
last four terms of same A.P is 112. The first term of the
1 1 A.P is 11. Find the number of terms in that A.P.
+ equals [IJSO-2013]
2012 2013 [IJSO-2015]
(A) 7 (B) 8
1 1 1 1 (C) 11 (D) 13
(A) + + +.....................+
1006 1007 1008 2013
17. There are ten numbers in a certain A.P. The sum of first
1 1 1 1 three terms is 321. The sum of last three numbers is
(B) + + +...............+ 405. Find the sum of all the ten numbers.
1007 1008 1009 2013
[IJSO-2015]
1 1 1 1 (A) 1165 (B) 1210
(C) + + +.................+ (C) 1221 (D) 1252
1006 1007 1008 2012

1 1 1 1 18.. If a, b > 0 then [IJSO-2016]


(D) + + +..................+
1007 1008 1009 2012 (A) a + b  ab

11. A number of bacteria are placed in a glass. 1 second (B) a + b > ab


later each bacterium divides in three, the next second
each of the resulting bacteria divides in three again, (C) a + b  2ab
and so on. After one minute the glas, is full. When was (D) None of the above inequalities will hold
1/9th of the glas full ? [IJSO-2014]
(A) 15 sec (B) 45 sec 19. A triangle has perimeter 316 and its sides are of integer
(C) 58 sec (D) 38 sec length. The maximum possible area for such a triangle
is achieved for [IJSO-2016]
12. A number is said to be triangular number if it is the (A) Single triangle
sum of consecutive numbers beginning with 1. Which (B) Two triangles
one of the following is not a triangular number : (C) Three triangles
[IJSO-2014] (D) More than three triangle
(A) 1431 (B) 190
(C) 28 (D) 506
1 1
20. If for x, y > 0 we have  = 2 then the minimum
x y
value of xy is [IJSO-2016]
13. The houses of a row are numbered consecutively from (A) 2 (B) 1
1 to 49. Find the value of x such that the sum of the (C) 4 (D) 2
numbers of houses preceding the house numbered x
is equal to the sum of the numbers of the houses
following it. [IJSO-2014]
(A) 25
(B) 35
(C) 37
(D) No such value exists

PAGE # 73
TRIGONOMETRY

ANGLE (ii) Centesimal system : In this system a right angle is


divided into 100 equal parts, called grades. Each grade
An angle is the amount of rotation of a revolving line is sub divided into 100 minutes, and each minute into
with respect to a fixed line. If the rotation is in 100 seconds.
anticlock-wise sense, then the angle measured is Thus, 1 right angle = 100 grades (100g)
positive and if the rotation is in clock-wise sense, 1 grade = 100 minutes (100’)
then the angle measured is negative. 1 minute = 100 seconds (100”)

(iii) Circular system : In this system the unit of


measurement is radian. One radian, written as 1c, is
the measure of an angle subtended at the centre of a
circle by an arc of length equal to the radius of the
circle.

QUADRANTS

Let X’OX and YOY’ be two lines at right angles in a The number of radians is an angle subtended by an
plane. These lines divide the plane into four equal length of arc
arc of a circle at the centre is equal to .
parts are known as quadrants. The lines X’OX and radius
YOY’ are known as X-axis and Y-axis respectively. s
 =
These two lines taken together are known as the co- r
ordinate axes. The regions XOY, YOX’, X’OY’ and Y’OX Where,  = angle in radian, s = arc length and r = radius.
are known as first, second, third and fourth quadrants
respectively. (b) Relation Between Three System of
Measurement of Angles :
D G 2R
 
90 100 
Where, D = number of degrees,
G = number of grades,
and R = number of radians.

 NOTE :
(i) The angle between two consecutive digits in a clock
is 30º = (/6 radians).

(ii) The hour hand rotates through an angle of 30º in


one hour, i.e. (1/2)º in one minute.
(a) Systems of measurement of angles :
(iii) The minute hand rotates through an angle of 6º in
(i) Sexagesimal system (ii) Centesimal system one minute.
(iii) Circular system
Ex.1 Express in radians 47º 25’ 36”.
(i) Sexagesimal system : In this system a right angle ' '
is divided into 90 equal parts called degrees. Each  36   3
Sol. 47º 25’   = 47º  25 
degree is divided into 60 equal parts called minutes  60   5
and each minute is divided into 60 equal parts called ' º º
 128   128 1   32 
seconds. = 47º   = 47º    =  47 
 5   5 60   75 
Thus, 1 right angle = 90 degrees ( 90º) º
1º = 60 minutes (60’)  3557  3557 c 3557 c
=   =  = .
1’ = 60 seconds (60”)  75  75 180 13500

PAGE # 74
Ex.2 Express in degrees : Ex.6 The angles in one regular polygon is to that in another
c as 3 : 2, also the number of sides in the first is twice
 2 
(a)   (b) (– 2)c . that in the second ; how many sides do the polygons
 15 
c º have ?
 2   2 180 
Sol. (a)       = 24º Sol. Suppose the second regular polygon has number of
 15   15  
side = x.
º
 180   The first regular polygon will have number of side = 2x.
(b) (– 2)c =   2  
  
( 4 x  4)
 Each angle of the first polygon = right angle
 180
º
  6
º 2x
=   7  ( 2)  =   114  And each angle of the second polygon
 22   11 
( 2x  4)
6 
'
 8
' = right angle
= – 114º   60  = – 114º  32  x
 11   11  4 x  4 2x  4
 : =3:2
'' 2x x
8 
= – 114º 32’   60  4 x  4 6 x  12
 11  

x x
= – 114º 32’ 44”.
 4x – 4 = 6x – 12
Ex.3 Express in radians 345g 25’ 36”.
Sol. 345g 25’ 36” = 345.2536g  2x = 8  x = 4.
 The number of sides in the first and second polygons
3452536 c
=  = 1.726268 c respectively are 8 and 4.
2000000
2x Ex.7 The radius of a certain circle is 30 cm, find the
Ex.4 One angle of a triangle is grades another is approximately length of an arc of this circle ; if the length
3
3x x of the chord of the arc be 30 cm.
degrees, whilst the third is radians ; express Sol. Let ABC be the circle whose centre is O and AC is
2 75
all angles in degrees. chord.
2 g 2  9º  3 In AOC, AO = OC = AC = 30 cm.
Sol. x = x    xº
3 3  10  5 
 AOC = 60º =
3
x c x 12 x º
And  180º = Hence,
75 75 5
 
3 3 12 arc AC = radius × = 30 × = 10 = 31.4159 cm.
But xº + xº + xº = 180º 3 3
5 2 5
 6xº + 15xº + 24xº = 1800 TRIGONOMETRY
45xº = 1800
x = 40º Trigonometry means, the science which deals with
Hence, three angles of the triangle are 24º, 60º and 96º. the measurement of triangles.
Ex.5 The angles of a triangle are in A.P. and the number of Trigonometric Ratios :
degrees in the least is to the number of radians in the
greatest is 60 to c. Find the angles in degrees.
Sol. The three angles in A.P. ; if y is common difference, let
these angles be (x – y)º, xº and (x + y)º.
 x + y + x + x – y= 180º
 x = 60º
According to the question.
(x  y) 60 A right angled triangle is shown in Figure.  B is of

c  90º. Side opposite to B is called hypotenuse. There
(x  y)
180 are two other angles i.e. A and C. If we consider

or (x – y) = (x + y) × 60º C as , then opposite side to this angle is called
180 º
 3 (x – y) = x + y perpendicular and side adjacent to  is called base.
 4y = 2x
(i) Six Trigonometric Ratios are :
 y = x/2
60 º Perpendicular P
 y= = 30º. AB
2 sin  = Hypotenuse = =
Hence three angles are 30º, 60º and 90º. H AC

PAGE # 75
13
Hypotenuse H AC Ex.9 If cosec A = , then prove that :
cosec  = = = 5
Perpendicular P AB tan A – sin A = sin4A sec2 A.
2 2

13 Hypotenuse
Base B BC Sol. We have cosec A = = .
cos  = = = 5 Perpendicular
Hypotenuse H AC
So, we draw a right triangle ABC, right angled at C
such that Hypotenuse AB = 13 units and perpendicular
Hypotenuse H AC
sec  = = = BC = 5 units
Base B BC
By pythagoras theorem,
AB2 = BC2 + AC2  (13)2 = (5)2 + AC2
Perpendicu lar P AB 2
AC = 169 – 25 = 144
tan  = = =
Base B BC
AC = 144 = 12 units

Base B BC BC 5 BC 5
cot  = = = tan A = = and sin A = =
Perpendicular P AB AC 12 AB 13
AB 13 B
(ii) Interrelationship in Basic Trigonometric Ratios : and sec A = =
AC 12
1 1 L.H.S tan2 A – sin2 A
tan  =  cot  = 13
cot  tan  2 2
5
 5   5 
=   –  
1 1  12   13 
cos  =  sec  =
sec  cos  A
25 25 C 12
= –
144 169
1 1
sin  = cos ec   cosec  = 25(169  144 )
sin  =
144  169
We also observe that
25  25
sin  cos  =
tan  = and cot  = 144  169
cos  sin 
R.H.S. sin4A × sec2 A
m 4 2
Ex.8 If tan  = , then find sin   5   13 
n =   ×  
 13   12 
Sol. Let P = m and B = n 5 4  13 2
=
13 4  12 2
P m 54 25  25
 tan  = = = =
B n
13  12 2
2 144  169
So, L.H.S = R.H.S Hence Proved.

Ex.10 In  ABC, right angled at B, AC + AB = 9 cm and


BC = 3 cm. Determine the value of cot C, cosec C, sec C.
Sol. In  ABC, we have
(AC)2 = (AB)2 + BC2
 (9 – AB)2 = AB2 + (3)2 C
[ AC + AB = 9cm  AC = 9 – AB]
H2 = P2 + B2
 81 + AB2 – 18AB = AB2 + 9
 72 – 18 AB = 0
5cm
H2 = m22 + n22 72 3cm
 AB = = 4 cm.
18
H =  m 2  n2
Now, AC + AB = 9 cm
A B
P m AC = 9 – 4 = 5 cm 4cm
 sin  = = BC 3 AC 5
H  m2  n 2 So, cot C =  , cosec C =  ,
AB 4 AB 4
m AC 5
sin  = . sec C =  .
m  n2
2
BC 3

PAGE # 76
TRIGONOMETRIC TABLE TR IG O N O M E T R I C RA TIO S O F
COMPLEMENTARY ANGLES
sin (90 – ) = cos  cos (90 – ) = sin 
tan (90 – ) = cot  cot (90 – ) = tan 
sec (90 – ) = cosec  cosec (90 – ) = sec 

cot 54 º tan 20 º
Ex.13  – 2.
tan 36 º cot 70 º
cot 54 º tan 20 º
Sol.  –2
tan 36 º cot 70 º
cot( 90 º 36 º ) tan( 90 º 70 º )
= + –2
tan 36 º cot 70 º
tan 36 º cot 70 º
= + –2
tan 36 º cot 70 º
Ex.11 Given that cos (A – B) = cos A cos B + sin A sinB, find
[cot (90 – ) = tan and tan (90 – ) = cot ]
the value of cos15º.
= 1 + 1 – 2 = 0.
Sol. Putting A = 45º and B = 30º
We get 2 sin 68 º 2 cot 15 º
Ex.14. 
cos (45º – 30º) = cos 45º cos 30º + sin 45º sin 30º cos 22º 5 tan 75 º

1 3 1 1 3 tan 45 º tan 20º tan 40º tan 50 º tan 70º


 – .
 cos 15º =  + 5
2 2 2 2
2 sin 68 º 2 cot 15 º
3 1 Sol. 
 cos 15º = . cos 22º 5 tan 75 º
2 2
3 tan 45 º tan 20º tan 40º tan 50 º tan 70º
Ex.12 A Rhombus of side of 10 cm has two angles of 60º –
5
each. Find the length of diagonals and also find its 2 sin( 90 º 22 º ) 2 cot( 90 º 75 º )
area. = 
cos 22º 5 tan 75 º
Sol. Let ABCD be a rhombus of side 10 cm and
3(1)(tan 20 º tan 70 º )(tan 40 º tan 50 º )
BAD = BCD = 60º. Diagonals of parallelogram –
5
bisect each other.
2 cos 22 º 2 tan 75 º
So, AO = OC and BO = OD = 
cos 22º 5 tan 75 º
In right triangle AOB
D C 3[tan( 90 º 70 º ) tan 70 º ][tan( 90 º 50 º ) tan 50 º ]
OB –
sin 30º = 5
AB
2 3
1 OB = 2 –  (cot 70º tan 70º) (cot 50º tan 50º)
 = O 5 5
2 10 [ t an ( 90 º –  ) = c ot  , c ot ( 90 º –  ) = ta n &
 OB = 5 cm 30º sin (90º – ) = cos]
 BD = 2(OB) A B 2 3
 BD = 2 ( 5 ) =2–  = 2 – 1 = 1.
5 5
 BD = 10 cm
Ex.15 If sin 3A = cos (A – 26º) where 3A is an acute angle,
OA
cos 30º = find the value of A.
AB Sol. sin 3A = cos (A – 26º)
3 OA  cos (90º – 3A) = cos (A – 26º)
 =
2 10 [ sin = cos(90º – ]
 OA = 5 3  90º – 3A = A – 26º
 4A = 116º
 AC = 2(OA)  A = 29º

 AC = 2 ( 5 3 ) = 10 3 cm Ex.16 If tan 2A = cot (A – 18º), where 2A is an acute angle,


find the value of A.
So, the length of diagonals AC = 10 3 cm & BD = 10 cm.
Sol. tan 2A = cot (A – 18º)
1  cot (90º – 2A) = cot (A – 18º)
Area of Rhombus = × AC × BD
2
1  90º – 2A = A – 18º
= × 10 3 × 10 = 50 3 cm2. 90º + 18º = A + 2A
2 
 3A = 108º  A = 36º.

PAGE # 77
2
AREA OF TRIANGLE y x2  y2 r2
(ii) 1 + tan2 = 1    = =
x x2 x2
In a  ABC, a & b are the length of 2 sides of triangle 2
r
and  is the included angle between them. =   = sec2 
1 x
Then, Area of triangle = ab sin  1 + tan2 =sec2 
2
Proof : x
2
y2  x2 r2
(iii) 1 + cot2 = 1    = =
y y2 y2
2
r
=   = cosec2 
y
 1 + cot =cosec2 
2

1  cos 
Ex.17 Prove : = cosec + cot
1 – cos 

Const. : Draw a line from B perpendicular to AC, 1  cos 


Sol. LHS
i.e.BD  AC 1 – cos 
BD 1  cos  1  cos 
sin =  BD = a sin = 
a 1 – cos  1  cos 
1
 Area of triangle =  base  height
2 1  cos 2
=
1 1 – cos 2 
=  b  BD
2
1  cos 
1 1 =
=  b  a sin = ab sin sin 
2 2
= cosec  + cot 
TRIGONOMETRIC IDENTITIES
1  cos   sin 2 
(i) sin2  + cos2  = 1 Ex.18 Prove that : = cot 
(A) sin2  = 1 – cos2  (B) cos2  = 1 – sin2 
sin (1  cos )

(ii) 1 + tan2  = sec2  [where   90º] 1  cos   sin 2 


(A) sec2  – 1 = tan2  (B) sec2  – tan2  = 1 Sol. LHS
(C) tan2  – sec2  = – 1
sin (1  cos )

(iii) 1 + cot2  = cosec2  [where   0º] 1  sin 2   cos 


(A) cosec2  – 1 = cot2  (B) cosec2  – cot2  = 1 =
sin (1  cos )
(C) cot2  – cosec2  = – 1
Proof : Consider a right-angled ABC in which cos 2   cos 
B = 90º and A = º. Let AB = x units, BC = y units and =
sin (1  cos )
AC = r units. C
cos (1  cos )
r =
y sin (1  cos )
= cot  RHS
A B
x sin  – 2 sin3 
Ex.19 Prove that : = tan
Then, by Pythagoras theorem, we have 2 cos 3  – cos 
x2  y2  r 2 sin   2 sin 3 
Now, Sol. LHS
2 cos 3   cos 
2 2
y x  y2 x2 
(i) sin2   cos 2          2  2  sin (1  2 sin2 )
r r r r  =
 cos (2 cos 2   1)
(x 2  y 2 ) r2
=  1 [ x 2  y 2  r 2 ]
r2 r2 tan (1  2 sin2 )
2 2
sin  + cos  = 1 =
(1  2 sin2 )

= tan R.H.S. Hence proved

PAGE # 78
1 Ex.21 A man is standing on the deck of a ship, which is 8 m
Ex.20 If 7 sin2 + 3 cos2= 4, show that tan  = . above water level. He observes the angle of elevation
3 of the top of a hill as 60º and the angle of depression of
Sol. 7 sin2 + 3 cos2= 4 the base of the hill as 30º. Calculate the distance of the
Divide by cos2 hill from the ship and the height of the hill.
Sol. Let x be distance of hill from man and h + 8 be height of
7 sin2  3 cos 2  4 hill which is required.
 + =
2
cos  cos 2
cos 2 

 7tan2 + 3 = 4 sec2
 7tan2 + 3 = 4 (1 + tan2)
 7tan2 + 3 = 4 + 4tan2
 3tan2 = 1

1
 tan2 =
3

1 In right triangle ACB,


 tan = .
3 AC h
 tan 60º = 
BC x

ANGLE OF ELEVATION h
 3 = x
In order to see an object which is at a higher level In right triangle BCD,
compared to the ground level we are to look up. The CD 8
line joining the object and the eye of the observer is tan 30º = 
BC x
known as the line of sight and the angle which this line 1 8

of sight makes with the horizontal drawn through the 
3 x x=8 3
eye of the observer is known as the angle of elevation.
 Height of hill = h + 8
Therefore, the angle of elevation of an object
helps in finding out its height (Figure). = 3 .x + 8 =  3  8 3  + 8 = 32 m.
Distance of ship from hill = x = 8 3 m.
Ex.22 A vertical tower stands on a horizontal plane and is
surmounted by a vertical flag staff of height 5 meters.
At a point on the plane, the angle of elevation of the
bottom and the top of the flag staff are respectively 30º
and 60º. Find the height of tower.
Sol. Let AB be the tower of height h metre and BC be the
height of flag staff surmounted on the tower.
Let the point on the plane be D at a distance x meter
from the foot of the tower
In  ABD
ANGLE OF DEPRESSION AB 1 h
tan 30º =  =
AD 3 x
When the object is at a lower level than the observer’s
eyes, he has to look downwards to have a view of the  x= 3h .....(i)
object. In that case, the angle which the line of sight
makes with the horizontal through the observer’s eye
is known as the angle of depression (Figure).

PAGE # 79
In  ADC 2196
= m/sec
AC 15
tan 60º =
AD 2196 18
=  Km/hr
5h 15 5
 3 = = 527.04 Km/hr
x
5h Hence, the speed of aeroplane is 527.04 Km/hr.
 x= ......(ii)
3
Ex.24 If the angle of elevation of a cloud from a point h
From (i) and (ii)
metres above a lake is  and the angle of depression
5h of its reflection in the lake is , prove that the distance
 3 h=
3 of the cloud from the point of observation is
 3h = 5 + h  2h = 5
2h sec 
5 .
 h= = 2.5 m tan   tan 
2 Sol. Let AB be the surface of the lake and let C be a point of
So, the height of tower = 2.5 m
observation such that AC = h metres. Let D be the
Ex.23 The angle of elevation of an aeroplane from a point position of the cloud and D’ be its reflection in the lake.
on the ground is 45º. After a flight of 15 sec, the elevation Then BD = BD’.
changes to 30º. If the aeroplane is flying at a height of In  DCE
3000 metres, find the speed of the aeroplane.
Sol. Let the point on the ground is E which is y metres from
point B and let after 15 sec flight it covers x metres
distance.
In  AEB

AB
tan 45º =
EB DE
tan  =
CE
3000
 1= y H
 CE = ......(i)
tan 
 y= 3000 m .......(i)
In  CED’
In  CED
ED'
CD tan  =
 tan 30º = EC
ED
hHh
1 3000  CE =
 = xy ( AB = CD) tan 
3
2h  H
 CE = .......(ii)
 x + y = 3000 3 .......(ii) tan 
From equation (i) and (ii) From (i) & (ii)
 x + 3000 = 3000 3 H 2h  H
 =
tan  tan 
 x = 3000 3 – 3000 x = 3000 ( 3 – 1)  H tan  = 2h tan  + H tan 
 x = 3000 × (1.732 – 1) x = 3000 × 0.732  H tan  – H tan  = 2h tan 
 x = 2196 m.  H (tan  – tan ) = 2h tan 
Dis tan ce covered 2h tan 
Speed of Aeroplane =  H= ........(iii)
Time taken tan   tan 

PAGE # 80
In  DCE  1
6. If  +  = and sin  = , then sin  is :
DE 2 3
sin  =
CD 2 2 2
(A) (B)
DE 3 3
 CD =
sin  2 3
(C) (D)
H 3 4
 CD =
sin  
7. If 7 sin  = 24 cos ; 0 <  < , then value of
Substituting the value of H from (iii) 2
14 tan  – 75 cos  – 7 sec  is equal to :
2h tan 
CD = (A) 1 (B) 2
tan   tan  sin 
sin  (C) 3 (D) 4
2h
cos 
 CD =  
tan   tan  sin  
tan 

8. If tan  = 4, then   is equal to :
2h sec   sin 3  
CD =   sin  cos  
tan   tan   cos  
Hence, the distance of the cloud from the point of (A) 0 (B) 2 2
2h sec  (C) 2 (D) 1
observation is . Hence Proved.
tan   tan 
9. The area of a triangle is 12 sq. cm. Two sides are 6 cm
and 12 cm. The included angle is :

 1  1
NOTE : More than one correct option may be possible. (A) cos–1  3  (B) cos–1  6 
   
SYSTEM OF MEASUREMENT OF ANGLE/
C O NV E R SI O N  1  1
(C) sin–1  6  (D) sin–1  
  3
1. Radian measure of 175º 45’ is :
TRIGONOMETRIC ANGLE
700 703
(A)  (B) 
720 720 10. If tan 15º = 2 – 3 , then the value of cot2 75º is :
(A) 7 + 3 (B) 7 – 2 3
705 710
(C)  (D)  (C) 7 – 4 3 (D) 7 + 4 3
720 720
4 3
 1
c 11. If a = cot2 30º + 3 sin2 60º – 3 cosec2 60º – tan2 30º
3 4
2. Degree measure of   is :
4 and b = 3 tan2 45º + cos 0º – cot 90º then logb(a) is :
(A) 15º 19’5” (B) 14º 19’ 5” (A) 2 (B) –1
(C) 15º 18º 6” (D) 14º 18º 6”
1 1
7 (C) (D)
3. Degree measure of is : 2 2
6
(A) 210° (B) 240° 12. The angles of the triangles ABC and DEF are given as
(C) 270° (D) None follows : A = 900, B = 300, D = 900 and E = 300. If the side
4. The difference between two angles is 19º and their BC is twice the side EF, which of the following statement
890 is true?
sum is grades. Find the greater angle.
9 (A) Sin B = 2 Sin E (B) Sin E = 2 Sin B
(A) 63º (B) 35º
(C) Sin B = Sin E (D) Sin A = Sin B
(C) 27º (D) 54º
TRIGONOMETRIC RATIOS 13. The value of the expression

a a sin   b cos  4 3
5. If tan  = then value of is = cot2 30º + 3sin2 60º – 2cosec2 60º – tan2 30º is :
b a sin  – b cos  3 4

a2  b2 a 2 – b2 20
(A) 1 (B) –
(A) 2 2 (B) 2 2 3
a –b a b
10
a 1 (C) (D) 5
(C) (D) 3
2 2 2 2
a b a b

PAGE # 81
14. The value of the expression 22. If tan 2A = cot (A – 18º), where 2A is an acute angle,
5 sin2 30 º  cos 2 45 º 4 tan 2 60º find the value of A.
is : (A) 36º (B)63º
2 sin 30 º cos 60 º  tan 45 º
(A) 4 (B) 9 (C)26º (D) 62º

53 55 23. Evaluate :
(C) (D)
12 6 sec 2 54 º  cot 2 36 º
+ 2 sin 2 38º sec 2 52º
15. Find the value of x from the equation : cos ec 2 57 º  tan 2 33 º
 
cot 2 sec tan
 2
 2  6 3 4 – sin2 45º + tan 17º tan 60º tan 73º.
x sin cos  3
6 4 cos ec 2  cos ec 
4 6 9 7
(A) (B)
2 2
(A) 4 (B) 6
(C) – 2 (D) 0 3 1
(C) (D)
16. If  and  are angles in the first quadrant, tan 2 2
HEIGHT AND DISTANCE
1 1
= , sin  = , then using the formula sin (A + B)
7 10 24. An aeroplane when flying at a height 2500 m from the
= sin A cos B + cos A sin B, one can find the value of ( ground, passes vertically above another aeroplane. At
+ 2) to be : an instant when the angles of elevation of the two
(A) 0º (B) 45º aeroplanes from the same point on the ground are 45º
(C) 60º (D) 90º and 30º respectively, then the vertical distance between
the two aeroplanes at that instant is :
17. If each of ,  and  is a positive acute angle such that (A) 1158 m (B) 1058 m
1 1 (C) 1008 m (D) none
sin ( +  –  ) = , cos( +  – ) = and
2 2 25. The shadow of a tower is 30 metres when the sun’s
tan (  +  – ) = 1, then the values of , and  is :
altitude is 30º. When the sun’s altitude is 60º, then the
(A) 45º, 45º & 90º (B) 60º, 45º & 75º length of shadow will be :
1 1 (A) 60 m (B) 15 m
(C) 37  , 45° & 52  (D) none
2 2 (C) 10 m (D) 5m
COMPLEMENTARY ANGLE 26. The angles of elevation of the top of a vertical tower
from two points 30 metres apart, and on the same
18. The value of tan 5º tan 10º tan 15º tan 20º...... tan 85º is : straight line passing through the base of tower, are
(A) 1 (B) 2 300 and 600 respectively. The height of the tower is :
(C) 3 (D) None (A) 10 m (B) 15 m

19. If  +  = 90º and  = 2, then cos2  + sin2  equals to : (C) 15 3 m (D) 30 m
27. If the angle of elevation of a cloud from a point 200
1
(A) (B) 0 metres above a lake is 30º and the angle of depression
2
of its reflection in the lake is 60º, then the height of the
(C) 1 (D) 2 cloud (in metres) above the lake is :
1 (A) 200 (B) 300
20. If tan (A – B) = and tan (A + B) = 3 ,
3 (C) 500 (D) none
0º < A + B  90º, A > B. Then the value of A and B is :
28. The angle of elevation of the top of tower from the top
(A) 45º, 30º (B) 45º, 15º
and bottom of a building h metre high are  and , then
(C) 60º, 30º (D) none
the height of tower is :
21. If A, B, C are the interior angles of a triangle ABC, then (A) h sin cos / sin ( + )
A B (B) h cos  cos / sin(–)
cos   equals to : (C) h cos  sin / sin(–)
 2 
C C (D) None of these
(A) cos (B) sec
2 2 29. When a eucalyptus tree is broken by strong wind, its
C C top strikes the ground at an angle of 30º to the ground
(C) cosec (D) sin
2 2 and at a distance of 15 m from the foot. What is the
height of the tree?
(A) 15 3 m (B) 10 3 m
(C) 20 m (D) 10 m
PAGE # 82
30. A man at the top of a vertical lighthouse, observes a 6. If cos + sin = 2 cos, then cos – sin = ?
boat coming directly towards it.If it takes 20 minutes [NSTSE 2009]
for the angle of depression to change from 30º to 60º, (A) 2 tan (B) 2 sin
the time taken by the boat to reach the lighthouse from 2
(C) (D) none of these.
the point when the angle of depression was 30º, is : cos   sin 
(A) 30 minutes (B) 20 minutes
7. The tops of two poles of heights 20m and 14m are
(C) 10 minutes (D) 5 minutes
connected by a wire. If the wire makes an angle of 30º
with the horizontal, then the length of the wire is :
[NSTSE 2009]
COMPETITIVE EXAM PREVIOUS YEAR (A) 40 m (B) 12 m
QUESTION (C) 28 m (D) 68 m
1. The expression (1 – tan A + sec A) (1 – cot A + cosec A) 2 3 4 x 1
8. If sin 2  . . ......  = 1, 0º < x < 100º, then the
has value : [IJSO-2008] 1 2 3 x 2
(A) – 1 (B) 0 value of x is equal to : [NSTSE 2010]
(C) + 1 (D) + 2 (A) 91º (B) 80º
(C) 49º (D) 46º
2. A person on the top of a tower observes a scooter
moving with uniform velocity towards the base of the 1 – sin x 1 – sin x cos x
9. If p = ,q= ,r= , then
tower. He finds that the angle of depression changes 1  sin x cos x 1  sin x
Which one of the following statement is correct ?
from 30º to 60º in 18 minutes. The scooter will reach
[NSTSE-2010]
the base of the tower in next : [IJSO-2008]
(A) p = q  r (B) q = r  p
(A) 9 minutes
(C) r = p  q (D) p = q = r
(B) 18 / ( 3 – 1) minutes
(C) 6 3 minutes 10. If sin  + cosec = 2, then [sin8 + cosec8 ] will have
(D) the time depends upon the height of the tower the value [IJSO-2010]
(A) 2 (B) 24
3. In the diagram, PTR and QRS are straight lines. Given
(C) 26 (D) 28
4
that, tan xº = and "T" is the midpoint of PR, calculate 11. An aeroplane is flying horizontally at a height of 3150
3
the length of PQ, in cm. [NSTSE 2009] m above a horizontal plane ground. At a particular in-
S stant it passes another aeroplane vertically below it. At
this instant, the angles of elevation of the planes from

a point on the ground are 30º and 60º. Hence, the dis-
3 cm

tance between the two planes at that instant is


T
R P [IJSO-2011]
(A) 1050 m. (B) 2100 m.
6 cm

(C) 4200 m. (D) 5250 m.


2 2
12. If cot (1–3sec + 2sec ) =1 [IJSO-2012]
(A) 120º (B) 210o
(C) 300º (D) 330º
Q
(A) 8 (B) 9 13. If sin x + sin y = a and cos x – cos y = b.Then find the
1
(C) 59 (D) 10 value of (2 – a2 – b2) [IJSO-2012]
2
(A) cos(x + y) (B) cos(x – y)
4. The trigonometric expression
(C) sin(x + y) (D) sin(x – y)
 sec  – 1  sin  – 1  14. Two friends A and B watched a car from the top of their
cot2   + sec2   1  sec   has the value buildings. Angle of depression for A was 10° more than
 1  sin    
angle of depression for B, then [IJSO-2016]
[IJSO-2009] (A) A's apartment is taller than B's apartment
(A) –1 (B) 0 (B) B's apartment is taller than A's apartment
(C) 1 (D) 2 (C) A's apartment and B's apartment have same height
(D) We cannot compare the heights of the two apart-
5. (1 + tan2 ) / (1 + cot2 ) = [IJSO-2009] ments
(A) tan2  (B) cot2 
(C) sec2  (D) cosec2 

PAGE # 83
CO-ORDINATE GEOMETRY

 REMARKS :
RECTANGULAR CO-ORDINATES (i) Abscissa is the perpendicular distance of a point
from y-axis. (i.e., positive to the right of y-axis and
Take two perpendicular lines X’OX and Y’OY intersecting negative to the left of y-axis).
at the point O. X’OX and Y’OY are called the co-ordinate
(ii) Ordinate is the perpendicular distance of a point
axes. X’OX is called the X-axis, Y’OY is called the
from x-axis. (i.e., positive above x-axis and negative
Y-axis and O is called the origin. Lines X’OX and Y’OY
below x-axis).
are sometimes also called rectangular axes.
(iii) Abscissa of any point on y-axis is zero.

(iv) Ordinate of any point on x-axis is zero.

(v) Co-ordinates of the origin are (0,0).

DISTANCE BETWEEN TWO POINTS

Let two points be P (x1, y1) and Q (x2, y2).


Take two mutually perpendicular lines as the coordinate
axis with O as origin. Mark the points P (x1, y1) and
(a) Co-ordinates of a Point : Q (x2, y2). Draw lines PA, QB perpendicular to X-axis,
Let P be any point as shown in figure. Draw PL and PM from the points P and Q, which meet the X-axis in
perpendiculars on Y- axis and X - axis, respectively. points A and B, respectively.
The length LP (or OM) is called the x - coordinate or Y
the abscissa of point P and MP is called the
D Q(x2, y2)
y-coordinate or the ordinate of point P. A point whose
abscissa is x and ordinate is y named as the point
C
-----------------

(x, y) or P (x, y). P(x1, y1) R

B X
O A

Draw lines PC and QD perpendicular to Y-axis, which


meet theY-axis in C and D, respectively. Produce CP to
meet BQ in R.
Now, OA = abscissa of P = x1
Similarly, OB = x2, OC = y1 and OD = y2
The two lines X’OX and Y’OY divide the plane into four
Therefore, we have
parts called quadrants. XOY, YOX’, X’OY’ and Y’OX
PR = AB = OB – OA = x2 – x1
are, respectively, called the first, second, third and fourth
Similarly, QR = QB – RB = QB – PA = y2 – y1
quadrants. The following table shows the signs of the Now, using Pythagoreus Theorem, in right angled
coordinates of points situated in different quadrants: triangle PRQ, we have
PQ2 = PR2 + RQ2
or PQ2 = (x2 – x1)2 + (y2 – y1)2
Quadrant X - coordinate Y - coordinate Point Since the distance or length of the line-segment PQ is
First quadrant + + (+, +) always non-negative, on taking the positive square root,
Second quadrant – + (–, +)
we get the distance as

Third quadrant – – (–, –)


PQ = (x 2 –x1)2  (y 2 –y1)2
Fourth quadrant + – (+, –)
This result is known as distance formula.
Corollary : The distance of a point P (x1, y1) from the
origin (0, 0) is given by OP = x 12  y 12 .

PAGE # 84
Some useful points : Ex.3 Using distance formula, show that the points (–3, 2),
1. In questions relating to geometrical figures, take the (1, –2) and (9, –10) are collinear.
Sol. Let the given points (–3, 2), (1, –2) and (9, –10) be
given vertices in the given order and proceed as
denoted by A, B and C, respectively. Points A, B and
indicated. C will be collinear, if the sum of the lengths of two
(i) For an isosceles triangle : We have to prove that at line-segments is equal to the third.
least two sides are equal. Now,
(ii) For an equilateral triangle : We have to prove that
AB = (1  3 )2  (– 2 – 2) 2  16  16  4 2
three sides are equal.
(iii) For a right-angled triangle : We have to prove that BC = (9 – 1)2  (– 10  2) 2  64  64  8 2
the sum of the squares of two sides is equal to the
AC = (9  3 )2  (–10 – 2) 2  144  144  12 2
square of the third side.
(iv) For a square : We have to prove that the four sides Since, AB + BC = 4 2 + 8 2 = 12 2 = AC, the points
s
are equal, two diagonals are equal. A, B and C are collinear.
(v) For a rhombus : We have to prove that four sides Ex.4 Find a point on the X-axis which is equidistant from
are equal (and there is no need to establish that two the points (5, 4) and (–2, 3).
diagonals are unequal as the square is also a Sol. Since the required point (say P) is on the X-axis, its
rhombus). ordinate will be zero. Let the abscissa of the point be x.
Therefore, coordinates of the point P are (x, 0).
(vi) For a rectangle : We have to prove that the opposite
Let A and B denote the points (5, 4) and (– 2, 3),
sides are equal and two diagonals are equal. respectively.
(viI) For a parallelogram : We have to prove that the Since we are given that AP = BP, we have
opposite sides are equal ( and there is no need to AP2 = BP2
establish that two diagonals are unequal as the i.e., (x – 5)2 + (0 – 4)2 = (x + 2)2 + (0 – 3)2
rectangle is also a parallelogram).  x2 + 25 – 10x + 16 = x2 + 4 + 4x + 9
 –14x = –28
2. For three points to be collinear : We have to prove
 x = 2.
that the sum of the distances between two pairs of
Thus, the required point is (2, 0).
points is equal to the third pair of points.
Ex.5 The vertices of a triangle are (– 2, 0), (2, 3) and (1, – 3).
Ex.1 Find the distance between the points (8, –2) and Is the triangle equilateral, isosceles or scalene ?
(3, –6). Sol. Let the points (–2, 0), (2, 3) and (1, –3) be denoted by A,
Sol. Let the points (8, –2) and (3, –6) be denoted by P and B and C respectively. Then,

Q, respectively. AB = (2  2) 2  (3 – 0) 2  5
Then, by distance formula, we obtain the distance PQ as
BC = (1 – 2)2  (–3 – 3) 2  37
PQ = (3 – 8)2  (– 6  2)2  (–5)2  (–4)2  41 unit.
And AC = (1  2) 2  (–3 – 0 )2  3 2
 1 1 Clearly, AB  BC  AC.
Ex.2 Prove that the points (1, –1),  – ,  and (1, 2) are
 2 2 Therefore, ABC is a scalene triangle.
the vertices of an isosceles triangle.
Ex.6 The length of a line-segment is 10. If one end is at
 1 1 (2, – 3) and the abscissa of the second end is 10,
Sol. Let the point (1, –1),  – ,  and (1, 2) be denoted by
 2 2 show that its ordinate is either 3 or – 9.
P, Q and R, respectively. Now, Sol. Let (2, – 3) be the point A. Let the ordinate of the second
2 2 end B be y. Then its coordinates will be (10, y).
 1  1  18 3
PQ =  – – 1    1 =  2
 2  2  4 2  AB = (10 – 2)2  ( y  3) 2  10 (Given)

2 2  64 + 9 + y2 + 6y = 100
 1  1 18 3
QR = 1     2 –    2  y2 + 6y + 73 – 100 = 0
 2  2 4 2
 y2 + 6y – 27 = 0
2 2
PR = (1 – 1)  (2  1)  9 = 3  (y + 9)(y – 3) = 0
From the above, we see that PQ = QR. Therefore, y = – 9 or y = 3.
i.e., The ordinate is 3 or – 9.
 The triangle is isosceles.

PAGE # 85
Ex.7 Show that the points (– 2, 5), (3, – 4) and (7, 10) are the AP AH PH
  
vertices of a right triangle. BP PK BK
Sol. Let the three points be A (– 2, 5), B (3, – 4) and C (7, 10). m x – x1 y – y1
  
Then, AB2 = (3 + 2)2 + (–4 – 5)2 = 106 n x2 – x y2 – y
BC2 = (7 – 3)2 + (10 + 4)2 = 212
m x – x1
AC2 = (7 + 2)2 + (10 – 5)2 = 106 Now, 
n x2 – x
We see that
 mx2 – mx = nx – nx1
 BC2 = AB2 + AC2  mx + nx = mx2 + nx1
 212 = 106 + 106  212 = 212 mx 2  nx1
 x=
 A = 900. mn
Thus, ABC is a right triangle, right angled at A. m y – y1
And n  y  y
2
Ex.8 If the distance of P (x, y) from A (5, 1) and B (–1, 5) are
equal, prove that 3x = 2y.  my2 – my = ny – ny1
 my + ny = my2 + ny1
Sol. P (x, y), A (5, 1) and B (–1, 5) are the given points.
my 2  ny1
AP = BP [Given]  y=
mn
 AP2 = BP2  AP2 – BP2 = 0
 mx 2  nx 1 my 2  ny 1 
 {(x – 5)2 + (y – 1)}2 – {(x + 1)2 + (y – 5)2} = 0 Thus, the coordinates of P are  , .
 mn mn 
 x2 + 25 – 10x + y2 + 1 – 2y – x2 – 1 – 2x – y2 – 25 + 10y = 0  REMARK :
If P is the mid-point of AB, then it divides AB in the ratio
 –12x + 8y = 0
 x1  x 2 y1  y 2 
 3x = 2y. 1 : 1, so its coordinates are  , .
 2 2 

SECTION FORMULAE (b) Formula for External Division :

(a) Formula for Internal Division : The coordinates of the point which divides the line
segment joining the points (x1, y1) and (x2, y2) externally
The coordinates of the point (x, y) which divides the in the ratio m : n are given by :
line segment joining the points (x1, y1) and (x2, y2)
internally in the ratio m : n are given by
mx 2  nx1 my 2  ny1
x , y .
mn mn
Proof :
mx 2  nx1 my 2  ny1
x , y .
Let O be the origin and let OX and OY be the X-axis and mn mn
Y-axis respectively. Let A (x1, y1) and B (x2, y2) be the Ex.9 Find the coordinates of the point which divides the line
segment joining the points (6, 3) and (–4, 5) in the ratio 3 : 2
given points. Let (x, y) be the coordinates of the point P
(i) internally (ii) externally.
which divides AB internally in the ratio m : n. Draw Sol. Let P(x, y) be the required point.
A L  O X, B M  O X, P N  O X. Also, draw AH and PK (i) For internal division, we have
perpendiculars from A and P on PN and BM respectively. 3  –4  2  6 35  23
Then, x= And y=
32 32
OL = x1, ON = x, OM = x2, AL = y1, PN = y and BM = y2.
 AH = LN = ON – OL = x – x1, PH = PN – HN
21
 x = 0 and y = .
5
21
So the coordinates of P are (0, ).
5
(ii) For external division, we have
3  –4 – 2  6 35 – 23
x= and y =
3–2 3–2

PN – AL = y – y1, PK = NM = OM – ON = x2 – x
And BK = BM – MK = BM – PN = y2 – y.  x = –24 and y = 9
Clearly,  AHP and  PKB are similar.. So the coordinates of P are (–24, 9).

PAGE # 86
Ex.10 In what ratio does the point (–1, –1) divides the line   x  x3   y  y3 
segment joining the points (4, 4) and (7, 7)?  1.x1  2 2  1.y1  2 2 
  2 ,  2 
Sol. Suppose the point C (–1, –1) divides the line joining  1 2 1 2 
 
the points A(4, 4) and B(7, 7) in the ratio k : 1. Then, the
 
 7k  4 7k  4 
coordinates of C are  ,   x  x 2  x 3 y1  y 2  y 3 
 k 1 k 1  = 1 , 
 3 3 
But, we are given that the coordinates of the point
C are (–1, –1).

7k  4 5
 = –1 k = –
k 1 8
Thus, C divides AB externally in the ratio 5 : 8.

Ex.11 In what ratio does the X-axis divide the line segment
joining the points (2, –3) and (5, 6)?
Sol. Let the required ratio be  : 1. Then the coordinates of
 5  2 6 – 3 
the point of division are  ,  . But, it is a
  1  1 
 x1  x 3 y1  y 3 
point on X-axis on which y-coordinate of every point is The coordinates of E are  ,  . The
 2 2 
zero.
coordinates of a point dividing BE in the ratio 2 : 1 are
6 – 3
 =0
 1  2( x1  x 3 ) 2( y1  y 3 ) 
1  1.x 2  1.y 2  
  =  2 , 2 
2  1 2 1 2 
1  
Thus, the required ratio is : 1 or 1 : 2.  
2  x  x  x y  y  y 3 
= 1 2 3
, 1 2

Ex.12 Determine the ratio in which the line 3x + y – 9 = 0  3 3 
divides the segment joining the points (1, 3) and (2, 7). Similarly the coordinates of a point dividing CF in the
Sol. Suppose the line 3x + y – 9 = 0 divides the line segment
 x  x 2  x 3 y1  y 2  y 3 
joining A (1, 3) and B (2, 7) in the ratio k : 1 ratio 2 : 1 are  1 , 
 3 3 
at point C. Then, the coordinates of C are
Thus, the point having coordinates
 2k  1 7k  3 
 ,  . But, C lies on 3x + y – 9 = 0, therefore
 k 1 k 1   x1  x 2  x 3 y1  y 2  y 3 
 ,  is common to AD, BE
 2k  1  7k  3  3 3 
3  –9 0 and CF and divides them in the ratio 1 : 2.
 k 1  k 1
 6k + 3 + 7k + 3 – 9k – 9 = 0 Hence, medians of a triangle are concurrent and the
3 coordinates of the centroid are
 k=
4  x 1  x 2  x 3 y1  y 2  y 3 
So, the required ratio is 3 : 4 internally.  , .
 3 3 
CENTROID OF A TRIANGLE IN – CENTRE OF A TRIANGLE

Prove that the coordinates of the centroid of the triangle The coordinates of the in-centre (intersection point
of angle bisector segment) of a triangle whose
whose vertices are (x1, y1), (x2, y2) and (x3, y3) are
vertices (x 1 , y1 ), (x 2 , y2 ) and (x 3 , y3 ) are
 x1  x 2  x 3 y1  y 2  y 3 
 ,  . Also, deduce that the  ax 1  bx 2  cx 3 ay 1  by 2  cy 3 
 3 3   , .
 abc abc 
medians of a triangle are concurrent.
Proof :
Let A (x 1, y1), B (x 2, y2) and C (x 3, y3) be the vertices
of  ABC whose medians are AD, BE and CF
respectively. So D, E and F are respectively the
mid-points of BC, CA and AB.
 x2  x3 y2  y3 
Coordinates of D are  , .
 2 2  Where a, b, c be the lengths of the sides BC, CA, AB
Coordinates of a point dividing AD in the ratio 2 : 1 respectively.
are
PAGE # 87
EX – CENTRE OF A TRIANGLE (ii) Let A (0, 6), B (8, 12) and C (8, 0) be the vertices
of triangle ABC.
Let A (x1, y1), B (x2, y2), C (x3, y3) be the vertices of the Then c = AB = (0  8)2  (6  12)2 = 10, b = CA
triangle ABC and let a, b, c be the lengths of the sides
= (0  8)2  (6  0)2 = 10
BC, CA, AB respectively.
The coordinates of ex-centre I1 (centre of exscribed And a = BC = (8  8)2  (12  0)2 = 12.
circle opposite to the angles A) are given by
The coordinates of the in-centre are
 – ax 1  bx 2  cx 3 – ay 1  by 2  cy 3 
 , 
–abc –abc  ax 1  bx 2  cx 3 ay 1  by 2  cy 3 
   , 
 abc abc 

 12  0  10  8  10  8 12  6  10  12  10  0 
or  , 
 12  10  10 12  10  10 

 160 192 
or  ,  or (5, 6).
 32 32 

AREA OF A TRIANGLE

Let ABC be any triangle whose vertices are A (x1 , y1),


B (x 2 , y2) and C(x 3 , y3). Draw BL, AM and CN
perpendiculars from B, A and C respectively, to the
The coordinates of I2 and I3 (centres of exscribed circles
X – axis. ABLM, AMNC and BLNC are all trapeziums.
opposite to the angles B and C respectively) are given by
Y
 ax 1 – bx 2  cx 3 ay 1 – by 2  cy 3 
I2  ,  and A(x1 , y1 )
 a–bc a–bc 
 ax 1  bx 2 – cx 3 ay 1  by 2 – cy 3 
B(x2 , y2 )

I3  ,  respectively.. C(x3 , y3 )
 ab–c ab–c 

 NOTE :
(i) Incentre divides the angle bisectors in the ratio,
(b + c) : a; (c + a) : b & (a + b) : c. X
O L M N

(ii) Orthocenter, Centroid & Circumcenter are always


Area of ABC = Area of trapezium ABLM + Area of
c o lli nea r & c e ntr oid di vi des th e l ine jo ini ng
trapezium AMNC – Area of trapezium BLNC
orthocenter & circumcenter in the ratio 2 : 1
1
respectively. We know that, Area of trapezium = (Sum of
2
(i ii ) I n a n i s o s c e les tr ian gle Ce nt rod (G ), parallel sides) (distance b/w them)
Orthocenter (O), Incenter (I & Circumcenter (C) lie 1 1
Therefore, Area of ABC = (BL + AM) (LM) + (AM +
on the same line and in an equilateral triangle, all 2 2
these four points coincide. 1
CN) MN – (BL + CN) (LN)
2
Ex.13 Find the coordinates of (i) centroid (ii) in-centre of 1 1
Area of ABC = (y2 + y1)(x1 – x2) + (y1 + y3) (x3 – x1) –
the triangle whose vertices are (0, 6), (8, 12) and 2 2
(8, 0). 1
(y + y3) (x3 – x2)
2 2
Sol. (i) We know that the coordinates of the centroid of a
triangle whose angular points are (x1, y1), (x2, y2)
Area of ABC =
 x1  x 2  x 3 y1  y 2  y 3

(x3, y3) are  , . 1
 3  3 [ x1 ( y 2  y 3 )  x 2 ( y 3  y1 )  x 3 ( y1  y 2 )]
2
So the coordinates of the centroid of a triangle
whose vertices are (0, 6), (8, 12) and (8, 0) are (a) Condition for collinearity :
Three points A (x1, y1), B (x2, y2) and C (x3, y3) are
 0  8  8 6  12  0   16 
 ,  or  , 6  .
 3 3   3  collinear if Area of ABC = 0.

PAGE # 88
AREA OF QUADRILATERAL LOCUS AND EQUATION OF THE LOCUS

Let the vertices of Quadrilateral ABCD are A (x1, y1), LOCUS : The curve described by a point which moves
B (x2, y2) , C (x3, y3) and D (x4, y4) under given condition or conditions is called the locus.
So, Area of quadrilateral ABCD = Area of  ABC + For example, suppose C is a point in the plane of the
Area of ACD. paper and P is a variable point in a plane of the paper
D C
(x4 , y4 ) (x3, y3) such that its distance from C is always equal to a (say).
It is clear that all the positions of the moving point P lie
on the circumference of a circle whose centre is C and
whose radius is a.

A B EQUATION OF THE LOCUS OF A POINT : The equation


(x1, y1) (x2, y2)
to the locus of a point is the relation which is satisfied
Ex.14 The vertices of ABC are (–2, 1), (5, 4) and (2, – 3)
by the co - ordinates of every point on the locus of the
respectively. Find the area of triangle.
point.
Sol. A (–2, 1), B (5, 4) and C (2, – 3) be the vertices of
triangle. ALGORITHM TO FIND THE LOCUS OF A POINT :
So, x1 = –2, y1 = 1 ; x2 = 5, y2 = 4 ; x3 = 2, y3 = –3 STEP 1 : Assume the co-ordinates of point say (h, k)
Area of  ABC whose locus is to be found.
1
= x1( y 2  y 3 )  x 2 ( y 3  y1)  x 3 ( y1  y 2 ) STEP 2 : Write the given condition in mathematical
2
1 form involving h, k.
= ( 2)( 4  3)  (5)(3  1)  2(1  4)
2
1 STEP 3 : Eliminate the variables if any.
=  14  ( 20)  ( 6)
2
1 STEP 4 : Replace h by x and k by y in the result obtained
=  40 = 20 Sq. unit.
2 in step 3.
The equation obtained is the locus of the point which
Ex.15 Find the area of quadrilateral whose vertices, taken
moves under some stated condition(s).
in order, are A(–3, 2), B(5, 4), C (7, – 6) and D (–5, –4).
Sol. Area of quadrilateral = Area of ABC + Area of  ACD Ex.16 The sum of the squares of the distances of a moving
D C point from two fixed points (a, 0) and (–a, 0) is equal to
(–5,–4) (7,–6)
a constant quantity 2c2. Find the equation of its locus.
Sol. Let P (h, k) be any position of the moving point and let
A (a, 0), B (–a, 0) be the given points. Then
PA2 + PB2 = 2c2 (Given )
A B
(–3,2) (5,4)  (h – a)2 + (k – 0)2 + (h + a)2 + (k – 0)2 = 2c2
 h2 – 2ah + a2 + k2 + h2 + 2ah + a2 + k2 = 2c2
So, Area of ABC
 2h2 + 2k2 + 2a2 = 2c2
1  h2 + k 2 = c 2 – a2
= ( 3)( 4  6)  5( 6  2)  7( 2  4)
2 Hence, locus of (h, k) is x2 + y2 = c2 – a2.
1 1
=  30  40  14 =  84
2 2 Ex.17 Find the locus of a point, so that the join of (– 5, 1)
= 42 Sq. units and (3, 2) subtends a right angle at the moving point.
Sol. Let P (h, k) be a moving point and let A (– 5, 1) and
Area of  ACD
B (3, 2) be given points. By the given condition.
1
=  3(  6  4)  7( 4  2)  ( 5 )(2  6) APB = 90º
2
1  PAB is a right angled triangle
=  6  42  40
2  AB2 = AP2 + PB2
1
=  76  (3 + 5)2 + (2 – 1)2 = (h + 5)2 + (k – 1)2 + (h – 3)2 + (k – 2)2
2
= 38 Sq. units  65 = 2 (h2 + k2 + 2h –3k) + 39

So, Area of quadrilateral ABCD = 42 + 38 = 80 Sq.  h2 + k2 + 2h – 3k – 13 = 0

units. Hence, locus of (h, k) is x2 + y2 + 2x – 3y – 13 = 0.

PAGE # 89
Ex.18 What is the slope of a line whose inclination with the
positive direction of X-axis is :
A straight line is a curve such that every point on the (i) 0º (ii) 90º (iii) 120º (iv) 150º
line segment joining any two points on it lies on it. Sol. (i) Here,  = 0º
Slope = tan  = tan 0º = 0. [line is parallel to x –axis]
(a) Slope (Gradient) of a Line :
(ii) Here  = 90º
The trigonometrical tangent of the angle that a line
Slope = tan  = tan 90º =  .
makes with the positive direction of the x-axis in
 The slope of line is not defined.
anticlockwise sense is called the slope or gradient
[line is parallel to y – axis]
of the line.
(iii) Here = 120º
 Slope = tan  = tan 120º

= tan (180º – 60º) = – tan 60º = – 3


(iv) Here  = 150º
 Slope = tan  = tan 150º
1
= tan (180º – 30º) = – tan 30º = –.
3
Ex.19 Find the slope of the line passing through the
The slope of a line is generally denoted by m. Thus, points
m = tan . (i) (1, 6) and (– 4, 2) (ii) (5, 9) and (2, 9)
Since a line parallel to x-axis makes an angle of 0º Sol. (i) Let A (1, 6) and B (– 4, 2)
with x-axis, therefore its slope is tan 0º = 0. A line
26 4 4
parallel to y-axis i.e., perpendicular to x-axis makes  Slope of AB = = =
 4 1 5 5

an angle of 90º with x-axis, so its slope is tan = .  y  y1 
2  U sin g slope  2 
Also the slope of a line equally inclined with axis is  x 2  x 1 

1 or –1 as it makes 45º or 135º angle with x-axis. The
(ii) Let A (5, 9), B (2, 9)
angle of inclination of a line with the positive direction
of x-axis in anticlockwise sense always lies between 99 0
 Slope of AB = = = 0.
25 3
0º and 180º.

(b) Slope of a Line in Term of Coordinates


of any two Points on it :

Let P (x1, y1) and Q (x2, y2) be two points on a line (a) The Slope Intercept Form of a Line :

making an angle  with the positive direction of x-axis.

The equation of a line with slope m and making an


intercept c on Y-axis is y = mx + c.
QN y 2 – y1
In  PQN, tan = 
PN x 2 – x1 The equation of a line with slope m and making an

Thus, if (x1, y1) and (x2, y2) are coordinates of any two intercept d on x-axis is y = m(x – d).

points on a line, then its slope is Ex.20 Find the equation of a line with slope – 1 and cutting
off an intercept of 4 units on negative direction of y-axis
y 2 – y1 Difference of ordinates
m = x – x  Difference of abscissa . Sol. Here m = –1 and c = – 4. So, the equation of the line is
2 1

y = mx + c, y = – x – 4 or x + y + 4 = 0.

PAGE # 90
Ex.21 Find the equation of a line which cuts off intercept (d) The Intercept Form of a Line :
4 at x - axis and makes an angle 60º with positive
direction of the x-axis.
Sol. Slope m of the line = tan 60º = 3 a nd t he
x - intercept = 4.
Therefore, the equation of the line is y = 3 (x – 4).

(b) The Point-Slope Form of a Line :


The equation of a line which cuts off intercepts a and b
x y
respectively from the x and y-axis is   1.
a b
Ex.24 Find x-intercept & y-intercept of the line 2x – 3y + 5 = 0.
Sol. Here, a = 2, b = – 3, c = 5

c 5 5
x-intercept = – =– and y-intercept = .
a 2 3
(e) Perpendicular / Normal form :
The equation of a line which passes through the point x cos + y sin = p (where p > 0, 0  < 2 ) is the
(x1, y1) and has the slope ‘m’ is y – y1 = m(x – x1). equation of the straight line where the length of the
Ex.22 Find the equation of a line passing through (2, –3) perpendicular from the origin O on the line is p and
this perpendicular makes an angle  with positive
and inclined at an angle of 135º with the positive
xaxis. y
direction of x-axis. B
Sol. Here, m = slope of the line = tan 135º
= tan (90º + 45º) = – cot 45º = –1,
and x1 = 2, y1 = –3. Q

So, the equation of the line is y – y1 = m(x – x1) P


i.e. y – (–3) = –1(x – 2) or y + 3 = – x + 2 or x + y + 1 = 0. x' x
(c) The Two-Point Form of a Line : O A
y'

Ex.25 Find the equation of the line which is at a distance


3 from the origin and the perpendicular from the
origin to the line makes an angle of 30º with the
positive direction of the x-axis.
Sol. Here p = 3,  = 30º
 Equation of the line in the normal form is
x cos 30º + y sin 30º = 3
3 y
or x+ = 3 or 3 x + y = 6.
2 2
The equation of a line passing through two points
(f) Parametric Form :
 y 2  y1 
(x1, y1) and (x2, y2) is y – y1 =   (x – x ). P (r) = (x, y) = (x 1 + r cos , y1 + r sin )
 x 2  x1 
1

x  x1 y  y1
Ex.23 Find the equation of the line joining the points (– 1, 3) or  = r is the equation of the line in
cos  sin 
and (4, – 2). parametric form, where ‘r’ is the parameter whose
Sol. Here the two points are (x 1 , y 1 ) = (–1, 3) and ab s ol ute v alu e i s t he di s ta nc e of an y poi nt
(x2, y2) = (4, –2). (x, y) on the line from the fixed point (x 1, y1) on the
So, the equation of the line in two-point form is line.
(g) Equations of straight lines passing through A
3  ( 2 ) given point and making a given angle with a given
y–3= (x + 1)
 1 4 line
Equations of the straight lines which pass through
 y–3=–x–1
a given point (x1, y1) and make a given  with the
 x + y – 2 = 0.
given straight line y = mx + c are
m  tan 
y – y1 = 1  m tan  (x – x1)

PAGE # 91
Ex.26 Find the equation of the line through the point (b) Condition of Perpendicularity of Two
A (2, 3) and making an angle of 45º with the x-axis. Lines :
Also determine the length of intercept on it between If two lines y = m1x + c1 and y = m2x + c2 of slopes m1
A and the line x + y + 1 = 0. and m2 are perpendicular, then the angle  between
Sol. The equation of a line through A and making an them is of 90º.
angle of 45º with the x-axis is 1 m m
1 2
 cot  = 0  m – m = 0  m1m2 = –1
2 1
x2 y 3 x2 y3
= or = Thus when two lines are perpendicular, the product of
cos 45 º sin 45 º 1 1
their slopes is –1. If m is the slope of a line, then the
2 2
or x–y+1=0 slope of a line perpendicular to it is – (1/m).

Suppose this line meets the line x + y + 1 = 0 at Ex.28 A line passing through the points (a, 2a) and (– 2, 3)
P such that AP = r. Then the coordinates of P are is perpendicular to the line 4x + 3y + 5 = 0, find the
value of ‘a’.
x2 y 3
given by = =r Sol. Let m 1 be slope of the line joining A ( a, 2a) and
cos 45 º sin 45 º
2a  3
 x = 2 + r cos 45º, y = 3 + r sin 45º B (–2, 3). Then m1 = 
a2
r r Let m2 be slope of the line 4x + 3y + 5 = 0.
 x=2+ ,y=3+
2 2 4
Then, m2 =  
 r r  3
Thus, the coordinates of P are  2  ,3 
 Since the two lines are perpendicular, then
 2 2
Since P lies on x + y + 1 = 0, m1m2 = – 1.
r r 2a  3 4
so 2 + +3+ +1=0    1
2 2 a2 3
18
 2 r = – 6  r = –3 2  8a – 12 = 3a + 6 a = 
5
 length AP = | r | = 3 2
Thus, the length of the intercept = 3 2.
ANGLE BETWEEN TWO LINES
The general equation of a straight line is Ax + By + C = 0
The angle  between the lines having slopes m1 and which can be transformed to various standard forms
as discussed below :
m 2 – m1
m2 is given by tan  = 1  m m . (a) Transformation of Ax + By + C = 0 in the
1 2
Slope Intercept Form (y = mx + c) :
Ex.27 If A (– 2,1), B (2, 3) and C (– 2, – 4) are three points,
find the angle between BA and BC.  A  C
y =  – x   –  ,
Sol. Let m1 and m2 be the slopes of BA and BC respectively.  B  B
3 –1 2 1 This is of the form y = mx + c,
Then, m1 =  
2 – (–2) 4 2 A C
where m = – ,c= – .
–4 – 3 7 B B
and m2 =  Thus, for the straight line Ax + By + C = 0,
–2–2 4
A Coeff. of x
Let  be the angle between BA and BC. Then, m = slope = – = – ,
B Coeff. of y
7 1 10
– C Const. term
m 2 – m1 4 2  8 2
tan  =  and Intercept on y-axis = – – .
1  m1m 2 7 1 15 3 B Coeff . of y
1 
4 2 8 (b) Transformation of Ax + By + C = 0 in
2
  = tan–1   . x y 
3 Intercept Form    1 :
 a b 
(a) Condition of Parallelism of Lines :
We have, Ax + By + C = 0  Ax + By = –C
It two lines y = m1x + c1 and y = m2x + c2 of slopes m1 Ax By
and m2 are parallel then the angle  between them is   1
–C –C
of 0º.
x y
m 2 – m1    1.
 tan  = tan 0º = 0  =0  –C  –C
1  m1m 2    
 A   B 
 m2 = m1

PAGE # 92
(c) Transformation of Ax + By + C = 0 in the This is the normal form of the line. So,
normal form (x cos + ysin = p) : 3 1
We have Ax + By + C = 0 ....(i) cos  = – , sin  = – and p = 4.
2 2
Let x cos + y sin – p = 0 ....(ii) Since, sin  and cos  both are negative, therefore 
Equation (i) & (ii) represent the same straight line  7
C is in the third quadrant and is equal to  + = .
A B 6 6
 = =
cos  sin  p 7
Hence, for a given line  = and p = 4.
Bp  Ap
6
 sin = , cos =
C C

C A
 p = ± 2 2 , cos = , NOTE : More than one correct option may be possible.
A B A 2  B2
DISTANCE FORMULA
B
sin = 1. Find the point P(x, y), if its distance from (–3, 0) & (3, 0)
A 2  B2 is 4 units individually :
So, equation (ii) take the form
 
(A) 0, 5 
(B) 0, 5 
A B C
x+ y= (C) 0, 7  (D) (1, 0)
A B2 2
A  B2
2 A 2  B2 2. The point on the y - axis which is equidistant from
A (–5, –2) and B (3, 2) is :
This is the equation required normal form of the
(A) (– 4, 0) (B) (0, – 2)
Ax + By + C = 0. (C) (–2, 0) (D) (0, – 4)
Ex.29 Transform the equation of the line 3 x + y + 8 = 0 to 3. If two opposite vertices of a square are (5, 4) and
(i) slope intercept form and find its slope and (1, –6), then the coordinates of its remaining two
y - intercept (ii) intercept form and find intercepts on the vertices is :
coordinate axis (iii) normal form and find the inclina- (A) (–2, 2) & (5, 3) (B) (8, –3) & (–2, 1)
(C) (8, 6) & (3, 5) (D) (1, –3) & (2, 5)
tion of the perpendicular segment from the origin on
the line with the axis and its length. 4. If two vertices of an isosceles triangle are (2, 0) and
(2, 5) and length of the equal sides is 3, then the third
Sol. (i) 3 x+y+8=0 vertex is :
(A) ( 2, 6) & (– 5, 3) (B) ( 8, 3) & ( 5, 1)
 y=– 3 x–8 11 5 14 7
(C) (2 ± , ) (D) (3 ± , )
This is the slope intercept form of the given line. 2 2 2 2
5. If the point (0, 2) is equidistant from the points (3, k)
 Slope = – 3 , y - intercept = – 8. and (k, 5), then the value of k is :
(A) 0 (B) 2
(ii) 3 x+y+8=0 (C) – 2 (D) None of these
6. If the distance between the points (a, 2) and (3, 4) be
 3 x+y =–8 8 then a =

x y (A) 2  3 15 (B) 2  3 15
  1
8/ 3 8 (C) 2  3 15 (D) 3  2 15

8 7. Value of ‘a’, when the distance between the points


So, x - intercept =  and y - intercept = – 8. (3, a) and (4, 1) is 10 is :
3
(A) 4 or – 2 (B) 2 or 4
(iii) 3 x+y+8=0 (C) 6 or 2 (D) None

 – 3 x– y=8 8. In the rectangle shown, the value of a – b is :


Y
3 1 8
 – x – = C (a,13)
2 2 2 2 2
 3 1  3 12  3 1 B (15, b)
(5,5)D
3 1 8
 – x – y= A (9,2)
2 2 2

O X
3 1
 – x – y=4
2 2 (A) – 3 (B) – 1
(C) 3 (D) 1

PAGE # 93
9. In the diagram, PQR is an isoscles triangle and 18. If the three vertices of a parallelogram are (a + b, a – b),
QR = 5 units. (2a + b, 2a – b) and (a – b, a + b), then the fourth vertex
Y
is :
Q (A) (– a, a) (B) (– a, – a)
(C) (– b, – b) (D) None

19. If (3, – 4) and (– 6, 5) are the extremities of the diagonal


X of a parallelogram and (– 2, 1) is its third vertex, then
P(–2,0) O R(4,0)
its fourth vertex is :
The coordinates of Q are :
(A) (– 1, 0) (B) (0, – 1)
(A) (1,5) (B) (3,4)
(C) (2,4) (D) (1,4) (C) (– 1, 1) (D) None of these.

10. The points A(–4, –1), B (–2, –4), C (4, 0) and D(2, 3) 20. If the mid point of the line-segment joining the points
are the vertices of : (– 7, 14) and (K, 4) is (a, b), where 2a + 3b = 5, then the
(A) Parallelogram (B) Rectangle value of K is :
(C) Rhombus (D) None of these (A) K = 15 (B) K = – 7

11. The three points (– 2, 2), (8, – 2) and (– 4, –3) are the
(C) K = – 15 (D) K = 10
vertices of :
(A) An isosceles triangle 21. The points which trisect the line segment joining
(B) An equilateral triangle the points (0, 0) and (9, 12) are :
(C) A right angled triangle (A) (3, 4) (B) (8, 6)
(D) None of these (C) (6, 8) (D) (4, 0)
SECTION FORMULA
22. Value of m for which the point P (m, 6) divides the join
12. If the y – axis divides the line joined by (2, 4) & (–3, 5), of A (– 4, 3) and B (2, 8) is :
then the ratio is :
3
(A) 2 : 3 (B) 2 : 5 (A) 5 (B)
2
(C) 3 : 2 (D) 5 : 2
2
13. The ratio in which the line segment joining (3, 4) and (C)  (D) None
5
(– 2, – 1) is divided by the x-axis is :
(A) 3 : 2 (B) 4 : 1 23. A (1,1) and B (2, –3) are two points and D is a point on
(C) 4 : 3 (D) None of these AB produced such that AD = 3 AB. Then the

14. The line segment joining the points (3, – 4) and (1, 2) co–ordinates of D is :
is trisected at the points P and Q. If the coordinates of (A) (4, 11) (B) ( 4, – 11)
P and Q are (p, – 2) and (5/3, q) respectively, then the (C) (– 2, 5) (D) ( – 4, – 11)
value of p and q is :
24. Four points are on a line segment. If AB : BC = 1 : 2 and
(A) p = 7/3, q = – 3 (B) p = 7/3, q = 0
(C) p = 5/3, q = 0 (D) None of these BC : CD = 8 : 5, then AB : BD equals :
(A) 4 : 13 (B) 1 : 13
15. The coordinates of one end of a diameter of a
(C) 1 : 7 (D) 3 : 13
circle are (5 ,  7) . If the coordinates of the centre
be (7 , 3) , the coordinates of the other end of the 25. If the line joining A (2, 3) and B(– 5, 7) is cut by x-axis at
diameter are : P, then AP : PB is :
(A) (6 ,  2) (B) (9 , 13) (A) 3 : 7 (B) – 3 : 7
(C) ( 2 , 6) (D) (13 , 9) (C) 7 : 3 (D) 7 : – 3
16. The point (11 , 10) divides the line segment joining
26. If P (1, 2), Q (4, 6), R (5, 7) and S (a, b) are the vertices
the points (5 ,  2) and (9 , 6) in the ratio :
of a parallelogram PQRS, then :
(A) 1 : 3 internally (B) 1 : 3 externally
(C) 3 : 1 internally (D) 3 : 1 externally (A) a = 2, b = 4 (B) a = 3, b = 4
(C) a = 2, b = 3 (D) a = 3, b = 5
17. If A & B are the points ( 3, 4) and (2, 1), then the
coordinates of the point C on produced AB such 27. If A (2, 2), B (–4, –4), C( 5, –8) are the vertices of any
that AC = 2 BC are : triangle, the length of median passes through C will be :
(A) (2, 4) (B) (3, 7) (A) 65 (B) 117
 1 5 (C) 85 (D) 113
(C) (7, 2) (D)   , 
 2 2

PAGE # 94
CENTROID, CIRCUMCENTRE, AREA OF TRIANGLE AND QUARDILATRAL
ORTHOCENTRE INCENTRE AND EXCENTRE
36. The points which are not collinear are :
28. If the middle points of the sides of a triangle be (–2, 3), (A) (0, 1), (8, 3) and (6, 7)
(4, – 3) and (4, 5), then centroid of triangle is : (B) (4, 3), (5, 1) and (1, 9)
(C) (2, 5), (–1, 2) and (4, 7)
5  5 
(A)  , 2  (B)  ,1 (D) (–3, 2), (1, – 2) and (9, – 10)
3  6 
37. Value of p, for which the points (– 5, 1), (1, p) and
 5  5 (4, – 2) are collinear is :
(C) 1,  (D)  2, 
 6  3 (A) 0 (B) 2
29. If the vertices of a triangle be (2, 1), (5, 2) and (3, 4) then (C) – 1 (D) None of these
its circumcenter is :
38. If the points A (6, 1), B (8, 2), C (9, 4) and D ( P, 3) are
 13 9   13 9  the vertices of a parallelogram taken in order. Then the
(A)  ,  (B)  , 
 2 2  4 4 value of P is :
 9 13  (A) 7 (B) 8
(C)  ,  (D) None (C) 4 (D) 9
4 4 
30. If the coordinates of the centroid of a triangle are 39. The area of the triangle whose vertices are (a,a),
(1, 3) and two of its vertices are (– 7, 6) and (8, 5), then (a + 1, a + 1) and (a +2, a) is :
the third vertex of the triangle is : (A) a3 (B) 1
(C) 2a (D) 21/2
 2 14   2 14 
(A)  ,  (B)   ,  40. Area of the triangle with vertices P(5, 2), Q(– 9, –3)
3 3   3 3 
and R(–3, –5) (in sq. units) is :
(C) (2, – 2) (D) (– 2, 2) (A) 58 (B) 14
31. The orthocentre of the triangle ABC is 'B' and the (C) 29 (D) None
circumcentre is 'S' (a, b). If A is the origin then the 41. Area of the quadrilateral (in sq. units) with vertices
coordinates of C are : A(–1, 6), B(–3, –9), C(5, –8) and D(3, 9) is :
a b (A) 48 (B) 96
(A) (2a, 2b) (B)  ,  (C) 24 (D) None of these
2 2
42. If the points (k, 2 – 2k), (1 – k, 2k) and (–k –4, 6 – 2k)
 2 2 
(C)  a b , 0  (D) None be collinear, the possible values of k are :
 
32. The incentre of the triangle formed by (0, 0), (5, 12), 1 1
(A) – (B)
2 2
(16, 12) is :
(A) (7, 9) (B) (9, 7) (C) 1 (D) –1
(C) (–9, 7) (D) (–7, 9) 43. The area of a triangle is 5. Two of its vertices are
(2, 1) and (3, –2). If the third vertex lies on y = x + 3., then
33. If the centroid and circumcenter of a triangle are
the third vertex is :
(3, 3) and (6, 2) respectively, then the orthocentre is :
(A) (–3, 5) (B) (–3, 1)  7 13  3 3
(A)  ,  or  , 
(C) (3, –1) (D) (9, 5)  2 2   2 2

34. The incentre of the triangle with vertices (1, 3 ), (0, 0)  7 15  3 3


(B)   ,  or  , 
and (2, 0) is :  2 2  2 2
 3  2 1 
(A) 1, 2  (B)  3 ,
 
  9 21  3 3
   3 (C)   ,  or  , 
 2 2  2 2
2 3   1 
(C)  3 , 2  (D) 1,
   (D) None of these

   3
35. Orthocentre of triangle with vertices (0, 0), (3, 4) and 44. The points (– a, – b), (0, 0), (a, b) and (a2, ab) are :
(A) Collinear
(4, 0) is :
(B) Vertices of a parallelogram
 3 (C) Vertices of a rectangle
(A)   3,  (B) (3, 12)
 4 (D) None of these.
 3 45. The area of a triangle whose vertices are (a, c + a),
(C)  3,  (D) (3, 9)
 4 (a, c) and (– a, c – a) are :
(A) a2 (B) b2
2
(C) c (D) a2 + c2

PAGE # 95
55. Find the equation of the line which cuts off an intercept
46. If four points A (6, 3), B (–3, 5), C (4, – 2) and D (x, 3x) are
4 on the positive direction of x-axis and an intercept
Area(DBC) 1 3 on the negative direction of y-axis .
given in such a way that  , then the (A) x + 3y + 3 = 0 (B) 3x – 4y = 12
Area(ABC) 2
(C) 4x +3 y + 1 = 0 (D) None
value of x is :
56. If the straight line, 2x + 3ay  1 = 0 & 3x + 4y + 1= 0 are
3 14 mutually perpendicular, then the value of 'a' will be :
(A) or (B) 2 or – 3 (A) 8/9 (B) – 1/2
8 8
(C) – 8/9 (D) 1/2
11 3 57. If the length of the perpendicular from the origin to a line
(C) or . (D) None of these.
8 8 is 7 and the line makes an angle of 150o with the positive
47. Find the value for "x", so that the three points, (2, 7), direction of y-axis, then the equation of the line is :
(6, 1), (x, 0) are collinear. (A) x + 3 y – 14 = 0 (B) x – y – 3 =0
1
(A) 7 (B) 4 (C) y – 3 x – 14 = 0 (D) 3 x + 3 y+4=0
2
2 58. Equation of the straight line that has y-intercept 4 and
(C) 10 (D) 6
3 is parallel to the straight line 2x – 3y = 7 is :
48. If the co-ordinates of two points A and B are (3, 4) and (A) 2x + 3y + 12 = 0 (B) 2x – 3y + 16 = 0
(C) 2x – 3y + 12 = 0 (D) 2x – 3y – 12 = 0
(5, –2) respectively, then the co-ordinates of any point
P if PA = PB and Area of PAB = 10 is : 59. Two opposite vertices of a rectangle are (1, 3) and
(A) (7, 2) or (1, 0) (B) (– 7, 2) or (3, 0) (5, 1). If the other two vertices of the rectangle lie on the
line y – x +  0,then =
(C) (7, – 2) or (5, 0) (D) (7, –2) or (–1, 0)
(A) – 1 (B) 2
STRAIGHT LINE (C) 1 (D) None of these

49. If the point (2, –3) lies on kx2 – 3y2 + 2x + y – 2 = 0, then 60. A straight line through P (1, 2) is such that its
k is equal to : intercept between the axis is bisected at P. Its
equation is :
1 (A) x + 2y = 5 (B) x – y + 1 = 0
(A) (B) 16
7 (C) x + y – 3 = 0 (D) 2x + y – 4 = 0
(C) 7 (D) 12
61. A line L is perpendicular to the line 5x – y = 1 and the
50. Given the points A (0, 4) and B (0,  4), the equation of the area of the triangle formed by the line L and
locus of the point P (x, y) such that AP  BP = 6 is : coordinate axes is 5. The equation of the line L is :
(A) x + 5y = 5 (B) x + 5y = ± 5 2
(A) 9x2  7y2 + 63 = 0 (B) 9x2  7y2  63 = 0
(C) 7x2  9y2 + 63 = 0 (D) 7x2  9y2  63 = 0 (C) x – 5y = 5 (D) x – 5y = 5 2

51. Equation to the locus of a point equidistant from the 62. On e s ide o f a re c ta ngl e lie s a lon g t he li ne
4x + 7y + 5 = 0. Two of its vertices are (–3, 1) and
points A (1, 3) and B (– 2, 1) is :
(1, 1). Then the equations of other sides are :
(A) 4x + 6y = 5 (B ) 6x + 4y = 5
(A) 7x – 4y + 25 = 0 (B) 7x + 4y + 25 = 0
(C) – 4x + 6y = 8 (D) None (C) 7x – 4y – 3 = 0 (D) 4x + 7y = 11
52. Locus of a point such that the sum of its distance from 63. The points on the line x + y = 4 which lie at a unit
the points (0, 2) and (0, – 2) is 6 : distance from the line 4x + 3y = 10, are :
(A) 4x2 + 6y2 = 5 (B ) 5x2 + 9y2 = 50 (A) (3, 1) (B) (7, 11)
2 2
(C) 9x + 5y = 45 (D) None (C) (–7, 11) (D) (1, 3)

53. Equation of a line with slope – 6 and cutting off an 64. If PS is the median of the triangle with vertices P (2, 2),
intercept of 7 units on negative direction of y-axis is : Q (6, –1) and R (7, 3). The equation of the line passing
(A) 6x – 6y – 4 = 0 (B) 6x – y – 7 = 0 through (1, –1) and parallel to PS is :
(C) 6x – y – 4 = 0 (D) 6x + y + 7 = 0 (A) 2x – 9y – 7 = 0 (B) 2x – 9y – 11 = 0
(C) 2x + 9y – 11 = 0 (D) 2x + 9y + 7 = 0
54. Equation of a line passing through (3, 2) and inclined
65. A line passes through (2, 2) and is perpendicular to
at an angle of 120º with the positive direction of
the line 3x + y = 3 . Its y  intercept is :
x-axis is :
(A) 1/3 (B) 2/3
(A) 3 x + y = 2 + 3 3 (B) 2x – 3y – 4 = 0 (C) 1 (D) 4/3

(C) 3 x – y = 2 – 3 3 (D) 3x – 2y + 4 = 0 66. The point (7,2) and (–1,0) lie on a line 34:
(A) 7y = 3x – 7 (B) 4y = x +1
(C) y = 7x + 7 (D) x = 4y + 1

PAGE # 96
67. Which of the following pair(s) of straight lines are 2. The centre of a clock is taken as origin. At 4.30 pm, the
mutually perpendicular ? equation of line along minute hand is x = 0. Therefore,
(A) 3x  4y  12 = 0 and 4x + 3y  6 = 0 at this instant the equation of line along the hour hand
(B) 4x + 5y  8 = 0 and 3x  2y + 4 = 0 will be [IJSO-2011]
(A) x – y = 0 (B) x + y = 0
x  x1 y  y1 xx 2 yy 2
(C) = & = 2
2 3 3 x
x  x1 y  y1 xx 2 yy 2 (C) y = 2x (D) y =
(D) = 4 & = 2
5 4 5
3. Find the equation of the line parallel to 4x + 3y = 5 and
68. Write the equation of the perpendicular bisector of the having x-intercept (-3) [IJSO-2012]
line joining the points (2, 1) and (4, 3) is : (A) 3x + 4y + 12 = 0 (B) 3x + 4y = 12
(A) x + y = 7 (B) x + y = 5 (C) 4x + 3y –12 = 0 (D) 4x + 3y + 12 = 0
(C) x = 5 (D) None
4. In the xy-plane let A be the point (5,0) and L be the line
x
y= . The number of points Pon the line L such that
3
COMPETITIVE EXAM PREVIOUS YEAR triangle OAP is isosceles is (O being the origin)
QUESTION [IJSO-2013]
1. Consider the point A(a, b+c), B(b, c+a) and C(c, a + b ). (A) 2 (B) 3
The area of ABC is [IJSO-2011] (C) 4 (D) 5
2 2 2
a b c
(A) 2(a2 + b2 + c2) (B) 5. If A(p, q + r), B(q, r + p) and C(r, p + q) are points then
6 area of triangle ABC [IJSO-2015]
(C) 2(ab + bc + ca) (D) none of these (A) p2+q2+r2 (B)(p + q + r)2
1
(C) (pq + qr + rp) (D) zero
2



PAGE # 97
SETS

(b) Set Builder Method :

In this method, a set is described by a characterizing


A well defined collection of objects is known as sets.
property P(x) of its elements x. In such a case the set
If a is an element of a set A, then we write a  A and
is described by {x : P(x) holds } or, {x | P(x) holds,}
say a belongs to A. If a does not belong to A, then a  A
which is read as ‘the set of all x such that P(x) holds’.
is written.
The symbol ‘|’ or ‘:’ is read as ‘such that’.
For example : The collection of all states in the Indian
Ex. 5 Write the set A = {0,1,4,9,16,........} in set builder form.
union is a set but collection of good cricket players of
Sol. A = {x2 : x  Z).
India is not a set, since the term “good player is vague
and it is not well defined. 1 1 1 1
Ex.6 Write the set X = {1, , , , , ........} in the set
4 9 16 25
Some letters are reserved for the sets as listed builder form.
below :
Sol. W e observe that the elements of set X are the
N : For the set of Natural numbers. reciprocals of the squares of all natural numbers. So,

Z : For the set of Integers.  1 


the set X in set builder form is X =  2 ; nN .
 n 
Z+ :For the set of all positive Integers.

Q : For the set of all Rational numbers. TYPES OF SETS

Q+ : For the set of all positive Rational numbers.


(a) Empty Set :
R : For the set of all Real numbers.
A set is said to be empty or null or void set if it has no
R+ : For the set of all Positive real numbers. element and it is denoted by or { }.
C : For the set of all Complex numbers.
Ex. 7 Write {x  N : 5 < x < 6} in roster form.
Sol. A = { }.

(b) Singleton Set :


A set is often described in the following two ways :
A set consisting of a single element is called a
(a) Roster Method :
singleton set.
In this method a set is described by listing elements,
separated by commas, within braces { }. Ex. 8 Write the set {x : x  N and x2 = 9} in roster form.
Sol. Let B is the set. So B is a singleton set equal to {3}.
Ex.1 Write the set of vowels of English alphabet in roster
form. (c) Finite Set :
Sol. A = {a, e, i, o, u}.
A set is called a finite set if it is either void set or its
Ex.2 Write the set of even natural numbers in roster form. element can be listed (counted labelled) by natural
Sol. B = {2,4,6,.....}. numbers 1, 2, 3 ....... and the process of listing
Ex.3 Write the set of all prime numbers less than 11 in terminates at a certain natural number n (say).
roster form. For example : Set of all persons on the earth is a finite
Sol. C = {2,3,5,7}. set.

Ex.4 Write the set A = {x  z, x2 < 20} in the roster from. (d) Infinite Set :
Sol. We observe that the squares of integers 0, ± 1, ± 2, ± 3,
A set whose elements cannot be listed by natural
± 4 are less than 20. Therefore, the set A in roster form
numbers 1, 2, 3,...... for any natural number n is called
i s A = { – 4, – 3, – 2, –1, 0, 1, 2, 3, 4}.
an infinite set.
 NOTE :
For example : Set of all points in a plane is an infinite
The order in which the element are written in a set
set.
makes no difference.

PAGE # 98
Ex. 9 Which of the following sets are finite and which are (ii) The empty set is a subset of every set.
infinite ?
(iii) The total number of subsets of a finite set
(a) Set of concentric circle in a plane.
containing n element is 2n.
(b) Set of letters of English alphabets.
(iv) The total number of proper subsets of a finite set
(c) { x  N, x > 5 } containing n element is 2n–1.
(d) { x  R, 0 < x < 1 }
(i) Universal Set :
(e) { x  N, x < 200}
Sol. (a) Infinite set (b) Finite set (c) Infinite set A set that contains all sets in a given context is called
(d) Infinite set (e) Finite set the Universal Set.

(e) Cardinal Number of a Finite Set : Ex.11 If A = {1,2,3}, B = {2,4,5,6} and C = {1,3,5,7}, then find
the universal set.
The number n in the above definition is called the
Sol. U = {1, 2, 3, 4, 5, 6, 7} can be taken as the universal
cardinal number or order of a finite set A and is denoted
set.
by n(A).
( j) Power Set :
(f) Equivalents Set :
Let A be a set. Then the collection or family of all
Two finite sets A and B are equivalent if their cardinal
subsets of A is called the power set of A and is denoted
numbers are same. i.e. n(A) = n(B).
by P(A).
For example : A = {1,2,3} and B = {a,b,c} are equivalent
sets. Ex.12 Let A = {1,2,3}. Then find the power set of A.
Sol. Subset of A are : , {1}, {2}, {3}, {1,2}, {1,3}, {2,3} and
(g) Equal Set :
{1,2,3}.
Two sets A and B are said to be equal if every element Hence, total number of subset are = 23 = 8.
of A is a member of B, and every element of B is Hence, P(A) ={ {1}, {2}, {3}, {1,2}, {1,3}, {2,3}, {1,2,3} }.
member of A.
Ex.13 Let A = {1,2,3,4}, B = {1,2,3} and C = {2,4}. Find sets X
 NOTE : satisfying each pair of conditions.
Equal sets are equivalents but equivalent sets need (i) X  B and X  C
not be equal. (ii) X  B, X  B and X  C
Ex. 10 Are the following sets equal ? (iii) X  A, X  B and X  C
A = { x : x is a letter in the word reap } Sol. We have
B = { x : x is a letter in the word paper }. (i) X  B and X  C
Sol. A = { r, e, a, p}  X is subset of B but X is not a subset of C
B = { p, a, e, r }  X  P (B) but X  P (C)
So, A and B are equal sets.  X = {1}, {3}, {1,2}, {1,3}, {2,3}, {1,2,3}.

(h) Subset : (ii) We have,

Let A and B be two sets. If every element of A is an X  B, X  B and X  C


element of B, then A is called a subset of B. If A is a  X is a subset of B other than B itself and X is not a
subset of B, we write A  B, which is read as “A is a subset of C
subset of B” or “A is contained in B”. Thus,  X  P(B), X  P(C) and X  B
A  B if a  A  a  B. The symbol “” stands for  X = {1}, {3}, {1,2}, {1,3}, {2,3}.
“implies”. If A is not a subset of B, we write (iii) We have,
A  B. X  A, X  B and X  C

 NOTE :  X  P(A), X  P(B) and X  X (C)


The subset consisting of all elements of a given set is  X is a subset of A, B and C
called improper subset. Every set is a subset of itself  X = , {2}
and it is consider to be an improper subset . A subsets
A of a set B is called a proper subset of B if A  B and
we write A  B. So empty set () is consider as a proper Diagram drawn to represent sets are called Venn-Euler
subset. diagram or simply Venn diagram. In Venn-diagram
the universal set U is represented by points within a
SOME RESULTS ON SUBSET :
rectangle and its subsets are represented by points in
(i) Every set is a subset of itself closed curves (usually circles) within the rectangle.

PAGE # 99
Ex.16 If A = { 2, 3, 4, 5, 6, 7 } and B = { 3, 5, 7, 9, 11, 13 }, then
find A – B and B – A.
(a) Union of Sets : Sol. A – B = { 2, 4, 6 } and B – A = { 9, 11, 13 }.
Let A and B be two sets. The union of A and B is the set
(e) Symmetric Difference of Two Sets :
of all those elements which belong either to A or to B
or to both A and B. Let A and B be two sets. The symmetric difference of
Thus, A B = { x : x  A or x  B}. sets A and B is the set (A – B)  (B – A) and is denoted
by A  B.

Ex.14 If A {1, 2, 3} and B = {1, 3, 5, 7}, then find A  B. Ex.17 If A = { x  R : 0 < x < 3 }, B = { x  R : 1  x  5 }, then
Sol. A  B = {1, 2, 3, 5, 7}.
find A  B.
(b) Intersection of Sets : Sol. A – B = { x  R : 0 < x < 1 }, B – A = { x  R : 3  x  5 }
Let A and B be two sets. The intersection of A and B is AB= {xR:0<x<1}{xR:3  x  5}
the set of all those elements that belong to both A and B.
A  B = { x  R : 0 < x < 1 or 3  x  5 }.

(f) Complement of a Set :

Let U be the universal set and let A be a set such that


A  U. Then, the complement of A with respect to U is
denoted by A’ or Ac or U– A and is defined the set of all
Thus, A  B = {x : x  A and x  B}. those elements of U which are not in A.
Ex.15 If A = { 1, 2, 3, 4, 5 } and B = { 1, 3, 9, 12 }, then Thus, A’ = {x  U : x  A}.
find A 
Sol. A = { 1, 3 }.
(c) Disjoint Sets :
Two sets A and B are said to be disjoint, if A B = .
If A  B  , then A and B are said to be intersecting or
overlapping sets. Clearly, x  A’  x  A.
(d) Difference of Sets :
Ex.18 Let U = { 1, 2, 3, 4, 5, 6, 7, 8, 9 }, A = { 1, 2, 3, 4 },
Let A and B be two sets. The difference of A and B, B = { 2, 4, 6, 8 } and C = { 3, 4, 5, 6 }. Find
written as A – B, is the set of all those elements of
(i) Ac (ii) ( A  C )c (iii) ( A  B )c (iv) (B – C)c
A which do not belong to B. c
Sol. (i) A = U – A = { 1, 2, 3, 4, 5, 6, 7, 8, 9 } – { 1, 2, 3, 4 }
= { 5, 6, 7, 8, 9 }.
u c
(ii) ( A  C ) = U – ( A  C )
= { 1, 2, 3, 4, 5, 6, 7, 8, 9 } – { 3, 4 }
A–B
= { 1, 2, 5, 6, 7, 8, 9 }.
A B (iii) ( A  B )c = U – ( A  B )
= { 1, 2, 3, 4, 5, 6, 7, 8, 9 } – { 1, 2, 3, 4, 6, 8 } = { 5, 7, 9 }.
Thus, A – B = {x : x  A and x  B}
(iv) (B – C)c = U – ( B – C )
or, A – B = {x  A : x  B}.
Clearly, x A – B  x  A and x  B. = { 1, 2, 3, 4, 5, 6, 7, 8, 9 } – { 2, 8 } = { 1, 3, 4, 5, 6, 7, 9 }.
Similarly, the difference B – A is the set of all
those elements of B that do not belong to A LAWS OF ALGEBRA OF SETS
i.e. B – A = {x  B : x  A}.
Some fundamental laws of algebra of sets are :
u (a) Idempotent Laws :

For any set A, we have (i) A  A = A (ii) A  A = A.


Proof : (i) A  A = {x : x  A or x  A} = {x : x  A} = A
A B (ii) A A = {x : x  A and x  A} = {x : x  A} = A.

PAGE # 100
(b) Identity Laws : Proof : (i) Let x be an arbitrary element of A ( B C).
Then,
For any set A, we have (i) A   = A (ii) A  U = A. x A ( B C)
i.e.  and U are identity elements for union and  x  A or x  (B  C)
intersection respectively.  x  A or (x  B and x  C)
Proof : (i) A   = {x : x  A or x  } = {x : x  A} = A  (x  A or x  B) and (x  A or x  C)
[ ‘or’ is distributive over ‘and’]
(ii) A  U = {x : x  A and x  U } = {x : x  A} = A
 x  (A  B ) and x  (A C)
(c) Commutative Laws :  x  ((A  B)  (A  C))
 A  (B  C)  (A  B) (A C).
For any two set A and B, we have
Similarly, (A B) (A  C)  A (B C).
(i) A  B = B  A (ii) A  B = B  A. Hence, A (B C) = (A B) (A C).
i.e. union and intersection are commutative.
(ii) Let x be an arbitrary element of A  (B  C). Then,
Proof : Two set X and Y are equal iff X  Y and Y  X. x A ( B C)
Also, X  Y if every element of X belongs to Y.  x  A and x  (B  C)
(i) Let x be an arbitrary element of A  B. Then  x  A and (x  B or x  C)
x  A  B  x  A or x  B  x  B or x  A  x  B  A  (x  A and x  B) or (x  A and x  C)
 A  B  B  A.  x  (A  B ) or x  (A C)
 x  (A B)  (A  C)
Similarly, B  A  A  B.
 A  (B  C)  (A  B) (A C).
Hence, A  B = B  A. Similarly, (A B) (A  C)  A (B C).
(d) Associative Laws : Hence, A (B C) = (A B) (A C).
(f) De - Morgans Laws :
If A, B and C are any three sets, then
(i) (A  B)  C = A  (B  C) If A and B are any two sets, then
(ii) A  (B  C) = (A  B)  C) (i) (A  B)’ = A’  B’ (ii) (A  B)’ = A’  B’.
i.e. union and intersection are associative.
Proof : (i) Let x be an arbitrary element of (A  B)’.
Proof : (i) Let x be an arbitrary element of (A  B)  C. Then,
Then, x  (A  B)  C x (A B)’
 x  (A  B) or x  C  x  (A B)
 (x  A or x  B) or x  C  x  A and x B
 x  A’ and x  B’
 x  A or (x  B or x  C)
 x  A’  B’.
 x  A or (x  B C)
 (A  B)’  A’  B’.
 x  A (B C)
Again, let y be an arbitrary element of A’  B’. Then,
 (A  B)  C  A  (B  C).
y  A’  B’
Similarly, A (B C )  (A B ) C.  y  A’ and y  B’
Hence, (A B) C = A (B C).  y  A and y  B
 y  A B.
(ii) Let x be an arbitrary element of A  (B  C). Then,
 y  (A  B)’.
x  (A  B)  C
 A’  B’ (A  B)’.
 x  A and x  (B  C) Hence, (A  B)’ = A’  B’
 x  A and (x  B and x  C)
(ii) Let x be an arbitrary element of (A  B)’. Then,
 (x  A and x  B) and x  C x (A B)’
 x  (A  B ) and x  C  x  (A B)
 x  (A  B)  C  x  A or x B
 A  (B  C)  (A  B) C.  x  A’ or x  B’
Similarly, (A B) C  A (B C).  x  A’  B’.
 (A  B)’  A’  B’.
Hence, A (B C) = (A B) C.
Again, let y be an arbitrary element of A’  B’. Then,
(e) Distributive Laws : y  (A’  B’)
If A, B & C are any three sets, then  y  A’ or y  B’
 y  A or y  B
(i) A (B C) = (A B) (A C)
 y  (A B).
(ii) A (B C) = (A B) (A C).
 y  (A  B)’.
i.e. union and intersection are distributive over  A’  B’ (A  B)’. Hence, (A  B)’ = A’  B’.
intersection and union respectively.

PAGE # 101
Ex.19 Let A and B be sets, If A  X = B  X =  and Ex.22 There are 40 students in a chemistry class and 60
A  X = B  X for some set X, prove that A = B. students in a physics class. Find the number of
Sol. A  X = B  X for some set X students which are either in physics class or chemistry

 A A  X) = A  B  X ) class in the following cases:


(i) the two classes meet at the same hour.
 (A A)  (A X) = A B  X)
(ii) the two classes meet at different hours and 20
 A = ( A B )  ( A X )
students are enrolled in both the subjects.
 A = ( A B )   [  A  X =  (given) ]
Sol. Let A be the set of students in chemistry class and B be
 A = A B
the set of students in physics class.
 A B .....(i)
It is given that n (A) = 40 and n(B) = 60.
Again, A  X = B  X (i) If two classes meet at the same hour, then there will
 B A  X) = B  B  X ) not be a common student sitting in both the classes.
 ( B A )  ( B X ) = (B B)  (B X) Therefore, n (A  B) = 0
 ( B A )  ( B X ) = B  n (A  B) = n(A) + n(B) – n(A  B)
 ( B A )   = B [  B  X =  (given) ]  n (A  B) = 40 + 60 – 0 = 100
 B A = B (ii) If two classes meet at different timings then there
 B A .....(ii) can be some student sitting in both the classes.

From (i) and (ii) A = B. Therefore, n (A  B) = 20


 n (A  B)= n(A) + n(B) – n(A  B)
Ex.20 Using properties of sets, show that for any two sets = 40 + 60 – 20 = 80.
A and B, ( A  B ) ( A  B’ ) = A.
Sol. ( A  B ) ( A  B’ ) = (( A  B )A)  (( A  B ) B’ )
= A  (( A  B ) B’ ) NOTE : More than one correct option may be possible.
= A  (( A  B’ ) ( B  B’ )) REPRESENTATION OF SETS, TYPES OF
= A  ( A  B’ ) = A . SETS, SUBSETS
1. If A = {x  N | x2  36}, then A is equal to :
MORE RESULTS ON OPERATIONS ON SETS (A) {1, 2, 3, 4, 5, 6} (B) {6}
(C) {0, 1, 3, 4, 5, 6}
If A, B and C are finite sets and U be the finite universal (D) {– 6, – 5.... – 1, 0, 1, 2,...6}
set, then
2. The number of subsets of an empty set is :
(i) A – B = A B’ = A – (AB) (A) 0 (B) 1
(ii) A – B = A AB =  (C) 2 (D) 3
(iii) (A – B) ( B – A ) = ( A  B ) – (AB) 3. Which of the following sets are disjoint ?
(vi) n (A  B) = n(A) + n(B) – n (A  B) (A) The set of natural numbers and set of integers.
(vii) n (A  B) = n(A) + n(B)  A, B are disjoint non-void (B) The set of natural numbers divisible by 3 and set of
sets. natural numbers divisible by 7.
(C) The set of all multiples of 3 and set of all multiples
(viii) n (A – B) = n(A) – n(A  B).
of 5.
Ex. 21 In a group of 800 people, 550 can speak Hindi and (D) The set of odd natural numbers and set of even
natural numbers.
450 can speak English. How many can speak both
Hindi and English ? 4. How many subsets can be formed from the set {p, q, r}
Sol. Let H denote the set of people speaking Hindi and is ?
(A) 6 (B) 7
E denote the set of people speaking English.
(C) 8 (D) 9
n(H) = 550, n(E) = 450 and n(H  E) = 800.
5. Which one of the following statements is correct ?
n(H  E) = n(H) + n(E) – n (H  E)
(A) Finite sets are equal
 n(H  E) = n(H) + n(E) – n(H  E) (B) Two sets are disjoint, if there are no common
 n(H  E) = 550 + 450 – 800 = 200. elements in both the sets
Hence, 200 people can speak both Hindi and English. (C) The set of integers is also called the set of natural
number.
(D) A set having no element is called infinite set.

PAGE # 102
6. If A is a void set then n[P(P(P(A)))] is : 19. The set of all those elements of A and B which are
(A) 0 (B) 1 common to both is called :
(C) 3 (D) 4 (A) Union of two sets
(B) Intersection of two sets
7. Given K = { B, A, N, T, I }. Then the number of subsets of
(C) Disjoint sets
K, that contain both A, N is :
(D) None of these.
(A) 8 (B) 16
(C) 24 (D) 32 THEOREM ON CARDINAL NUMBER

8. A = { 1, 2, {3, 4}}, number of proper subsets of A is : 20. In a group of 500 people, 200 can speak Hindi alone
(A) 16 (B) 15 while only 125 can speak English alone. The number
(C) 3 (D) 7 of people who can speak both Hindi and
English is :
9. If A be a finite set having n elements and P(A) is its
(A) 175 (B) 325
power set, then total number of elements of P (P(A)) is:
(C) 300 (D) 375
(A) 2n (B) 4n
n 21. A small town has a population 8000 out of which 3500
(C) 22 (D) 22n
people read ‘Times of India’ and 3000 people read
10. A = {1, 2, {3, 4}, {5}}. Which of the following is true? ‘Indain Express’ and 800 people read both. How many
(A) {3, 4}  A (B) 3  A neither read both newspapers.
(C) n(A) = 5 (D) {5}  A (A) 800 (B) 1500
11. If A = {  , {  }, 1, {1,  }, 2 } then which of the following (C) 2300 (D) 2850

is true ? 22. If n(A) = 115, n(B) = 326, n(A – B) = 47 then n A  B  is


(A) { 1 }  A (B) 2  A equal to :
(C) { {2}, {1} }  A (D) {  ,{  }, {1,  } }  A (A) 373 (B) 165
(C) 370 (D) None
ALGEBRA OF SETS
Directions : Q. 23 - 28
12. A  B = A is true if : In a survey of 25 students, it was found that 15 had
(A) A  B (B) A  B taken mathematics, 12 had taken physics and 11 had
(C) B  A (D) B =  taken chemistry, 5 had taken mathematics and
chemistry, 9 had taken mathematics and physics, 4
13. The relation A  B = A  B can be true ; when :
had taken physics and chemistry and 3 had taken all
(A) A = B (B) A = 
the three subjects.
(C) A  B
(D) A and B are disjoint sets. 23. Find the number of students that had taken only
mathematics.
14. A’ is the complement of a set A in the universal set U.
(A) 2 (B) 3
Then which of the following statements is not true ?
(C) 4 (D) 5
(A) A A’ = U (B) A A’ = 
(C) IfA’ = then A’ U = A 24. Find the number of students that had taken physics
(D) IfA’ = then A’ U = A and chemistry but not mathematics.
(A) 1 (B) 2
15. A – B is equal to :
(C) 3 (D) 10
(A) A  B (B) A  B’
(C) A’  B’ (D) A’  B’ 25. Find the number of students that had taken at least
one of the three subjects.
16. If U = {1, 2, 3, 4, 5, 6, 7}, A = {1, 2, 3, 4, 5}, B = {1, 3, 5, 7}
(A) 20 (B) 22
then A’ – B is equal to :
(C) 23 (D) 25
(A) {1, 3, 5} (B) {4, 6 }
(C) {3, 4, 6} (D) {6} 26. Find the number of students that had taken none of the
subjects.
17. If A = {1, 2, 3,.....9} and B = {2, 3, 4, 5, 7, 8}, then A – B
(A) 0 (B) 2
is given by :
(C) 3 (D) 5
(A) {1, 6, 7, 8} (B) {1, 6, 9}
(C) {1, 9} (D) {6, 9} 27. Find the number of students that had taken only one of
the subjects.
18. Which one of the following statements is true ?
(A) 10 (B) 11
(A) A  = A (B) A  = 
(C) 12 (D) 15
(C) A U =  (D) A  = U

PAGE # 103
28. Find the number of students that had taken exactly two 34. The shaded region in venn’s diagram given below
of the three subjects. represents :
(A) 9 (B) 10
(C) 11 (D) 12

29. Let n(U) = 700, n(A) = 200, n(B) = 300 and


n(A  B) = 100, then n(Ac  Bc) =
(A) 400 (B) 600 (A) A – B (B) B – A
(C) 300 (D) 200 (C) A’ – B (D) A – B’
35. The shaded region in venn’s diagram given below
30. In a class of 55 students, the number of students
represents :
studying different subjects are 23 in Mathematics and
24 in Physics, 19 in Chemistry, 12 in Mathematics
and Physics, 9 in Mathematics and Chemistry, 7 in
Physics and Chemistry and 4 in all the three
subjects. The number of students who have taken
exactly one subject is :
(A) 20 (B) 21
(C) 23 (D) 22 (A) A  B  C (B) A  B  C
(C) A  B  C (D) None of these
31. Out of 800 boys in a school, 224 played cricket, 240
played hockey and 336 played basketball. Of the total, 36. 3 sets A, B, C are represented by 3 circles in the given
64 played both basketball and hockey ; 80 played venn diagram. W hat does the shaded region
cricket and basketball and 40 played cricket and hockey represents :
24 player all the three games. The number of boys (A) A B   C
A B
who did not play any game is :
(A) 128 (B) 216 (B) A  (B  C)
(C) 240 (D) 160 (C) A  B   C C

32. In a hotel, 60% had vegetarian lunch while 30% had


(D) A  (B  C)
non-vegetarian lunch and 15% had both types of lunch.
If 96 people were present, how many did not eat either
type of lunch ?
(A) 20 (B) 24
(C) 26 (D) 28

VENN DIAGRAM 1. If set X consists of three elements then the number of


elements in the power set of power set of X is
33. The shaded region in the venn’s diagram given below [IJSO-2016]
represents : (A) 33 (B) 23
(C) 38 (D) 28

(A) (A  B)  C (B) (A  B)  C
(C) (A  B)  C (D) (A  B)  C

PAGE # 104
PLANE GEOMETRY - I

than 90º but less than 180º is called an obtuse angle.

A line has length but no width and no thickness.

An angle is the union of two non-collinear rays with a 90º < AOB < 180º.
common initial point. The common initial point is called
(iv) Straight angle : An angle whose measure is 180º
the ‘vertex’ of the angle and two rays are called the
is called a straight angle.
‘arms’ of the angles.

(v) Reflex angle : An angle whose measure is more


than 180º is called a reflex angle.
 REMARK :

Every angle has a measure and unit of measurement


is degree.
One right angle = 90º
1º = 60’ (minutes)
1’ = 60” (Seconds)
180º < AOB < 360º.
Angle addition axiom : If X is a point in the interior of
(vi) Complementary angles : Two angles, the sum of
BAC, then m BAC = m BAX + m XAC.
whose measures is 90º are called complementary
angles.

AOC & BOC are complementary as their sum is


(a) Types of Angles : 90 º.

(i) Right angle : An angle whose measure is 90º is (vii) Supplementary angles : Two angles, the sum of
called a right angle. whose measures is 180 º , are called the
supplementary angles.

AOC & BOC are supplementary as their sum is


(ii) Acute angle : An angle whose measure is less 180 º.
than 90º is called an acute angle. (viii) Angle Bisectors : A ray OX is said to be the bisector
of AOB , if X is a point in the interior of AOB, and
B
AOX = BOX.

O A
00 < BOA < 900
(iii) Obtuse angle : An angle whose measure is more
(ix) Adjacent angles : Two angles are called adjacent

PAGE # 105
angles, if angles if both lie either above the two lines or below
(A) they have the same vertex, the two lines, in figure 1 & 5, 4 & 8, 2 & 6,
(B) they have a common arm, 3 & 7 are the pairs of corresponding angles.
(C) non common arms are on either side of the If a transversal intersects two parallel lines then the
common arm. corresponding angles are equal i.e.  1 =  5,
4 = 8, 2 = 6 and 3 = 7.

(ii) Alternate interior angles : 3 & 5, 2 & 8, are
the pairs of alternate interior angles.
If a transversal intersects two parallel lines then the
each pair of alternate interior angles are equal i.e.
AOX and BOX are adjacent angles, OX is common 3 = 5 and 2 = 8.
arm, OA and OB are non common arms and lies on
(iii) Co- interior angles : The pair of interior angles on
either side of OX.
the same side of the transversal are called pairs of
(x) Linear pair of angles : Two adjacent angles are consecutive or co - interior angles. In figure
said to form a linear pair of angles, if their non common 2 &5, 3 & 8, are the pairs of co-interior angles.
arms are two opposite rays. If a transversal intersects two parallel lines then each
pair of consecutive interior angles are supplementary
i.e. 2 + 5 = 180º and 3 + 8 = 180º.

Ex.1. Find the measure of an angle, if six times its


complement is 12º less than twice its supplement.
Sol. Let the measure of the required angle be xº.
AOC + BOC = 180º.
Then, measure of its complement = (90 – x)º.
(xi) Vertically opposite angles : Two angles are called Measure of its supplement = (180 – x)º.
a pair of vertically opposite angles, if their arms form  6 (90 – x) = 2 (180 – x) – 12
two pairs of opposite rays.  540 – 6x = 360 – 2x – 12
 4x = 192  x = 48º.
Hence, the required angle is 48º.

Ex.2. In the adjoining figure, ABC = 100º, EDC = 120º


and AB || DE. Then, find BCD .

 AOC &  BOD form a pair of vertically opposite D


E
angles. Also AOD & BOC form a pair of vertically
B
A
opposite angles.
If two lines intersect, then the vertically opposite angles xº
are equal i.e. AOC = BOD and BOC = AOD.
D C
(b) Angles Made by a Transversal with two E
Parallel Lines : B
Sol. A F
Transversal : A line which intersects two or more given
parallel lines at distinct points is called a transversal xº
of the given lines.
C
Produce AB to meet CD at F.
BFD = EDF = 120° [alternate interior s]
 BFC = (180° – 120°) = 60°
 CBF = (180° – 100°) = 80°
 BCF = 180°– (60° + 80°) = 40°.

POLYGON

A closed plane figure bounded by line segments is


(i) Corresponding angles : Two angles on the same called a polygon.
side of a transversal are known as the corresponding A polygon is named according to the number of sides
it has :

PAGE # 106
No. of sides 3 4 5 6 7 8 10 1620 º
Figure Triangle Quadrilateral Pentagon Hexagon Heptagon Octagon Decagon
 2n – 4 = 90 º
 2n – 4 = 18
In general, a polygon having n sides is called 'n' sided
polygon.  2n = 22
 Diagonal of Polygon :  n = 11.
Line segment joining any two non-consecutive vertices
of a polygon is called its diagonal.
 Minute hand and hour hand coincides once in
 Convex Polygon : every hour. They coincide 11 times in 12 hours
and 22 times in 24 hours.
If all the interior angles of a polygon are less than 1800,  They coincide only one time between 11 to 1 O’
it is called a convex polygon.
clock. at 12 O’ clock.
 Minute hand and hour hand are opposite once in
every hour. They do it 11 times in 12 hours and 22
times in 24 hours.
 They opposite only one time between 5 to 7 O’
 Concave Polygon : clock. at 6 O’ clock.
If one or more of the interior angles of a polygon is  Both hands (minute and hour) are perpendicular
greater than 1800 i.e. reflex, it is called a concave twice in every hour. 22 times in 12 hours and 44
polygon. times in 24 hours.
 In one minute, hour hand moves 1/2º and minute
hand moves 6º. In one hour, hour hand moves 30º
and minute hand moves 360º.
 In an hour, minute hand moves 55 minutes ahead
of hour hand.

Ex.4 At what time between 3 O’Clock and 4 O’Clock will


 Regular Polygon :
the two hands coincide ?
A polygon is called a regular polygon if all its sides Sol. At 3 O’clock the distance between the two hands
have equal length and all the angles have equal is 15 minutes when they coincide with each other
measure. the distance between the two hands will be 0 min.
So, the time taken (15 + 0 ) = 15 minutes.
 Minute hand is 55 min. ahead of hour hand in
60 min.
 Minute hand is 1 min. ahead of hour hand in
60 min.
 REMARKS : 55
 Minute hand is 15 min. ahead of hour hand in
(1) The sum of the interior angles of a convex polygon of n
sides is (2n – 4) right angles or (2n – 4) 90º. 60  15 180 4
= = 16 min.
55 11 11
(2) The sum of the exterior angles of a convex polygon is
4 right angles or 360º. 4
Hence the right time is 16 minute past 3.
11
(3) Each interior angle of a n-sided regular polygon is
2n  4 90º Ex.5 At what time between 2 O’clock and 3 O’clock will
n
the two hands be opposite ?
Sol. At 2 O’clock the distance between the two hands
(4) Each exterior angle of a regular polygon of n sides
is 10 minutes. When they are at 30 minutes
 3600  distance, they are opposite to each other. The time
 
=  n  taken (30 + 10 ) = 40 min.
 
 Minute hand is 55 min. ahead of hour hand in
(5) If a polygon has n sides, then the number of diagonals 60 min.
nn  3   Minute hand is 1 min. ahead of hour hand in
of the polygon = . 60
2
min.
Ex.3 If the sum of interior angles of a polygon is 1620º, find 55
its number of sides.  Minute hand is 40 minutes ahead of hour hand
Sol. We know that sum of all interior angles of a n sided 60  40 480 7
polygon = (2n – 4) right angles in = = 43 min.
55 11 11
= [(2n – 4) 900]
but given sum of interior angles = 1620º 7
Hence, the right time is 43 min. past 2.
 (2n – 4) 900 = 1620º 11

PAGE # 107
Ex.6 At what time between 4 O’clock and 5 O’clock will
the hands are perpendicular ? TRIANGLE
Sol. At 4 O’clock the distance between the two hands
A plane figure bounded by three lines in a plane is
is 20 min. When they are at 15 minutes distance,
called a triangle. Every triangle have three sides and
they are perpendicular to each other.
three angles. If ABC is any triangle then AB, BC & CA are
Case-I When the time taken (20 – 15) = 5 min. three sides and A,B and C are three angles.
 Minute hand is 55 min. ahead of hour hand in
60 min.
Minute hand is 5 min. ahead of hour hand in
60  5 60 5
= = 5 min.
55 11 11
5
Hence, the right time is 5 min. past 4.
11  Types of triangles :
Case-II When the time taken (20 + 15) = 35 min. A. On the basis of sides we have three types of triangle.
 Minute hand is 55 min. ahead of hour hand in
60 min. 1. Scalene triangle – A triangle in which no two sides
Minute hand is 35 min. ahead of hour hand in are equal is called a scalene triangle.
60  35 420 2 2. Isosceles triangle – A triangle having two sides equal
= = 38 min. is called an isosceles triangle.
55 11 11
2 3. Equilateral triangle – A triangle in which all sides are
Hence, the right time is 38 min. past 4.
11 equal is called an equilateral triangle.
Mirror Image of Clock B. On the basis of angles we have three types :

 If the time is between 1 O’clock to 11 O’clock, 1. Right triangle – A triangle in which any one angle is
then to find the mirror image, time is subtracted right angle is called right triangle.
from 11 : 60. 2. Acute triangle – A triangle in which all angles are
 If the time is between 11 O’clock to 1 O’clock, acute is called an acute triangle.
then to find the mirror image, time is subtracted 3. Obtuse triangle – A triangle in which any one angle
from 23 : 60. is obtuse is called an obtuse triangle.
Ex.7 The time in the clock is 4 : 46, what is the mirror
 SOME IMPORTANT THEOREMS :
image ?
Sol. (11 : 60) – (4 : 46) = 7 : 14. Theorem : The sum of interior angles of a triangle is
180º.
Ex.8 The time in the clock is 12 : 35, then find its mirror
image. Theorem : If the bisectors of angles ABC and ACB
Sol. (23 : 60) – (12 : 35) = 11 : 25. of a triangle ABC meet at a point O, then
1
BOC = 90º + A.
Angle between Two Hands 2
Exterior Angle of a Triangle :
 Angle are of two types : If the side of the triangle is produced, the exterior angle
Positive angle : It is obtained by moving from hour so formed is equal to the sum of two interior opposite
hand to minute hand moving in clockwise
angles.
direction.
Negative angle : It is obtained by moving from Corollary : An exterior angle of a triangle is greater
minute hand to hour hand. than either of the interior opposite angles.

 Both types of angles are 360º in total. If one angle Theorem : The sides AB and AC of a ABC are produced
is known, other can be obtained by subtracting to P and Q respectively. If the bisectors of PBC and
from 360º. 1
QCB intersect at O, then BOC = 90º – A.
2
Ex.9 At 4 : 30, what is the angle formed between hour
Ex.10 In figure, TQ and TR are the bisectors of Q and R
hand and minute hand ?
Sol. At 4 O’ clock angle between hour and min. hand respectively. If QPR = 80º and PRT = 30º, determine
is of 120º. TQR and QTR.
In 30 min. minute hand make an angle of 180º. Sol. Since the bisectors of Q and R meet at T.
So, the resultant angle is 180º – 120º = 60º. P
But in 30 min. hour hand will also cover an angle 80º
of 15º.
Hence, the final angle between both hands is T
60º – 15º = 45º.
30
º

Q R
1
 QTR = 90º + QPR
2

PAGE # 108
1 (ii) ASA Congruence Criterion :
 QTR = 90º + (80º) A
2 P
 QTR = 90º + 40º = 130º
In QTR, we have
TQR + QTR + TRQ = 180º
 TQR + 130º + 30º = 180º [ TRQ = PRT = 30º ]
 TQR = 20º B C Q R
Thus, TQR = 20º and QTR = 130º. Two triangles are congruent if two angles and the
included side of one triangle are equal to the
corresponding two angles and the included side of
the other triangle.
The figures are called congruent if they have same
shape and same size. In other words, two figures are (iii) AAS Congruence Criterion :
called congruent if they are having equal length, width
A P
and height.

B C Q R
If any two angles and a non included side of one triangle
are equal to the corresponding angles and side of
another triangle, then the two triangles are congruent.
Fig.(i) Fig.(ii)
(iv) SSS Congruence Criterion :
In the above figures {fig.(i) and fig.(ii)} both are equal in A P
length, width and height, so these are congruent
figures.
(a) Congruent Triangles :

Two triangles are congruent if and only if one of them B Q


C R
can be made to superimposed on the other, so as to
cover it exactly. Two triangles are congruent if the three sides of one
triangle are equal to the corresponding three sides of
the other triangle.
( v ) RHS Congruence Criterion :
A P

If two triangles ABC and DEF are congruent then


B C Q R
A = D, B = E, C = F and AB = DE, BC = EF,
AC = DF. Two right triangles are congruent if the hypotenuse
If two ABC & DEF are congruent then we write and one side of one triangle are respectively equal to
the hypotenuse and one side of the other triangle.
ABC   DEF, we can not write as ABC  DFE or
 NOTE :
ABC  EDF.
If two triangles are congruent then their corresponding
Hence, we can say that “two triangles are congruent if
sides and angles are also congruent by CPCT
and only if there exists a one-one correspondence (corresponding parts of congruent triangles are also
between their vertices such that the corresponding congruent).
sides and the corresponding angles of the two
triangles are equal. Theorem : If the bisector of the vertical angle bisects
(b) Sufficient Conditions for Congruence of two the base of the triangle, then the triangle is isosceles.
Triangles : Ex.11 In figure, line m is the bisector of an angle A and O
is any point on m. OQ and OP are perpendiculars from
(i) SAS Congruence Criterion :
O to the arms of A. Prove that :
A P
P
m
O

B C Q R
Two triangles are congruent if two sides and the A
included angle of one are equal to the corresponding Q
sides and the included angle of the other triangle.
(i) AOQ AOP (ii) OQ = OP.

PAGE # 109
Sol. (i) In AOQ and AOP, (ii) If two sides of a triangle are unequal, then the longer
OAQ = OAP [ line m is the bisector of A] side has greater angle opposite to it
AQO = APO = 90º [Given] i.e. in ABC, if AB > AC then C > B.
and AO = AO [Common]
By AAS congruency (iii) Of all the line segments that can be drawn to a
 AOQ  AOP given line, from a point, not lying on it, the perpendicular
(ii) OQ = OP [C.P.C.T.] line segment is the shortest. i.e. in PMN,
P
Ex.12 If D is the mid-point of the hypotenuse AC of a right
1
triangle ABC, prove that BD = AC.
2
Sol. Given : ABC is a right triangle such that B = 900 and
D is mid point of AC.
1
To prove : BD = AC. M N
2 (A) PM < PN.
Construction : Produce BD to E such that BD = DE and
join EC. (iv) The difference between any two sides of a triangle
Proof : is less than its third side. i.e. in ABC,
(A) a – b < c (B) b – c < a (C) a – c < b

(v) In a right angle triangle the sum of squares of two


smaller sides is equal to the square of its third side.
i.e. in ABC, a2 + b2 = c2.

(vi) If sum of squares of two smaller sides is greater


In ADB and CDE than the square of its third side then that triangle is
AD = DC [Given]
acute angled triangle.
BD = DE [By construction]
And, ADB = CDE [Vertically opposite angles] i.e in ABC, a2 + b2 > c2.
 By SAS criterion of congruence we have (vii) If sum of squares of two smaller sides is lesser
ADB  CDE
than the square of its third side then that triangle is
 EC = AB and CED = ABD ... (i) [By CPCT]] obtuse angled triangle.
But CED & ABD are alternate interior angles
i.e in ABC, a2 + b2 < c2.
 CE || AB
 ABC + ECB = 1800 Ex.13 From which given triplet we can make the sides a
[Consecutive interior angles] triangle.
0
 90 + ECB = 180 (i) {15, 7, 8} (ii) {3.5, 4.5, 5.5}
 ECB = 900. Sol. As we know sum of two sides is always greater then
Now, In ABC & ECB we have third side.
AB = EC [By (i)]
BC = BC [Common] (i) we can not make the triangle because
And, ABC = ECB = 900 here the sum of two side is equal to third side
 By SAS criterion of congruence i.e 7+8=15
ABC  ECB (ii) We can make the triangle because sum of two
 AC = EB [By CPCT] sides is always greater then third side
1 1 3.5 +4.5 >5.5
 AC = EB
2 2 3.5 +5.5 >4.5
1 4.5 +5.5 >3.5
 BD = AC. Hence Proved.
2 Ex.14 Prove that any two sides of the triangle are together
greater than twice the median drawn to the third side.
Sol. Given :ABC and AD is the median.
To prove : AB + AC > 2AD
In triangle ABC, AB = c, BC = a & CA = b then,
A A

c b

B a C D
B C
(i) The sum of any two sides of a triangle is greater
than its third side. i.e. in ABC,
(A) a + b > c (B) b + c > a (C) a + c > b

PAGE # 110
Construction : Produce AD to E such that AD = DE. (ii) If two triangles are equiangular, then the ratio of the
Join EC. corresponding sides is same as the ratio of the
Proof : In ADB and CDE corresponding angle bisector segments.
AD = DE [By construction]
(iii) If two triangles are equiangular then the ratio of the
BD = DC [AD is the median]
corresponding sides is same as the ratio of the
 ADB =  CDE [Vertically opposite angles]
corresponding altitudes.
ADB  CDE [By SAS congruency]
(iv) If one angle of a triangle is equal to one angle of
So, by CPCT
another triangle and the bisectors of these equal
AB = EC
angles divide the opposite side in the same ratio, then
In AEC the triangles are similar.
AC + EC > 2AD.
(v) If two sides and a median bisecting one of these
[Sum of two sides of a triangle is always greater than
the third side] sides of a triangle are respectively proportional to the
So, AC + AB > 2AD. [As EC = AB]. two sides and the corresponding median of another
triangle, then the triangles are similar.

(vi) If two sides and a median bisecting the third side


SIMILAR FIGURES of a triangle are respectively proportional to the
Two geometric figures having the same shape and corresponding sides and the median of another
size are known as congruent figures. Geometric triangle, then two triangles are similar.
figures having the same shape but different sizes are  NOTE :
known as similar figures. If the ratio of sides of triangle is a : b : c, then ratio of
1 1 1
SIMILAR TRIANGLES their altitudes is : : .
a b c
Two triangles ABC and DEF are said to be similar if their
(i) Corresponding angles are equal.
THALES THEOREM
D (BASIC PROPORTIONALITY THEOREM)

A Statement : If a line is drawn parallel to one side of a


triangle to intersect the other two sides in distinct points,
then the other two sides are divided in the same ratio.

Given : A triangle ABC in which a line parallel to side


BC intersects other two sides AB and AC at D and
B C E F
E respectively.
i.e. A = D, B = E, C = F
A
(ii) Corresponding sides are proportional. i.e. N M

AB BC AC E
  . D
DE EF DF
(a) Characteristic Properties of Similar
Triangles : B C

(i) (AAA Similarity) If two triangles are equiangular, AD AE


To Prove : = .
then they are similar. DB EC
Construction : Join BE and CD and draw DM  AC and
(ii) (SSS Similarity) If the corresponding sides of two EN  AB.
triangles are proportional, then they are similar.
1
Proof : Area of  ADE = base × height
(iii) (SAS Similarity) If in two triangles, two pairs of 2
corresponding sides are proportional and the included 1
= AD × EN.
angles are equal then the two triangles are similar. 2
Area of ADE is denoted as ar(ADE)
(b) Results Based Upon Characteristic
1
Properties of Similar Triangles : So, ar(ADE) = AD × EN
2
(i) If two triangles are equiangular, then the ratio of the 1
corresponding sides is the same as the ratio of the And ar(BDE) = DB × EN,
2
1 AD  EN
corresponding medians. ar( ADE) 2 AD
Therefore, = 1 DB  EN = ... (i)
ar(BDE) 2 DB
PAGE # 111
1 (vii) If the diagonals of a quadrilateral divide each other
Similarly, ar(ADE) = AE × DM and ar(DEC) proportionally, then it is a trapezium.
2
1 (viii) Any line parallel to the parallel sides of a
= EC × DM.
2 trapezium divides the non-parallel s ides
1 AE  DM proportionally.
ar( ADE) 2 AE
And = 1 EC  DM = ... (ii)
ar(DEC) 2 EC (ix) If three or more parallel lines are intersected by
Note that  BDE and  DEC are on the same base DE two transversals, then the intercepts made by them on
and between the two parallel lines BC and DE. the transversals are proportional.
So, ar(BDE) = ar(DEC) ... (iii) Ex.15 In a ABC, D and E are points on the sides AB and AC
Therefore, from (i), (ii) and (iii), we have : respectively such that DE || BC. If AD = 4x – 3,
AD AE AE = 8x – 7, BD = 3x – 1 and CE = 5x – 3, find the value
 Hence Proved. of x.
DB EC
Sol. In ABC, we have
COROLLARY
DE || BC
If in a ABC, a line DE || BC, intersects AB in D and AC AD AE
in E, then    [By Basic Proportionality Theorem]
DB EC
DB EC AB AC
(i)  (ii)  4 x  3 8x  7
AD AE AD AE  
3x  1 5x  3
AD AE AB AC
(iii)  (iv) 
AB AC DB EC

DB EC
(v) 
AB AC
A

D E  20x2 – 15x – 12x + 9 = 24x2 – 21x – 8x + 7


 20x2 – 27x + 9 = 24x2 – 29x + 7
 4x2 – 2x – 2 = 0
B C  2x2 – x – 1 = 0
 (2x + 1) (x – 1) = 0
(a) Converse of Basic Proportionality
1
Theorem :  x = 1 or x = –
2
If a line divides any two sides of a triangle in the So, the required value of x is 1.
same ratio, then the line must be parallel to the third 1
side. [x = – is neglected as length can not be negative].
2
(b) Some Important Results and Theorems :
Ex.16 D and E are respectively the points on the sides AB
(i) The internal bisector of an angle of a triangle divides and AC of a ABC such that AB = 12 cm ,
the opposite side internally in the ratio of the sides AD = 8 cm, AE = 12 cm and AC = 18 cm, show that
containing the angle. DE || BC.
(ii) In a triangle ABC, if D is a point on BC such that D Sol. We have,
divides BC in the ratio AB : AC, then AD is the bisector AB = 12 cm, AC = 18 cm, AD = 8 cm and AE = 12 cm.
of A.  BD = AB – AD = (12 - 8) cm = 4 cm
CE = AC - AE = (18 – 12) cm = 6 cm
(iii) The external bisector of an angle of a triangle divides
the opposite sides externally in the ratio of the sides AD 8 2
Now,  
containing the angle. BD 4 1

(iv) The line drawn from the mid-point of one side of a AE 12 2


And,  
triangle parallel to another side bisects the third side. CE 6 1

(v) The line joining the mid-points of two sides of a AD AE


 
triangle is parallel to the third side. BD CE

(vi) The diagonals of a trapezium divide each other Thus, DE divides sides AB and AC of ABC in the same
proportionally. ratio. Therefore, by the converse of basic proportionality
theorem, we have DE ||BC.

PAGE # 112
Ex.17 In a trapezium ABCD, AB ||DC and DC = 2AB. EF Ex.18 In ABC, if AD is the bisector of A, prove that
drawn parallel to AB cuts AD in F and BC in E such that Area ( ABD) AB
 .
BE 3 Area ( ACD) AC
 . Diagonal DB intersects EF at G. Prove that
EC 4 Sol. In ABC, AD is the bisector of A.
7FE = 10AB.
Sol. In DFG and DAB,
1 = 2 [Corresponding s  AB ||FG]
FDG = ADB [Common]
 DFG ~ DAB [By AA rule of similarity]

DF FG
  … (i)
DA AB

AB BD
  .....(i)
AC DC
[By internal bisector theorem]
From A draw AL  BC

1
BD.AL
Area ( ABD) 2 BD AB
 = = = [From (i)]
FDG = ADB [Common] Area ( ACD) 1 DC AC
DC.AL
 DFG ~ DAB [By AA rule of similarity] 2
Hence Proved.
DF FG
  … (i) Ex.19 In triangle ABC, BAC = 90º, AD is its bisector. If
DA AB
Again in trapezium ABCD DE  AC, prove that DE × (AB + AC) = AB × AC.
EF ||AB ||DC Sol. It is given that AD is the bisector of A of  ABC.
AF BE AB BD
   =
DF EC AC DC
AF 3  BE 3 
   EC  4 ( given) AB BD
DF 4    +1= +1 [Adding 1 on both sides]
AC DC
AF 3
 1 1
DF 4 AB  AC BD  DC
 =
AF  DF 7 AC DC
 
DF 4
AD 7 DF 4 AB  AC BC
    … (ii)  = .....(i)
DF 4 AD 7 AC DC
From (i) and (ii), we get A
FG 4 4
 i.e., FG = AB … (iii)
AB 7 7
90º
In BEG and BCD, we have E
BEG = BCD [Corresponding angle  EG || CD]
GBE = DBC [Common]
 BEG ~ BCD [By AA rule of similarity]
B D C
BE EG
 
BC CD In 's CDE and CBA, we have
3 EG DCE = BCA [Common]
 
7 CD DEC = BAC [Each equal to 90º]
 BE 3 EC 4 EC  BE 4  3 BC 7  So, by AA-criterion of similarity
 EG  7 i.e., BE  3  BE 
3
 
BE 3 
  CDE ~  CBA
3 3 CD DE
 EG = CD  (2 AB)  CD  2AB (given)  =
7 7 CB BA
6
 EG = AB ... (iv) AB BC
7  = .....(ii)
Adding (iii) and (iv), we get DE DC
4 6 10 From (i) and (ii), we have
FG + EG = AB  AB  AB
7 7 7 AB  AC AB
 =
10 AC DE
 EF = AB i.e., 7EF = 10AB. Hence proved.
7  DE × (AB + AC) = AB × AC.

PAGE # 113
(a) Properties of Areas of Similar
AREAS OF SIMILAR TRIANGLES
Triangles :
Theorem : The ratio of the areas of two similar triangles (i) The areas of two similar triangles are in the ratio of
is equal to the square of the ratio of their corresponding the squares of corresponding altitudes.
sides.
(ii) The areas of two similar triangles are in the ratio of
Given : Two triangles ABC and PQR such that the squares of the corresponding medians.
 ABC ~  PQR [Shown in the figure]
(iii) The area of two similar triangles are in the ratio of
P the squares of the corresponding angle bisector
A segments.
Ex.20 Prove that the area of the equilateral triangle
described on the side of a square is half the area of
the equilateral triangle described on its diagonal.
B C Q R
M N Sol. Given : A square ABCD. Equilateral triangles BCE
2 2 2
and ACF have been described on side BC and
ar( ABC)  AB   BC   CA  diagonal AC respectively.
To prove : =  =  =  .
ar( PQR )  PQ   QR   RP 
1
To Prove : Area (BCE) = . Area (ACF)
Construction : Draw altitudes AM and PN of the 2
triangle ABC and PQR. Proof : Since BCE and ACF are equilateral.
1 Therefore, they are equiangular ( each angle being
Proof : ar(ABC) = BC × AM equal to 60º) and hence BCE ~ ACF.
2
1
And ar(PQR) = QR × PN
2

1 BC  AM
ar( ABC) 2 BC  AM
So, = 1 QR  PN = ... (i)
ar(PQR) 2 QR  PN

Now, in  ABM and  PQN,


And B = Q [As  ABC   PQR]
M = N [ 90º each ]
So,  ABM  PQN [AA similarity criterion]

AM AB
Therefore, = ... (ii) Area( BCE) BC 2
PN PQ  =
Area( ACF) AC 2
Also,  ABC ~  PQR [Given] Area( BCE) BC 2 1
 = 2 =
Area( ACF)  2BC 2
AB BC CA
So, = = ... (iii) Area( BCE)
PQ QR RP 1
 = . Hence Proved.
Area( ACF) 2
ar( ABC) BC AB
Therefore, =  [From (i) and (ii)]
ar(PQR) QR PQ
Statement : In a right triangle, the square of the
AB AB
= × [From (iii)] hypotenuse is equal to the sum of the squares of the
PQ PQ
other two sides.
2
 AB  Given : A right triangle ABC, right angled at B.
= 
To prove : AC2 = AB2 + BC2
 PQ 
Construction : BD  AC
Now using (iii), we get

2 2 2
ar(ABC)  AB   BC   CA 
=  =    
ar(PQR)  PQ   QR   RP 

Proof :  ADB &  ABC


DAB = CAB [Common]

PAGE # 114
BDA = CBA [90º each] then AC2 = AB2 + BC2 + 2BC. BD
So,  ADB ~  ABC [By AA similarity]

AD AB
= [Sides are proportional]
AB AC

or, AD . AC = AB2 ... (i)


Similarily  BDC ~  ABC

CD BC
So, =
BC AC
(ii) In the given figure, if B of ABC is an acute angle
or CD . AC = BC 2
... (ii) and AD  BC, then AC2 = AB2 + BC2 – 2BC . BD
Adding (i) and (ii),
AD . AC + CD . AC = AB2 + BC2
or, AC (AD + CD) = AB2 + BC2
or, AC. AC = AB2 + BC2
or, AC2 = AB2 + BC2 Hence Proved.

(a) Converse of Pythagoras Theorem :

Theorem : In a triangle, if the square of one side is (iii) In any triangle, the sum of the squares of any two
equal to the sum of the squares of the other two sides, sides is equal to twice the square of half of the third
then the angle opposite to the first side is a right angle. side together with twice the square of the median which
A bisects the third side.
D
(iv) Three times the sum of the squares of the sides of
a triangle is equal to four times the sum of the
squares of the medians of the triangle.

E Ex.21 BL and CM are medians of ABC right angled at A.


B C F
Prove that 4 (BL2 + CM2) = 5 BC2.
Given : A triangle ABC such that AC2 = AB2 + BC2 Sol. In BAL
Construction : Construct a triangle DEF such that BL2 = AL2 + AB2 … (i) [Using Pythagoras theorem]
and, In CAM
DE = AB, EF = BC and  E = 90º.
CM2 = AM2 + AC2… (ii) [Using Pythagoras theorem]
Proof : In order to prove that  B = 90º, it is sufficient to Adding (1) and (2) and then multiplying by 4, we get
show  ABC ~  DEF. For this we proceed as follows. B

Since  DEF is a right-angled triangle with right angle


at E. Therefore, by Pythagoras theorem, we have
DF2 = DE2 + EF2 M

 DF2 = AB2 + BC2


[ DE = AB and EF = BC (By construction)]
C
 DF = AC2
2
[ AB2 + BC2 = AC2 (GIven)] A L

 DF = AC ......(i) 4(BL2 + CM2) = 4(AL2 + AB2 + AM2 + AC2)


Thus, in  ABC and  DEF, we have = 4{AL2 + AM2 + (AB2 + AC2)}
[ABC is a right triangle]
AB = DE, BC = EF [By construction]
= 4(AL2 + AM2 + BC2)
And AC = DF [From equation (i)]
= 4(ML2 + BC2) [ LAM is a right triangle ]
  ABC   DEF [By SSS criteria of congruency] 2
= 4ML +4 BC 2

  B =  E = 90º [A line joining mid-points of two sides is parallel to


third side and is equal to half of it, ML =BC/2]
Hence,  ABC is a right triangle, right angled at B.
= BC2 + 4BC2 = 5BC2. Hence proved.
(b) Some Results Deduced From
P y t ha g o r as T h e o r e m :
(i)In the given figure ABC is an obtuse triangle, obtuse
angled at B. If AD  CB,

PAGE # 115
Ex.22 In the given figure, BC  AB, AE  AB and DE  AC. Sol. Let CD  AB. Then, CD = p
Prove that DE. BC = AD. AB.
1
Sol. In ABC and EDA,  Area of ABC = (Base × height)
2
E
1 1
C = (AB × CD) = cp
2 2

A
D
A B
We have c
D b
ABC = ADE [Each equal to 90°]
ACB = EAD [Alternate angles] p
 By AA Similarity
ABC ~ EDA
B a C
BC AD
  Also,
AB DE
 DE . BC = AD. AB. Hence Proved. 1 1
Area of ABC = (BC × AC) = ab
2 2
Ex.23 O is any point inside a rectangle ABCD (shown in the
1 1
figure). Prove that OB2 + OD2 = OA2 + OC2.  cp = ab
2 2
Sol. Through O, draw PQ || BC so that P lies on AB and Q
 cp = ab.
lies on DC.
A D (ii) Since  ABC is a right triangle, right angled at C.
 AB2 = BC2 + AC2
P Q  c 2 = a2 + b2
O
2
 ab   ab 
B C
   = a2 + b2  cp  ab  c  
 p   p 
Now, PQ || BC a 2b 2
Therefore,  = a2 + b2
p2
PQ  AB and PQ  DC [ B = 90º and  C = 90º] 1 1
1
So,  BPQ = 90º and  CQP = 90º  = +
p2 b2 a2
Therefore, BPQC and APQD are both rectangles.
1 1 1
Now, from  OPB,  = + .
p2 a2 b2
OB2 = BP2 + OP2 ... (i)
Similarly, from  ODQ, Ex.25 In an equilateral triangle ABC, the side BC is trisected
OD2 = OQ2 + DQ2 ... (ii) at D. Prove that 9 AD2 = 7AB2.
From  OQC, we have Sol. ABC be an equilateral triangle and D be point on BC
OC2 = OQ2 + CQ2 ... (iii) such that
And from  OAP, we have 1
BD = BC (Given)
OA2 = AP2 + OP2 ... (iv) 3
Adding (i) and (ii) Draw AE  BC, Join AD.
OB2 + OD2 = BP2 + OP2 + OQ2 + DQ2 BE = EC ( Altitude drown from any vertex of an equilateral
= CQ2 + OP2 + OQ2 + AP2 [As BP = CQ and DQ = AP] triangle bisects the opposite side)
= CQ2 + OQ2 + OP2 + AP2 A

= OC2 + OA2 [From (iii) and (iv)] Hence Proved.

Ex.24 ABC is a right triangle, right-angled at C. Let BC = a,


CA = b, AB = c and let p be the length of perpendicular
form C on AB, prove that
(i) cp = ab

1 1 1 B D E C
(ii) 2 = 2 + 2 BC
p a b
So, BE = EC =
2

PAGE # 116
In  ABC Special Quadrilaterals :
AB2 = AE2 + EB2 ......(i) (i) Parallelogram : A parallelogram is a quadrilateral
AD2 = AE2 + ED2 ......(ii) in which both pairs of opposite sides are parallel. In
From (i) and (ii) figure, AB || DC, AD || BC therefore, ABCD is a
AB2 = AD2 – ED2 + EB2 parallelogram.
BC2 BC2 D C
 AB2 = AD2 – +
36 4
BC BC BC BC
(  BD + DE =  + DE =  DE = )
2 3 2 6 A B
2 2
BC BC BC Properties :
 AB2 + – = AD2 ( EB = )
36 4 2 (a) A diagonal of a parallelogram divides it into two
2
congruent triangles.
AB AB2
 AB2 + – = AD2 ( AB = BC) (b) In a parallelogram, opposite sides are equal.
36 4
(c) The opposite angles of a parallelogram are equal.
36 AB 2  AB 2  9 AB 2
 = AD2 (d) The diagonals of a parallelogram bisect each other.
36
(ii) Rectangle : A rectangle is a parallelogram, in which
28 AB 2 each of its angle is a right angle. If ABCD is a rectangle
 = AD2 then A = B = C = D = 90°, AB = CD, BC = AD and
36
diagonals AC = BD.
 7AB2 = 9AD2.
D C
QUADRILATERAL
A quadrilateral is a four sided closed figure. 900
D A B
(iii) Rhombus : A rhombus is a parallelogram in which
all its sides are equal in length. If ABCD is a rhombus
then, AB = BC = CD = DA.
A C

B
Let A, B, C and D be four points in a plane such that :
The diagonals of a rhombus are perpendicular to each
(i) No three of them are collinear. other.
(ii) The line segments AB, BC, CD and DA do not (iv) Square : A square is a parallelogram having all
intersect except at their end points, then figure sides equal and each angle equal to right angle. If ABCD
obtained by joining A, B, C & D is called a quadrilateral. is a square then AB = BC = CD = DA, diagonal AC = BD
and A = B = C = D = 90°.
Convex and Concave Quadrilaterals :
(i) A quadrilateral in which the measure of each interior
angle is less than 180° is called a convex
quadrilateral. In figure, PQRS is convex quadrilateral.
R

S
The diagonals of a square are perpendicular to each
other.
(v) Trapezium : A trapezium is a quadrilateral with only
P Q one pair of opposite sides parallel. In figure, ABCD is
(ii) A quadrilateral in which the measure of one of the a trapezium with AB || DC.
interior angles is more than 180° is called a concave D C
quadrilateral. In figure, ABCD is concave quadrilateral.
D B

C A B

PAGE # 117
(vi) Kite : A kite is a quadrilateral in which two pairs of A quadrilateral become a parallelogram when :
adjacent sides are equal. If ABCD is a kite then AB = AD (i) Opposite angles are equal.
and BC = CD. C
(ii) Both the pair of opposite sides are equal
(iii) A pair of opposite side is equal as well as parallel
B D (iv) Diagonals of quadrilateral bisect each other.
 REMARK :
(i) Square, rectangle and rhombus are all
parallelograms.
A
(ii) Kite and trapezium are not parallelograms.
(vii) Isosceles trapezium : A trapezium is said to be an
(iii) A square is a rectangle.
isosceles trapezium, if its non-parallel sides are equal.
Thus a quadrilateral ABCD is an isosceles trapezium, (iv) A square is a rhombus.
if AB || DC and AD = BC. (v) A parallelogram is a trapezium.
Ex.26 The diagonals of a parallelogram ABCD intersect at
O. A line through O intersects AB at X and DC at Y. Prove
that OX = OY.
In isosceles trapezium A = B and C =D. Sol. Since AB || CD.
D Y C
PROPERTIES 2
4
Theorem 1 : The sum of the four angles of a O
3
quadrilateral is 360°.
1
A X B
Theorem 2 : A diagonal of a parallelogram divides it
into two congruent triangles. In OAX and OCY

Theorem 3 : In a parallelogram, opposite sides are 1 = 2 [Alternate angles]


equal. OA = OC
and, 3 = 4 [Vertically opposite angles]
Theorem 4 : The opposite angles of a parallelogram
are equal. So, by ASA criterion of congruence,
OAX  OCY
Theorem 5 : The diagonals of a parallelogram bisect
each other.  OX = OY.
Ex.27 The angle bisectors of a parallelogram form a
Theorem 6 : Each of the four angles of a rectangle is a
right angle. rectangle.
Sol. A parallelogram ABCD in which bisectors of angles
Theorem 7 : Each of the four sides of a rhombus is of
the same length. A, B, C, D intersect at P, Q, R, S to form a quadrilateral
PQRS.
Theorem 8 : Each of the angles of a square is a right A D
angle and each of the four sides is of the same length. R

Theorem 9 : The diagonals of a rectangle are of equal S Q


length.
P
Theorem 10 : The diagonals of a rhombus are per- B C
pendicular to each other. Since, ABCD is a parallelogram, so AD || BC and
Theorem 11 : The diagonals of a square are equal transversal AB intersects them at A and B respectively.
and perpendicular to each other. A + B = 180º
Theorem 12 : Parallelogram and Triangles on the 1 1
 A + B = 90º
same base (or equal bases) and between the same 2 2
parallels, then area of parallelogram is twice the area  BAS + ABS = 90º ....(i)
of triangle. [AS and BS are bisectors of A and B respectively]
Theorem 13 : Two triangles on the same base (or equal
BAS + ABS +ASB = 180º
bases) and between the same parallels are equal in
area.  90º + ASB = 180º
Theorem 14 : Median of a triangle divides it into two
 ASB = 90º
triangles of equal area.
Theorem 15 : Parallelograms on the same base and  RSP = 90º
between the same parallels are equal in area.
[ASB and RSP are vertically opposite angles]
Similarly, SRQ = 90º, RQP = 90º and SPQ = 90º.
Hence, PQRS is rectangle.

PAGE # 118
Ex.28 ABCD is a trapezium in which AB || CD and AD = BC.  Area of parallelogram ABCD = AD × BN = AB × DM
Show that :  AD × 5 = 8 × 4
(i) A = B (ii) C = D 84
(iii)  ABC   BAD (iv) diagonal AC = diagonal BD.  AD = 5
= 6.4 cm.
Sol.
A B E Ex.30 ABCD is a quadrilateral. A line through D, parallel to
AC, meets BC produced in P as shown in figure. Prove
that ar (ABP) = ar(quad. ABCD).
A

D C
D
(i) Extend AB and draw a line through C parallel to DA
intersecting AB produced at E.
Since, AD || CE and transversal AE cuts them at A and E
B C P
respectively.
Sol. Since s ACP and ACD are on the base AC and
 A + E = 180º
between the same parallels AC and DP.
 180º – E = A  ar(ACP) = ar(ACD)
Since, AB || CD and AD || CE  ar(ACP) + ar(ABC) = ar(ACD) + ar(ABC)
 AECD is a parallelogram.  ar(ABP) = ar(quad. ABCD).
 AD = CE Ex.31 If the medians of a  ABC intersect at G, show that
 BC = CE 1
ar(  AGB) = ar(  AGC) = ar(  BGC) = ar(  ABC).
3
Thus, in BCE
Sol. Given : A  ABC and its medians AD, BE and CF
BC = CE
intersect at G.
 CBE = CEB
To prove :
 180º – B = E 1
ar(  AGB) = ar(  AGC) = ar(  BGC) = ar(  ABC).
 180º – E = B 3
Proof : A median of a triangle divides it into two triangles
 A = B.
of equal area.
(ii) Consecutive interior angles on the same side of
a transversal are supplementary. In  ABC, AD is the median.
 A + D = 180º and E + C = 180º  ar(  ABD) = ar(  ACD) ...(i)
A
 A + D = E + C
 B + D = E + C
 D = C [B = E ]
F E
(iii) In ABC and BAD G
AB = BA
B = A
B D C
and BC = AD
In  GBC, GD is the median.
So, by SAS congruence criterion
ABC  BAD  ar(  GBD) = ar(  GCD) ...(ii)
(iv) Since, ABC  BAD Subtract equation (ii) from (i), we get
AC = BD ar(  ABD) – ar(  GBD) = ar(  ACD) – ar(  GCD)
Hence, diagonal AC = diagonal BD.  ar(  AGB) = ar(  AGC) ..(iii)
Similarly, ar(  AGB) = ar(  BGC) ..(iv)
Ex.29 In a parallelogram ABCD, AB = 8 cm. The altitudes
From (iii) & (iv)
corresponding to sides AB and AD are respectively
4 cm and 5 cm. Find AD. ar(  AGB) = ar(  AGC) = ar(  BGC)
Sol. Area of a ||gm = Base × corresponding altitude But, ar(ABC) = ar(  AGB) + ar(  AGC) + ar(  BGC)
= 3 ar (  AGB)
1
 ar(  AGB) = ar(  ABC).
3
Hence,
1
ar(  AGB) = ar(  AGC) = ar(  BGC) = ar(  ABC).
3

PAGE # 119
side AB and PQ is parallel to BC.
To prove : PQ bisects the third side AC i.e., AQ = QC.
In a triangle, the line segment joining the mid-points
Construction : Through C, draw CR parallel to BA, which
of any two sides is parallel to the third side and is
half of it. meets PQ produced at point R.
Proof : Since, PQ || BC i.e., PR || BC [Given]
and CR || BA i.e., CR || BP. [By construction]
 Opposite sides of quadrilateral PBCR are parallel.
 PBCR is a parallelogram

Given : A triangle ABC in which P is the mid-point of  BP = CR


side AB and Q is the mid-point of side AC. Also, BP = AP [As, P is mid-point of AB]
To Prove : PQ is parallel to BC and is half of it  CR = AP
1  AB || CR and AC is transversal, PAQ = RCQ
i.e., PQ || BC and PQ = BC.
2
[Alternate angles]
Construction : Produce PQ upto point R such that
PQ = QR. Join R and C.  AB || CR and PR is transversal, APQ = CRQ
Proof : In APQ and CRQ [Alternate angles]
PQ = QR [By construction]
In  APQ and  CRQ
AQ = QC [Given]
And, AQP = CQR [Vertically opposite angles] CR = AP, PAQ = RCQ and APQ = CRQ
So, APQ  CRQ [By SAS]   APQ   CRQ [By ASA]
 AP = CR [By cpct]
 AQ = QC Hence Proved.
And, APQ = CRQ [By cpct]
But, APQ and CRQ are alternate angles and
whenever the alternate angles are equal, the lines are Ex.32 In the adjoining figure, D, E, F are the midpoints of
parallel. the sides BC, CA and AB of ABC. If BE and DF
 AP || CR intersect at X while CF and DE intersect at Y, prove
1
 AB || CR that XY = BC.
4
 BP || CR Sol. In ABC, F and E are the midpoints of AB and AC
AP = BP [Given, P is mid-point of AB] respectively.
 CR = BP [As, AP = CR] A
Now, BP = CR and BP || CR
 BCRP is a parallelogram. F E
[When any pair of opposite sides are equal and
X Y
parallel, the quadrilateral is a parallelogram] C
B D
BCRP is a parallelogram and opposite sides of a 1
parallelogram are equal and parallel.  FE || BC and FE = BC = BD.
2
 PR = BC and PR || BC  FE || BD and FE = BD.
Since, PQ = QR So, BDEF is a parallelogram whose diagonals BE
1 1 and DF intersect each other at X.
 PQ = PR = BC [As, PR = BC]  X is the midpoint of DF.
2 2
Also, PQ || BC [As, PR || BC] Similarly, Y is the midpoint of DE.
Thus, in DEF, X and Y are the midpoints of DF and
1
 PQ || BC and PQ = BC Hence Proved. DE respectively.
2
1 1 1 1
Converse of the Mid-Point Theorem : The line drawn So, XY || FE and XY = FE = × BC = BC.
2 2 2 4
through the mid-point of one side of a triangle parallel
to the another side; bisects the third side. Ex.33 In the adjoining figure, ABCD is a trapezium in which
A AB || DC and AD = BC. If P, Q, R, S be respectively the
midpoints of BA, BD and CD, CA then show that PQRS
Q
P R is a rhombus.
Sol. In BDC, Q and R are the midpoints of BD and CD
respectively.
B C
Given : A triangle ABC in which P is the mid-point of

PAGE # 120
A

F E
G

1
 QR || BC and QR = BC. B C
2 D
1 Adding these, we get
Similarly, PS || BC and PS = BC.
2 2(AG + BG + CG) > AB + BC + CA
1
 PS || QR and PS = QR [Each equal to BC.] 2 2 2 
2 or 2 AD  BE  CF  > AB + BC + CA
 PQRS is a parallelogram.  3 3 3 
In ACD, S and R are the midpoints of AC and CD 4
or (AD + BE + CF) > AB + BC + CA
respectively. 3
1 1 or 4(AD + BE + CF) > 3(AB + BC + CA) ... (i)
 SR || AD and SR = AD = BC [ AD = BC]
2 2 Now, in ABD
 PS = QR = SR = PQ.
Hence, PQRS is a rhombus. AB + BD > AD
1
Ex.34 Prove that the line segment joining the midpoints of or, AB + BC > AD (Since AD is a median)
2
the diagonals of a trapezium is parallel to the parallel 1 1
sides and equal to half of their difference. Similarly, BC + CA > BE and CA + AB > CF
2 2
Sol. Let ABCD be a trapezium in which AB || DC, and let M 3
and N be the midpoints of the diagonals AC and BD Adding these, we get (AB + BC + CA) > AD + BE + CF
2
respectively. or 3(AB + BC + CA) > 2(AD + BE + CF) ... (ii)
Combining (i) and (ii), we get
2(AD + BE + CF) < 3(AB + BC + CA) < 4(AD + BE + CF).

Ex.36 In a ABC, AC > AB. The bisector of A meets BC at


E. Prove that CE > BE.
Join CN and produce it to meet AB at E. Sol. Given : A ABC, in which AC > AB and AE bisects A
In CDN and EBN, we have
To Prove : CE > BE.
DN = BN [ N is midpoint of BD]
DCN = BEN [Alt. Int. ] Construction : Take a point F on AC, such that AF = AB.
CDN = EBN [Alt. Int. ] Join BF and EF. Let BF intersect AE at O.
 CDN EBN [AAS-criteria] Proof : ABF is an isosceles
 DC = EB and CN = NE [c.p.c.t]
(Since AB = AF, by construction)
Thus; in CAE, the points M and N are the midpoints of
AC and CE respectively. AE bisects A (given)
1  AE is the perpendicular bisector of BF, so any point
 MN || AE and MN = AE  MN || AB || DC
2 on AE is equidistant from B and F. In particular,
1 1 1 A
and MN = AE = (AB – EB) = (AB – DC)
2 2 2
[ EB = DC].
Ex.35 AD, BE and CF be medians of a ABC. Prove that
2(AD + BE + CF) < 3(AB + BC + CA) < 4(AD + BE + CF). F
O
Sol. In ABC, AD, BE and CF are medians. We know that
these are concurrent.Let these meet at G.
Also AG : GD = BG : GE = CG : GF = 2 : 1 B E C
2 2 2
So, AG = AD, BG = BE, CG = CF.. BE = FE ... (i)
3 3 3
Because in triangle, the sum of two sides is greater So, in BEF,
than the third, EBF = EFB = , say (angles opposite equal sides)
So, in AGB, AG + BG > AB So, BEO = 90° –  = FEO (= , say)
in BGC, BG + CG > BC
Thus AE also bisects BEF.
in CGA, CG + AG > CA
Now, because in a triangle, exterior angle is the sum
of interior opposite angles, so

PAGE # 121
EFC =  +  and ext B =  +  1
area (CYX) = area(DBC)
So, EFC = ext B. But B = 2 + C, so EFC = 2 + C. 4
Therefore EFC > C 1
= area (parallelogram ABCD). .. (i)
8
So, CE > FE
1
Hence CE > BE (using (i)). [Since ar(DBC) = area (parallelogram ABCD)]
2
Ex.37 If the medians of ABC intersects at G, show that Parallelogram ABCD and ABX are between the
1
ar(AGB) = ar(AGC) = ar(BGC) = ar(ABC). 1
3 same parallels AD and BC and BX = BC.
Sol. Given : A ABC such that its medians AD, BE & CF 2
interest at G. A 1
 area (ABX) = area (parallelogram ABCD) ... (ii)
4
1
F E Similarly, area (AYD) = area (parallelogram ABCD)
G 4
Now, ar(AXY) = ar(parallelogram ABCD) – [ar(ABX)
+ ar(AYD) + ar(CYX)]
B D C
ar(AXY) = ar (parallelogram ABCD)
To prove : ar(AGB) = ar(AGC) = ar(BGC)  1 1 1
1 –     ar(ABCD)
= ar(ABC). 4 4 8
3
 5
Proof : We know that medians of a triangle divides it ar(AXY) = 1–  ar(parallelogram ABCD)
into two triangles of equal area.  8
In ABC, AD is the median 3
= ar(parallelogram ABCD).
 ar(ABD) = ar(ACD) .... (i) 8
In GBC, GD is the median Ex.39 In the figure AE : EB = 1: 2 and BD : DC = 5 : 3.
 ar(GBD) = ar(GCD) .... (ii) Find EG : GC. A
Subtracting (ii) from (i), we get 1
E
ar(ABD) – ar(GBD) = ar(ACD) – ar(GCD)
2
 ar(AGB) = ar(AGC) ... (iii) G
Similarly, ar(AGB) = ar(GBC) ... (iv) C
B 5 D 3
From (iii) and (iv) we get
Sol. Through E, we draw a line parallel to AD, intersecting
ar(AGB) = ar(AGC) = ar(BGC)
BC at M.
But, ar(AGB) + ar(AGC) + ar(BGC) = ar(ABC)
1
 3 ar(AGB) = ar(ABC)  ar(AGB) = ar(ABC)
3
1
Hence, ar(AGB) = ar(AGC) = ar(BCG) = ar(ABC).
3
Ex.38 ABCD is a parallelogram. X and Y are mid–points of
BC and CD respectively. Prove that 2  1
BM = 5  and MD = 5 
3 3
  3
area (AXY) = area(parallelogram ABCD).
8 5
Sol. Since X and Y are the mid–points of sides BC and EG : GC = MD : DC = : 3 = 5 : 9.
3
CD respectively in BCD.
1
 XY || BD and XY = BD.
2

PAGE # 122
7. If the sum of all angles except one of a convex polygon
is 2180º, then the number of sides of the polygon is :
NOTE : More than one correct option may be possible. (NTSE Stage-II/2009)
(A) 19 (B) 17
LINES AND ANGLES
(C) 15 (D) 13
1. In Figure, D and E are the mid-points of sides AB and
AC respectively of ABC.Find EDB. 8. At what time between 5 O’clock & 5 : 30 will the
A hands of a clock be at right angle ?
(A) 110º
60° 10
(A) 10 minutes past 5.
(B) 120º 11
D E
(C) 70º 5cm 5
(B) 11 minutes past 5.
11
(D) 80º 50°
B C 10
(C) 9 minutes past 5.
2. In ABC, what is sum of the angles a + b + c + d + e + 11
f+g+h+i? A
9
(D) 10 minutes past 5.
(A) 360º a 11

(B) 540º 9. Find the angle between the two hands of a clock
d

e
of 15 minutes past 4 O’clock
(C) 600º (A) 38.50 (B) 36.50
h
i g f c 0
(D) Cannot be determined B C (C) 37.5 (D) None of these

3. P is a point inside ABC. If PBA = 20ºBAC = 50º CONGRUENT TRIANGLES & SIMILAR
and, PCA = 35º, then the measure of BPC is : TRIANGLES

(A) 65º 10. A square board side 10 centimeters, standing vertically,


(B) 75º is tilted to the left so that the bottom-right corner is
raised 6 centimeters from the ground.
(C) 90º

(D) 105º
4. In the given figure OBC and OKH are straight lines. If
AH = AK, b = 80º and c = 30º then the value of d is :
(A) 20º 6 cm

(B) 25º

(C) 30º By what distance is the top-left corner lowered from its
original position ?
(D) 45º (A) 1 cm (B) 2 cm
5. In given figure find the values of x and y, if QS = RQ. (C) 3 cm (D) 0.5 cm
R
11. In a right angled triangle ABC, P is mid point of AC.
S Which one is true ?

103º AC AC
Yº (A) PA = (B) PB =
148º xº 2 2
X P Q
(C) PA = PB (D) All of these
(A) x = 36º, y = 32º (B) x = 45º, y = 32º
(C) x = 32º, y = 45º (D) x = 45º, y = 26º 12. The sides of a triangle are in the ratio 4 : 6 : 11. Which
6. In the figure, AB = BC = CD = DE = EF = FG = GA, then of the following words best described the triangle?
find DAE (approximately) (A) obtuse (B) isosceles
(C) acute (D) impossible
(A) 24º
(B) 25º
(C) 26º
(D) None of these

PAGE # 123
13. In the given diagram B = C = 65º and D = 30º, then 19. In the below diagram, ABCD is a rectangle with
the true statement is : AE = EF = FB. What is the ratio of the areas ofCEF
A
(A) BC = CA and that of the rectangle ?
1 D C
(B) CA > CD (A)
6
(C) BD > AD 1
65º 65º 30º (B)
(D) AC = AD B D 8
C
1
14. In a PQR, PS is bisector of P, Q = 70º and (C)
9 A E F B
R = 30º, then : (D) None of these
(A) QR < PR > PQ (B) QR > PR > PQ
(C) QR = PR = PQ (D) QR < PR = PQ 20. In the figure DF || AG, DE || AB, AB = 15, CD = 8,
AD = x, DE = 10, FG = y and CG = 6. The ratio x : y equal
15. In the diagram given below, ABD = CDB to :
= PQD = 90º. If AB : CD = 3 : 1, the ratio of CD : PQ is :
(A) 1 : 2
(A) 1 : 0.69
(B) 1: 3
(B) 1 : 0.75
(C) 2 : 1
(C) 1 : 0.72
(D) 3 : 2
(D) None of the above.
21. In the figure A = CED, CD = 8 cm CE = 10 cm,
16. In the right-angled triangle QPR given below, PS is the
altitude to the hypotenuse. The figure is followed by BE = 2cm, AB = 9cm, AD = b and DE = a. The value of
three possible inferences. a + b is :

I. Triangle PQS and (A) 13 cm


Triangle RPS are similar.
(B) 15 cm
II. Triangle PSQ and
Triangle RSP are congruent.
(C) 12 cm
III. Triangle PSQ and
triangle RPQ are similar. (D) 9 cm
Mark the correct option 22. Let X be any point on the side BC of a triangle ABC. If
XM, XN are drawn parallel to BA and CA meeting CA, BA
(A) I and II are correct (B) I and III are correct
in M,N respectively ; MN meets CB produced in T. Then
(C) only II is correct (D) All three are correct A
(A) TB2 = TX  TC
17. In the figure below, AL is perpendicular to BC and CM
is perpendicular to AB. If CL = AL = 2BL, find
(B) TC2 = TB  TX M
MC/AM. A
N
(A) 2 (C) TX2 = TB  TC

(B) 3 M (D) TX2 = 2(TB  TC) T C


B X
(C) 4
23. The altitude of the given triangle are AP = 3, BQ = 4
(D) Cannot be determined C L
B and CR = 5. What is the ratio of BC : AC : AB ?

18. In the given figure, ABC is a right angled triangle. Also (A) 20 : 15 : 12
FG || DE || BC and AG = GE = EB. If DE = 12 cm, then the
measure of BC is : (B) 20 : 12 : 15

(A) 12 cm (C) 15 : 12 : 20
(B) 18 cm
(D) 3 :4 : 5
(C) 24 cm
(D) 30 cm 24. If the sides of a right triangle are 9, 12 and 15 cm long,
then the sum of squares of medians is :
(A) 227.5 (B) 337.5
(C) 537.5 (D) None of these

PAGE # 124
25. In a ABC, AB = AC = 2.5 cm, BC = 4 cm. Find its height 32. In a triangle ABC, if B = 90º and D is the point in BC
from A to the opposite base : such that BD = 2 DC, then :
(A) 1.5 cm (B) 1 cm
(A) AC2 = AD2 + 3 CD2 (B) AC2 = AD2 + 5 CD2
(C) 2 cm (D) 3 cm 2 2 2
(C) AC = AD + 7 CD (D) AC2 = AB2 + 5 BD2
26. In figure, PA, QB and RC are each perpendicular to AC.
33. Let XOY be a right angled triangle with XOY = 90º.
1 1 Let M and N be the midpoints of legs OX and OY, re-
Then :  
x z spectively. Given that XN = 19 and YM = 22, the length
(A) 1 XY is equal to
(B) y (A) 24 (B) 26
(C) 28 (D) 34
1
(C) y 34. If CD = 15, DB = 9, AD bisects A,ABC = 90º, then
(D) None AB has length :

27. A square ABCD is constructed inside a triangle PQR


having sides 10, 17 and 21 as shown in figure. Find
the perimeter of the square ABCD.

(A) 28
(B) 23.2
(C) 25.4
(D) 28.8 (A) 32 (B) 18
(C) 7 (D) 24
28. Two triangles ABC and PQR are similar, if
35. The perimeters of two similar triangles are 25 cm and
QR
BC : CA : AB = 1 : 2 : 3, then is : 15 cm respectively. If one side of first triangle is
PR
2 1 9 cm, then the corresponding side of the other triangle
(A) (B) is :
3 2
1 2 (A) 6.2 cm (B) 3.4 cm
(C) (D)
2 3 (C) 5.4 cm (D) 8.4 cm
29. In the given figure, DE || BC and AD : DB = 5 : 4, find 36. In a triangle ABC, a straight line parallel to BC intersects
area( DFE) A
. AB and AC at point D and E respectively. If the area of
area( CFB)
ADE is one-fifth of the area of ABC and BC = 10 cm,
(A) 5 : 9 D E
(B) 25 : 16 then DE equals :
F
(C) 25 : 81 (A) 2 cm (B) 2 5 cm
B C
(D) None of these (C) 4 cm (D) 4 5 cm
30. In a triangle ABC, the internal bisector of the angle A
meets BC at D. If AB = 4, AC = 3 and A = 60º, then the 37. ABC and DEF are two similar triangles such that
length of AD is : BC = 4 cm, EF = 5 cm and Area of ABC = 64 cm2 .
Then, the area of DEF is :
12 3
(A) 2 3 (B) (A) 50 cm2 (B) 75 cm2
7 2
(C) 100 cm (D) None of these
15 3
(C) (D) None of these
8 Q UA D R I LAT E R A LS
PYTHAGORAS THEOREM 38. ABCD is a trapezium in which AB || CD. If ADC =
31. In the following figure, AE  BC, D is the mid point of 2ABC, AD = a cm and CD = b cm, then the length (in
BC, then x is equal to cm) of AB is :
A
1 2 2 a2  a
(A) a b – d – 4  (A) + 2b (B) a + b
  2
b
hd c
(B) h d 2 2
3
(C) a+b (D) a + b
cd–h 3 3
(C) B
2 E x D C
a 2  b 2  d2 – c 2 a
(D)
4

PAGE # 125
39. E is the midpoint of diagonal BD of a parallelogram 45. In the trapezium shown, AB II DC, and E and F are
ABCD. If the point E is joined to a point F on DA such the midpoints of the two diagonals. If DC = 60 and
1 EF = 5 then the length of AB is equal to :
that DF = DA, then the ratio of the area of DEF to the
3
area of quadrilateral ABEF is :
(A) 1 : 3 (B) 1: 4
(C) 1 : 5 (D) 2 : 5 (A) 40 (B) 45
(C) 50 (D) 55
40. In the figure, the area of square ABCD is 4 cm2 and E
any point on AB. F, G, H and K are the mid point of DE, 46. In the figure, the quadrilateral ABCD is a rectangle, P
CF, DG, and CH respectively. The area of KDC is - lies on AD and Q on AB. The triangles PAQ, QBC and
PCD all have the same area, and BQ = 2. The length of
AQ, is :
(A) 3  5 (B) 2 3
(C) 5  1
(D) not uniquely determined

1 1 2
(A) cm2 (B) cm
4 8
COMPETITIVE EXAM PREVIOUS YEAR
1 1 QUESTION
(C) cm2 (D) cm2 1. L, M and N are mid points of sides AB, BC and CA of
16 32
triangle ABC. If area of triangle ABC is 48 units, the
41. The sides of rectangle are all produced in order, in
such a way that the length of each side is increased by area of triangle LMN will be : [IJSO-2008]
‘k’ times itself. The area of the new quadrilateral formed (A) 6 units (B) 8 units
(C) 12 units (D) 24 units
1
becomes 2 times the area of the original rectangle. 2. The adjacent sides of a parallelogram are 15 cm and
2
Find the value of ‘k’. 10cm. If length of one diagonal of this parallelogram is
20 cm, the length of other diagonal will be :
1 5 [IJSO-2008]
(A) (B)
2 2 (A) 30 cm (B) 5 10
3 (C) 10 5 cm (D) 4 30 cm
(C) (D) None of these
2 3. In the parallelogram, the value of “x” is :
42. The line joining the mid points of the diagonals of a [NSTSE-2009]
trapezium has length 3. If the longer base is 97, then 80º
the shorter base is :
(A) 94 (B) 92 xº 150º
(C) 91 (D) 90
(A) 30º (B) 50º
43. In the adjoining figure DP is parallel to AC, then the (C) 70º (D) 80º
ratio of area of triangle PCB and quadrilateral ABCD 4. Given two similar triangles one of which has twice the
is : perimeter of the other, by what factor is the area of the
(A) 1 : 1 larger triangle bigger than the smaller ?
[NSTSE-2009]
(B) 1 : 2 (A) 2 (B) 4
(C) 1 : 4 (C) 2 (D) 2 2

(D) 2 : 3 5. In the figure shown, PQ || RS and SM || TN. Then


44. One side of a parallelogram has length 3, and another measure of angle  is : [IJSO-2009]
side has length 4. Let a and b denote the lengths of the L M
diagonals of the parallelogram. Which of the following
quantities can be determined from the given N
information? 29º
R
I. a + b II a2 + b2 III a3 + b3 S
(A) Only I (B) Only II 62º
(C) Only III (D) Only I and II P T Q
(A) 58º (B) 118º
(C) 89º (D) 91º

PAGE # 126
6. The diagonals of a quadrilateral are equal and bisect 14. All the three sides of a  ABC have lengths in integral
each other. The quadrilateral has to be : [IJSO-2009] units, with AB = 2001 units and BC = 1002 = units. The
(A) any parallelogram (B) any rectangle possible number of triangles with this condition is :
[IJSO-2010]
(C) any trapezium (D) any rhombus
(A) 2001 (B) 2002
7. ABCD is a parallelogram. 'P' is a point on AD such that (C) 2003 (D) 2004
1
AP = AD and 'Q' is a point on BC such that 15. If the altitudes of a triangle are in the ratio 2 : 3 : 4, then
3
1 the lengths of the corresponding sides are in the ratio:
CQ = BC. Then AQCP is a : [NSTSE-2010] [IJSO-2010]
3
(A) Parallelogram (B) Rhombus (A) 2 : 3 : 4 (B) 6 : 4 : 3
(C) Rectangle (D) Square (C) 3 : 2 : 4 (D) 3 : 2 : 1
8. The degree measure of each of the three angles of a
triangle is an integer. Which of the following could not 16. In th figure shown below, DE || BC and AD = 3x – 2,
be the ratio of their measures ? [NSTSE-2010] AE = 5x – 4, BD = 7x – 5 and CE = 5x – 3. Therefore , the
(A) 2: 3 : 4 (B) 3 : 4 : 5 value of x is [IJSO-2011]
(C) 5 : 6 : 7 (D) 6: 7 : 8 A
9. ABC is a right angled triangle, where B = 90º. CD
and AE are medians. If AE = x and CD = y then, correct
statement is : [NSTSE-2010]
D E
(A) x2 + y2 = AC2 A

(B) x2 + y2 = 2AC2 B C
x 7
3 D (A) only 1 (B) only
(C) x + y = AC2
2 2
y 10
2
7 10
5 (C) 1 or (D)
C 10 7
(D) x2 + y2 = 4 AC2 B E

10. If the hypotenuse of a right angled triangle is 41 cm 17. In ABC, D is a point on side AC such that ABD = ½
and the area of the triangle is 180 sq cm, then the ABC. If AB = 36, BC = 48, CD = 28, then the length
difference between the lengths of the legs of the triangle DA will be [IJSO-2011]
must be : [NSTSE 2010] (A) 20 (B) 21
(A) 22 cm (B) 25 cm (C) 22 (D) 24
(C) 27 cm (D) 31 cm
11. The sides of a triangle are in the ratio 6 : 8 : 9 . Therefore: 18. Consider triangles having integer sides such that no
[IJSO-2010] side is greater than 4 units. How many such triangles
(A) Angles of the triangle are in the ratio 6 : 8 : 9 are possible? [IJSO-2012]
(B) It is an acute angled triangle (A) 13 (B) 17
(C) It is a right angled triangle (C) 24 (D) 64
(D) It is an obtuse angled triangle
19. In the adjoining figure AQ = 2, QB = 4, BP = 3, PC = 5,
12. In the figure shown AB is parallel to DE. The difference CR = 6 and RA = 4. [IJSO-2012]
between angles x and y is : [IJSO-2010] Find the area of triangle PQR.
A B
35º
y c
x

53º
D E
(A) 0º (B) 4º
(C) 10º (D) 12º (A) 4.8 (B) 5.2
(C) 5.8 (D) 6.2
13. AD, BE and CF are the medians of ABC. The sum
20. Let ABC be a triangle in which AB = AC. Let D be a point
of lengths of segments BE and CF is :[IJSO-2010]
on BC such that AD bisects angle A. Value of the ratio
3 5 BD
(A) < BC (B) > BC is [IJSO-2013]
2 3 DC

3 2 (A) 1:1 (B) 1:2


(C) > BC (D) < BC (C) 2:1 (D) none of these
2 3

PAGE # 127
21. Let m be the number of distinct (non congruent) inte- 25. The adjacent sides of a parallelogram are 30 cm and
ger-sided triangles each with perimeter 15 and n be 20 cm. The length of one of the diagonal is 40 cm.
the number of distinct (non congruent) integer sided What is the length of the other diagonal? [IJSO-2015]
triangles each with perimeter 16.
Then m-n equals [IJSO-2013] (A) 60 cm (B) 10 10 cm
(A) –2 (B) 0
(C) 20 5 cm (D) 8 30 cm
(C) 2 (D) – 4

22. At what time (to the nearest second) immediately after 26. In the adjoining figure segment AD, BE and CF are the
4 O’ clock will angle between the hands of the clock be altitudes of triangle ABC. Find AD × BC if AB × AC =
the same as that at 4 O’ clock ? [IJSO-2013] 409.6, BE × CF = 202.5.
(A) 4h 42m 505 (B) 4h 43m 38s
h m S
(C) 4 43 40 (D) 5h 5m 27s

23. A ray of light is incident on system of mirror as shown


in the adjacent figure. What is the total deflection (d) of [IJSO-2015]
the ray when it emerges out after two reflections ?

d (A) 225 (B) 256


(C) 288 (D) 312
[IJSO-2014]
27. The angle between the hour arm and the minute arm
70° of a clock at 2:10 a.m. is [IJSO-2016]
(A) zero (B) 4º
(C) 5º (D) 6º
(A) 220° (B) 180°
(C) 120° (D) 140° 28. If ABCD is a rhombus and  ABC = 60 º then
[IJSO-2016]
24. If the distance between A and B is 230 km, B and C is (A) The point A, B, C, D are concylic
120 km, C and A is 350 km. Also, if the distance be- (B) The quadrilateral has exactly half the area of the
tween C and D is 200 km, distance between D and B square with same sides as ABCD
is 330 km and distance from A to E is 100 km and
distance between D and E is 570 km. The diagram 3
(C) The quadrilateral has area AB2
(not drawn to scale) that represents this graphically is: 2
[IJSO-2014] (D) The diagonals of the quadrilateral ABCD are equal
and bisect each other at right angle
A
E D
E B
(A) (B)
A B C
D C

(C) (D) none of these


D
A B C

PAGE # 128
PLANE GEOMETRY - II

Major arc PQ

 Circle : The collection of all the points in a plane,


which are at a fixed distance from a fixed point in the Q P Minor arc PQ Q
P
plane, is called a circle.
 Circumference : The length of the complete circle
The fixed point is called the centre of the circle and the is called its circumference.
fixed distance is called the radius of the circle.
Segment : The region between a chord and either
of its arcs is called a segment of the circular region or
O simply a segment of the circle. There are two types of
segments which are the major segment and the minor
P segment (as in figure).
In figure, O is the centre and the length OP is the radius
of the circle. So the line segment joining the centre
Major segment
and any point on the circle is called a radius of the
circle.
P Minor segment Q
 Chord : If we take two points P and Q on a circle, then
the line segment PQ is called a chord of the circle.
Sector : The region between an arc and the two radii,
joining the centre to the end points of an arc is called a
O sector. Minor arc corresponds to the minor sector and
the major arc corresponds to the major sector. When
P Q
two arcs are equal, then both segments and both
sectors become the same and each is known as a
 Diameter : The chord which passes through the semicircular region.
centre of the circle, is called the diameter of the circle.

Major sector Semicircular


B O region
O O
P Q
Minor Semicircular
A sector region
P Q

Theorem : Equal chords of a circle subtend equal


A diameter is the longest chord and all diameters of
angles at the centre.
same circle have the same length, which is equal to
two times the radius. In figure, AOB is a diameter of Given : AB and CD are the two equal chords of a circle
circle. with centre O.
To Prove : AOB = COD.
 Arc : A piece of a circle between two points is called
an arc. The longer one is called the major arc PQ and
the shorter one is called the minor arc PQ. The minor
A O D
arc PQ is also denoted by PQ and the major arc PQ by
`
QP . When P and Q are ends of a diameter, then both
arcs are equal and each is called a semi circle. B C
Proof : In  AOB and  COD,
OA = OC [Radii of a circle]
OB = OD [Radii of a circle]
AB = CD [Given]

PAGE # 129
 AOB  COD [By SSS congruency] We know that two lines (perpendicular bisectors) can
 AOB = COD. [By CPCT] Hence Proved. intersect at only one point, so we can draw only one
circle with radius OA. In other words, there is a unique
Converse :If the angles subtended by the chords of a circle passing through A, B and C. Hence Proved.
circle at the centre are equal, then the chords are equal.
Ex.1The radius of a circle is 13 cm and the length of one of
Theorem : The perpendicular from the centre of a circle its chords is 10 cm. Find the distance of the chord from
to a chord bisects the chord. the centre.
Sol. Let O be the center of the circle of radius 13 cm and AB
O is the chord of length 10 cm
OC AB
A M B AB 10
AC = = =5
Given : A circle with centre O. AB is a chord of this circle. 2 2
OM  AB. [Line perpendicular from centre to chord bisect the
To Prove : MA = MB. chord]
Construction : Join OA and OB. In AOC
Proof : In right triangles OMA and OMB, (OC)2 + (AC)2 = (AO)2
OA = OB [Radii of a circle] (OC)2 = (13)2 – (5)2
OM = OM [Common]
OC = 12 cm.
OMA = OMB [90º each]
  OMA   OMB [By RHS] Ex.2 In figure,  and O is the centre of the circle.
 MA = MB [By cpctc]
Prove that OA is the perpendicular bisector of BC.
Hence Proved.
Converse : The line drawn through the centre of a circle
to bisect a chord is perpendicular to the chord.
Theorem : There is one and only one circle passing
through three given non-collinear points.
Proof : Take three points A, B and C, which are not in
the same line, or in other words, they are not collinear
[as in figure ]. Draw perpendicular bisectors of AB and
BC say, PQ and RS respectively. Let these Sol. Given : In figure,  and O is the centre of the
perpendicular bisectors intersect at one point O.(Note circle.
that PQ and RS will intersect because they are not To Prove : OA is the perpendicular bisector of BC.
parallel) [as in figure]. Construction : Join OB and OC.
C
P Proof :
R O   [Given]
S
A  chord AB = chord AC.
[  If two arcs of a circle are congruent, then their
corresponding chords are equal]
Q B  AOB = AOC ...(i)
 O lies on the perpendicular bisector PQ of AB. [ Equal chords of a circle subtend equal angles at
 OA = OB the centre]
[ Every point on the perpendicular bisector of a line
In  OBD and  OCD,
segment is equidistant from its end points]
Similarly, DOB = DOC [From (i)]
 O lies on the perpendicular bisector RS of BC. OB = OC [Radii of the same circle]
 OB = OC OD = OD [Common]
[ Every point on the perpendicular bisector of a line  OBD  OCD [By SAS congruency]
segment is equidistant from its end points]  ODB = ODC ...(ii) [By CPCT]
So, OA = OB = OC And, BD = CD ...(iii) [By CPCT]
i.e., the points A, B and C are at equal distances from But BDC = 180º
the point O.  ODB + ODC = 180º
So, if we draw a circle with centre O and radius OA it  ODB + ODB = 180º [From equation(ii)]
will also pass through B and C. This shows that there
 2ODB = 180º
is a circle passing through the three points A, B and C.

PAGE # 130
 ODB = 90º  PE  QE [By CPCT]
1 1
 ODB = ODC = 90º ...(iv) [From (ii)]  PE – AB = QE – CD [  AB = CD (Given)]
2 2
So, by (iii) and (iv), OA is the perpendicular bisector of
 PE – PB = QE – QD
BC. Hence Proved.
 EB = ED. Hence Proved.
Theorem : Equal chords of a circle (or of congruent
circles) are equidistant from the centre (or centres).  REMARK :
C Angle Subtended by an Arc of a Circle :
A In figure, the angle subtended by the minor arc PQ at O
O
N is POQ and the angle subtended by the major arc PQ
M
at O is reflex angle POQ.
B D
Given : A circle have two equal chords AB & CD. i.e.
AB = CD and OM  AB, ON  CD. O
To Prove : OM = ON
Construction : Join OB & OD
P Q
Proof : AB = CD (Given)
[ The perpendicular drawn from the centre of a circle
to a chord bisects the chord] Theorem : The angle subtended by an arc at the centre
1 1 is double the angle subtended by it at any point on the
 AB = CD remaining part of the circle.
2 2
 BM = DN Given : An arc PQ of a circle subtending angles POQ at
In  OMB &  OND the centre O and PAQ at a point A on the remaining part
OMB = OND = 90º [Given] of the circle.
OB = OD [Radii of same circle] To Prove : POQ = 2PAQ.
BM = DN [Proved above] Construction : Join AO and extend it to a point B.
  OMB   OND [By R.H.S. congruency]
 OM = ON [By CPCT]
Hence Proved.
 REMARK :
Chords equidistant from the centre of a circle are equal
in length.
Ex.3 AB and CD are equal chords of a circle whose centre
is O. When produced, these chords meet at E. Prove
(A) (B) (C)
that EB = ED.
Sol. Given : AB and CD are equal chords of a circle whose Proof : There arises three cases:-
centre is O. When produced, these chords meet at E.
(A) arc PQ is minor
To Prove : EB = ED.
(B) arc PQ is a semi-circle
Construction : From O draw OP  AB and OQ  CD.
Join OE. (C) arc PQ is major.
Proof :  AB = CD [ Given] In all the cases,
 OP = OQ BOQ = OAQ + AQO ...(i)
[  Equal chords of a circle are equidistant from the [  An exterior angle of a triangle is equal to the sum of
centre] A the two interior opposite angles]
P
B In OAQ,
O E OA = OQ [Radii of a circle]
D  OQA = OAQ ...(ii)
Q
C [Angles opposite equal sides of a triangle are equal]
From (i) and (ii)
Now in OPE and OQE,
OPE = OQE [Each 90º] BOQ = 2OAQ ...(iii)
OE = OE [ Common] Similarly,
OP = OQ [ Proved above] BOP = 2OAP ...(iv)
  OPE   OQE [By RHS congruency] Adding (iii) and (iv), we get
BOP + BOQ = 2(OAP + OAQ)
 POQ = 2PAQ. ...(v)

PAGE # 131
 AOB = 60º
 NOTE :
1
For the case (C), where PQ is the major arc, (v) is  ACB = AOB = 30º
2
replaced by reflex angles. Consider arc ACB.
Thus, reflex POQ = 2PAQ. Clearly, it makes 360º – 60º = 300º at the centre O.
Ex.5. In the given figure, the chord ED is parallel to the
Theorem : Angles in the same segment of a circle are
equal. diameter AC. Find CED.
Proof : Let P and Q be any two points on a circle to B
form a chord PQ, A and C any other points on the 50º
remaining part of the circle and O be the centre of the
circle. Then, P O
A C
1 2

3
E D

Sol. CBE = 1 [Angles in the same segment]


 1 = 50º …(i) [  CBE = 50º]
 AEC = 90º ..(ii)
POQ = 2PAQ ...(i) [Angle in a semicircle is a right angle]
And POQ = 2PCQ ...(ii)
[Angle subtended at the centre is double than the angle
subtended by it on the remaining part of the circle]
From equation (i) & (ii)
2PAQ = 2PCQ
 PAQ = PCQ. Hence Proved.
Theorem : Angle in the semicircle is a right angle.

Now, in AEC,
1 + AEC + 2 = 180º
 50º + 90º + 2 = 180º
Proof :  PAQ is an angle in the segment, which is  2 = 180º – 140º = 40º
a semicircle. Thus  2 = 40º ...(iii)
1 1 Also, ED || AC [Given]
 PAQ = POQ = × 180º = 90º
2 2  2=3 [Alternate angles]
[ POQ is straight line angle or POQ = 180º]   3 = 40º
If we take any other point C on the semicircle, then Hence,  CED = 40º.
again we get
1 1
PCQ = POQ = × 180º = 90º.
2 2
Hence Proved.
A quadrilateral ABCD is called cyclic if all the four
Ex.4. A chord of a circle is equal to the radius of the circle,
vertices of it lie on a circle.
find the angle subtended by the chord at a point on the
minor arc and also at a point on the major arc.
Sol.
C

A B
Theorem : The sum of either pair of opposite angles of
D
a cyclic quadrilateral is 180º.
Let AB be a chord of the circle with centre at O such that
OA = OB = AB. Given : A cyclic quadrilateral ABCD.
OAB is equilateral.

PAGE # 132
D mentary]
C 130º + ABC = 180º
ABC = 50º
b = 90º [Angle in a semicircle]
A a = 180º – ABC – b
= 180º – 50º – 90º
B a = 180º– 140º = 40º.
To Prove : A + C = B + D = 180º. So, a = 40º and b = 90º.
Construction : Join AC and BD. Ex.8 If the nonparallel side of a trapezium are equal, prove
Proof : ACB = ADB ...(i) that it is cyclic.
And BAC = BDC ...(ii) Sol. Given : ABCD is a trapezium whose two non-parallel
[Angles of same segment of a circle are equal] sides AD and BC are equal.
Adding equation (i) & (ii) To Prove : Trapezium ABCD is a cyclic.
 ACB + BAC = ADB + BDC Construction : Draw BE || AD.
ACB + BAC = ADC.
Adding ABC to both sides, we get
ACB + BAC + ABC = ADC + ABC.
 ADC + ABC = 180º
Proof :  AB || DE [Given]
i.e., D + B = 180º and AD || BE [By construction]
 A + C = 360º – (B + D) = 180º  Quadrilateral ABED is a parallelogram.
[ A  B  C  D  360 º ] Hence Proved.  BAD = BED ...(i) [Opp. angles of a ||gm]
And , AD = BE ...(ii) [Opp. sides of a ||gm]
Ex.6 If a side of a cyclic quadrilateral is produced, then the But AD = BC ...(iii) [Given]
exterior angle is equal to the interior opposite angle. From (ii) and (iii),
Sol. Let ABCD be a cyclic quadrilateral inscribed in a circle BE = BC
with centre O. The side AB of quadrilateral ABCD is  BCE = BEC ...(iv)
produced to E. Then, we have to prove that [Angles opposite to equal sides]
CBE = ADC. BEC + BED = 180º [Linear Pair Axiom]
 BCE + BAD = 180º [From (iv) and (i)]
 Trapezium ABCD is cyclic.
[ If a pair of opposite angles of a quadrilateral is 180º,
then the quadrilateral is cyclic] Hence Proved.

Ex.9 In figure, O is the centre of the circle. Prove that


x + y = z.
Since the sum of opposite pairs of angles of a cyclic 1 1
Sol. EBF = EOF = z
quadrilateral is 180° 2 2
 ABC + ADC = 180° [ Angle subtended by an arc of a circle at the centre is
But ABC + CBE = 180° twice the angle subtended by it at any point of the
[ ABC and CBE form a linear pair]
remaining part of the circle]
 ABC + ADC = ABC + CBE
 ADC = CBE
or CBE = ADC. Hence proved
Ex.7 Find the value of a & b.
D C
130º b

A a 1
B  ABF = 180º – z ...(i) [Linear Pair Axiom]
O
2
1 1
EDF = EOF = z
2 2
[ Angle subtended by any arc of a circle at the centre
Sol. In ADC
AD = DC is twice the angle subtended by it at any point of the
DAC = DCA = x (say) remaining part of the circle]
130º + x + x = 180º 1
2x = 50º  x = 25º  ADE = 180º – z ...(ii)
2
ADC + ABC = 180º [Linear Pair Axiom]
[Opposite angles in a cyclic quadrilateral are supple- BCD = ECF = y [Vertically Opp. Angles]

PAGE # 133
BAD = x
In quadrilateral ABCD
ABC + BCD + CDA + BAD = 2 × 180º

1 1
 180º – z + y + 180º – z + x = 2 × 180º
2 2
Given : A circle C (O, r) and a tangent AB at a point P.
 x + y = z. Hence Proved. To prove : OP  AB

Ex.10 AB is a diameter of the circle with centre O and chord Construction : Take any point Q, other than P on the
CD is equal to radius OC. AC and BD produced meet tangent AB. Join OQ. Suppose OQ meets the circle at R.
at P. Prove that CPD = 60º. Proof : Among all line segments joining the point O to
Sol. Given : AB is a diameter of the circle with centre O and a point on AB, the shortest one is perpendicular to AB.
chord CD is equal to radius OC. AC and BD produced So, to prove that OP  AB, it is sufficient to prove that OP
meet at P. is shorter than any other segment joining O to any
To Prove : CPD = 60º. O point of AB.
A B
Construction : Join AD. Clearly OP = OR (Radius)
Proof : In  OCD, Now, OQ = OR + RQ
C D
OC = OD ...(i)  OQ > OR
[Radii of the same circle]  OQ > OP ( OP = OR)
P
OC = CD ...(ii) [Given] Thus, OP is shorter than any other segment joining O
From (i) and (ii), to any point of AB.
OC = OD = CD Hence, OP  AB.
 OCD is an equilateral triangle.
 COD = 60º
Theorem : Lengths of two tangents drawn from an
1 1
 CAD = COD = (60º) = 30º external point to a circle are equal.
2 2
[ Angle subtended by any arc of a circle at the centre
is twice the angle subtended by it at any point of the
remaining part of the circle]
 PAD = 30º ...(iii)
And,ADB = 90º ...(iv) [Angle in a semi-circle]
 ADB + ADP = 180º [Linear Pair Axiom] Given : AP and AQ are two tangents drawn from a point
 90º + ADP = 180º [From (iv)] A to a circle C (O, r).
 ADP = 90º ...(v) To prove : AP = AQ
In ADP, Construction : Join OP, OQ and OA.
APD + PAD + ADP = 180º Proof : In AOQ and APO
 APD + 30º + 90º = 180º [From (iii) and (v)] OQA = OPA [Tangent at any point of a circle is perp.
 APD + 120º = 180º to radius through the point of contact]
 APD = 180º – 120º = 60º AO = AO [Common]

 CPD = 60º. Hence Proved. OQ = OP [Radius]


So, by R.H.S. criterion of congruency AOQ  AOP
SECANT AND TANGENT  AQ = AP [By CPCT] Hence Proved.

Secant to a circle is a line which intersects the circle in RESULTS :


two distinct points.
(i) If two tangents are drawn to a circle from an external
A tangent to a circle is a line that intersects the circle in point, then they subtend equal angles at the centre.
exactly one point. QOA = POA [By CPCT]

Theorem : A tangent to a circle is perpendicular to the (ii) If two tangents are drawn to a circle from an external
radius through the point of contact. point, they are equally inclined to the segment, joining
the centre to that point OAQ = OAP [By CPCT]

PAGE # 134
Ex.11 If all the sides of a parallelogram touches a circle, Ex.13 Prove that the segment joining the points of contact
show that the parallelogram is a rhombus. of two parallel tangents passes through the centre.
Sol. Let PAQ and RBS be two parallel tangents to a circle
Sol. Given : Sides AB, BC, CD and DA of a || gm ABCD touch a
with centre O. Join OA and OB. Draw OC||PQ.
circle at P, Q, R and S respectively.
Now, PA || CO
To prove : ||gm ABCD is a rhombus.
D R C

S Q
O

A P B
Proof : AP = AS ....(i)  PAO + COA = 180º
BP = BQ ....(ii) [Sum of co-interior angle is 180º]
CR = CQ ....(iii)  90º + COA = 180º [ PAO = 90º ]
 COA = 90º
DR = DS ....(iv)
Similarly, COB = 90º
[Tangents drawn from an external point to a circle are
 COA + COB = 90º + 90º = 180º
equal] Hence, AOB is a straight line passing through O.
Adding (1), (2), (3) and (4), we get
 AP + BP + CR + DR = AS + BQ + CQ + DS SEGMENTS OF A CHORD
 (AP + BP) + (CR + DR) = (AS + DS) + (BQ + CQ)
Let AB be a chord of a circle, and let P be a point on AB
 AB + CD = AD + BC
inside the circle. Then, P is said to divide AB internally
 AB + AB = AD + AD
into two segments PA and PB.
[In a ||gm ABCD, opposite sides are equal] Theorem : If two chords of a circle intersect inside or outside
 2AB = 2AD or AB = AD the circle when produced, the rectangle fromed by two
But AB = CD and AD = BC [Opposite sides of a || gm] segments of one chord is equal in area to the rectangle
 AB = BC = CD = DA formed by the two segments of another chord.
Hence, || gm ABCD is a rhombus.

Ex.12 Prove that the tangents at the extremities of any chord D


make equal angles with the chord. A P
Sol. Let AB be a chord of a circle with centre O, and let AP
C B
and BP be the tangents at A and B respectively.
Suppose, the tangents meet at point P. Join OP. Fig. (i)
Suppose OP meets AB at C. Given : Two chords AB and CD of a circle C(O, r)
intersecting at P, inside in fig. (i) and outside in fig.(ii).
To prove : PA . PB = PC. PD
Construction : Join AC and BD.
Proof :

Case 1. When AB and CD intersect at P inside the


We have to prove that PAC = PBC
circle.
In triangles PCA and PCB,
In ’s APC and BPD,
PA = PB
[ Tangent from an external point are equal] APC = BPD [Vertically opposite angles]
APC = BPC PAC = PDB [Angles in the same segment]
[  PA and PB are equally inclined to OP]  APC ~ BPD
And PC= PC [Common] PA PC
Hence, = `
So, by SAS criteria of congruence PD PB
 PA × PB = PC × PD.
PAC  PBC
 PAC = PBC [By CPCT] Case 2. When AB and CD when produced intersect at
P outside the circle.

PAGE # 135
In PBD and PAC, PBD = ACP 35
OB = = 10 cm.
[ABCD is a cyclic quadrilateral and exterior angle is 3 .5
equal to the interior opposite angle] Ex.15 In ABC, AP  BC and BQ  AC which intersect each
PDB = CAP other at O. Prove that AO × OP = BO × OQ.
[ABCD is a cyclic quadrilateral and exterior angle is Sol. We have, APB = 90º
equal to the interior opposite angle] and AQB = 90º [ AP  BC and BQ  AC]
 PBD ~ PCA
PB PD
Hence, = `
PC PA
 PA × PB = PC × PD.

Theorem : If PAB is a secant to a circle intersecting the


circle at A and B and PT is a tangent segment, then
PA × PB = PT2.
Given : A circle with centre O; PAB is a secant
intersecting the circle at A and B and PT is a tangent
segment to the circle. In  AOQ &  BOP
2
To prove : PA × PB = PT .  AOQ =  BOP (Vertically opp. angles)
Construction : Join OA, OP and OT. Draw OD  AB.
 OQA =  OPB = 90º
T
By, AA Similarity
 AOQ ~  BOP
O
AO OQ
P B 
A D OB OP
Proof : Since OD  chord AB
 AO  OP = OB  OQ
 AD = DB.
Ex.16 In ABC, the bisector of A intersects the base BC at
Now PA × PB = (PD – AD) (PD + DB) the point D. Prove that AB × AC = BD × DC + AD2.
= (PD – AD)(PD + AD) [ AD = DB] Sol. Draw a circumcircle of ABC. Produce AD to meet the
2 2
= PD – AD circle at E. Join EC.
= (OP2 – OD2) – AD2 In s ABD and AEC, BAD = EAC
[From right-angled ODP] [ AD is the bisector of BAC]
ABD = AEC [s in the same segment]
= OP2 – (OD2 + AD2)
 ABD ~ AEC.
= OP2 – OA2
[From right-angled ODA]
= OP2 – OT2  OA = OT [Radii]
2
= PT
[From right-angled OTP]
Hence, PA × PB = PT2.

Ex.14 Two chords AB and CD of a circle intersect each


AD AB
other at O internally. If AO = 3.5 cm, CO = 5 cm and DO  =
AC AE
= 7 cm, find OB.  AB × AC = AD × AE = AD × (AD + DE)
AB × AC = AD2 + AD × DE = AD2 + BD × BC
[ AD × DE = BD × DC]

Sol. ANGLES IN THE ALTERNATE SEGMENTS

Let PAQ be a tangent to a circle at point A and AB be a


chord. Then, the segment opposite to the angle formed
by the chord of a circle with the tangent at a point is
called the alternate segment for that angle.
We have, AO × OB = CO × OD
 (3.5) × OB = 5 × 7 = 35

PAGE # 136
Theorem : A line touches a circle and from the point of Sol. (a) ATQ = 180º – (70º + 45º)
contact a chord is drawn. Prove that the angles which [ Sum of the adjacent s = 180º]
the chord makes with the given line are equal = 180º – 115º = 65º.
respectively to angles formed in the corresponding
 ABT = ATQ = 65º
alternate segments.
[s in the alternate segment]
Given : A circle with centre O and PAT is a tangent to the
(b) SRT = 90º
circle at A. A chord AB is drawn. Let C and D be the
[Angle in a semi circle]
points on the circumference on opposite sides of AB.
To prove : BAT = ACB and BAP = ADB. NowRST = TRQ = 30º
Construction : Draw the diameter AOE and Join EB. [s in the alternate segments]
Proof : Since AE is the diameter,  RTS = 90º – 30º = 60º.
 ABE = 90º [Angle in semi circle] Ex.18 ABCD is a cyclic quadrilateral and PQ is a tangent to
 BEA + EAB = 90º ...(i) the circle at C. If BD is a diameter and DCQ = 40º and
[Remaining s of BEA] ABD = 60º ; find the measure of the following angles :
But since EA  AT
(i) DBC (ii) BCP (iii) ADB.
[ Radius is  to the tangent]
Sol. Since BD is a diameter of the circle.
 EAB + BAT = 90º ...(ii)
 BAD = 90º
Hence from (i) and (ii)
BEA + EAB = EAB + BAT ...(iii) and also BCD = 90º

(i) DBC = DCQ = 40º


[s is the alternate segments]
(ii) BCP + BCD + DCQ = 180º
 BCP + 90º + 40º = 180º
 BCP = 50º

(iii) Similarly from BAD, 60º + BAD + ADB = 180º


 60º + 90º + ADB = 180º
ADB = 30º.
Ex.19 AB is diameter and AC is a chord of a circle such that
 BEA = BAT
BAC = 30º. The tangent at C intersects AB produced
But BEA = ACB [Angles in the same segment]
at D. Prove that BC = BD.
ACB = BAT
Sol. Since AB is the diameter of the circle
i.e. BAT = ACB.
 ACB = 90º [Angle in a semi circle]
This proves the first part.
 ABC = 90º – 30º = 60º ...(i)
Again, since ADBC is a cyclic quadrilateral.
Again, BCD = CAB = 30º
 ACB + ADB. = 180º
[s in the alternate segments]
Also BAT + BAP = 180º [Linear pair]
Now in CBD,
 ACB + ADB =BAT + BAP
 BAT + ADB = BAT + BAP
[ ACB = BAT]
 ADB = BAP
 BAP = ADB
Hence, BAT = ACB and BAP = ADB.

Ex.17 Two figures are given below :


ext.CBA = BCD + BDC
 60º = 30º + BDC
 BDC = 30º
Hence, BCD = BDC
 BC = AD.

Ex.20 In the adjoining figure, AP is a tangent to the circle at


P. ABC is a secant such that PD is the bisector of BPC.
(a) In fig (i) PQ is a tangent to the circle at T, BTA = 45º Prove that
and PTB = 70º. Find ABT. (i) ADP = APD
(b) In fig (ii) O is the centre and PQ is a tangent to circle (ii) AP = AD
at R. Calculate STR. 1
(iii) BPD = (ABP – APB)
2

PAGE # 137
Sol. AP is a tangent to the circle and PB is a chord through Ex.22 In a right triangle ABC, the perpendicular BD on the
the point of contact P. hypotenuse AC is drawn. Prove that
 1 = 2 [s in the alternate segments]
(i) AC × AD = AB2 (ii) AC × CD = BC2
 1 + 4 = 2 + 3
[ 3 = 4 since PD bisects CPB] Sol. We draw a circle with BC as diameter. Since  BDC = 90º.
 APD = 2 + 3 ..(i)  The circle on BC as diameter will pass through D.
But from PCD, ext. PDA = 2 + 3 Again
Hence from (i), APD = PDA  BC is a diameter and AB  BC.
 AB is a tangent to the circle at B.
Since AB is a tangent and ADC is a secant to the circle.

Thus (i) is proved.


Again APD = PDA  AC × AD = AB2 This proves (i)
 AP = AD. Again
Further, from PCB, ext. PBA = sum of the two AC × CD = AC × (AC – AD) = AC2 – AC × AD
opposite interior angles viz.2 + (3 + 4) = AC2 – AB2 [Using (i)]
= 1 + (4 + 4) [ 2 =1 & 3 = 4]
= BC2 [ABC is a right triangle]
= APB + 2BPD
2
 2BPD = PBA – APB Hence, AC × CD = BC . This proves (ii).
1 Ex.23 In the given figure, PAT is a tangent to the circumcircle
 BPD = (PBA – APB). of a ABC at the vertex A. A line parallel to PAT intersects
2
the sides AB and AC at the points D and E respectively.
Ex.21 In a right-angled ABC, a circle with side AB as
diameter is drawn to intersect the hypotenuse AC in L.
Prove that the tangent to the circle at L bisects the side
BC.
Sol. Let the tangent at L meet BC in M. Join BL.
A
Prove that (a) ABC ~ AED (b) AB · AD = AC · AE
2 L
Sol. TAC =  TAE =  ABC … (i)
3 [Angles in the alternate segment are equal
O
Similarly,  PAB =  PAD =  ACB … (ii)
1
[Angles in the alternate segment are equal]
B C Also  TAE =  AED (alternate angles) … (iii)
M
Firstly, ALB = 90º [ in a semi circle]
 BLC = 90º
Since the lengths of tangents from an external point
are equal.
 MB = ML
Now LM is a tangent to the circle at L and LB is a chord  PAD =  ADE [alternate angles]… (iv)
through the point of contact L From (i) to (iv), we get
 3 = 2  ABC =  AED
Further,MLC = BLC – 3 = 90º – 3  ACB =  ADE
and LCM = ACB = 90º – 2 = 90º – 3 [ 2 = 3]   ABC ~  AED [By AA similarity]
MLC = LCM
Since corresponding sides of similar triangles are
 ML = MC
proportional.
Thus BM = LM = MC.
Hence, M is the mid-point of BC. AB AC
=  AB × AD = AC × AE
AE AD
Hence, the required results are proved.

PAGE # 138
Ex.24 Two circles touch externally at P and a common
tangent touches them at A and B. Prove that
(i) the common tangent at P bisects AB.
(ii) AB subtends a right angle at P.
Sol. Let PT be the common tangent at any point P. Since the
tangent to a circle from an external point are equal,
 TA = TP, TB = TP  TA = TB
i.e. PT bisects AB at T (a) In fig. (i) d > r1 + r2 i.e. two circles do not intersect.
TA = TP gives  TAP =  TPA (from PAT) In this case, four common tangents are possible.
The tangent lines l and m are called direct common
tangents and the tangent lines p and q are called
indirect (transverse) common tangents.

(b) In fig. (ii), d = r1 + r2. In this case, two circles touch


externally and there are three common tangents.
(c) In fig.(iii) d < r1 + r2. In this case two circles intersect
in two distinct points and there are only two common
tangents.
TB = TP gives  TBP =  TPB [from PBT]
(d) In fig. (iv), d = r1 – r2 (r1 > r2), in this case, two circles
  TAP +  TBP = TPA + TPB = APB
touch internally and there is only one common tangent.
  TAP +  TBP + APB = 2  APB
(e) In fig. (v), the circle C(O2, r2) lies wholly in the circle
 2  APB = 180º [sum of s of a  = 180º]
C(O1, r1) and there is no common tangent.
  APB = 90º Hence proved
Ex.25 Two circles touch each other externally. Their radii
are 9 cm and 4 cm and the distance between their
Definition : A line which touches the two given circles is centres is 13 cm. Find the length of their common
called common tangent to the two circles. Let tangent segment.
C(O1, r1), C(O2, r2) be two given circles. Let the distance Sol. Given : Two circles C(O, 9) and C(O’, 4) touch externally
between centres O1 and O2 be d i.e., O1O2 = d. at S.
 OO’ = 9 cm + 4 cm = 13 cm.
PQ is the common tangent segment.
Construction : Join OP and O’Q. Then OP  PQ and
O’Q  PQ. From O’ draw O’R  OP.
Proof : Clearly PQ = O’R
Also, RP = O’Q = 4 cm.
 RP = OP – RP = 9 – 4 = 5 cm.

Now from right-angled ORO’, OO’2 = OR2 + O’R2


 132 = 52 + PQ2
 PQ2 = 132 – 52 = (13 – 5)(13 + 5)
 PQ2 = 8 × 18 = 9 × 16
 PQ = 3 × 4 = 12 cm.

PTOLEMY’S THEOREM

In cyclic quadrilateral, the product of the diagonals is


equal to the sum of the product of the opposite sides.
A
D

B
C
AC × BD = AB × DC + BC × AD

PAGE # 139
8. In figure,  BDC =
(A) 95°
(B) 105°
NOTE : More than one correct option may be possible.
(C) 100°
PROBLEMS BASED ON CHORD (D) 110°
1. If a diameter of a circle bisects each of the two chords
of a circle then the chords are : 9. In figure, O is centre, then reflex  AOC =
(A) intersecting (B) concurrent
(C) collinear (D) parallel (A) 190°
(B) 160º
2. OD is perpendicular to a chord AB of a circle whose
(C) 200°
centre is O. If BC is a diameter, if CA = k × OD then k
(D) 185°
equals to

10. In figure, O is centre, then  BXD =

(A) 65°

(B) 60°

(A) 2 (B) 2.5


(C) 70°
(C) 3 (D) 3.5
3. Two chords AB & CD of lengths 6 cm & 8 cm of a circle
are parallel and are on the same side of its centre. If (D) 55°
the distance between them is 1 cm. Find the radius of
the circle. 11. In figure, O is centre, then  AEB =
(A) 3 cm (B) 4 cm (A) 60°
(C) 5 cm (D) 2 cm (B) 65°
4. AB and AC are two chords of the circle of (C) 70°
radius r. If AB = 2AC and the perpendicular drawn from (D) 75°
the centre on those chords are of lengths a and b. If
b2 = k (a2 + 3r2) then k =
(A) 1 (B) 2
12. In figure, PQ is a diameter of a circle with centre at O
(C) 3 (D) 1/4
and OR  PQ, where R is a point on the circle. If S is
5. In the figure given above, A and B are the centers of the another point on the circle such that  RPS = 32°, then
two congruent circles with radius 17 units. If AB = 30  QRS is :
(A) 13°
units, the length of the common chord DC is :
(B) 26°
(A) 25 units
(C) 45°
(B) 18 units (D) None of these
(C) 10 units
(D) 16 units 13. In figure, PQRS is a cyclic quadrilateral. Its diagonals
PR and QS intersect each other at T.
6. Let P be a point on the circumference of a circle. Per- If  PRS = 80º and RQS = 50º, calculate  PSR.
pendiculars PA and PB are drawn to points A and B on (A) 30°
two mutually perpendicular diameters. If AB = 36 cm,
the diameter of the circle is : (B) 50°
(A) 16 cm (B) 24 cm
(C) 70°
(C) 36 cm (D) 72 cm

PROBLEMS BASED ON ANGLES (D) 90°


7. A, B, C are three points on the circumference of a circle
14. C is the centre of a circle passing through the points P,
with centre O such that  OAC = 53° and  CBO = 32°,
Q, R and S taken in order. If  PSR = 120º and PQ is a
then  AOB =
diameter, then  QPR =
(A) 100° (B) 120°
(A) 30º (B) 40º
(C) 150° (D) 170°
(C) 45º (D) 50º

PAGE # 140
15. In figure,  ADC = 22. In the given figure, O, O’ are centres of two circles
(A) 60º intersecting at B and C. ACD is a straight line. Find x.

(B) 70º
(A) 130º
(C) 75º (B) 50º
(D) 80º (C) 40º
(D) None of these
16. In figure, PQ = PR, find  QTR.
(A) 35º
23. ABCD is a cyclic quadrilateral inscribed in a circle with
(B) 70º the centre O. Then OAD is equal to
C
(C) 110º (A) 30º
50º
(D) None of these D
(B) 40º O
17. In figure, CDEF is a cyclic quadrilateral, DE and CF are

30º
produced to A and B respectively such that AB || CD. If 40º
 FED = 80°, find  FBA. (C) 50º A B
(A) 30º
(D) 60º
(B) 60º
(C) 80º 24. In the given figure find the value of AOC.

(D) None of these


(A) 130º
18. ABC is an equilateral triangle. Find  BEC.

(A) 30º (B) 140º

(B) 60º
(C) 150º
(C) 90º
(D) 160º
(D) 120º

19. Find  ASR. 25. O is the centre of the circle in the figure given below.
The correct relationship between a, b and c is :
(A) 52º D

(B) 78º (A) a + b = c C

(C) 102º
(B) a = b + c 2 1

b
(D) None of these O
3
(C) a + c = b 4
a
20. ABCD is a cyclic quadrilateral. Then, find xº as given
in the figure, below. E (D) None of these
D 80º
(A) 50º C
26. In the given figure, AB is the diameter of a circle C (O, r).
(B) 80º Chord CD is equal to radius OC. If AC and BD intersect
(C) 90º at P, find APB.
A B
x
(D) 100º (A) 30º
F O
A B

21. In the given figure O is the centre of the circle. The angle (B) 45º
subtended by the arc BCD at the centre is 1400 . Find C D

BAD . (C) 60º P


(A) 40º
(B) 50º (D) 90º
(C) 20º
(D) 70º

PAGE # 141
27. In the given figure value of 'a' is : 32. In the given figure, find the value of 'a' is :

D C (A) 170º
(A) 30º 130º b

(B) 20º
a
(B) 40º A
O
B

(C) 10º
(C) 60º
(D) 5º

(D) 90º 33. In the given figure, find the measure of ABC, if ABCD
is a cyclic quadrilateral.
28. In the adjoining figure, chord ED is parallel to the A
diameter AC of the circle. If CBE = 65º, then what is (A) 120º
B
the value of DEC ?
Y 62º
(B) 118º
(A) 35º 82º
46º
(C) 138º D C
(B) 55º

(C) 45º (D) 128º

34. Given, a quadrilateral ABCD is inscribed in a circle as


(D) 25º
shown in the figure below.
29. In the adjoining figure, the chords XY and SR are If B = 125º, then E is equal to _____
E
produced to meet outside the circle at P. The chords
XR and YS meet inside the circle at T. If X = xº and D C
XTS = yº, then P is equal to :
A
B
(A) (x + y)º
(A) 55º (B) 125º
(B) (y – x)º (C) 130º (D) 62.5º

(C) 2(y – x)º PROBLEM BASED ON SECANTS AND


TA N G E N T S
35. In the circle with centre ‘O’ as shown, chord AB and CD
(D) (y – 2x)º
intersect at P and are perpendicular to each other. If
30. If PR is a diameter of the circle and the angles RPS, AP = 4, PB = 6 and PC = 2, then the area of the circle is
QPR, QRP and PRS forms an AP with RPS = 15º, then D
the value of the angle QRP is :

(A) 55º O
Q

(B) 75º A P B
P R
C
(C) 60º (A) 45 (B) 49
S

(D) 35º (C) 50 (D) 41

36. In the circle shown AB = 24, and the perpendicular


31. In the given figure, BCP is equal to : chord CD bisects AB. If DM is 4 times as long as CM
then the length of BD, is
(A) 60º
C B

(B) 100º M
A

(C) 50º
D
(A) 8 5 (B) 12 5

(D) 130º (C) 16 5 (D) 20 5

PAGE # 142
37. In the given figure, O is the centre of a circle and BD is 42. In a triangle ABC, AB = 130, AC = 200 and BC = 260.
a diameter. AB and AC are tangents touching the circle Point D is chosen on BC so that the circles inscribed
in triangle ABD and ADC are tangent to AD at the same
at B & C respectively. If BAC = 700 then OBC is :
point. Length of BD is equal to :
(A) 300 A

(B) 350

(C) 400

(D) 450 B C
D
(A) 105 (B) 95
38. In the given figure AQ and AR are the tangents drawn (C) 90 (D) 85
from A to the circle with centre O. If the measure of
43. ABC is a right angled triangle, right angled at B such
QOR is 120º, then QAR is equal to : that BC = 6 cm and AB = 8 cm. A circle with centre O is
inscribed in ABC. The radius of the circle is :
(A) 45º
(A) 1 cm (B) 2 cm
(B) 50º (C) 3 cm (D) 4 cm
44. A circle is inscribe in trapezoid PQRS. If PS = QR = 25
(C) 65º
cm, PQ = 18 cm and SR = 32 cm, what is the length of
(D) 60º the diameter of the circle ?
39. Triangle PAB is formed by three tangents to circle O P Q
(A) 14 cm
and APB = 40º, then angle AOB
R (B) 25 cm

B (C) 24 cm
O S R
Q (D) 674 cm
40º 45. PQ is a chord of a circle. The tangent XR at X on the
P
A T circle cuts PQ produced at R. If XR = 12 cm, PQ = x cm,
QR = x – 2 cm, then x in cm is :
(A) 45º (B) 50º (A) 6 X
(B) 7
(C) 60º (D) 70º 12
(C) 10
40. In the figure, AB is a diameter of the circle. TD is a (D) 14 x x– 2
R
P Q
tangent. If AHD = 36º, CDT is :

(A) 120º
COMPETITIVE EXAM PREVIOUS YEAR
(B) 116º QUESTION
1. A circle of radius 25 units has a chord going through a
point that is located 10 units from the centre. What is
(C) 106º
the shortest possible length that chord could have ?
[NSTSE-2009]
(D) 126º
(A) 25 units (B) 525 units

41. Two circles of radii 4 cm and 14 cm have a common (C) 40 units (D) 2100 units
external tangent of length 24 cm. The distance between 2. Given a chord AB in a circle as shown. If two more
the centres of these circles (in cm) is : chords AD and BE are drawn perpendicular to AB as
shown in the figure, then : [NSTSE-2009]

D E

A B

(A) 24 (B) 25 (A) AD = BE (B) AD = 2BE


(C) 26 (D) 27 (C) 2AD = BE (D) AD  BE

PAGE # 143
3. In the figure shown, the bigger circle has radius 1 unit. 9. PQ is a chord of a circle. The tangent XR at X on the
Therefore, the radius of smaller circle must be : circle cuts PQ produced at R. If XR = 12 cm, PQ = x cm,
[IJSO-2009] QR = x – 2 cm, then x in cm is : [NSTSE 2010]
(A) 2 +1 (A) 6
X
1 (B) 7
(B) 12
2 (C) 10
x x– 2
1 (D) 14 P
R
Q
(C)
2
1
(D) 10. A point P is outside a circle at a distance of 13 cm from
2 1 its centre. A secant from P cuts the circle in Q and R
4. Two circles of radii 2 and 3 cm touch each other such that QR = 7 cm and the segment PQ of the secant
externally. The length of direct common tangent to the exterior to the circle is 9 cm. Therefore, the radius of
circle is : [IJSO-2010]
two circles will be : [IJSO-2009]
(A) 3 cm (B) 4 cm
(A) 2 6 cm (B) 26 cm (C) 5 cm (D) 6 cm
(C) 5 cm (D) 2.4 cm
11. The hypotenuse of a right angled triangle is 10 cm and
5. In the figure shown [IJSO-2009]
the radius of its inscribed circle is 1cm. Therefore,
perimeter of the triangle is : [IJSO-2010]
(A) 22 cm (B) 24 cm
O (C) 26 cm (D) 30 cm

140º 12. In the figure shown, chord ED is parallel to diameter AC


of a circle. If CBE = 60º, then DEC must be :
[IJSO-2010]
(A) + = 110º (B)  = = 55º
(C) = 110º (D) Both A & C B
6. Two circles with centres A and B have radii 8 cm and 60º
1 cm respectively. The distance AB is 13 cm . A third
A C
circle with centre C and radius r cm touches both circles O
externally. If angle ACB is 90º, then value of r is :
[IJSO-2009]
E D
(A) 11 (B) 4
(C) 5 (D) 2
(A) 15º (B) 30º
7. In the diagram, PQR is a tangent to the circle at Q and (C) 10º (D) 20º
"O" is the centre of the circle. Find the value of "X".
13. In the figure shown below, m ABD = 2x, m ADB = 3x,
[NSTSE 2009]
m APB = 4x and m CBQ = 7x. Therefore, m BCD is
(A) 25º T

S [IJSO-2011]
O
(B) 30º
70º 50º
(C) 35º
P Q R

(D) 40º

8. A circle is inscribe in trapezoid PQRS. If PS = QR = 25


cm, PQ = 18 cm and SR = 32 cm, what is the length of
(A) 90º (B) 85º
the diameter of the circle ? [NSTSE 2009] (C) 72º (D) 96º
(A) 14 cm P Q
14. If ABCD is a cyclic quadrilateral. AB = 204, BC = 104,
(B) 25 cm CD = 195, DA = 85 and BD = 221.
Then find AC [IJSO-2012]
(C) 24 cm
S R (A) 205 (B) 210
(D) 674 cm
(C) 220 (D) 225

PAGE # 144
15. A circle is inscribed in an isosceles trapezium ABCD 17. Two parallel chords 96 cm and 28 cm long are on the
in which AB is parallel DC. If AB = 10 and DC = 30. Find opposite side of the centre of the circle with radius 50
the area of the circle. [IJSO-2012] cm. Find the area of the quadrilateral whose vertices
(A) 45 (B) 50 are the end points of the chords. [IJSO-2015]
(C) 60 (D) 75
(A) 3488 (B) 3848
(C) 3844 (D) 3484
16. Two circles each of radius 3 touch each other exter-
nally in the plane. Inhow many ways can a circle of
radius 8 be placed inthe plane touching each of these 18. In a n - sided regular polygon, the radius of the circum-
two circles ? [IJSO-2013] circle is equal in length to the shortest diagonal. The
(A) 2 (B) 4 number of values of n < 60 for which this can happen
is [IJSO-2016]
(C) 6 (D) 8
(A) 0 (B) 1
(C) 10 (D) 2



PAGE # 145
M E N S U R AT I O N

(C) Hypotenuse, d = b 2  h 2 [Pythagoras theorem]


Perimeter : The perimeter of a plane figure is the length (iv) Isosceles right-angled triangle
of its boundary. The unit of perimeter is same as the
unit of length i.e cm, m, etc. For an isosceles right-angled triangle, let a be the
Area : The area of plane figure is the measure of the equal sides, then
surface enclosed by its boundary. The unit of area is
cm2, m2, etc. (A) Hypotenuse = a2  a2 = 2a

AREA & PERIMETER OF TRIANGLES (B) Perimeter = 2a + 2a


A

a 2a

D
B a C

(i) Scalene triangle 1 1 1 2


(C) Area = (Base × Height) = (a × a) = a.
Perimeter = a + b + c 2 2 2

1 1 (v) Equilateral triangle


Area = × Base × Height = ah
2 2
3
Heron’s formula : Area = (side)2, Perimeter = 3(side)
4

Area = s( s  a)( s  b)( s  c ) Ex.1 The sides of a triangle containing the right triangle are
5x cm and (3x – 1) cm. If the area of the triangle is
abc 60 cm 2, calculate the lengths of the sides of the
Where, s = semi-perimeter =
2 triangle.
(ii) Isosceles triangle Sol. Since, area of a right angled triangle

1
1 1 = × product of its sides containing the right triangle
Area =  base  (equal side )2  (base )2 2
2 4
1
 60 = × 5x × (3x – 1)
(iii) Right-angled triangle. 2
 120 = 15 x2 – 5x  15x2 – 5x – 120 = 0
For an right-angled triangle, let b be the base, h be the
 3x2 – x – 24 = 0
perpendicular and d be the hypotenuse. Then
8
(A) Perimeter = b + h + d On solving we get x = 3 and x =  .
3
8
1 1 Since, x =  will give negative values of the sides of
(B) Area = (Base × Height) = bh 3
2 2 the triangle, which is impossible; therefore, x = 3.
A
 Length of the sides = 5 (3) = 15 cm and (3 × 3 – 1) = 8 cm.
Ex.2 Find the perimeter of an equilateral triangle whose
d area is equal to that of a triangle with sides 21 cm,
h
16 cm and 13 cm. Answer correct to 2 decimal places.
Sol. For the given triangle a = 21 cm, b = 16 cm and
c = 13 cm.
B b C abc
 s=
2
21  16  13
= = 25 cm.
2

PAGE # 146
So, Area = s(s  a)(s  b)(s  c ) 1 1
= AC × BX + AC × DX
2 2
= 25(25  21)(25  16)(25  13)
1 1
= 60 3 cm2 = AC ( BX + DX ) = AC × BD
2 2
Given :
Area of equilateral triangle = Area of this triangle 1
= × The product of the diagonals
3 2
 ( side )2 = 60 3
4
 (side)2 = 240
 side = 240 = 4 15 cm
1. Rectangle :
Perimeter of the equilateral triangle = 3 (side)
= 3 × 4 15
= 12 × 3.873
= 46.48 cm (Approx.)

AREA & PERIMETER OF QUADRILATERALS 

1. When one diagonal and perpendiculars to this Perimeter = 2 (+ b)


diagonal from the remaining vertices are given. Area =  × b
Length of diagonal (d) = 2  b2
A D 2. Square :

B C

In quadrilateral ABCD, the diagonal AC and Perimeter = 4a


perpendiculars BX and DY to AC from the remaining Area = a2
vertices B and D respectively are given, then the
Length of diagonal = a 2
Area of quadrilateral ABCD = Ar. of ABC + Ar. of ADC
3. Parallelogram :
1 1
= AC × BX + AC × DY
2 2
1
= AC ( BX + DY )
2
1
 Area of quadrilateral = × one diagonal × sum of
2
the lengths of the perpendiculars drawn on it from the
remaining two vertices
2. When two diagonals of a quadrilateral cut each Perimeter = 2 (a + b)
other at right angles. Area = ah1 = bh2
D 4. Rhombus :

A C
X

B
Area of Quadrilateral ABCD
= Ar. of ABC + Ar. of ADC
Perimeter = 4a = 2 d12  d22

1
Area = dd
2 1 2

PAGE # 147
5. Trapezium : Ex.5 The base of a parallelogram is thrice its height. If the
b area is 876 cm 2, find the base and height of the
D C
parallelogram.
Sol. Let the height of the parallelogram be x cm. Then,
h
base = 3x cm.
 Area of the parallelogram= (x × 3x) cm2 = 3x2 cm2
A B But, area of the parallelogram is given as 867 cm2
a
1  3x2 = 867  x2 = 289  x2 = 172  x = 17
Area = h (a + b) Thus, height = 17 cm and base = (3 × 17) cm = 51 cm.
2
Ex.3 The length and breadth of a rectangular field are in the Ex.6 The area of a rhombus is 72 cm2. If its perimeter is 32
ratio 3 : 2. If the area of the field is 3456 m2, find the cost cm, find its altitude.
of fencing the field at Rs 3.50 per metre. Sol. We have, perimeter of the rhombus = 32 cm
Sol. Let the length and breadth of the rectangular field be  4 (side) = 32 cm [ Perimeter = 4 (side)]
3x and 2x metres respectively. Then, 32
Area of the rectangular field = (3x  2x)m2 = 6x2 m2.  side = cm = 8 cm
4
Also, area of the rectangular field = 3456 m2 Now, area of the rhombus = 72 cm2

3456  (Side  Altitude) = 72


 6x2 = 3456  x2 =
6  8  Altitude = 72
72
 x2 = 576  x= 576 = 24  Altitude = 8 cm = 9 cm
 length = (3  24) m = 72 m, breadth = (2  24) m Ex.7 If the area of a rhombus be 48 cm2 and one of its
= 48 m diagonal is 12 cm, find its altitude.
 Perimeter of the field = 2 (length + breadth) Sol. Let ABCD be a rhombus of area 48 cm2 and diagonal
= [2  (72 + 48)]m = 240 m BD = 12 cm.
Rate of fencing = Rs 3.50 per metre Now, Area = 48 cm2 D C
 Cost of fencing = Rs (240  3.50) = Rs 840. 1
  AC  BD = 48
Ex.4 A grassy plot is 80 m × 60 m . Two cross paths each 4 2 O
m wide are constructed at right angles through the 1
  AC  12 = 48
centre of the field , such that each path is parallel to 2 A B
one of the sides of rectangle. Find the total area used  6  AC = 48
as path. Also, find the cost of gravelling them at Rs 5 48
per square metre.  AC = cm = 8 cm
6
Sol. Let ABCD and EFGH be the cross paths. Since the diagonal of a rhombus bisect each other at
We have , AB = 80 m and BC = 4 m. right angles.
 Area of path ABCD = (80 × 4) m2 = 320 m2 1 1
Again, EF = 60 m and FG = 4 m  OA = AC = 4 cm, OB = BD = 6 cm.
2 2
 Area of path EFGH = (60 × 4) m2 = 240 m2. Also, AB2 = OA2 + OB2 [Using pythagoras theorem]
Clearly, area PQRS is common to both the paths. 2 2 2 
 AB = 4 + 6 AB2 = 16 + 36
We have,
Area PQRS = (4 × 4) m2 = 16 m2  AB = 52
 Total area used as path = Area of path ABCD + Area Since a rhombus is a parallelogram also, therefore,
of path EFGH – Area PQRS Area of rhombus = AB  Altitude
= (320 + 240 – 16) m2 = 544 m2 48
Rate of gravelling the path = Rs 5 per square metre  48 = 52  Altitude  Altitude = cm .
52
 Total cost of gravelling the path
= Rs ( 5 × 544) = Rs 2720 Ex.8 Find the area of a trapezium whose parallel sides are
25 cm, 13 cm and other sides are 15 cm and 15 cm.
E H
Sol. Let ABCD be the given trapezium in which AB = 25 cm,
CD = 13 cm, BC = 15 cm and AD = 15 cm.
4m Draw CE || AD.
S R
D C
60 m

D 13 cm
C
A B
P Q
4m
15 cm

F G
A E L B
80 m 25 cm

PAGE # 148
Now, ADCE is a parallelogram is which AD || CE and (i) Circumference = 2r or d, where d = 2r is the
AE || CD. diameter of the circle.
 AE = DC = 13 cm (ii) Area = r2.
and BE = AB – AE = 25 – 13 = 12 cm.
r 2
In BCE, we have (iii) Area of semi-circle = .
2
15  15  12 (iv) Perimeter of the semi-circle = r + 2r.
s= = 21
2
(v) Area enclosed by two concentric circles
 Area of BCE = s( s  a)( s  b)(s  c ) = R2 – r2 =  (R2 – r2) = (R + r) (R – r)
Where R and r and radii of two concentric circles.
 Area of BCE = 21( 21  15 )( 21  15 )(21  12 )
 Area of BCE = 21 6  6  9
= 18 21 cm2 ...(i)
Let h be the height of BCE, then
r R
1
Area of BCE = (Base × Height)
2
1
= × 12 × h = 6h ..(ii) NOTE :
2
From (i) and (ii), we have, 1. If two circles touch each other externally, then the
distance between their centres is equal to sum of their
6h = 18 21
radii.
 h = 3 21 cm
Clearly, the height of trapezium ABCD is same as 2. If two circles touch each other internally, then the
that of BCE distance between their centres is equal to difference
of their radii.
1
 Area of trapezium = (AB + CD) × h
2 3. The distance moved by a rotating wheel in one
1 revolution is equal to the circumference of the wheel.
 Area of trapezium = (25 + 13) × 3 21 cm2
2
Ex.9 If the perimeter of a semi-circular protractor is 66 cm,
= 57 21 cm2. 22
find the diameter of the protractor (Take = ).
7
Sol. Let the radius of the protractor be r cm.
PERIMETER AND AREA OF A CIRCLE
Then, Perimeter = 66 cm
Circle :The collection of all points in a plane, which are  r  2r  66
at a fixed distance from a fixed point in the plane, is
called a circle.  22 
The fixed point is called the centre of the circle and the  r   2  66
 7 
fixed distance is called the radius of the circle. The
diameter of a circle is twice its radius.
 36 
In figure, O is the centre and the length OP is the radius  r   66
of the circle.  7 

66  7
 r
36

77
 r
6
The length of the boundary of a circle is called its
77
circumference or its perimeter. We know that the ratio  Diameter = 2r = .
of the circumference of a circle to its diameter is always 3
a constant. This constant ratio is denoted by the greek Ex.10 Two circles touch externally. The sum of their areas
letter . is 130  sq. cm. and the distance between their centres
Circumference is 14 cm. Find the radii of the circles.
= Sol.
Diameter
Circumference =  × 2r = 2r [Diameter = 2r]
The exact value of  is not known, because  is an r1 r2
irrational number. For all practical purposes, the value C1 C2
22
of  is approximately taken as or 3.14
7
If r is the radius of a circle, then

PAGE # 149
If two circles touch externally, then the distance between Ex.13 A wire is looped in the form of a circle of radius 28
their centres is equal to the sum of their radii. cm. It is re-bent into a square form. Determine the
Let the radii of the two circles be r1 cm and r2 cm
length of the side of the square.
respectively.
Let C1 and C2 be the centres of the given circles. Then, Sol. We have,
Length of the wire = Circumference of the circle
C1C2 = r1 + r2  ClC 2  14 cm (given )
 14 = r1 + r2  22 
 r1 + r2 = 14 ...(i) Length of the wire = 2   28cm
 7 
It is given that the sum of the areas of two circles is
equal to 130  cm2. Length of the wire = 176 cm ....(i)
 r12 + r22 = 130  Let the side of the square be x cm.
 r12  r22  130 ...(ii) Then,
Now, (r1 + r2)2 = r12  r12  2 r1r2 Perimeter of the square = Length of the wire
 142 = 130 + 2 r1r2 [Using (i) and (ii)]
 4x = 176  x = 44 cm
 196 – 130 = 2r1r2
 r1r2 = 33 ...(iii) Hence, the length of the side of the square is 44 cm.

Now, (r1 – r2)2 = r12  r12 – 2 r1r2 SECTOR OF A CIRCLE AND ITS AREA
 (r1 – r2)2 = 130 – 2 × 33 [Using (ii) and (iii)]
The region bounded by an arc of a circle and its two
 (r1 – r2)2 = 64
bounding radii is called a sector of the circle.
 r1 – r2 = 8 ...(iv)
Solving (i) and (iv), we get r1 = 11 cm and r2 = 3 cm.
Hence, the radii of the two circles are 11 cm and 3 cm.
Ex.11 A race track is in the form of a ring whose inner
circumference is 352 m, and the outer circumference
is 396 m. Find the width of the track.
Sol. Let the outer and inner radii of the ring be R metres
and r metres respectively.

If the arc is a minor arc then the corresponding sector


r
is called the minor sector and the remaining part
O bounded by the major arc is called the major sector.
In figure shaded region OAMB is the minor sector and
R
the remaining portion OANB is the major sector.
Length of Arc and Area of sector :
Then, 2R = 396 and 2r= 352
Let r be the radius of the circle with centre O and AOB
22 22 be a sector of the circle such that AOB = .
 2  R  396 and 2   r  352
7 7 0 <  < 180°, then the arc AB is a minor arc of the circle.
7 1 7 1
 R = 396   and r  352  
22 2 22 2
 R = 63 m and r = 56 m
Hence, width of the track = (R – r) m
= (63 – 56) m = 7 m.

Ex.12 The diameter of a cycle wheel is 28 cm. How many


revolution will it make in moving 13.2 km ? Now, if  increases, the length of arc AB is also
Sol. Distance travelled by the wheel in one revolution increases and if  becomes 180°, then arc AB
becomes the circumference of a semi-circle.
22 28
=2r = 2 × × = 88 cm When an arc subtends an angle 180° at the centre,
7 2
and the total distance covered by the wheel then the length of the arc of the semi-circle = r.
= 13.2 × 1000 × 100 cm When an arc subtends angle  at the centre, then length
= 1320000 cm r   r
of the arc = = .
180 180 
 Number of revolutions made by the wheel r
If L be the length of the arc AB, then L=
180 
1320000 Again, when an arc subtends angle 180° at the centre,
= = 15000.
88 the corresponding sector is a semi-circular region of
1 2
area r .
2

PAGE # 150
 When an arc subtends an angle 180° at the centre, Sol. Let A1 and A2 be the areas of sectors
2 OAB and OCD respectively. Then,
r
then the area of the corresponding sector is . A1 = Area of a sector of angle 30° in a circle of radius
2
 When an arc subtends an  at the centre, then 7 cm

r 2  1   r 2   30  22  7 2 
A1 =  [Using : A =

× r2]
area of the sector = =  
360
2 180 360   360 7 
r 2  77
If A be the area of the sector, then A = and length  A1 = cm2
360  6
r
of an arc L = A2 = Area of a sector of angle 30° in a circle of radius
180 
3.5 cm.
A r 2  180  r
 = × =  30 22 2
L 360 r 2  A2 =  360  7  (3.5) 
 
Lr
Hence, A =
2  1 22 7 7  77
NOTE :  A2 = 12  7  2  2  = cm2.
  24
(i) Angle described by minute hand in 60 minutes  Area of the shaded region = A1 – A2
= 360°.
 Angle described by minute hand in one minute  77 77 
=  6 – 24  cm2
 
 360 
=  60  = 6°. 77
  = × (4 – 1) cm2
Thus, minute hand rotates through an angle of 6° in 24
one minute. 77
= cm2 = 9.625 cm2.
(ii) Angle described by hour-hand in -12 hours. = 360°. 8
 Angle described by hour hand in one hour Ex.16 The minute hand of a clock is 10 cm long. Find the
 360  area of the face of the clock described by the minute
=  12  = 30°. hand between 9 A.M. and 9.35 A.M.
 
 Angle described by hour hand in one minute Sol. We have,
 30   1  Angle described by the minute hand in one minute
=  60  =  2  . = 6°.
   
 1   Angle described by the minute hand in 35 minutes
Thus, hour hand rotates through an angle of  2  in = (6 × 35)° = 210°
 
one minute.  Area swept by the minute hand in 35 minutes.
= Area of a sector of angle 210° in a circle of radius
Ex.14 A sector is cut from a circle of radius 21 cm. The 10 cm
angle of the sector is 150°. Find the length of its arc
 210 22 2
and area. =  360  7  (10)  cm2 = 183.3 cm2.
 
Sol. The length or arc  and area A of a sector of angle  in
a circle of radius r are given by SEGMENT OF A CIRCLE AND ITS AREA
  2r
 =
360 The region enclosed by an arc and a chord is called
  r 2 the segment of the circle.
and A = respectively..
360 N
Here, r = 21 cm and  = 150
 150 22 
  =  360  2  7  21 cm = 55 cm Major segment
Major arc
 
O
 150 22 2
and A =  360  7  (21)  cm2
 
1155
= cm = 577.5 cm2.
2
A B
2
Ex.15 In figure, there are shown sector of two concentric M Minor arc
Minor segment
circles of radii 7 cm and 3.5 cm. Find the area of the The segment containing the minor arc is called a minor
22 segment and the remaining segment containing the
shaded region. (Use = ). major arc is called the major segment.
7
In the figure, the shaded region is the minor segment
and the remaining part of the circle is major segment.
Area of a Segment of a Circle :
Let r be the radius of a circle with centre O and let AB be
an arc subtending an angle  at the centre O. we shall
find the area of the shaded segment AMB.

PAGE # 151
AREAS OF COMBINATIONS OF PLANE FIGURES

In our daily life we come across various plane figures,


O
P which are combinations of two or more plane figures.
r r For example, window designs, flower beds, circular
paths etc. In this section, we shall discuss problems
A B
of combinations of plane figures.
M
Let AP  OB. Important Formulae :
Now, area of the segment AMB In any triangle ABC, the circle which pass through
= Area of the sector OAMB – area of OAB the vertex of triangle is known Circumcircle and its
r 2  – 1 radius is called circumradius.
= × OB × AP In any triangle ABC, the circle which touches the
360 2
sides of triangle is known incircle and its radius is
r 2  – 1 called inradius.
= r × OA sin 
360 2
Area of triangle = r s,
AP
[ From AOP, sin  =  AP = OA sin  ]
OA
r 2  – 1 2 abc
= r sin  [ OA = OB = r] Area of triangle =
360 2 4R
r 2  – 1 2
Hence, area of the segment = r sin .
360 2 Where a,b and c represent the length of side of
Tr ian gle ,s re pre s en t t he s em ipe ri m et er of
Ex.17 A chord of circle 14 cm makes an angle of 60º at the
tr ian gle an d r, R re pre s e nt the in rad iu s a nd
center of the circle. Find : circumradius respectively.
(i) area of minor sector
(ii) area of the minor segment Ex.18 ABCP is a quadrant of a circle of radius 14 cm. With
(iii) area of the major sector AC as diameter, a semicircle is drawn. Find the area of
(iv) area of the major segment the shaded portion. (figure)
Sol. Given, r = 14 cm,  = 60º
Q A Q

O 14cm
14
cm
cm
14 60º

A B C
B 14cm
P
 Sol. In right angled triangle ABC,
(i) Area of minor sector OAPB = r
360 º we have
60 º AC2 = AB2 + BC2
= × 3.14 × 14 × 14
360 º AC2 = 142 + 142
= 102.57 cm2
AC = 2  14 2 = 14 2 cm
2 2
r  r Now required Area
(ii) Area of minor segment APB = – sin
360 º 2
= Area APCQA
14  14
= 102.57 – sin 60º = Area ACQA – Area ACPA
2
= Area ACQA – (Area ABCPA – Area of ABC)
3
= 102.57 – 98 × 2
2 1  14 2  1 1 
= ××     (14 ) 2   14  14 
= 17.80 cm2. 2  2  –  4 2 
 
(iii) Area of major sector = Area of circle – Area of minor
sector OAPB = (14)2 – 102.57 1 22 1 22
= 615.44 – 102.57 = 512.87 cm2 = × × 7 2 ×7 2 – × × 14 × 14 + 7 × 14
2 7 4 7
(iv) Area of major segment AQB = 154 – 154 + 98 = 98 cm2.
= Area of circle – Area of minor segment APB
= 615.44 – 17.80 = 597.64 cm2.

PAGE # 152
Ex.19 Find the area of the shaded region in figure, where Area of the incircle =  (OD)2
radii of the two concentric circles with centre O are  22 2
7 cm and 14 cm respectively and AOC = 40°.  
 
=  7  4 3  cm2
Sol. We have,
 22 
Area of ring =  (R2 – r2) =   48  cm2
7 
=  × (142 – 72)
= 150.85 cm2
= 462 cm2
3
Area of the triangle ABC = (Side)2
4
3
(24)4 = 249.4 cm2
=
4
 Area of the remaining portion of the triangle
O = (249.4 – 150.85) cm2 = 98.55 cm2.
7cm
40° D Ex.21 A horse is placed for grazing inside a rectangular
B
field 70 m by 52 m and is tethered to one corner by a
rope 21 m long. On how much area can it graze ?
cm

Sol. Shaded portion indicates the area which the horse


14

A C can graze. Clearly, shaded area is the area of a


quadrant of a circle of radius r = 21 m.
Area of the region ABDC
1 2
= Area of sector AOC – Area of sector BOD  Required area = r
4
 40 22 40 22  C B
=  360  7  14  14  360  7  7  7  cm2
 

1 1  52 m
=  9  22  14  2  9  22  7  1 cm2 Q
 

22  28  7  154
= cm2 = cm2
9 3 O 21 m A
70 m
Hence, Required shaded area
1 2 222 
 154 
=  462  3  cm2
 Required area =  4  7  (21)  cm
 
 
693
1232  Required area = cm 2  346.5 cm 2
= cm2 = 410.67 cm2 2
3
Ex.22 In figure , AOBCA represents a quadrant of a circle of
Ex.20 In an equilateral triangle of side 24 cm, a circle is
radius 3.5 cm with centre O. Calculate the area of the
inscribed touching its sides. Find the area of the
22
remaining portion of the triangle [Take 3 = 1.732]. shaded portion (Take  = ).
B 7
Sol. Let ABC be an equilateral triangle of side 24 cm, and
let AD be perpendicular from A on BC. Since the triangle
is equilateral, so AD bisects BC. D C
A

2 cm
cm
24

O 3.5 m A
O

B 12cm D 12cm C Sol. We have,


1 2
 BD = CD = 12 cm Area of quadrant AOBCA  r
4
The centre of the inscribed circle will coincide with the
centroid of ABC. 1 22
   ( 3 .5 ) 2
AD 4 7
 OD = = 9.625 cm2
3
In  ABD, we have 1
AB2 = AD2 + BD2 [Using Pythagoras Theorem] Area of AOD =  Base  Height
2
 242 = AD2 + 122 1
 AD = 24 2  12 2 = (3.5)(2) = 3.5 cm2
2
= 24  1224  12 Hence, Area of the shaded portion

= = Area of quadrant – Area of AOD


36  12 = 12 3 cm.
= (9.625 – 3.5) cm2 = 6.125 cm2
1 1 
 OD = AD =  3  12 3  cm = 4 3 cm
3  

PAGE # 153
T.S.A. = C.S.A. + circular top & bottom
= 2  rh + 2  r2
If any figure such as cuboid, which has three = 2  r (h + r) sq. units.
dimensions length, width and height are known as (iii) Volume of cylinder :
three dimensional figures. Where as rectangle has Volume = Area of base × height
only two dimensions i.e. length and width. Three = (  r2 ) × h
dimensional figures have volume in addition to areas
=  r2h cubic units
of surface from which these solid figures are formed.
(d) Cone :
Some of the main solid figures are :
(a) Cuboid :
Total Surface Area (T.S.A.) : The area of surface from
which cuboid is formed. There are six faces
(rectangular), eight vertices and twelve edges in a
cuboid.
(i) C.S.A. =  r
(ii) T.S.A. = C.S.A. + other area
=  r +  r2
=  r ( + r)

(i) Total Surface Area (T.S.A.) = 2 [ × b + b × h + h × ] 1


(iii) Volume =  r2h
(ii) Lateral Surface Area (L.S.A.) = 2 [b × h + h × ] 3
(or Area of 4 walls) = 2 h [ + b] Where, h = height
r = radius of base
(iii) Volume of Cuboid = (Area of base) × height  = slant height
= ( × b) × h
(e) Sphere :
(iv) Length of diagonal =  2  b 2  h2
T.S.A. = S.A. = 4  r2
(b) Cube :
Cube has six faces. Each face is a square. 4
Volume =  r3
3

(f) Hemisphere :
C.S.A. = 2  r2
T.S.A. = C.S.A. + other area
= 2  r2 +  r2
(i) T.S.A. = 2 [x  x + x  x + x  x]
= 2 [x2 + x2 + x2] = 2 (3x2) = 6x2 = 3  r2

(ii) L.S.A. = 2 [x2 + x2] = 4x2 2


Volume =  r3
3
(iii) Volume = (Area of base) × Height)
= (x2)  x = x3 (a) Frustum of a Cone :
(iv) Length of diagonal = x 3 When a cone is cut by a plane parallel to base, a small
(c) Cylinder : cone is obtained at top and other part is obtained at
bottom. That other part is known as ‘Frustum of Cone’.
Curved surface area of cylinder (C.S.A.) : It is the
area of surface from which the cylinder is formed. When A
we cut this cylinder, we will find a rectangle with length
2r and height h units.
 1 –  h1– h
1
D Er h1
2
h 
r1
B
C
 ABC ~ ADE
(i) C.S.A. of cylinder = (2  r) × h = 2  rh. AC AB BC
  
(ii) Total Surface Area (T.S.A.) : AE AD DE
h1 1 r
  1
h1  h  1   r2

PAGE # 154
h1  1 r Ex.24 A field is 70 m long and 40 m broad. In one corner of
Or   1
h  r1  r2 the field, a pit which is 10 m long, 8 m broad and
5 m deep, has been dug out. The earth taken out of it is
Volume of Frustum evenly spread over the remaining part of the field. Find
the rise in the level of the field.
1 1
=  r12 h1 –  r22 (h1 – h) Sol. Area of the field = (70 × 40) m2 = 2800 m2.
3 3
Area of the pit = (10 × 8) m2 = 80 m2
Area over which the earth is spread over = (2800 – 80) m2
1
=  [r 2 h – r 2 (h – h)] = 2720 m2
3 1 1 2 1
70 m
1  2  r1h   rh 
= 3 r1  r  r   r2 2  1  h 

40 m
  1 2  r 
 1 2r  10 m
8m
3 3
1  r1  r2 
= 3 h   Volume of the earth dug out = (10 × 8 × 5) m3 = 400 m3.
 r1  r2 
 Volume of the earth dug out 
1 2 2 Rise in level of the field =  Area on which the earth is spread 
=
3

h r1  r2  r1r2   

Curved Surface Area of Frustum  400 


=  m
= r11 – r2( 1–)  2720 
  r1   r   400  100 
= r1   r2  1    =   cm
  r1  r2   r1  r2   2720 

 r2 2   250 
r =   cm = 14.70 cm
=   1  2   17 
 r1  r2 r1  r2 
Ex. 25 A well of diameter 2 m is dug 14 m deep. The earth
=  (r1 + r2) taken out of it is spread evenly all around it to a width of
Total Surface Area of a Frustum 5 m to form an embankment. Find the height of the
= CSA of frustum +  r1 2 +  r22 embankment.
=  (r1 + r2) +  r1 2 +  r22 Sol. Let h be the required height of the embankment.
The shape of the embankment will be like the shape
Slant height of a Frustum = h 2  (r1  r2 ) 2
of a cylinder of internal radius 1 m and external radius
where, (5 + 1) m = 6 m.
h = height of the frustum The volume of the embankment will be equal to the
r1 = radius of larger circular end
volume of the earth dug out from the well. Now, the
r2 = radius of smaller circular end
Ex.23 If v is the volume of a cuboid of dimensions a, b, and volume of the earth dug out
c and s is its surface area, then prove that = volume of the cylindrical well
1 2 1 1 1 =  × 12 × 14 m3 = 14  m3
    .
v sa b c Also, the volume of the embankment
Sol. Volume of cuboid (v) = abc cubic unit. =  (62 – 12) h cm3 = 35  h m3
Surface area of cuboid (s) = 2 (ab + bc + ca) sq. units
6m 1m
1 1
L.H.S. = = ... (i)
v abc h
2  1 1 1
R.H.S. =    
sa b c
14 m
2  bc  ca  ab 
=  
2(ab  bc  ca)  abc 

1
= .. (ii)
abc
1 2 1 1 1
From (i) and (ii)     . Hence proved Hence, we have 35  h = 14 
v sa b c

 h = 14  2 = 0.4
35 5
Hence, the required height of the embankment = 0.4 m.

PAGE # 155
Ex.26 The ratio of the volumes of the two cones is 4 : 5 and
22  21
the ratio of the radii of their bases is 2 : 3. Find the ratio = (202 + 20 × 14 + 142) cm3
73
of their vertical heights.
Sol. Let the radii of bases, vertical heights and volumes of = 22 × 876 cm3 = 19272 cm3
the two cones be r1, h1, v1 and r2, h2, v2 respectively. 19272
According to the question, = litres = 19.272 litres.
1000
v1 4 Now,
v2 = 5 ... (i)
= (r1 – r2 )2  h 2 = (20 – 14 )2  212 cm
r1 2
r2 = 3 ... (ii) = 6 2  212 cm = 36  441 cm
= 477 cm = 21.84 cm.
1 2  Total surface area of the bucket (which is open at the top)
r1 h1 4
3 =  (r1 + r2) +  r22 = [(r1+ r2)  + r22 ]
From (i), we have =
1 2 5
r2 h 2 22
3 = 20  14  21.84  14 2
  = 2949.76 cm2.
2 7
r12h1 4  r1  h1 4
=     Required cost of the tin sheet at the rate of Rs 1.50

r22h 2 5  r2  h2 = 5
per dm2 i.e., per 100 cm2
2 2 1.50  2949 .76
 2 h1 4 h1 4 3 = Rs Rs 44.25.
100
h2 = 5  h = [Using (ii)]
    
3 2 5  2
Ex.29 A cone is divided into two parts by drawing a plane
h1 9 through a point which divides its height in the ratio
 h =
2 5
1 : 2 starting from the vertex and the plane is parallel to
Hence the ratio of their vertical height is 9 : 5.
the base. Compare the volume of the two parts.
Ex.27 The surface area of a sphere of radius 5 cm is five Sol. Let the plane XY divide the height AD of cone ABC such
times the area of the curved surface of a cone of radius
that AE : ED = 1 : 2, where AED is the axis of the cone.
4 cm. Find the height of the cone.
Sol. Surface area of sphere of radius 5 cm = 4 (5)2 cm2 Let r2 and r1 be the radii of the circular section XY and
Area of the curved surface of cone of radius 4 cm = (4)  cm2 the base BC of the cone respectively and let h1 – h and
where  cm is the slant height of the cone. h1 be their heights [figure].
According to the question,
4 (5)2 = 5[(4)]   = 5 cm
 r 2  h2 = 5  r2 + h2 = 25
 (4)2 + h2 = 25  16 + h2 = 25
 h2 = 9  h = 3.
Hence, the height of the cone is 3 cm.
Ex.28 A bucket is 40 cm in diameter at the top and 28 cm in
diameter at the bottom. Find the capacity of the bucket in
litres, if it is 21 cm deep. Also, find the cost of tin sheet
used in making the bucket, if the cost of tin is Rs 1.50 per
sq dm.
Sol. Given : r1 = 20 cm, r2 = 14 cm and h = 21 cm

h1 3 3
Then,   h1 = h
h 2 2
3
h
r1 h1
And   2 =3
r2 h1  h 1
h
2
 r1 = 3r2

Volume of cone AXY


Now, the required capacity (i.e., volume ) of the bucket
1 1 3
h 2 =  r 2 (h1 – h) =  r22 ( h – h)
= (r + r1 r2 + r22) 3 2 3 2
3 1
1 2
= r h
6 2

PAGE # 156
Volume of frustum XYBC 5. Three parallel lines 1, 2 and 3 are drawn through the
vertices A, B and C of a square ABCD. If the distance
1
= h(r12 + r22 + r1r2) between 1 and 2 is 7 and between 2 and 3 is 12,
3
then the area of the square ABCD is :
1 (A) 193 (B) 169
= h(9r22 + r22 + 3r22)
3 (C) 196 (D) 225
1
= h(13r22) 6. A rectangle is inscribed in a square creating four
3
isosceles right triangle. If the total area of these four
1 2 triangles is 200. The length of the diagonal of the
r2 h
Volume of cone AXY 6 rectangle is :
So, 
Volume of frustum XYBC 13 2
r2 h
3
Volume of cone AXY 1
Volume of frustum XYBC = 26 .

i.e., the ratio between the volume of the cone AXY and
the remaining portion BCYX is 1 : 26. (A) 10 (B) 15
(C) 20 (D) 25

7. In the given figure, the diameter of the biggest


semi-circle is 56 cm and the radius of the smallest
NOTE : More than one correct option may be possible. circle is 7 cms. The area of the shaded portion is :
PERIMETER AND AREA OF PLANE FIGURES

1. ABC is an isosceles right triangle with area P. The


radius of the circle that passes through the point A, B
and C is
P (A) 482 cm2 (B) 462 cm2
(A) P (B) (C) 654 cm2 (D) 804 cm2
2

P 8. In the given figure, OPQR is a rhombus, three of whose


(C) (D) 2P
2 vertices lie on a circle with centre O. If the area of
rhombus is 32 3 cm2. The radius of circle is :
2. All the 3 sides of a right triangle are integers and one (A) 8
side has a length 11 units. Area of the triangle in square
(B) 9
units lies between
(A) 1 and 100 (B) 100 and 200 (C) 10
(C) 200 and 300 (D) More than 300
(D) 11

3. A triangle EFG is inscribed in a unit square ABCD with 9. A wire in the shape of an equilateral triangle encloses
E on AB, F on DA, G on CD such that AE = DF = CG = an area of S sq. cm. If the same wire is bent to form of
1/3. The area of the triangle EFG is : a circle. The area of the circle will be :
5 1 S2 3S2
(A) (B) (A) (B)
18 3 9 
3S 3 3S
(C) (D)
5 4  
(C) (D)
9 9
10. ABCD is a rectangle and lines DX, DY and XY are
4. ABCD is a rectangle, P lies on AD and Q on AB. The drawn as shown. Area of AXD is 5, Area of BXY is 4
triangles PAQ, QBC and PCD all have the same area, and area of CYD is 3. If the area of DXY can be
and BQ = 2. The length of AQ, is :
expressed as x where x  N then x is equal to
(A) 3  5 (B) 2 3
X
A B
(C) 5 1
(D) not uniquely determined Y

D C

(A) 72 (B) 75
(C) 84 (D) 96

PAGE # 157
CUBE AND CUBOID 19. A corn cob, shaped somewhat like a cone, has the
radius of its broadest end as 2.1 cm and length (height)
11. The cost of white washing the four walls of a room is
as 20 cm. If each 1 cm2 of the surface of the cob carries
Rs. 25. The cost of white-washing a room twice in
an average of four grains, find how many grains
length, breadth and height will be :
(approximately) you would find on the entire cob.
(A) Rs. 50 (B) Rs. 75
(A) 511 (B) 531
(C) Rs. 100 (D) Rs. 200
(C) 551 (D) 571
12. Three equal cubes are placed adjacently in a row. Find
the ratio of the total surface area of the new cuboid to 20. The radius of the base of a conical tent is 7 m. The tent
that of the sum of the surface areas of three cubes. is 24 m high. Find the cost of the canvas required to
(A) 5 : 9 (B) 3 : 5 make the tent, if one square meter of canvas costs
(C) 2 : 5 (D) 7 : 9 Rs. 180 (Take  = 22/7).
(A) Rs 99000 (B) Rs 98000
13. Water in a canal, 30 dm wide and 12 dm deep, is (C) Rs 95000 (D) Rs 97000
flowing with a speed of 10 km/hour. How much area
will it irrigate in 30 minutes, if 8 cm of standing water is 21. A right triangle with its sides 9 cm, 12 cm and 15 cm is
required for irrigation. revolved about the side 12 cm. Find the volume of the
(A) 220500 m2 (B) 22500 m2 solid so formed.
(C) 220000 m2 (D) 225000 m2 (A) 1018.28 cm
3
(B) 702.57 cm
3

3 3
(C) 1200 cm (D) 1218.28 cm
C Y L IN D E R
22. A cone is divided into two parts by drawing a plane
14. The ratio of the height of a circular cylinder is to the through the mid point of its axis parallel to its base
diameter of its base is 1 : 2. Then the ratio of the areas then the ratio of the volume of two parts is :
of its curved surface to the sum of the areas of its two (A) 1 : 3 (B) 1 : 7
ends is : (C) 1 : 8 (D) 1 : 9
(A) 1 : 1 (B) 1 : 2
(C) 2 : 1 (D) 1 : 3 SPHERE AND HEMISPHERE
15. The diameter of a garden roller is 1.4 m and it is 2 m long.
23. Find the difference between total surface area &curved
How much area will it cover in 5 revolutions?
surface area of a hemisphere of radius 21 cm.
(Use  = 22/7) (A) 1376 cm2 (B) 1386 cm2
(A) 40 m2 (B) 41.40 m2 (C) 1396 cm 2
(D) 1404 cm2
(C) 44 m2 (D) 42.40 m2
24. The hollow sphere, in which the circus motorcyclist
16. An iron pipe 20 cm long has exterior diameter equal to performs his stunts, has diameter of 7 m. Find the
25 cm. If the thickness of the pipe is 1 cm, find the area available to motorcyclist for riding.
whole surface area of the pipe. (A) 154 m2 (B) 144 m2
2
(A) 3168 cm2 (B) 3160 cm2 (C) 38.5 m (D) 176 m2
2
(C) 3068 cm (D) 3268 cm2
25. A hemispherical dome of a building needs to be
painted. If circumference of the base of the dome is 44
17. Savitri had to make a model of a cylindrical kaleidoscope
m, find the cost of painting it, given the cost of painting
for her science project. She wanted to use chart paper
is Rs 2 per 100 cm2.
to make the curved surface of the kaleidoscope. What (A) Rs 62600 (B) Rs 63000
would be the area of chart paper required by her, if she (C) Rs 61000 (D) Rs 61600
wanted to make a kaleidoscope of length 25 cm with a
3.5 cm radius ? You may take  = 22/7). 26. How many spherical bullets can be made out of a solid
(A) 540 cm2 (B) 520 cm2 cube of lead whose edge measures 44 cm, each bullet
2
(C) 550 cm (D) 560 cm2
being 4 cm in diameter.
CONE AND FRUSTUM (A) 2451 (B) 2541
18. The radius and height of a cone are in the ratio 4 : 3. (C) 2304 (D) 2536
The area of the base is 154 cm2. Find the area of the 3
27. A hemispherical tank of radius 1 m is full of water. It
curved surface. 4
is connected with a pipe which empties it at the rate of
(A) 192.5 cm2 (B) 195 cm2
2
7 liters per second. How much time will it take to empty
(C) 190.5 cm (D) 185.5 cm2
the tank completely?
(A) 26.74 min. (B) 26.54 min.
(C) 26.4 min. (D) 26 min.

PAGE # 158
MISC ELL ANEOUS 4. The radius of a cone is 2 times the height of the cone.
A cube of maximum possible volume is cut from the
28. A sphere, a cylinder and a cone are of the same radius same cone. What is the ratio of the volume of the cone
and same height. Find the ratio of their curved surfaces. to the volume of the cube ? [NSTSE-2010]
(A) 3.18  (B) 2.25 
(A) 4 : 4 : 5 (B) 4 : 5 : 5
(C) 2.35
(C) 3 : 4 : 5 (D) 2 : 3 : 2 5 (D) can’t be determined
5. A right circular cone has for its base a circle having the
29. The surface area of a sphere of radius 5 cm is five same radius as a given sphere. The volume of the
times the area of the curved surface of a cone of radius cone is one-half that of the sphere. The ratio of the
4 cm. Find the height of the cone. altitude of the cone to the radius of its base is :
(A) 3 cm (B) 2 cm [NSTSE-2010]
(C) 4 cm (D) 5 cm 1 1
(A) (B)
1 2
30. The shape of a solid is a cylinder surmounted by a
cone. If the volume of the solid is 40656 cm3, the 2 2
(C) (D)
diameter of the base is 42 cm and the height of the 1 3
cylinder is 20 cm, find the slant height of the conical
portion. 6. The perimeter of an isosceles right angled triangle is
(A) 45 cm (B) 35 cm 2p. Its area is [IJSO-2010]
(C) 40 cm (D) 50 cm
(A) 2  2 p 2
  
(B) 2 – 2 p2

31. A piece of wire in the form of a rectangle 15 cm long
(C) 3 – 2 2  p 2 2
and 7 cm broad is reshaped and bent into the form of 
(D) 3  2 2 p
a circle. Find the radius of the circle.
(A) 5 cm (B) 7 cm
(C) 8 cm (D) 9 cm 7. The area of a circle is doubled when its radius r is
increased by a. Therefore, radius r equals.
[IJSO-2010]

(A)  2  1a (B)  2 – 1a


COMPETITIVE EXAM PREVIOUS YEAR
(C) a (D) 2 – 2  a
QUESTION
1. A cylinder of radius 6 cm and height h cm is filled with 8. In case of a right circular cylinder the radius of base
ice cream. The ice cream is then distributed among and height are in the ratio 2 : 3. Therefore, the ratio of
10 children in identical cones having hemispherical lateral surface area to the total surface area is :
tops. The radius of the base of the cone is 3 cm and its [IJSO-2010]
height is 12cm. Then the height h of the cylinder must (A) 5 : 3 (B) 3 : 5
be : [IJSO-2008] (C) 2 : 5 (D) 2 : 3
(A) 100/7 cm (B) 18 cm
(C) 15 cm (D) 200 / 11 cm 9 A sphere and cube have equal surface areas.The ratio
of their volumes is [IJSO-2011]
2. A regular hexagon of maximum possible area is cut off
from an equilateral triangle. The ratio of area of
 6
triangle to the area of hexagon will be : [IJSO-2009] (A) (B)
6 
3 6
(A) (B)
2 2 6 
(C) (D)
3 3  3
(C) (D)
2 2 10. An equilateral triangle has area A 3 .Three circles
3. Three cubes with sides in the ratio 3 : 4 : 5 are melted are drawn with their centres at the vertices of the
to form a single cube whose diagonal is 12 3 cm. triangle.Diameter of each circle is equal to the length
The sides of the cubes are : [NSTSE-2010] of each side of the triangle.The area of the triangle
(A) 3 cm, 4 cm, 5 cm NOT included in any of the three circles is
(B) 6 cm, 8 cm, 10 cm [IJSO-2011]
(C) 9 cm, 12 cm, 15 cm

(D) None of these (A) A ( 3 – ). (B) A( – 3 ).
6

(C) A(3– 3 ) (D) A( 3 – ).
2

PAGE # 159
11. A conical vessel of radius 6 cm and height 8 cm is 17. A piece of wire 60 cm long is cut into two parts, one of
completely filled with water. A metal sphere is now low- them being 24 cm long. Each part is then bent to form
a square. The ratio of the area of the larger to the
ered into the water. The size of the sphere is such that
smaller square is : [IJSO-2014]
when it touches the inner surface, it just gets (A) 9/4 (B) 7/4
immersed.The fraction of water that overflows from the (C) 3/2 (D) 11/3
conical vessel is [IJSO-2011]
18. If set of marbles, of radius 5 cm, is poured into a cube
3 5 of side 1 m. The maximum number of marbles that
(A) (B) can be filled into the box are [IJSO-2014]
8 8
7 5 (A) 2000 (B) 1000
(C) (D) (C) 1500 (D) 3000
8 16
12. An equilateral triangle has area A cm2. A regular hexa-
19. A round table cover has six equal designs as shown in
gon of maximum area is cut off from the triangle. If the
the adjacent figure. If the radius of the cover is 4 cm,
area of the hexagon is 320 cm2, the area A is
then cost of making the designs at the rate of Rs 10.00
[IJSO-2011]
per cm2 (round off your answer to a nearest rupee) is
(A) 640 cm2 (B) 480 cm2
[IJSO-2014]
(C) 600 cm2 (D) 400 cm2

13. What is the radius of the circumcircle of a triangle


whose sides are 30cm, 36cm and 30cm in length
[IJSO-2012]
(A) 18 cm (B) 18.25cm
(C) 18.50cm (D) 18.75cm
(A) Rs 85 (B) 86
14. If the radius of the base of a cone is doubled then the
(C) 90 (D) 87
slant area becomes 3 times the original slant area.
Suppose when the radius of the base of the cone is
quadrupled (that is increased to 4 times), the slant 20. Total surface area of a sphere S with radius 2+ 3
area becomes k times the original slant area. Then cm is [IJSO-2016]
the integer closest to k is [IJSO-2013] (A) 400  (5 + 2 6 ) sqmm
(A) 6 (B) 8
(B)  ( 2 + 3 )2 sqcm
(C) 10 (D) 11
(C) 2 ( 2 + 3 )2 sqcm
15. Which is of semicircular shape a pavement of uniform
width is laid. If the ratio of the area of the lawn to the (D) 40 (5 + 2 6 ) sqmm
area of the pavement is 25:24 , then the ratio of the
outer and inner perimeters of the pavement is 21. The probability of a point within an equilateral triangle
[IJSO-2013] with side 1- unit lying outside its in-circle (inscribed
circle) is [IJSO-2016]
7 6
(A) (B)
5 5
1 
5 (A) 1 – (B) 1 –
(C)
5
(D)
 
2 3  
3 3
4 2 6
 2
16. The adjacent figure is a modification of the Switzer- (C) 1 – (D) 1 –
2 3 3 3
land flag to suit the problem ! Five identical small
squares from the central cross. The length of each
side of the big square is 10 m. If the area of the white 22. A craft teacher reshapes the wax from a cylinder of
cross is 20% of the area of the flag, then the length of candle with section diameter 6 cm and the height 6 cm
into a sphere. The radius of this sphere will be
the side of the small square is : [IJSO-2014]
[IJSO-2016]
(A) r = 6 3 / 2 cm (B) r = 6 cm

(A) 2m
+ (B) 2.25
(C) r = 3 3 3 / 2 cm (D) r = 6 3 2 cm

(C) 1.6 m (D) 1.75 m

PAGE # 160
PERMU TATI ON A N D C OMBI NATI ON

FUNDAMENTAL PRINCIPLE OF COUNTING

Principle of Multiplication:
The continued product of first n natural numbers is
called the “ n factorial’ and is denoted by n!. If an event can occur in ‘m’ different ways, following
i.e. n! = 1 × 2 × 3 × 4 × ....× (n – 1) × n. which another event can occur in ‘n’ different ways,
Thus, 3! = 1 × 2 × 3 = 6 ; 4! = 1 × 2 × 3 × 4 = 24, then total number of different ways of simultaneous
5! = 1 × 2 × 3 × 4 × 5 = 120 etc. occurrence of both the events in a definite order is
Clearly, n! is defined for positive integers only. m ×n.

 NOTE : Principle of Addition:

(i) Factorials of negative integers are not defined. If an event can occur in ‘m’ different ways, and
another event can occur in ‘n’ different ways, then
(ii) 0 ! = 1 ! = 1 exactly one of the events can happen in m + n ways.
(iii) nP n = n ! = n. (n - 1) !
Ex. 3 There are 8 buses running from Kota to Jaipur and
n
(iv) (2n) ! = 2 . n ! [1. 3. 5. 7... (2n - 1)] 10 buses running from Jaipur to Delhi. In how many
ways a person can travel from Kota to Delhi via
1 1 x Jaipur by bus.
Ex. 1 If   ,find x.
9! 10! 11! Sol. Let E 1 be the event of travelling from Kota to Jaipur
& E 2 be the event of travelling from Jaipur to Delhi
Sol. We have, by the person.
1 1 x E 1 can happen in 8 ways and E 2 can happen in
 
9! 10! 11! 10 ways.
Since both the events E 1 and E 2 are to be happened
1 1 x in o rder, simultaneously, the num ber of ways
  =
9! 10  9! 11 10  9! = 8 × 10 = 80.
1 1  x  1 Ex. 4 How many numbers between 10 and 10,000 can
 1 =  
9!  10   11 10  9! be formed by using the digits 1, 2, 3, 4, 5 if
(i) No digit is repeated in any number.
1 x
 1 + = (ii) Digits can be repeated.
10 11 10
Sol. (i) Number of two digit numbers = 5 × 4 = 20
11 x Number of three digit numbers = 5 × 4 × 3 = 60
 = Number of four digit numbers = 5 × 4 × 3 × 2 = 120
10 11 10
Total = 200
 x = 11 × 11 = 121
(ii) Number of two digit numbers = 5 × 5 = 25
n! n! Number of three digit numbers = 5 × 5 × 5 = 125
Ex. 2 If and are in the ratio 2 : 1, find Number of four digit numbers = 5 × 5 × 5 × 5 = 625
2! (n  2)! 4 ! (n  4) !
the value of n. Total = 775
Sol. We have, Ex. 5 How many numbers are there between 100 and 1000
n! n! in which all the digits are distinct ?
: =2:1 Sol. A number between 100 and 1000 has three digits. So,
2! (n  2)! 4! (n  4) !
we have to form all possible 3-digit numbers with
n! 4 ! ( n  4) ! 2
 × = distinct digits.
2! (n  2)! n! 1 W e cannot have 0 at the hundred’s place. So, the
4! (n  4) ! 2 hundred’s place can be filled with any of the 9 digits 1,
 =
2! (n  2)  (n  3)  (n  4) ! 1 2, 3,...., 9. So, there are 9 ways of filling the hundred’s
place.
4  3  2! 2
 = Now, 9 digits are left including 0. So, ten’s place can
2! (n  2)(n  3) 1 be filled with any of the remaining 9 digits in 9 ways.
 (n – 2) (n – 3) = 6  n2 – 5n = 0 Now, the unit’s place can be filled with in any of the
 n (n – 5) = 0  n = 0, 5. remaining digits. So, there are 8 ways of filling the
unit’s place.
But, for n = 0, (n – 2) ! and (n – 4) ! are not
Hence, the total number of required numbers
meaningful. Therefore, n = 5.
= 9 × 9 × 8 = 648.

PAGE # 161
Ex. 8 In how many ways three different rings can be worn in
four fingers with at most one in each finger ?
Each of the arrangements which can be made by taking Sol. The total number of ways is same as the number of
some or all of a number of things is called a arrangements of 4 fingers, taken 3 at a time.
permutation. 4!
n So, required number of ways = 4P3 =
P r d eno tes t he num ber of pe rm u ta tio ns of ( 4  3)!
n different things, taking r at a time, then 4!
n! = = 4! = 24.
n
1!
Pr = n (n - 1) (n - 2)..... (n - r + 1) =
n  r ! . Ex. 9 How many different signals can be made by 5 flags
For example if there are three objects a, b & c, then the from 8 flags of different colours ?
permutations of these objects, taking two at a time, Sol. The total number of signals is the number of
are ab, ba, bc, cb, ac, ca arrangements of 8 flags by taking 5 flags at a time.
So, the number of permutations of three different things Hence, required number of signals
taken two at a time is 6. 8! 8! 8  7  6  5  4  3!
= 8P5 = = = = 6720.
 NOTE : (8  5)! 3! 3!
It should be noted that in permutations the order of Ex.10 How many words, with or without meaning, can be
arrangement is taken into account; when the order is formed using all the letters of the word EQUATION,
changed, a different permutation is obtained. using each letter exactly once.
Sol. There are eight letters in the word ‘EQUATION’. So, the
Ex. 6 If 10Pr = 5040, find the value of r.
total number of words is equal to the number of
Sol. 10Pr = 5040
arrangement of these letters, taken all at a time. The
10 ! number of such arrangement is 8P8 = 8!. Hence, the
 (10  r )! = 10 × 504 total number of words = 8!.
Ex.11 How many numbers of three digits can be formed
10 ! using the digits 1, 2, 3, 4, 5, without repetition of
 (10  r )! = 10 × 9 × 8 × 7 digits. How many of these are even.
Sol. Three places are to be filled with 5 different objects.
10 ! 10  9  8  7  6!  Number of ways = 5P3 = 5 × 4 × 3 = 60
 (10  r )! = 6! For even numbers unit digit can be filled in two ways
& the remaining two digits can be filled in 4P2 ways.
10 ! 10!  Number of even numbers = 2 × 4P2 = 24.
 (10  r )! = 6!
(a) Permutations of Objects not all Distinct :
 (10 – r)! = 6!  10 – r = 6
 r = 4. So, far we were discussing permutations of distinct
objects (things) by taking some or all at a time. In this
Ex. 7 If 9P5 + 5. 9P4 = 10Pr ‘ find r. article we intend to discuss the permutations of a given
Sol. We have number of objects when objects are not all different.
9
P5 + 5. 9P4 = 10Pr For example, the number of arrangements of the
9! 9! 10! letters of the word MISSISSIPPI, the number of six digit
  5.  numbers formed by using the digits 1, 1, 2, 3, 3, 4 etc.
(9  5)! (9  4 )! (10  r )!
The following theorem is very helpful to determine the
9! 9! 10! number of such arrangements.
  5. =
4! 5! (10  r )!
Theorem : The number of mutually distinguishable
9! 9! 10! permutations of n things, taken all at a time, of which
  = p are alike of one kind, q alike of second such that
4! 4! (10  r )!

9! 10! n!
 2× = p + q = n, is .
4! (10  r )! p! q!
NOTE :
5  2  9! 10! The number of permutations of n things, of which
 = p1 are alike of one kind; p2 are alike of second kind;
5  4! (10  r )!
p3 are alike of third kind....pr are alike of r th kind such
10  9! 10!
 =
5! (10  r )! n!
that p1 + p2 +...... pr = n, is
p1!p2 !p3 !......pr !
10! 10!
 = The number of permutations of n things, of which p are
5! (10  r )!
alike of one kind, q are alike of second kind and
 (10 – r)! = 5! 10 – r = 5 r = 5. n!
remaining all are distinct, is .
p! q!

PAGE # 162
Suppose, there are r things to be arranged, allowing So these 4 even places can be occupied by 4 vowels
repetitions. Let further p1, p2, ... pr be the integers such 4!
that the first object occurs exactly p1 times, the second in 4 ! = 1 way. Now, we are left with 5 places in which
occurs exactly p2 times, etc. Then the total number of
5 letters, of which two are alike (2L’s) and other distinct,
permutations of these r objects to the above condition
5!
(p1  p 2 ......p r )! can be arranged in 2! ways.
is .
p1!p 2 !p 3 !......p r !
Hence, the total number of words in which vowels
Ex.12 How many different words can be formed with the occupy the even places
letters of the word ‘MISSISSIPPI’? 5! 4! 5!
Sol. There are 11 letters in the given word, of which 4 are = × = = 60.
2! 4! 2!
S’s, 4 are I’s and 2 are P’s. So, total number of words
is the number of arrangements of 11 things, of which (ii) Considering both L together and treating them as
4 are similar of one kind, 4 are similar of second kind one letter we have 8 letters out of which A repeats 4
times and others are distinct. These 8 letters can be
11!
and 2 are similar of third kind is 4! 4! 2! 8!
arranged in 4 ! ways.

11! So, the number of words in which both L come together


Hence, the total number of words = 4! 4! 2! = 34650. 8!
= 4 ! = 1680.
Ex. 13 (i) How many different words can be formed with the Hence, the number of words in which both L do not
letters of the word HARYANA ? come together
(ii) How many of these begin with H and end with N ? = Total number of words – Number of words in which
(iii) In how many of these H and N are together ? both L come together
Sol. (i) There are 7 letters in the word ‘HARYANA’ of which 3 = 7560 – 1680
are A’s and remaining all are each of its own kind. = 5880.
7! 7! (b) Circular Permutation :
So, total number of words = 3 !1!1!1!1! = 3! = 840.
The number of circular permutations of n different
(ii) After fixing H in first place and N in last place, we things taken all at a time is (n – 1) ! .
have 5 letters out of which three are alike i.e. A’s and
If clockwise & anti-clockwise circular permutations
remaining all are each of its own kind.
(n  1)!
5! are considered to be same, then it is .
So, total number of words = = 20. 2
3!
 NOTE :
(iii) Considering H and N together we have 7 – 2 + 1 = 6
Number of circular permutations of n things when
letters out of which three are alike i.e. A’s and others
p are alike and the rest are different, taken all at a
are each of its own kind. These six letters can be
time, distinguishing clockwise and anticlockwise
6!
arranged in ways. But H and N can be arranged (n 1)!
3! arrangement is .
p!
amongst themselves in 2! ways. Ex.15 In how many ways can we arrange 6 different
flowers in a circle. In how many ways we can form a
6!
Hence, the required number of words = × 2! garland using these flowers.
3!
Sol. The number of circular arrangements of 6 different
= 120 × 2 = 240. flowers = (6 – 1)! = 120.
Ex.14 How many different words can be formed by using When we form a garland, clockwise and anticlockwise
all the letters of the word ‘ALLAHABAD’? arrangements are similar. Therefore, the number of
(i) In how many of them vowels occupy the even 1
positions ? ways of forming garland = (6 – 1) ! = 60.
2
(ii) In how many of them both L do not come together ?
Ex.16 In how many ways 6 persons can sit at a round
Sol. There are 9 letters in the word ‘ALLAHABAD’ out of
which 4 are A’s, 2 are L’s and the rest are all distinct. table, if two of them prefer to sit together.
Sol. Let P1, P2, P3, P4, P5, P6 be the persons, where P1, P2
9! want to sit together.
So, the required number of words = 4 ! 2! = 7560.
Regard these person as 5 objects. They can be
(i) There are 4 vowels and all are alike i.e. A‘s. Also, arranged in a circle in (5 – 1)! = 24. Now P1, P2 can
there are 4 even places viz 2nd, 4th, 6th and 8th. be arranged in 2! ways. Thus the total number of
ways = 24 × 2 = 48.

PAGE # 163
Since the committee is formed in each case, therefore
by the fundamental principle of addition, the total
Each of the different selections made by taking some number of ways of forming the committee
or all of a number of objects, irrespective of their = 4C2 × 6C3 + 4C3 × 6C2 + 4C4 × 6C1 = 120 + 60 + 6 = 186
arrangements is called a combination. (ii) A committee of 5 persons, consisting of at most two
Theorem : The number of all combination of n distinct ladies, can be done in the following ways :
objects, taken r at a time is given by (a) Selecting 5 gents only out of 6. This can be done in
6
n
n! C5 ways.
Cr = .
(n  r)! r ! (b) Selecting 4 gents only out of 6 and one lady out of 4.
 NOTE : This can be done in 6C4 × 4C1 ways.
(i) nCr = nCn – r
(c) Selecting 3 gents only out of 6 and two ladies out of
(ii) nCr + nCr – 1 = n + 1Cr
4. This can be done in 6C3 × 4C2 ways.
(iii) nCr = 0 if r  {0, 1, 2, 3........, n} Since the committee is formed in each case, so, the
Ex.17 If the ratio 2nC3 : nC3 is equal to 11 : 1, find n. total number of ways of forming the committee
= 6C5 + 6C4 × 4C1 + 6C3 × 4C2 = 6 + 60 + 120 = 186.
Sol. We have, 2nC3 : nC3 = 11 : 1
Ex.20 How many diagonals are there in a polygon with
2n
C3 11 n sides ?
 n =
C3 1 Sol. A polygon of n sides has n vertices. By joining any two
vertices of a polygon, we obtain either a side or a
( 2n)! diagonal of the polygon. Number of line segments
( 2n  3 )! 3! 11 obtained by joining the vertices of a n sided polygon
 = taken two at a time.
n! 1
= Number of ways of selecting 2 out of n
(n  3 )! (3! )
( 2n)! (n  3)! 11 n(n  1)
  = = nC2 =
( 2n  3)! n! 1 2
Out of these lines, n lines are the sides of the
(2n)(2n  1)(2n  2)(2n  3)!
 polygon.
(2n  3)!  Number of diagonals of the polygon
(n  3)! 11 n(n  1) n(n  3 )
× = = –n= .
n(n  1)(n  2)(n  3 )! 1 2 2
Ex.21 Fifteen players are selected for a cricket match.
(2n)(2n  1)(2n  2) 11 (i) In how many ways the playing 11 can be selected
 =
n(n  1)(n  2) 1 (ii) In how many ways the playing 11 can be selected
4( 2n  1) 11 including a particular player.
 = (iii) In how many ways the playing 11 can be selected
n2 1
 8n – 4 = 11n – 22 excluding two particular players.
Sol. (i) 11 players are to be selected from 15
 3n = 18
Number of ways = 15C11 = 1365.
 n = 6.
(ii) Since one player is already included, we have to
Ex.18 If 49C3r – 2 = 49C2r + 1, find 'r'. select 10 from the remaining 14
Sol. nCr = nCs if either r = s or r + s = n. Number of ways = 14C10 = 1001.
Thus, 3r – 2 = 2r + 1 (iii) Since two players are to be excluded, we have
 r=3 to select 11 from the remaining 13.
or 3r – 2 + 2r + 1 = 49 Number of ways = 13C11 = 78.
 5r – 1 = 49 Ex.22 In how many ways we can select 4 letters from the
 r = 10 letters of the word MISSISSIPPI.
 r = 3, 10. Sol. M
IIII
Ex.19 A committee of 5 is to be formed out of 6 gents and
SSSS
4 ladies. in how many ways this can be done, when PP
(i) at least two ladies are included Number of ways of selecting 4 alike letters
(ii) at most two ladies are included ? = 2C1 = 2.
Sol. (i) A committee of person, consisting of at least 2 ladies, Number of ways of selecting 3 alike and 1 different
can be formed in the following ways : letters = 2C1 × 3C1 = 6.
(a) Selecting 2 ladies out of 4 and 3 gents out of 6. This Number of ways of selecting 2 alike and 2 alike
can be done in 4C2 × 6C3 ways. letters = 3C2 = 3.
(b) Selecting 3 ladies out of 4 and 2 gents out of 6. This Number of ways of selecting 2 alike & 2 different
can be done in 4C3 × 6C2 ways. = 3C1 × 3C2 = 9.
(c) Selecting 4 ladies out of 4 and 1 gents out of 6. This Number of ways of selecting 4 different = 4C4 = 1
can be done in 4C4 × 6C1 ways. Total = 2 + 6 + 3 + 9 +1 = 21.

PAGE # 164
Ex.23 In how many ways we can arrange 3 red flowers,
4 yellow flowers and 5 white flowers in a row. In
how many ways this is possible if the white flowers Number of ways in which atleast one object to be
are to be separated in any arrangement (Flowers of selected out of 'n' distinct objects is
n
same colour are identical). C1 + nC2 + nC3 +...............+ nCn = 2n – 1
Sol. Total we have 12 flowers 3 red, 4 yellow and 5 white. Number of ways in which atleast one object may be
selected out of 'p' alike objects of one type 'q' alike
12 ! objects of second type and 'r' alike of third type is
Number of arrangements = 3 ! 4 ! 5 ! = 27720.
(p + 1) (q + 1) (r + 1) – 1
For the second part, first arrange 3 red & 4 yellow Number of ways in which atleast one object may be
selected from 'n' objects where 'p' alike of one type
7! 'q' alike of second type and 'r' alike of third type and
This can be done in 3 ! 4 ! = 35 ways. rest n – (p + q + r) are different, is
(p + 1) (q + 1) (r + 1) 2 n – (p + q + r) – 1
Now select 5 places from among 8 places (including
extremes) & put the white flowers there. Ex.27 There are 12 different books on a shelf. In how
This can be done in 8C5 = 56. many ways we can select atleast one of them.
 The number of ways for the 2nd part = 35 × 56 = 1960. Sol. We may select 1 book, 2 books,........, 12 books.
 The number of ways = 12C1 + 12C2 + ....... + 12C12
Ex.24 In how m any ways the letters of the word = 2 12 – 1 = 4095.
"ARRANGE" can be arranged without altering the
relative positions of vowels & consonants. Ex.28 There are 12 fruits in a basket of which 5 are
Sol. The consonants in their positions can be arranged apples, 4 mangoes and 3 bananas (fruits of same
species are identical). How many ways are there to
4! select atleast one fruit.
in 2 ! = 12 ways. Sol. Let x be the number of apples being selected
y be the number of mangoes being selected and
The vowels in their positions can be arranged in
z be the number of bananas being selected.
3! Then x = 0, 1, 2, 3, 4, 5
2 ! = 3 ways y = 0, 1, 2, 3, 4
z = 0, 1, 2, 3
 Total number of arrangements = 12 × 3 = 36.
Total number of triplets (x, y, z) is 6 × 5 × 4 = 120
Exclude (0, 0, 0)
 Number of combinations = 120 – 1 = 119.

Number of ways in which (m + n + p) different things


can be divided into three different groups containing

m, n & p things respectively is


m  n  p  ! . FUNDAMENTAL PRINCIPLE OF COUNTING
m!n!p!
1. Different 7–digit telephone numbers that can be
If m = n = p and the groups have identical qualitative
formed from 0, 1, 2, 3, 4, 5, 6, 7, 8, 9 is :
(3n)! (A) 10 7 (B) 9 × 10 6
characteristic then the number of groups = .
n! n! n! 3! (C) 9! (D) None of these
However, if 3n things are to be distributed equally
am ong t hre e p eop le th en the nu m b er of 2. Number of ways in which 5 letters can be posted in 4
letter boxes is :
(3n)! (A) 5 4 (B) 20
ways = .
n!3 (C) 25 (D) 4 5

Ex.25 12 different toys are to be distributed to three 3. Total numbers divisible by 5 and lying between 4000 and
children equally. In how many ways this can be done. 5000 can be formed from the digits 4, 5, 6, 7 and 8 is :
Sol. The problem is to divide 12 different things into three (A) 125 (B) 120
different groups. (C) 25 (D) 625
12 ! 4. Numbers of different signals can be given using any
Number of ways = = 34650.
4!4! 4! number of flags from 5 flags of different colours is :
Ex.26 In how many ways 10 persons can be divided into (A) 325 (B) 240
5 pairs. (C) 60 (D) None of these
Sol. We have each group having 2 persons and the
qualitative characteristic are same (Since there is 5. Number of ways in which these scholarships can be
no purpose mentioned or names for each pair). awarded is :
(A) 12 (B) 60
10 !
Thus the number of ways = = 945. (C) 720 (D) None of these
( 2 ! )5 5 !

PAGE # 165
6. Number of ways in which one of these scholarships Directions Q.16 – Q. 18
can be awarded is : There are 5 friends : A, B, C, D and E. They wanted to
(A) 12 (B) 60 take a group photograph of all of them sitting in a single
(C) 720 (D) None of these row.
16. How many distinctly different photographs can be
7. Number of different ways can a person reach
clicked.
Mumbai from Ahmedabad if there are 4 different
(A) 60 (B) 120
routes from Ahmedabad to Surat and 3 different
(C) 240 (D) None of these
routes from Surat to Mumbai is :
(A) 7 (B) 12 17. In how many of these photographs would A be sitting
(C) 24 (D) 36 in the middle.
(A) 12 (B) 18
8. Number of different ways that can 3 travellers stay
(C) 24 (D) None of these
in 4 hotels when each one should stay in different
hotel is : 18. In how many of these would A and B be sitting next to
(A) 24 (B) 30 each other ?
(C) 36 (D) 48 (A) 24 (B) 30
(C) 36 (D) 48
9. A letter lock contains 4 rings, each ring containing 5
letters. If the lock opens in only one arrangements 19. A group of 6 students comprised of 3 boys and 3
of 4 letters, then number of unsuccessful events girls. Number of ways could they be arranged in a
possible is : straight line such that no two boys were sitting
(A) 120 (B) 625 together is :
(C) 624 (D) None of these (A) 36 (B) 72
10. There are 10 digits from 0 to 9 in the decimal (C) 108 (D) 144
system. How many 5 – digit numbers can be 20. Numbers of ways to arrange the letters of the word
formed, such that no 2 digits are the same. ‘FIGMENT’ when it starts with F is :
(A) 12216 (B) 26127 (A) 360 (B) 720
(C) 62716 (D) 27216 (C) 1080 (D) 1440
11. There are 10 digits from 0 to 9 in the decimal 21. Numbers of ways to arrange the letters of the word
system. How many 4 – digit numbers can be formed ‘FIGMENT’ when vowels at first and last positions is :
using these 10 digits? (A)120 (B) 240
(A) 4500 (B) 9000 (C) 360 (D) 720
(C) 10500 (D) 12000
22. Number of ways in which 7 boys can be seated at a
12. Number of different ways in which 5 different balls round table so that 2 particular boys are next to each
be distributed to 3 different boxes, when each box other is :
can hold any number of balls is : (A) 120 (B) 240
(A) 120 (B) 240 (C) 360 (D) 480
(C) 243 (D) 486 23. Let m be the number of ways in which two couples
can be seated on 4 chairs in a row so that no wife is
PERMUTATION
next to her husband and n be the number of ways in
10
13. Value of r when Pr = 720 is : which they can be seated in 4 chairs in a circle. In
(A) 3 (B) 5 the latter case, rotations are considered different
(C) 7 (D)None of these configurations. Then :
(A) m = n (B) m = 2n
14. Number of ways that can 3 scholarships of unequal
(C) m = 4n (D) m = 8n
value be awarded to 17 candidates, such that no
candidate gets more than one scholarship is : 24. How many four digit numbers will have all the four
(A) 17
P1 (B) 17
P17 digits distinct ?
(A) 4536 (B) 3024
17
(C) P3 (D) None of these (C) 5040 (D) 9000
15. Number of ways in which 17 billiard balls can be
25. How many six-digit numbers are there in which no
arranged in a row if 7 are black, 6 red and 4 white is :
digit is repeated, even digits appear at even places,
17 17 odd digits appear at odd places and the number is
(A) P7 17 P6 17 P4 (B) P3
divisible by 4?
17 ! 17 (A) 3600 (B) 2700
(C) (D) C7 17 C 6 17 C 4
7!6! 4! (C) 2160 (D) 1440

PAGE # 166
COMBINATION 36. Number of ways in which 7 boys can be seated at a
n n n n round table so that 2 particular boys are separated is :
26. If Pr  Pr 1 and Cr  Cr 1 ,then the values of (A) 120 (B) 240
n and r is : (C) 360 (D) 480
(A) n = 4, r = 2 (B) n = 4, r = 3
37. Four gentlemen and four ladies are to sit at a round
(C) n = 3, r = 2 (D) n = 3, r = 1
table so that no two gentlemen sit together. Then,
27. In an examination, a student has to answer 4 the number of ways in which this can be done is :
questions out of 5 questions, questions 1 and 2 are (A) (4!) (3!) (B) (4!)2
however compulsory. Number of ways in which the (C) 7! (D) 8!
student can make a choice is :
38. The number of rectangles that can be obtained by
(A) 6 (B) 3
(C) 2 (D) 60 joining four of the twelve vertices of a 12-sided regu-
lar polygon is :
28. Out of 7 consonants and 4 vowels, number of words of (A) 66 (B) 30
3 consonants and 2 vowels is : (C) 24 (D) 15
(A) 462 (B) 2800
(C) 25200 (D) 24540
Directions Q.29 – Q. 30
1. Figure shows a square gird of order 3, which of the
There are 3 candidates for a Classical, 5 for a following is correct formula for the total number of
Mathematical, and 4 for a Natural science scholarship. squares in a similar grid of order n.
n 2 n2
29. If C8 : P4 = 57 : 16, then the value of n is :
(A) 16 (B) 17
(C) 18 (D) 19
30. For the post of 5 teachers, there are 23 applicants, 2
posts are reserved for SC candidates and there are 7
SC candidates among the applicants. In how many [IJSO-2014]
ways can the selection be made.
(A) 11760 (B) 1170 n(n  1) n2 (n  1)2
(C) 16710 (D)None of these (A) (B)
2 4
31. A group of 6 students comprised of 3 boys and 3
n(n  1)(2n  1) n(n  1)(n  2)
girls. Number of ways could they be arranged in a (C) (D)
straight line such that the girls and the boys occupy 6 6
alternate positions is :
(A) 36 (B) 72 2. How many triangles are there in this figure?
(C) 108 (D) 144
32. Different words can be formed using the letter
‘HALLUCINATION’ if all vowels occupy odd places is :
[IJSO-2015]
7
6! 7! 7
7!
(A) C6 × × (B) C6 × 6 ! ×
2! 2! 2! 2! 2! 2!
6! (A) 50 (B) 70
(C) 7C6 × × 7! (D) 7C6 × 6! × 7!
2! 2! (C) 84 (D) 91
33. Total number of triangles that can be formed by joining 3. let the number of rectangles formed by 6 horizontal
the vertices of the polygon that has the same number and 4 vertical lines be n and those formed by 5 vertical
of diagonals as its sides is : and 5 horizontal lines be m then we have
(A) 5 (B) 9 [IJSO-2016]
(C) 8 (D) 10 (A) n = m (B) n  m + 1
Directions Q.34 – Q. 35 (C) m  n (D) m > n + 5
There are 10 points in a plane. Except for 4 points 4. The number of squares formed by 5 vertical and 4
which are collinear no three points are in a straight horizontal lines (all are equispaced) is
line. [IJSO-2016]
34. The number of straight line obtained by joining (A) 60 (B) 20
these points is : (C) 40 (D) 46
(A) 40 (B) 50
(C) 60 (D) 70
35. Number of triangles that can be formed with the
vertices as these points is :
(A) 120 (B) 117
(C) 116 (D) 110

PAGE # 167
ANSWER KEY
NUMBER SYSTEM

EXERCISE#1
Q. 1 2 3 4 5 6 7 8 9 10 11 12 13 14 15
Ans. B B C A C B A B D C D A A D C
Q. 16 17 18 19 20 21 22 23 24 25 26 27 28 29 30
Ans. B A A B D C B A A D C A A B A
Q. 31 32 33 34 35 36 37 38 39 40 41 42 43 44 45
Ans. D C C D B A C D C C D D A A C
Q. 46 47 48 49 50 51 52 53 54 55 56 57 58 59 60
Ans. D A A C C A B D B C B C B B B
Q. 61 62 63 64 65 66 67 68 69 70 71 72 73 74 75
Ans. A A B D A D B A A B B B B C A
Q. 76 77 78 79 80 81 82 83 84 85
Ans. A B C D D A C C C D

EXERCISE # 2
Que. 1 2 3 4 5 6 7 8 9 10 11 12 13 14 15
Ans. AD B D B D A B B B B D D D C A
Que. 16 17 18 19 20 21 22 23 24 25 26 27 28 29 30
Ans. B D D B B B D D D D A B C C C
Que. 31 32 33 34 35 36 37 38 39 40 41 42
Ans. A C D B D C B B D B C C

COMMERCIAL MATHEMATICS
EXERCISE#1
Q. 1 2 3 4 5 6 7 8 9 10 11 12 13 14 15
Ans. C A B D C B A A B D B A D B D
Q. 16 17 18 19 20 21 22 23 24 25 26 27 28 29 30
Ans. B B D B A A B B C D D D C A D
Q. 31 32 33 34 35 36 37 38 39 40 41 42 43 44 45
Ans. D B D C C B C B B A C B B C B
Q. 46 47 48 49 50 51 52 53 54 55 56 57 58 59 60
Ans. C C C B C C C A D A D D B D A
Q. 61 62 63 64 65 66 67 68 69 70 71 72 73 74 75
Ans. A D B C A B B B C C B A D C C
Q. 76 77 78 79 80 81 82 83 84 85 86 87 88 89 90
Ans. C A A D B A A A D A D B B A A
Q. 91 92 93 94 95 96 97 98 99 100 101 102 103 104 105
Ans. A D B B A A A B B A A A D C D
Q. 106 107
Ans. C A

EXERCISE#2
Q. 1 2 3 4 5 6 7 8
Ans. B C A A C C D D

PAGE # 168
POLYNOMIALS
EXERCISE # 1
Q. 1 2 3 4 5 6 7 8 9 10 11 12 13 14 15
Ans. B B C D B B A D A D D A B B B
Q. 16 17 18 19 20 21 22 23 24 25 26 27 28 29 30
Ans. D B B A A D D D B D B C A A A
Q. 31 32 33 34 35 36 37
Ans. B C A C C A B

EXERCISE # 2
Que. 1 2 3 4 5 6 7 8 9 10 11 12 13 14 15

Ans. D A B C C A B C C D B C B B C

LINEAR EQUATION IN TWO VARIABLES


EXERCISE#1

Q. 1 2 3 4 5 6 7 8 9 10 11 12 13 14 15
Ans. C B B D D A C A B C D A D B C
Q. 16 17
Ans. A D

EXERCISE#2
Que. 1 2 3 4 5 6 7 8 9

Ans. D D D B A D ABC C D

LINEAR INEQUATIONS
EXERCISE#1

Q. 1 2 3 4 5 6 7 8 9 10 11 12 13 14 15

Ans. D C A B A D C D D C B A A C D

Q. 16 17 18

Ans. A D A

EXERCISE#2
Que. 1 2 3 4

Ans. D A A C

PAGE # 169
QUADRATIC EQUATIONS
EXERCISE#1
Que. 1 2 3 4 5 6 7 8 9 10 11 12 13 14 15
Ans. C D C C D B A D A C C C C A C
Que. 16 17 18 19 20 21 22 23
Ans. C B D C A A B C

EXERCISE#2
Que. 1 2 3 4 5 6 7 8 9 10

Ans. B CD B C C B A D D A

PROGRESSIONS
EXERCISE#1
Q. 1 2 3 4 5 6 7 8 9 10 11 12 13 14 15
Ans. B D A B A A C C C C A A A A C
Q. 16 17 18 19 20 21 22 23 24 25 26 27
Ans. C B D B C D D A B B C D

EXERCISE#2
Que. 1 2 3 4 5 6 7 8 9 10 11 12 13 14 15

Ans. D A A B C A B C B B C D B D B

Que. 16 17 18 19 20

Ans. C B B A B

TRIGONOMETRY
EXERCISE#1
Q. 1 2 3 4 5 6 7 8 9 10 11 12 13 14 15
Ans. B B A D A B B D D C C C C D B
Q. 16 17 18 19 20 21 22 23 24 25 26 27 28 29 30
Ans. B C A A B D A A B C C D C A A

EXERCISE#2
Que. 1 2 3 4 5 6 7 8 9 10 11 12 13 14

Ans. D A D B A B B D D A B C A D

PAGE # 170
CO-ORDINATE GEOMETRY
EXERCISE#1
Q. 1 2 3 4 5 6 7 8 9 10 11 12 13 14 15
Ans. C B B C D D A D D B C A B B B
Q. 16 17 18 19 20 21 22 23 24 25 26 27 28 29 30
Ans. D C D A C AC C B A B C C D B C
Q. 31 32 33 34 35 36 37 38 39 40 41 42 43 44 45
Ans. A A A D C A C A B C B BD A A A
Q. 46 47 48 49 50 51 52 53 54 55 56 57 58 59 60
Ans. C D A C A B C D A B B C C C D
Q. 61 62 63 64 65 66 67 68
Ans. B A AC D D B ACD B

EXERCISE#2
Que. 1 2 3 4 5

Ans. D B D C D

S ET S
EXERCISE#1
Q. 1 2 3 4 5 6 7 8 9 10 11 12 13 14 15

An s. A B D C B D A D C D D A A D B

Q. 16 17 18 19 20 21 22 23 24 25 26 27 28 29 30

An s. D B A B A C A C A C B B A C D

Q. 31 32 33 34 35 36

An s. D B B B A D

EXERCISE#2
Que. 1

Ans. D

PLANE GEOMETRY - I
EXERCISE#1
Q. 1 2 3 4 5 6 7 8 9 10 11 12 13 14 15

Ans. A B D B D C C A C B D D C B B

Q. 16 17 18 19 20 21 22 23 24 25 26 27 28 29 30

Ans. B B B A C A C A B A C B B C B

Q. 31 32 33 34 35 36 37 38 39 40 41 42 43 44 45

Ans. A B B B C B C B C B A C A B C

Q. 46

Ans. C

EXERCISE#2
Q ue . 1 2 3 4 5 6 7 8 9 10 11 12 13 14 15

Ans . C B C B C B A D D D B B C C B
Q ue . 16 17 18 19 20 21 22 23 24 25 26 27 28

Ans . A B A C A C B D D B C C C

PAGE # 171
PLANE GEOMETRY - II
EXERCISE # 1
Q. 1 2 3 4 5 6 7 8 9 10 11 12 13 14 15

Ans. D A C D D D D B C C B A B A D

Q. 16 17 18 19 20 21 22 23 24 25 26 27 28 29 30

Ans. B C D C D D A D B B C B D D A

Q. 31 32 33 34 35 36 37 38 39 40 41 42 43 44 45

Ans. D D D A C B B D D D C B B C C

EXERCISE # 2
Que. 1 2 3 4 5 6 7 8 9 10 11 12 13 14 15
Ans. D A D A D B B C C C A B A C D
Que. 16 17 18
Ans. C C B

MENSURATION
EXERCISE#1

Q. 1 2 3 4 5 6 7 8 9 10 11 12 13 14 15
Ans. A D A C A C B A D C C D D A C
Q. 16 17 18 19 20 21 22 23 24 25 26 27 28 29 30
Ans. A C A B A A B B A D B A A A B
Q. 31
Ans. B

EXERCISE#2

Que. 1 2 3 4 5 6 7 8 9 10 11 12 13 14 15

Ans. C D B B C C A B C D A B D D A

Que. 16 17 18 19 20 21 22

Ans. A A B D A B C

PERMUTATION & COMBINATION


EXERCISE#1
Q. 1 2 3 4 5 6 7 8 9 10 11 12 13 14 15

Ans. B D C A B A B A C D B C A C C

Q. 16 17 18 19 20 21 22 23 24 25 26 27 28 29 30

Ans. B C D D B B B D A D C B C D A

Q. 31 32 33 34 35 36 37 38

Ans. B A D A C D A D

EXERCISE#2
Que. 1 2 3 4
Ans. C D D B

PAGE # 172

You might also like